You are on page 1of 455

Paper 1: Advanced Financial Reporting [CAP III]

Attempt Wise Compilation of Revision Test Paper from June 2017 to 2021 December

Table of Contents
1. RTP 2017 June 2
2. RTP 2017 Dec 43
3. RTP 2018 June 71
4. RTP 2018 Dec 107
5. RTP 2019 June 148
6. RTP 2019 Dec 188
7. RTP 2020 June 217
8. RTP 2020 Dec 248
9. RTP 2021 June 272
10. RTP 2021 Dec 302
Page 2 of 328
Paper1: Advanced Financial Reporting [RTP 2017 June]

Questions
1. On 1 January 20X6, Gardenbugs Co received a NRs. 30,000,000 government grant
relating to equipment which cost NRs. 90,000,000 and had a useful life of six years. The grant
was netted off against the cost of the equipment. On 1 January 20X7, when the equipment had a
carrying amount of NRs. 50,000,000 , its use was changed so that it was no longer being used in
accordance with the grant. This meant that the grant needed to be repaid in full but by 31
December 20X7, this had not yet been done.

Pass the required journal entries relating to refund of the grant in the books of Gardenbugs Ltd.
For the year ended 31 December 20X7
2.
a. Shiba Co entered into a non-cancellable four-year operating lease to hire a photocopier on 1
January 20X7. The terms of the lease agreement were as follows:
Operating lease rental 5,000 per annum
Cash back incentive received at the start of the lease 1,000
Useful life of the asset Eight years
What is the charge in the statement of profit or loss of Shiba Co for the year ended 31 December
20X7 in respect of this operating lease?

b. On 1 October 20X1, Bash Co borrowed NRs.6m for a term of one year, exclusively to finance
the construction of a new piece of production equipment. The interest rate on the loan is
6% and is payable on maturity of the loan. The construction commenced on 1 November
20X1 but no construction took place between 1 December 20X1 to 31 January 20X2 due
to employees taking industrial action. The asset was available for use on 30 September 20X2
having a construction cost of NRs.6m.
What is the carrying amount of the production equipment in Bash Co‘s statement of
financial position as at 30 September 20X2?

c. Petre owns 100% of the share capital of the following companies. The directors are unsure of
whether the investments should be consolidated.
Which of the following circumstances would the investment NOT be consolidated?

 Petre has decided to sell its investment in Alpha as it is loss-making; the directors
believe its exclusion from consolidation would assist users in predicting the group‘s future
profits
 Beta is a bank and its activity is so different from the engineering activities of the rest of the
group that it would be meaningless to consolidate it
 Delta is located in a country where local accounting standards are compulsory and these are
not compatible with NFRS used by the rest of the group
 Gamma is located in a country where a military coup has taken place and Petre has lost
control of the investment for the foreseeable future

3. Aphrodite Co has a year end of 31 December and operates a factory which makes computer
chips for mobile phones. It purchased a machine on 1 July 20X3 for NRs.80,000 which had
Page 3 of 328
Paper1: Advanced Financial Reporting [RTP 2017 June]

a useful life of ten years and is depreciated on the straight-line basis, time apportioned in
the years of acquisition and disposal. The machine was revalued to NRs.81,000 on 1 July 20X4.
There was no change to its useful life at that date.

A fire at the factory on 1 October 20X6 damaged the machine leaving it with a lower operating
capacity. The accountant considers that Aphrodite Co will need to recognise an impairment
loss in relation to this damage. The accountant has ascertained the following information at 1
October 20X6:
(1) The carrying amount of the machine is NRs.60,750.
(2) An equivalent new machine would cost NRs.90,000.
(3) The machine could be sold in its current condition for a gross amount of NRs.45,000.
Dismantling costs would amount to NRs.2,000.
(4) In its current condition, the machine could operate for three more years which gives it
a value in use figure of NRs.38,685.

Required:
a. In accordance with NAS 16 Property, Plant and Equipment, what is the depreciation charged
to Aphrodite Co‘s profit or loss in respect of the machine for the year ended 31 December
20X4?
b. What is the total impairment loss associated with Aphrodite Co‘s machine at 1 October
20X6?
c. On 1 July 20X7, it is discovered that the damage to the machine is worse than originally
thought. The machine is now considered to be worthless and the recoverable amount of the
factory as a cash-generating unit is estimated to be NRs.950,000.
At 1 July 20X7, the cash-generating unit comprises the following assets:
NRs.‘000
Building 500
Plant and equipment (including the damaged machine at a carrying amount of 335
NRs.35,000)
Goodwill 85
Net current assets (at recoverable amount) 250
1,170
In accordance with NAS 36, what will be the carrying amount of Aphrodite Co‘s plant and
equipment when the impairment loss has been allocated to the cash-generating unit?

4. After preparing a draft statement of profit or loss (before interest and tax) for the year ended 31
March 20X6 (before any adjustments which may be required by notes (i) to (iv) below), the
summarised trial balance of Triage Co as at 31 March 20X6 is:
NRs.‟000 NRs.‟000
Equity shares of NRs.1 each 50,000
Retained earnings as at 1 April 20X5 3,500
Draft profit before interest and tax for year ended 31 March 30,000
20X6
6% convertible loan notes (note (i)) 40,000
Leased property (original life 25 years) – at cost (note (ii)) 75,000
Plant and equipment – at cost (note (ii)) 72,100
Page 4 of 328
Paper1: Advanced Financial Reporting [RTP 2017 June]

Accumulated amortisation/depreciation at 1 April 20X5: leased 15,000


property
plant and equipment 28,100
Trade receivables (note (iii)) 28,000
Other current assets 9,300
Current liabilities 17,700
Deferred tax (note (iv)) 3,200
Interest payment (note (i)) 2,400
Current tax (note (iv) 700
187,500 187,500
The following notes are relevant:
(i) Triage Co issued 400,000 NRs.100 6% convertible loan notes on 1 April 20X5. Interest is
payable annually in arrears on 31 March each year. The loans can be converted to equity shares
on the basis of 20 shares for each NRs.100 loan note on 31 March 20X8 or redeemed at par for
cash on the same date. An equivalent loan without the conversion rights would have required an
interest rate of 8%.
The present value of NRs.1 receivable at the end of each year, based on discount rates of 6% and
8%, are:
End of year 6% 8%
1 0·94 0·93
2 0·89 0·86
3 0·84 0·79
(ii) Non-current assets:
The directors decided to revalue the leased property at NRs.66·3m on 1 October 20X5. Triage
Co does not make an annual transfer from the revaluation surplus to retained earnings to reflect
the realisation of the revaluation gain; however, the revaluation will give rise to a deferred tax
liability at the company‘s tax rate of 20%.
The leased property is depreciated on a straight-line basis and plant and equipment at 15% per
annum using the reducing balance method. No depreciation has yet been charged on any non-
current assets for the year ended 31 March 20X6.
(iii) In September 20X5, the directors of Triage Co discovered a fraud. In total, NRs.700,000
which had been included as receivables in the above trial balance had been stolen by an
employee. NRs.450,000 of this related to the year ended 31 March 20X5, the rest to the
current year. The directors are hopeful that 50% of the losses can be recovered from the
company‘s insurers.
(iv) A provision of NRs.2·7m is required for current income tax on the profit of the year to 31
March 20X6. The balance on current tax in the trial balance is the under/over provision of
tax for the previous year. In addition to the temporary differences relating to the information
in note (ii), at 31 March 20X6, the carrying amounts of Triage Co‘s net assets are NRs.12m more
than their tax base.

Required:
(a) Prepare a schedule of adjustments required to the draft profit before interest and tax (in
the above trial balance) to give the profit or loss of Triage Co for the year ended 31 March
20X6 as a result of the information in notes (i) to (iv) above.
(b) Prepare the statement of financial position of Triage Co as at 31 March 20X6.
Page 5 of 328
Paper1: Advanced Financial Reporting [RTP 2017 June]

(c) Calculate the diluted earnings per share for Triage Co for the year ended 31 March 20X6

5. The finance director of Downing Co has correctly calculated the company‘s basic and diluted
earnings per share (EPS) to be disclosed in the financial statements for the year ended 31
March 2016 at 14.82 and 11.94 cents respectively. The dilution of EPS is due to issue of
convertible loan notes due for redemption on 31 March 2018.
On seeing these figures, the chief executive officer (CEO) is concerned that the market will react
badly knowing that the company‘s EPS in the near future will be only 11.94, a fall of over 19%
on the current year‘s basic EPS.

Required:
Explain why and what aspect of Downing Co‘s capital structure is causing the basic EPS to be
diluted and comment on the validity of the CEO‘s concerns.

6. The following details relate to two items of property, plant and equipment (A and B) owned by
Purnabiram Ltd. which are depreciated on a straight-line basis with no estimated residual value:
Item A Item B
Estimated useful life at acquisition 8 years 6 years
NRs.‘000 NRs.‘000
Cost on 1 April 2010 240,000 120,000
Accumulated depreciation (two years) (60,000) (40,000)
Carrying amount at 31 March 2012 180,000 80,000
Revaluation on 1 April 2012:
Revalued amount 160,000 112,000
Revised estimated remaining useful life 5 years 5 years
Subsequent expenditure capitalised on 1 April nil 14,400
2013
At 31 March 2014 item A was still in use, but item B was sold (on that date) for NRs.70 million.
Note: Purnabiram Ltd. makes an annual transfer from its revaluation surplus to retained
earnings in respect of excess depreciation.

Required:
Prepare extracts from:
(i) Purnabiram Ltd.‘s statements of profit or loss for the years ended 31 March 2013 and 2014 in
respect of charges (expenses) related to property, plant and equipment;
(ii) Purnabiram Ltd.‘s statements of financial position as at 31 March 2013 and 2014 for the
carrying amount of property, plant and equipment and the revaluation surplus.

7. The following issues have arisen during the preparation of Skeptic‘s draft financial
statements for the year ended 31 March 2014:
(i) From 1 April 2013, the directors have decided to reclassify research and amortised
development costs as administrative expenses rather than its previous classification as cost of
sales. They believe that the previous treatment unfairly distorted the company‘s gross profit
margin.
(ii) Skeptic has two potential liabilities to assess. The first is an outstanding court case
concerning a customer claiming damages for losses due to faulty components supplied by
Page 6 of 328
Paper1: Advanced Financial Reporting [RTP 2017 June]

Skeptic. The second is the provision required for product warranty claims against 200,000 units
of retail goods supplied with a one-year warranty. The estimated outcomes of the two liabilities
are:
Court case Product warranty claims
10% chance of no damages awarded 70% of sales will have no claim
65% chance of damages of NRs.4 20% of sales will require a NRs.25
million repair
25% chance of damages of NRs.6 10% of sales will require a NRs.120
million repair

(iii) On 1 April 2013, Skeptic received a government grant of NRs.8 million towards the
purchase of new plant with a gross cost of NRs.64 million. The plant has an estimated life of 10
years and is depreciated on a straight-line basis.
One of the terms of the grant is that the sale of the plant before 31 March 2017 would trigger a
repayment on a sliding scale as follows:
Sale in the year Amount of
ended repayment
31 March 2014 100%
31 March 2015 75%
31 March 2016 50%
31 March 2017 25%
Accordingly, the directors propose to credit to the statement of profit or loss NRs.2 million
(NRs.8 million x 25%) being the amount of the grant they believe has been earned in the
year to 31 March 2014. Skeptic accounts for government grants as a separate item of
deferred credit in its statement of financial position. Skeptic has no intention of selling the
plant before the end of its economic life.

Required:
Advise, and quantify where possible, how the above items (i) to (iii) should be treated in
Skeptic‘s financial statements for the year ended 31 March 2014.

8. On 1 January 2014, Chandra Ltd. acquired 80% of the equity share capital of Bindu Ltd.. The
consideration was satisfied by a share exchange of two shares in Chandra Ltd. for every three
acquired shares in Bindu Ltd.. At the date of acquisition, shares in Chandra Ltd. and Bindu Ltd.
had a market value of NRs.3 and NRs.2·50 each respectively. Chandra Ltd. will also pay cash
consideration of 27·5 Chandra Ltd. on 1 January 2015 for each acquired share in Bindu Ltd..
Chandra Ltd. has a cost of capital of 10% per annum.

None of the consideration has been recorded by Chandra Ltd.

Below are the summarised draft financial statements of both companies.


Statements of profit or loss and other comprehensive income for the year ended 30 September
2014
Chandra Ltd. Bindu Ltd.
NRs.‘000 NRs.‘000
Page 7 of 328
Paper1: Advanced Financial Reporting [RTP 2017 June]

Revenue 62,600 30,000


Cost of sales (45,800) (24,000)
Gross profit 16,800 6,000
Distribution costs (2,000) (1,200)
Administrative expenses (3,500) (1,800)
Finance costs (200) (nil)
Profit before tax 11,100 3,000
Income tax expense (3,100) (1,000)
Profit for the year 8,000 2,000
Other comprehensive income:
Gain on revaluation of property (note 1,500 nil
(i))
Total comprehensive income 9,500 2,000

Statements of financial position as at 30 September 2014


Chandra Ltd. Bindu Ltd.
NRs.‘000 NRs.‘000
Assets
Non-current assets
Property, plant and equipment 18,700 13,900
Investments: 10% loan note from Bindu Ltd. 1,000 nil
(note (ii))
19,700 13,900
Current assets
Inventory (note (iii)) 4,300 1,200
Trade receivables (note (iv)) 4,700 2,500
Bank nil 300
9,000 4,000
Total assets 28,700 17,900
Equity and liabilities
Equity
Equity shares of NRs.1 each 10,000 9,000
Revaluation surplus (note (i)) 2,000 nil
Retained earnings 6,300 3,500
18,300 12,500
Non-current liabilities
10% loan notes (note (ii)) 2,500 1,000
Current liabilities
Trade payables (note (iv)) 3,400 3,600
Bank 1,700 nil
Current tax payable 2,800 800
7,900 4,400
Total equity and liabilities 28,700 17,900

The following information is relevant:


Page 8 of 328
Paper1: Advanced Financial Reporting [RTP 2017 June]

(i) At the date of acquisition, the fair values of Bindu Ltd.‘s assets and liabilities were equal to
their carrying amounts with the exception of Bindu Ltd.‘s property which had a fair value of
NRs.4 million above its carrying amount. For consolidation purposes, this led to an increase
in depreciation charges (in cost of sales) of NRs.100,000 in the post-acquisition period to 30
September 2014. Bindu Ltd. has not incorporated the fair value property increase into its entity
financial statements.
The policy of the Chandra Ltd. group is to revalue all properties to fair value at each year end.
On 30 September 2014, the increase in Chandra Ltd.‘s property has already been recorded,
however, a further increase of NRs.600,000 in the value of Bindu Ltd.‘s property since its
value at acquisition and 30 September 2014 has not been recorded.
(ii) On 30 September 2014, Chandra Ltd. accepted a NRs.1 million 10% loan note from Bindu
Ltd..
(iii) Sales from Chandra Ltd. to Bindu Ltd. throughout the year ended 30 September 2014 had
consistently been NRs.300,000 per month. Chandra Ltd. made a mark-up on cost of 25% on all
these sales. NRs.600,000 (at cost to Bindu Ltd.) of Bindu Ltd.‘s inventory at 30 September 2014
had been supplied by Chandra Ltd. in the post-acquisition period.
(iv) Chandra Ltd. had a trade receivable balance owing from Bindu Ltd. of NRs.1·2 million
as at 30 September 2014. This differed to the equivalent trade payable of Bindu Ltd. due to a
payment by Bindu Ltd. of NRs.400,000 made in September 2014 which did not clear Chandra
Ltd.‘s bank account until 4 October 2014. Chandra Ltd.‘s policy for cash timing differences is to
adjust the parent‘s financial statements.
(v) Chandra Ltd.‘s policy is to value the non-controlling interest at fair value at the date of
acquisition. For this purpose Bindu Ltd.‘s share price at that date can be deemed to be
representative of the fair value of the shares held by the non-controlling interest.
(vi) Due to rePaisa adverse publicity concerning one of Bindu Ltd.‘s major product lines, the
goodwill which arose on the acquisition of Bindu Ltd. has been impaired by NRs.500,000 as at
30 September 2014. Goodwill impairment should be treated as an administrative expense.
(vii) Assume, except where indicated otherwise, that all items of income and expenditure accrue
evenly throughout the year.

Required:
(a) Prepare the consolidated statement of profit or loss and other comprehensive income for
Chandra Ltd. for the year ended 30 September 2014.
(b) Prepare the consolidated statement of financial position for Chandra Ltd. as at 30 September
2014.

9. P Ltd. owns 80% of S and 40% of J and 40% of A. J is jointly controlled entity and
A is an
associate. Summarised Balance Sheets of four companies as on 31.03.1 5 are:
NRs. Lakhs
Particulars P Ltd. S Ltd. J Ltd. A Ltd.
Investment in S 800 - - -
Investment in J 600 - - -
Investment in A 600 - - -
Fixed assets 1000 800 1400 1000
Current assets 2200 3300 3250 3650
Page 9 of 328
Paper1: Advanced Financial Reporting [RTP 2017 June]

Total 5200 4100 4650 4650


Liabilities:
Share capital
Equity share 1000 400 800 800
Retained earnings 4000 3400 3600 3600
Trade payables 200 300 250 250
Total 5200 4100 4650 4650

P td. acquired shares in S‗ many years ago when S‗ retained earnings were 520 lakhs.
P td. acquired its shares in J‗ at the beginning of the year when J‗ retained earnings were 400
lakhs. P td. acquired its shares in A‗ on 01.04.14 when A‗ retained earnings were 400
lakhs.
The balance of goodwill relating to S had been written off three years ago. The value
of goodwill in J‗ remains unchanged.
Prepare the Consolidated Balance Sheet of P Ltd. as on 31.03.15.

10. Prepare the Consolidated Balance Sheet as on December 31, 2011 of group of companies A Ltd.,
B Ltd. and C Ltd. Their summarized balance sheets on that date are given below:

Particulars A Ltd. B Ltd. C Ltd.


Share Capital (share of ` 100 1,25,000 1,00,000 60,000
each)
Reserves 18,000 10,000 7,200
Profit & Loss A/c 16,000 4,000 5,000
Trade Payables 7,000 3,000 —
A Ltd. — 7,000 —
C Ltd. 3,300 — —
Total 1,69,300 1,24,000 72,200
Assets
Fixed Assets 28,000 55,000 37,400
Investments in shares-
B Ltd. 85,000 — —
C Ltd. — 53,000 —
Inventory 22,000 6,000 —
B Ltd. 8,000 — —
Trade Receivables 26,300 10,000 31,500
A Ltd. — — 3,300
Total 1,69,300 1,24,000 72,200
Other information:
(i) A Ltd. holds 750 shares in B Ltd. and B Ltd. holds 400 shares in C Ltd. These holdings were
acquired on 30th June, 2011
(ii) On 1st January, 2011 the following balances stood in the books of B Ltd. and C Ltd.

B Ltd. C Ltd.
Reserves 8,000 6,000
Page 10 of 328
Paper1: Advanced Financial Reporting [RTP 2017 June]

P & L Account 1,000 1,000

iii) C Ltd., sold goods costing ` 2,500 to B Ltd. for ` 3,100. These goods still remain unsold.

11. An entity purchases equipment from a foreign supplier for €6 million on March 31, 20X6, when
the ex-
change rate was €2 = $1. The entity also sells goods to a foreign customer for €3.5 million on
April 30,
20X6, when the exchange rate was €1.75 = $1. At the entity‘s year-end of May 31, 20X6, the
amounts
have not been paid. The closing exchange rate was €1.5 = $1. The entity‘s functional currency is
the
dollar.

Required
Calculate the exchange differences that would be recorded in profit or loss for the period ending
May 31,
20X6.

12. An entity commenced business on January 1, 20X6, with an opening share capital of $2 million.
The income statement and closing balance sheet follow:

Income Statement for the year ended December 31,


20X6
$m
Revenue 32
Cost of sales -10
Gross profit 22
Distribution costs -8
Administrative expenses -2
.Profit before tax
Profit before tax 12
Tax expense -4
Profit for period 8

Balance Sheet at December 31, 20X6


$m
Share capital 2
Retained earnings 8

Trade payables 4
Total equity and liabilities 14
Land (nondepreciable) acquired December
31, 20X6 8
Page 11 of 328
Paper1: Advanced Financial Reporting [RTP 2017 June]

Inventories 4
Trade receivables 2
Total assets 14

The functional currency is the dollar, but the entity wishes to present its financial statements
using the euro as its presentational currency. The entity translates the opening share capital at the
closing rate. The exchange rates in the period were
$1 =
January 1, 20X6 €1
December 31, 20X6 €2
Average rate €1.5

Required
Translate the financial statements from the functional currency to the presentational currency.

13. Hawa Pvt. Ltd. is a newly established enterprise. It was set up by an entrepreneur who is
generally interested in the business of providing engineering and operational support services to
aircraft manufacturers. Hawa Pvt. Ltd., through the contacts of its owner, received a confirmed
order from a well-known aircraft manufacturer to develop new designs for ducting the air
conditioning of their aircraft. For this project, Hawa Pvt. Ltd. needed funds aggregating to
NRs.1 million. It was able to convince venture capitalists and was able to obtain funding of
NRs.1 million from two Silicon Valley venture capitalists. The expenditures Hawa Pvt. Ltd.
incurred in pursuance of its research and development project follow, in chronological order:
• January 15, 20X5: Paid NRs.175,000 toward salaries of the technicians (engineers and
consultants)
• March 31, 20X5: Incurred NRs.250,000 toward cost of developing the duct and producing the
test model
• June 15, 20X5: Paid an additional NRs.300,000 for revising the ducting process to ensure that
product could be introduced in the market
• August 15, 20X5: Developed, at a cost of NRs.80,000, the first model (prototype) and tested it
with the air conditioners to ensure its compatibility
• October 30, 20X5: A focus group of other engineering providers was invited to a conference
for the introduction of this new product. Cost of the conference aggregated to NRs.50,000.
• December 15, 20X5: The development phase was completed and a cash flow budget was
prepared. Net profit for the year 20X5 was estimated to equal NRs.900,000.

Required
What is the proper accounting treatment for the various costs incurred during 20X5?

14. Commodity Traders Ltd. is a company engaged in commodities trading. The company recently
obtained NRs.5 million short-term borrowing which is secured by the company‘s inventory of
1,000 tons of copper which it purchased at a cost of NRs.5.2 million. The bank has obligated
Commodity Traders Ltd. to provide additional collateral in event the value of copper inventories
fall below NRs.5 million. On 1 January 2015, Commodity Traders Ltd. sold the inventories
forward by entering into a 12-month futures contract at price of NRs.5,200 per ton.
Page 12 of 328
Paper1: Advanced Financial Reporting [RTP 2017 June]

On 30 June 2015, i.e. the financial year end of Commodity Traders Ltd., price of copper fell to
NRs.4,900 per ton.
Identify the hedged instrument and the hedging instrument and journalize the transactions.

15. Entity C prepares financial statements to 30 September each year. On 1 October 2011 entity C
granted 200 options to 20 senior executives. The options only vest if entity C‘s share price
reaches NRs.20 on set dates and the relevant executives remain employed by the entity on those
dates. The options can vest:
 On 30 September 2013 if the share price reaches NRs.20 during the year ended 30
September 2013 and remains that at least that level until 30 September 2013.
 On 30 September 2014 if the share price reaches NRs.20 during the year ended 30
September 2014 and remains that at least that level until 30 September 2014.
 On 30 September 2015 if the share price reaches NRs.20 during the year ended 30
September 2015 and remains that at least that level until 30 September 2015.
 If the above conditions are not satisfied the options do not vest.

On 1 October 2011 the directors of entity C estimated that the fair value of a share option under
this scheme was NRs.18. This estimate was arrived at based upon the assumption that the options
would vest on 30 September 2015. In fact the share price reached NRs.20 on 1 June 2014 and
remained at or above that level until 30 September 2014. Therefore the options vested on 30
September 2014. The directors consistently estimated that all 20 executives would remain
employed by entity C over the relevant vesting period and all executives were in employment
when the options vested.
Show the impact of the share-based payment arrangement on the financial statements for the
years ended 30 September 2013 and 2014.

16. The Balance Sheet of Suyash Ltd .as on 31-3-2003 is as follows:


Liabilities Rs Assets Rs
Equity Shares of Rs 10 each 5,00,000 Building 5,00,000
10% Preference Shares of Rs 1,000,00 Machinery 3,00,000
100 each Stock 60,000
General Reserve 1,00,000 Debtors 50,000
Profit & Loss Account 50,000 Bank 50,000
Creditors 1,10,000
Provision for Income tax 40,000
Proposed Dividend 60,000
Total 9,60,000 Total 9,60,000

The profits of the company after charging depreciation but before income - tax @ 40% were as
follows for last five years;
1991 - Rs 1,00,000
2000 - Rs 1,40,000
2001 - Rs 1,60,000
2002 - Rs 1,70,000
2003 - Rs 1,80,000
Reasonable Return on Equity funds in this line of business is considered to be 10%.
Page 13 of 328
Paper1: Advanced Financial Reporting [RTP 2017 June]

 Find out the value of equity share under the Net Asset or Intrinsic Method after valuing
goodwill on the basis of five year‘s purchase of annual super profits.
 Also , ascertain the share value on the basis of profit earning capacity in relation to
 Average maintainable profits.
 Average dividend rate which was 8%, 9% ,7% for the last three years.
 Further calculate the number of shares to be purchased by an individual investor with an
amount of Rs 20,000 available for investment on the basis of appropriate fair value of the
relevant equity shares.

17. Famous Corporation has been preparing Value Added Statement for the past five years. The
Human Resource Manager of the Company has suggested introducing a Value Added Incentive
Scheme to motivate the Employees for their better performance.
To introduce the Scheme, it is proposed that the Best Index Performance (favourable to
Employer), i.e. Employee Costs to Added Value for the last five years, will be used as the Target
Index for future calculations of the bonus to be paid After the Target Index is determined, any
actual improvement in the Index will be rewarded. The Employer & the Employee will be
sharing any such improvement in the ratio 1:2. The bonus is given at the end of the year, after the
profit for the year is determined.

The following information is available: Value Added Statement for 5 years ( in Thousands)
Particulars 2010 2011 2012 2013 2014
Sales 5,600 7,600 9,200 10,400 12,000
Less: Cost of Bought in
Goods/Services 2,560 4,000 5,000 5,600 6,400
Added Value 3,040 3,600 4,200 4,800 5,600
Employee Costs 1,300 1,520 1,680 1,968 2,240
Dividend 200 300 400 480 600
Taxes 640 760 840 1 1,120
Depreciation 520 620 720 880 1,120
Debenture Interest 80 80 80 80 80
Retained Earning 300 320 480 392 440
Added Value 3,040 3,600 4,200 4,800 5,600

Summarized Profit and Loss Account for the year ended on 31st March 2015 (in Thousands)
Particulars Amount Amount
Income
Sales less Returns 13,600
Dividends and Interes 500
Miscellaneous Income 500 14,600
Expenditure:
Production
Cost of Materials 5,000
Wages & Salaries 1,800
Other Manufacturing Expenses 1,400 8,200
Page 14 of 328
Paper1: Advanced Financial Reporting [RTP 2017 June]

Administrative
Administration Salaries 600
Administration Expenses 600 1,200
Selling and Distribution Expenses
Selling and Distribution Salaries 120
Selling Expenses 400 520
Finance Cost
Debenture Interest 80
Depreciation 1,520
Total Expenditure 11,520
Profit before Taxation 3,080
Less: Provision for Taxation 770
Profit after Taxation 2,310

From the above information, prepare Value Added Statement for the year 2014–2015 and
determine the amount of Bonus Payable to Employees, if any.

18. Suppose that Tract Ore Co finishes its first year of operations in which all contract costs were
paid in cash and all progress billings and advances were received in cash. For contracts W, X and
Z only:
(a) contract costs include costs of materials purchased for use in the contract which have not
been used at the period end; and
(b) customers have advanced sums to the contractor for work not yet performed.
The relevant figures for all contracts at the end of Tract Ore's first year of trading are as follows.
W X Y Z Total
NRs. NRs.
Particulars m NRs. m NRs. m NRs. m m NRs. m
Contract revenue recognised 113.10 405.60 296.40 156.00 42.90 1,014.00
Contract expenses recognised (85.80) (351.00) (273.00) (195.00) (42.90) (947.70)
Expected losses recognised – – – (31.20) (23.40) (54.60)
Recognised profits less recognised losses 27.30 54.60 23.40 (70.20) (23.40) 11.70
Contract costs incurred in the period 85.80 437.60 386.10 195.00 85.80 1,190.30
Contract expenses recognised (85.80) (351.00) (73.00) (195.00) (42.90) (747.70)
Contract expenses that relate to future
activity recognised as an asset – 86.60 13.10 – 42.90 142.60
Contract revenue 113.10 405.60 296.40 156.00 42.90 1,014.00
Progress billings (78.00) (405.60) (296.40) (140.40) (42.90) (963.30)
Unbilled contract revenue 35.10 – – 15.60 – 50.70
Advances – 41.60 10.40 – 13.00 65.00

Required
Show the figures that should be disclosed under NAS 11.
Page 15 of 328
Paper1: Advanced Financial Reporting [RTP 2017 June]

19. These consolidated financial statements relate to the JYCE group for the year ended September
30,
20X4:
JYCE Group balance sheet at September 30, 20X4
NRs.m
Assets :
Noncurrent assets
Property, plant, and equipment 500
Goodwill 100
Investment in associate 70
670
Current assets 130
800
Equity and Liabilities:
Equity attributable to equity holders of
parent
Share capital 200
Retained earnings 400
600
Minority interest 50
Total equity 650
Noncurrent liabilities 60
Current liabilities 90
Total equity and liabilities 800

JYCE Group income statement for the year ended September 30, 20X4

Revenue 1800
Cost of sales -1200
Gross profit 600
Other income 60
Distribution costs -200
Administrative expenses -100
Other expenses -50
Finance costs -60
Share of profit of associates 10
Profit before tax 260
Income tax expense -70
Profit for the period 190
Attributable to:
Equity holders of the parent 176
Minority interest 14
190

This information is relevant to the production of the segmental information:


Page 16 of 328
Paper1: Advanced Financial Reporting [RTP 2017 June]

(a) The entity is organized for management purposes into three major operating divisions: office
furniture, office stationery, and computer products. There are other smaller operating divisions.
(b) The sales revenue for the major operating divisions is set out next.
Intersegment sales
Revenue eliminated on consolidation
NRs.m NRs.m
Office furniture 800 200
Office stationery 500 150
Computer products 400 80

There are no intersegment sales to the smaller operating divisions.


(c) The profit after taking into account the other income, distribution costs, and administrative
ex-
penses can be allocated in this way:

Percentage of
profit
Office furniture 50%
Office stationery 25%
Computer products 20%
Other divisions 5%
100%

(d) The ―other‖ expenses, finance costs, and income tax expense cannot be allocated to the seg-
ments on any reasonable basis.
(e) During the year, the office furniture division had purchased an investment in an associate.
The
profit shown in the income statement is after the elimination of intersegment profit of NRs.2
million.
(f) The next table shows the breakdown of segment assets and liabilities that are allocated to
segments.

Office Office
furniture stationery Computer products
NRs.m NRs.m NRs.m
Property, plant, and
equipment 300 100 80
Goodwill 60 30 10
Current assets 80 40 6
Noncurrent liabilities 30 21 4
Current liabilities 45 33 8

The remainder of the assets and liabilities relate to the other divisions except for an asset of
NRs.4 million and a liability of NRs.6 million that cannot be allocated.
Page 17 of 328
Paper1: Advanced Financial Reporting [RTP 2017 June]

Required
Produce a schedule that shows the information required for segment disclosures under NFRS 08,
Operating Segment.

20. A subsidiary sold goods costing NRs. 10 million to its parent for NRs. 11 million, and all of
these goods are still held in inventory at the year-end. Assume a tax rate of 30%. Explain the
deferred tax implications.

21. Write short notes on


a. What is economic value added and how is it calculated?
b. Corporate Social Reporting
c. Qualitative characteristics of Financial Statements
Page 18 of 328
Paper1: Advanced Financial Reporting [RTP 2017 June]

Suggested Answer

Solution to Q No. 1
The repayment of the grant must be treated as a change in accounting estimate. The
carrying amount of the asset must be increased as the netting off method has been used. The
resulting extra depreciation must be charged immediately to profit or loss.
Particulars Original As if no grant Adjustment
Cost 90,000,000 90,000,000
Grant (30,000,000) -
60,000,000 90,000,000
Depreciation (10,000,000) [1 yr] (30,000,000) [2 yr] 20,000,000
Carrying amount 50,000,000 [1/1/X7] 60,000,000 10,000,000
[31/12/X7]

Required Journal Entry


Dr. Depreciation Expense 20,000,000
Dr. PPE 10,000,000
Cr. Liability 30,000,000

Solution to Q No. 2 (a)


Answer:
Net total being paid over four years ((5,000 x 4 years) – 1,000) = 19,000
Annual charge spread evenly over the lease term (19,000/4 years) = 4,750

Solution to Q No. 2(b)


Answer
Production cost of PPE 6,000
Capitalisation of borrowing costs:
NRs. 6m x 6% x 9/12 = 270
Total cost capitalised (and carrying amount) at 30 September 20X2 = 6,270

Solution to Q No. 2(c)


Answer:
Gamma Ltd. The investment no longer meets the definition of a subsidiary (ability to control)
and therefore would not be consolidated

Solution to Q No. 3
Answers
a. Depreciation 1 January to 30 June 20X4 (80,000/10 x 6/12) = 4,000
Depreciation 1 July to 31 December 20X4 (81,000/9 x 6/12) = 4,500
Total depreciation = 8,500

b. Value in Use = 38,685


Fair Value less cost to sell = 45000-2000 = 43,000
Carrying Amount = 60,750
Page 19 of 328
Paper1: Advanced Financial Reporting [RTP 2017 June]

As VIU is lower than FV (less costs to sell), so impairment is 60,750 – 43,000 = NRs.17,750

c. The impairment loss of NRs.220 (1,170 – 950) is allocated as follows


NRs.‘000
Plant & Machinery (carrying amount of damaged 35
asset)
Goodwill 85
Total Impairment Loss recognized 120
Balance 100
Balance Amount of IL has to be charged proportionately to the remaining assets of the CGU
NRs.‘000
Component Carrying Amount Proportionate IL CA after
Impairment
Building 500 62.5 437.5
(500/800*100)
Plant & Machinery 300 37.5 262.5
Net Current Assets 250
Total 950

Solution to Q No. 4
Answer
(a) Triage Co – Schedule of adjustments to profit for the year ended 31 March 20X6

Draft profit before interest and tax per trial balance 30,000
Adjustments re:
Note (i)
Convertible loan note finance costs (w (i)) -3,023
Note (ii)
Amortisation of leased property (1,500 + 1,700 (w
(ii))) -3,200
Depreciation of plant and equipment (w (ii)) -6,600
Note (iii)
Current year loss on fraud (700 – 450 see below) -250
Note (iv)
Income tax expense (2,700 + 700 – 800 (w (iii))) -2,600
Profit for the year 14,327

The NRs.450,000 fraud loss in the previous year is a prior period adjustment (reported in the
statement of changes in equity).
The possible insurance claim is a contingent asset and should be ignored.

(b) Triage Co – Statement of financial position as at 31 March 20X6


Page 20 of 328
Paper1: Advanced Financial Reporting [RTP 2017 June]

Assets NRs.‘000 NRs.‘000


Non-current assets
Property, plant and equipment (64,600 + 37,400 (w 102,000
(ii)))
Current assets
Trade receivables (28,000 – 700 fraud) 27,300
Other current assets per trial balance 9,300 36,600
Total assets 138,600
Equity and liabilities
Equity
Equity shares of NRs.1 each 50,000
Other component of equity (w (i)) 2,208
Revaluation surplus (7,800 – 1,560 (w (ii))) 6,240
Retained earnings (w (iv)) 17,377 25,825
75,825
Non-current liabilities
Deferred tax (w (iii)) 3,960
6% convertible loan notes (w (i)) 38,415 42,375
Current liabilities
Per trial balance 17,700
Current tax payable 2,700 20,400
Total equity and liabilities 138,600

Diluted earnings per share (w (v)) 29 Paisa

Workings (monetary figures in brackets in NRs.‘000)


(i) 6% convertible loan notes
The convertible loan notes are a compound financial instrument having a debt and an equity
component which must both be quantified and accounted for separately:
Year ended 31 March outflow 8% present value
NRs.‘000 NRs.‘000
20X6 2,400 0·93 2,232
20X7 2,400 0·86 2,064
20X8 42,400 0·79 33,496
Debt component 37,792
Equity component (= 2,208
balance)
Proceeds of issue 40,000
–––––––
The finance cost will be NRs.3,023,000 (37,792 x 8%) and the carrying amount of the loan notes
at 31 March 20X6 will be NRs.38,415,000 (37,792 + (3,023 – 2,400)).

(ii) Non-current assets


Leased property
Page 21 of 328
Paper1: Advanced Financial Reporting [RTP 2017 June]

The gain on revaluation and carrying amount of the leased property is:
NRs.‘000
Carrying amount at 1 April 20X5 (75,000 – 15,000) 60,000
Amortisation to date of revaluation (1 October 20X5) (75,000/25 x (1,500)
6/12)
Carrying amount at revaluation 58,500
Gain on revaluation = balance 7,800
Revaluation at 1 October 20X5 66,300
Amortisation to year ended 31 March 20X6 (66,300/19·5 years x (1,700)
6/12)
Carrying amount at 31 March 20X6 64,600

Annual amortisation is NRs.3m (75,000/25 years); therefore the accumulated amortisation


at 1 April 20X5 of NRs.15m represents five years‘ amortisation. At the date of
revaluation (1 October 20X5), there will be a remaining life of 19·5 years.
Of the revaluation gain, NRs.6·24m (80%) is credited to the revaluation surplus and NRs.1·56m
(20%) is credited to deferred tax.

Plant and equipment


NRs.‘000
Carrying amount at 1 April 20X5 (72,100 – 28,100) 44,000
Depreciation for year ended 31 March 20X6 (15% reducing balance) (6,600)
Carrying amount at 31 March 20X6 37,400

(iii) Deferred tax


Provision required at 31 March 20X6:
Revalued property and other assets (7,800 + 12,000) x 3,960
20%)
Provision at 1 April 20X5 (3,200)
Increase in provision 760
Revaluation of land and buildings (7,800 x 20%) (1,560)
Balance credited to profit or loss (800)

(iv) Retained earnings


Balance at 1 April 20X5 3,500
Prior period adjustment (fraud) (450)
Adjusted profit for year (from (a)) 14,327
Balance at 31 March 20X6 17,377

(v) The maximum additional shares on conversion is 8 million (40,000 x 20/100), giving total
shares of 58 million. The loan interest ‗saved‘ is NRs.2·418m (3,023 (from (w (i)) above x
80% (i.e. after tax)), giving adjusted earnings of NRs.16·745m (14,327 + 2,418).
Therefore diluted EPS is
NRs.16,745,000 x 100/58 million shares = 29 Paisa

Solution to Q No. 5
Page 22 of 328
Paper1: Advanced Financial Reporting [RTP 2017 June]

Answer
The issue of convertible financial instruments is the reason why Downing Co has to disclose a
figure for diluted EPS in addition to its basic EPS. When the convertible loan notes are due for
redemption on 31 March 2018, there is the potential that they will be converted into equity
shares which would increase the number of equity shares in issue. At the same time there will
also be a saving in the after tax interest which will no longer be paid to the loan note
holders, however, this will be proportionately less per share than is currently generated from
earnings per share (otherwise there would not be a dilution) and thus the EPS will be diluted or
‗watered down‘. In reality, it is possible that the convertible loan notes, or a proportion of them,
will be redeemed for cash which would not cause a dilution, however, NAS 33 Earnings
per Share requires that maximum possible dilution has to be assumed when calculating the
diluted EPS.

Despite this, the CEO seems mistaken as to what the diluted EPS figure actually means; it does
not mean that this will be the EPS in the near future (or at the time of redemption). The future
EPS will be based on future earnings and the (weighted average) number of shares actually in
issue in that future year. Rather, the diluted EPS figure should be seen as a sort of warning. It
is saying that, based on existing circumstances, if the dilution had already taken place, i.e. that
the convertible shares had already been redeemed for equity (at the maximum possible number
of new shares), the diluted EPS as disclosed would have been the figure reported as the actual
(basic) EPS.

So, although the CEO does not fully understand what the diluted EPS figure means, it does
indicate to investors the possibility of a future dilution of EPS; and with the dilution being a
sizable 19% lower than the basic EPS, it may well cause an adverse reaction in the market price
of the Downing Co‘s shares.
Solution to Q No. 6

(i) Purnabiram Ltd. – Extracts from statement of profit or loss (see workings):
NRs.‘000
Year ended 31 March 2013
Plant impairment loss 20,000
Plant depreciation (32,000 + 54,400
22,400)
Year ended 31 March 2014
Loss on sale 8,000
Plant depreciation (32,000 + 58,000
26,000)

(ii) Purnabiram Ltd. – Extracts from statement of financial position (see workings):
NRs.‘000
As at 31 March 2013
Property, plant and equipment (128,000 + 217,600
89,600)
Revaluation surplus
Revaluation of item B (1 April 2012) 32,000
Page 23 of 328
Paper1: Advanced Financial Reporting [RTP 2017 June]

Transfer to retained earnings (32,000/5 years) (6,400)


Balance at 31 March 2013 25,600
As at 31 March 2014
Property, plant and equipment (item A only) 96,000
Revaluation surplus
Balance at 1 April 2013 25,600
Transfer to retained earnings (asset now sold)
(25,600)
Balance at 31 March 2014 nil

Workings:
Item A Item B
NRs.‟000 NRs.‟000
Carrying amounts at 31 March 2012 180,000 80,000
Balance = loss to statement of profit or loss (20,000)
Balance = gain to revaluation surplus 32,000
Revaluation on 1 April 2012 160,000 112,000
Depreciation year ended 31 March 2013 (160,000/5 years) (22,400) (112,000/5
(32,000) years)
Carrying amount at 31 March 2013 128,000 89,600
Subsequent expenditure capitalised on 1 April nil 14,400
2013
104,000
Depreciation year ended 31 March 2014 (unchanged) (32,000) (26,000) (104,000/4
years)
78,000
Sale proceeds on 31 March 2014 (70,000)
Loss on sale (8,000)
Carrying amount at 31 March 2014 96,000 nil

Solution to Q No. 7
Answer:
(i) Changing the classification of an item of expense is an example of a change in accounting
policy, in accordance with NAS 8 Accounting Policies, Changes in Accounting Estimates and
Errors. Such a change should only be made where it is required by an NFRS or where it would
lead to the information in the financial statements being more reliable and relevant. It may be
that this change does represent an example of the latter, although it is arguable that
amortised development costs should continue to be included in cost of sales as amortisation
only occurs when the benefits from the related project(s) come on-stream. If it is accepted
that this change does constitute a change of accounting policy, then the proposed treatment by
the directors is acceptable; however, the comparative results for the year ended 31 March 2013
must be restated as if the new policy had always been applied (known as retrospective
application).
Page 24 of 328
Paper1: Advanced Financial Reporting [RTP 2017 June]

(ii) The two provisions must be calculated on different bases because NAS 37 Provisions,
Contingent Liabilities and Contingent Assets distinguishes between a single obligation (the
court case) and a large population of items (the product warranty claims).
For the court case the most probable single likely outcome is normally considered to be the best
estimate of the liability, i.e. NRs.4 million. This is particularly the case as the possible outcomes
are either side of this amount. The NRs.4 million will be an expense for the year ended 31 March
2014 and recognised as a provision.

The provision for the product warranty claims should be calculated on an expected value basis at
NRs.3·4 million (((75% x nil) + (20% x NRs.25) + (10% x NRs.120)) x 200,000 units). This will
also be an expense for the year ended 31 March 2014 and recognised as a current liability (it is a
one-year warranty scheme) in the statement of financial position as at 31 March 2014.
(iii) Government grants related to non-current assets should be credited to the statement of profit
or loss over the life of the asset to which they relate, not in accordance with the schedule of any
potential repayment. The directors‘ proposed treatment is implying that the government grant is a
liability which decreases over four years. This is not correct as there would only be a liability if
the directors intended to sell the related plant, which they do not. Thus in the year ended
31 March 2014, NRs.800,000 (8 million/10 years) should be credited to the statement of profit
or loss and NRs.7·2 million should be shown as deferred income (NRs.800,000 current and
NRs.6·4 million non-current) in the statement of financial position

Solution to Q No. 8

Chandra Ltd.
Consolidated statement of profit or loss and other comprehensive income for the year ended 30
September 2014
NRs.‘000

Revenue (62,600 + (30,000 x 9/12) – (300 x 9 months intra-group 82,400


sales))
Cost of sales (w (i)) (61,320)
Gross profit 21,080
Distribution costs (2,000 + (1,200 x 9/12)) (2,900)
Administrative expenses (3,500 + (1,800 x 9/12) + 500 goodwill (5,350)
impairment)
Finance costs (200 + 135 (w (v))) (335)
Profit before tax 12,495
Income tax expense (3,100 + (1,000 x 9/12)) (3,850)
Profit for the year 8,645
Other comprehensive income
Gain on revaluation of property (1,500 + 600) 2,100
Total comprehensive income 10,745

Profit for year attributable to:


Equity holders of the parent (balance) 8,465
Page 25 of 328
Paper1: Advanced Financial Reporting [RTP 2017 June]

Non-controlling interest (w (ii)) 180


8,645
Other comprehensive income attributable to:
Equity holders of the parent (balance) 10,445
Non-controlling interest (180 above + (600 x 20%)) 300
10,745

Chandra Ltd. – Consolidated statement of financial position as at 30 September 2014


NRs.‘000
Assets
Non-current assets
Property, plant and equipment (w (iii)) 37,100
Intangible asset: goodwill (w (iv)) 5,200
42,300
Current assets
Inventory (4,300 + 1,200 – 120 URP (w (i))) 5,380
Trade receivables (4,700 + 2,500 – 1,200 intra-group) 6,000
Bank 300
11,680
Total assets 53,980

Equity and liabilities


Equity attributable to owners of the parent
Equity shares of NRs.1 each ((10, 000 + 4,800) w (iv)) 14,800
Other component of equity (share premium) (w (iv)) 9,600
Revaluation surplus (2,000 + (600 x 80%)) 2,480
Retained earnings (w (v)) 6,765
33,645
Non-controlling interest (w (vi)) 4,800
Total equity 38,445
Non-current liabilities
10% loan notes (2,500 + 1,000 – 1,000 intra-group) 2,500
Current liabilities
Trade payables (3,400 + 3,600 – 800 intra-group) 6,200
Current tax payable (2,800 + 800) 3,600
Deferred consideration (1,800 + 135 w (v)) 1,935
Bank (1,700 – 400 cash in transit) 1,300
13,035
Total equity and liabilities 53,980

Working Notes:
(i) Cost of sales 45,800
Plastik 18,000
Bindu Ltd. (24,000 x 9/12) -2,700
Intra-group purchases (300 x 9 months) 120
URP in inventory (600 x 25/125) 100
Page 26 of 328
Paper1: Advanced Financial Reporting [RTP 2017 June]

Additional depreciation on property 61,320

(ii) Non-controlling interests in Bindu td.‘s profit or loss


Bindu td.‘s profit as reported 2,000
9/12 post-acquisition = 1,500
Deduct: Additional depreciation on property -100
Goodwill impairment -500
Adjusted post-acquisition profit 900
x 20% non-controlling interest 180

(iii) Non-current assets


Plastik 18,700
Bindu Ltd. 13,900
Fair value increase at acquisition 4,000
Additional depreciation on property -100
Fair value increase since acquisition 600
37,100

(iv) Goodwill in Bindu Ltd.


Investment at cost
Shares (9,000 x 80% x 2/3 x NRs.3) 14,400
Deferred consideration (9,000 x 80% x 27·5 Chandra Ltd. x 1/1·1) 1,800
Non-controlling interest (9,000 x 20% x NRs.2·50) 4,500
20,700

Net assets (equity) of Bindu Ltd. at 30 September 2014 -12,500


Less post-acquisition profits (2,000 x 9/12) 1,500
Fair value adjustment: property -4,000

Net assets at date of acquisition -15,000


Goodwill on consolidation 5,700
Impairment as at 30 September 2014 -500
5,200

Note: The 4·8 million (9,000 x 80% x 2/3) shares issued by Plastik at NRs.3 each would be recorded as
share capital of NRs.4·8 million (4,800 x NRs.1) and share premium of NRs.9·6 million (4,800 x
NRs.2)

(v) Retained earnings


Plastik 6,300
Bindu td.‘s post-acquisition adjusted profit (900 (w (ii)) x 80%) 720
Finance costs on deferred consideration (1,800 x 10% x 9/12) -135
Unrealised profit in inventory (w (i)) -120
Page 27 of 328
Paper1: Advanced Financial Reporting [RTP 2017 June]

6,765

Alternative calculation 6,300


Plastik‘s retained earnings at 30 September 2014 -8,000
ess Plastik‘s profit for the year 8,465
Consolidated profit for the year from part (a) 6,765

(vi) Non-controlling interest in statement of financial position


At date of acquisition (w (iv)) 4,500
Post-acquisition from statement of profit or loss and other comprehensive
income 300
4,800

Solution to Q No. 9

Consolidated Balance Sheet of P Ltd. as on 31.3.15

Particulars Amount in
lacs
I. Equity and Liabilities
Shareholder's Funds
(a)Share Capital 1,000
(b)Reserves and Surplus 8,800
Minority Interest (in S Ltd.) 760
Current Liabilities
Trade Payables(200+ 300 + 40% of 250)
Total 11,160
II.Assets
Non-current assets
Fixed assets
Tangible assets [1,000 + 800 + 560 (1400 x 40%)] 2,360
Intangible assets (Goodwill) 120
Non-current investment (investment in A) 1,880
Current assets [2,200+3,300+1,300 (3,250x 40%)] 6,800
Total 11,160

Working Notes:

Computation of Goodwill
S (subsidiary)
in lacs
Cost of investment 800
Less: Paid up value of shares 320
acquired
Page 28 of 328
Paper1: Advanced Financial Reporting [RTP 2017 June]

Share in pre-acquisition 416 (736)


profits of S Ltd. (520 × 80%)
Goodwill 64

Minority Interest
Share Capital 400* 20% 80
Retained Earning 680
3400*20%
Total 760
J (Jointly Controlled Entity)

in lacs
Cost of Investment 600
Less:
Paid up value of shares 320
acquired (40% of 800)
Share in pre-acquisition 160 (480)
profits (40% of 400)
Goodwill 120
Note: Jointly controlled entity J‗ to be consolidated on proportionate basis

Associate A`
in lacs
Cost of investment 600
Less:Paid up value of shares acquired (800 x 320
40%)
Share in pre-acquisition profits (400 x 40%) 160 (480)
Goodwill 120

Goodwill shown in the Consolidated Balance Sheet


in lacs
Goodwill of J 120
Goodwill of S 64
ess:Goodwill written off of S (64)
Goodwill 120

Investment in Associate (Revaluation)


Share Capital 800
Reserves (pre acquisition + post 3600
acquisition)
Total 4400
P Ltd. Share 40 %
1760
Goodwill (on investment in associate) 120
Total 1880
Page 29 of 328
Paper1: Advanced Financial Reporting [RTP 2017 June]

Solution to Q No. 10
Solution
Workings Notes:
Shareholding Pattern
B Ltd. C Ltd.
Total Number of 1,000 600
Shares
A td‘s Holding 750 NA
B td‘s Holding NA 400
Minority Holding 250 200
Minority % 25 % 33.33%

(1) Analysis of Profit


Capital Revenue Revenue
C Ltd. Profit Reserve Profit
Reserve on 1.1.2011 6,000 0 0
Additional Reserve created in 2011 600 600 0
[1,200 X
[7,200 – 6,000] = 1,200 ½] 1,200 X ½]
P & L A/c, Balance on 1.1.2011 1,000 0 0
Profit for 2011 [5,000 – 1,000] = 4,000 2,000 0 2,000
[4,000 X [4,000 X
½] 0 ½]
9,600 600 2,000
3,200 200 667
Due to outsiders, 1/3 6,400 400 1,333
Share of B Ltd. (2/3)

B Ltd.
From C Ltd. 6,400 400 1,333
Reserve on 1.1.2011 8,000 — —
Additional Reserve created in 2011 1,000 1,000 —
[2,000 X 2,000 X
[10,000-8,000] = 2,000 ½] ½] —
Profit and Loss A/c:
Balance on 1.1.2011 1000 —
Profit during 2011 [4,000-1,000]=3,000 1,500 — 1,500
[3,000 X
[3,000 X ½] ½]
17,900 1,400 2,833
Due to outsiders (1/4) 4,475 350 708
Share of A Ltd. 13,425 1,050 2,125
Page 30 of 328
Paper1: Advanced Financial Reporting [RTP 2017 June]

A Ltd. 18,000 16,000


18125
Less: Inventory Reserve -600
Total 13,425 19,050 17,525

Notes:
(i) During 2011, 1,200 has been added to the Reserves of C Ltd., and 2,000 to the Reserves of B
Ltd. The profit must have been earned during the whole of the year; hence, half of these figures
(i.e., up to 30.6.2011) must be considered as capital pre-acquisition and the remaining revenue.
(ii) Total unrealised profit is ` 600, i.e., ` 3,100 less ` 2,500.

(2) Minority Interest


B Ltd. C Ltd.
Share Capital 25,000 20,000
Share of Capital Profits 4,475 3,200
Share of Revenue Reserves 350 200
Share of Revenue Profits 708 667
Total 30,533 24,067
Grand Total 54,600

(3) Cost of Control/Goodwill


Amount paid:
A Ltd. 85,000
B Ltd. 53,000 1,38,000
Less: Par value of shares in:
B Ltd. 75,000
C Ltd. 40,000
Capital Profits* 13,425 (1,28,425)
Cost of Control/Goodwill 9,575

Note ∗ The whole of this amount may preferably be adjusted against cost of control, instead of
being added
to the profits of B Ltd. Consequently capital profits will increase by ` 1,600 with a
corresponding
reduction in Minority interest.

(4) Since X Ltd. shows ` 8,000 against B Ltd. whereas B Ltd., shows only ` 7,000 in favour of A
Ltd., it
must be assumed that B Ltd., has remitted ` 1,000 to A Ltd.; not yet received by A Ltd. The
amount is in transit.
(5) If capital profit is increased by ` 1,600 cost of control will be ` 7,975.

Consolidated Balance Sheet of A Ltd.


Page 31 of 328
Paper1: Advanced Financial Reporting [RTP 2017 June]

and its subsidiaries B Ltd. and C Ltd.,


as on 31st December, 2011

Particulars Note Amount


I. Equity and Liabilities
(1) Shareholder's Funds
(a) Share Capital 125,000
(b) Reserves and Surplus 1 36,575
(2) Minority Interest (W.N
2) 54,600
(3) Current Liabilities
Trade Payables 2 10,000
Total 226,175.00

II. Assets
(1) Non-current assets

(i) PPE 3 120,400


(ii) Intangible assets 4 9,575
(2) Current assets
(a) Inventories 5 27,400
(b) Trade receivables 6 67,800
(c) Cash & Cash
equivalents 7 1,000
Total 226,175

Explanatory Notes/Schedules:
Note Particulars Amount Amount
1 Reserves and Surplus
Reserves (W.N.1) 19,050
Profit and Loss Account
(W.N.1) 17,525 36,575
2 Trade Payables
A Ltd. 7,000
B Ltd. 3,000 10,000
3 Tangible Assets
A Ltd. 28,000
B Ltd. 55,000
C Ltd. 37,400 120,400
4 Intangible assets
Goodwill (W.N 3)
5 Inventories
A Ltd. 22,000
B Ltd. 6,000
Page 32 of 328
Paper1: Advanced Financial Reporting [RTP 2017 June]

28,000
Less : Inventory reserve (600) 27,400
6 Trade Receivables
A Ltd. 26,300
B Ltd. 10,000
C Ltd. 31,500 67,800
7 Cash & Cash equivalents
Cash in transit 1000
Page 33 of 328
Paper1: Advanced Financial Reporting [RTP 2017 June]

Solution to Q No. 11

The entity records the asset at a cost of $3 million at March 31, 20X6, and a liability of the same
amount. At year-end, the amount has not been paid. Thus using the closing rate of exchange, the
amount payable would be retranslated at $4 million, which would give an exchange loss of $1
million to be reported in profit or loss. The cost of the asset remains at $3 million before
depreciation. Similarly, the entity will record a sale of $2 million and an amount receivable of
the same amount. At year-end, the receivable would be stated at $2.33 million, which would give
an exchange gain of $0.33 million, which would be reported in profit or loss.

Solution to Q No. 12

Income Statement for the year ended December 31, 20X6, at average rate
Amount (€1.5 =
Particulars $1)
Revenue 48
Cost of sales -15
.
Gross profit 33
Distribution costs -12
Administrative
expenses -3
.Profit before tax
Profit before tax 18
Tax expense -6
Profit for period 12

The exchange difference is calculated in this way:


The retained earnings if translated into euros would be €16 million. As the income statement has
been
translated using the average rate, the profit per that statement is €12 million, creating an
exchange differ-
ence of €4 million.
The total exchange difference of €4 million, is shown as a component of equity.

Balance Sheet at December 31, 20X6


Amount (€2 =
$1)
Share capital (closing rate) 4
Retained earnings (above) 12
Exchange difference (see below) 4
20
Trade payables 8
Total equity and liabilities 28
Land (nondepreciable) acquired December 16
Page 34 of 328
Paper1: Advanced Financial Reporting [RTP 2017 June]

31, 20X6
Inventories 8
Trade receivables 4
Total assets 28

Solution to Q No. 13
Treatment of various costs incurred during 20X5 depends on whether these costs can be
capitalized or expensed as per NAS 38. Although NAS 38 is clear that expenses incurred during
the research phase should be expensed, it is important to note that not all development costs can
be capitalized. In order to be able to capitalize costs, strict criteria established by NAS 38 should
be met. Based on the criteria prescribed by NAS 38, these conclusions can be drawn:

(1) It could be argued that the technical feasibility criterion was established at the end of August
20X5, when the first prototype was produced.
(2) The intention to sell or use criterion was met at the end of August 20X5, when the sample
was tested with the air-conditioning component to ensure it functions. But it was not until
October 20X5 that the product‘s marketability was established. The reason is attributable to the
fact that the entity had doubts about the new models being compatible with the air conditioners
and that the sample would need further testing, had it not functioned.
(3) In October 20X5, the existence of a market was clearly established.
(4) The financial feasibility and funding criterion was also clearly met because Hawa Pvt. Ltd.
has obtained a loan from venture capitalists and it had the necessary raw materials.
(5) Hawa Pvt. Ltd. was able to measure its cost reliably, although this point was not addressed
thoroughly in the question. Hawa Pvt. Ltd. can easily allocate labor, material, and overhead costs
reliably.

Therefore, the costs that were incurred before October 20X5 should be expensed.

The total costs that should be expensed = NRs.175,000 + NRs.250,000 + NRs.300,000 +


NRs.80,000 = NRs.805,000.

The costs eligible for capitalization are those incurred after October 20X5. However, conference
costs of NRs.50,000 would need to be expensed because they are independent from the
development process.

Thus there are no total costs to be capitalized in terms of NAS 38.

Solution to Q No. 14

Hedged instrument is the instrument whose fair value is shielded using the hedging strategy. In
this case, it is the copper inventory held by Commodity Traders Ltd. Hedging instrument on the
other hand is the derivative instrument which mitigates the fair value changes of hedged
instrument by reversely mimicking its fair value movement.
Page 35 of 328
Paper1: Advanced Financial Reporting [RTP 2017 June]

On 30 June 2015, the fair value of copper inventories held for trading shall be adjusted as
follows:

Profit and loss on Financial Instrument NRs.300,000


Inventories (NRs.5,200,000 - NRs.4,900 * 1,000) NRs.300,000

The loss on inventories shall be offset by corresponding gain on the forward transaction. Since
the forward transaction entitles Commodity Traders Ltd. to sell copper at NRs.5,200 per ton even
though the market price is NRs.4,900 per ton, it represents a NRs.300 gain per ton, which
translates into NRs.300,000 gain on 1,000 tons. The fair value change of the hedging instrument
is recognized as follows:

Derivative (asset) NRs.300,000


Profit and loss on Financial Instrument (1,000 * (NRs.5,200 -
NRs.300,000
NRs.4,900)

There shall be zero effect on the net value of inventories on Commodity Traders Ltd. balance
sheet even though copper price fell over the period by NRs.300 per ton. The hedging strategy
saves Commodity Traders Ltd. from furnishing additional security to the bank in wake of the fall
in fair value.

Solution to Q No. 15

Year ended 30 September 2013


The options do not vest in this period so the charge to profit or loss and the credit to equity is
based on the assumptions made when estimating the fair value of the options at the grant date –
ie a three-year period with a vesting date of 30 September 2015. Therefore the charge to profit or
loss and the credit to equity is NRs.24,000 (200 x 20 x NRs.18 x 1/3).

Year ended 30 September 2014


The options actually vest in this period so the total cumulative cost needs to be recognised in
equity by that date. The total amount to be recognised is NRs.72,000 (200 x 20 x NRs.18). This
means the charge to profit or loss in this accounting period is NRs.48,000 (NRs.72,000 –
NRs.24,000).

Alternative assumption
Assuming, as an alternative, that the share price remained below NRs.20 for the whole of the
three-year period ending 30 September 2015 so the options never vested, there would be a charge
to profit or loss, and a corresponding credit to equity, of NRs.24,000 for each of the three years
ending 30 September 2013, 2014 and 2015. Therefore there would be a charge to profit or loss
even though the options never vested. This would never happen for non-market vesting
conditions.

Solution to Q No. 16
Page 36 of 328
Paper1: Advanced Financial Reporting [RTP 2017 June]

 Computation of Goodwill
Capital Employed Rs Rs
Buildings 6,00,000
Machinery 2,50,000
Stock 60,000
Debtors 50,000
Bank 50,000
10,10,000
Less: Liabilities
Creditors 1,10,000
Provision for Income Tax 40,000
Proposed Dividend 60,000 2,10,000
Capital Employed 8,00,000

Average Maintainable Profits (before income tax)

(1,00,000 + 1,40,000 +1,60,000 + 1,70,000+1,80,000) 1,50,000


÷5
Less :Income tax @40% 60,000
Normal profit (10% of 8,00,000) 80,000
Super profit 10,000

Goodwill = Super Profit x 5 years


= Rs 10,000 x 5
= Rs 50,000

 Valuation of shares on Net Asset Basis


Capital employed 8,00,000
Add : Goodwill (as above ) 50,000
Assets available to Shareholders 8,50,000
Less : Preference share capital 1,00,000
Assets available to Equity Shareholder 7, 50,000

Value of Equity Share = Assets available to Equity Shareholder / Number of Equity


Shares
= 7,50,000 / 50,000
= Rs 15 per share

 Valuation of share on Yield Basis


Average Equity Dividend = 8 + 9 +7 / 3
= 8%
Value of Equity Share = (Average Dividend Rate ÷ Normal Dividend) x Paid up
value
= 8 x 10 / 10
= Rs 8 per share
Page 37 of 328
Paper1: Advanced Financial Reporting [RTP 2017 June]

 Fair Value of Equity Share = 15 + 8 / 2


= Rs 11.50 per share

 Number of shares which can be purchased = 20000 / 11.50


= 1,739 Equity Shares

Solution to Q No. 17

Calculation of Target Index


Year Ending 31st March 2010 2011 2012 2013 2014
Employee Cost 1,300 1,520 1,680 1,968 2,240
Value Added 3,040 3,600 4,200 4,800 5,600
Ratio (Employee Cost ÷ Value
Added) 42.76% 42.22% 40.00% 41.00% 40.00%
Target Index is taken as least of the above on conservative basis = 40% (Favorable to Employer)
Computation of Bonus for the Year ending 31st March 2015
Particulars Amount („000)
Sales 13,600
Less: Bought in Goods & Services:
Cost of Materials 5,000
Production Expenses 1,400
Administration Office Expenses 600
Sales Office Expenses 400 7,400
Value Added from Manufacturing and Trading Activities 6,200
Add: Other Income (500 + 500) 1,000
1. Net Value Added 7,200
2. Employees Cost, i.e. Wages & Salaries – [Wages & Salaries ` 1,800 +
Administration Salaries ` 600 + Selling & Distribution Salaries ` 120] 2,520
3. Employee cost to Net Value added for the year 35%
4. Improvement for the Year eligible for Bonus = Target 40% –
Ratio as above 35% 5%
5. Bonus Payable for the Year = Value Added ` 7,200 × 5% ×
Employee Share 2/3rd 240

Statement of Application of Value Added


Particulars Amount ‗000 %
1. To Employees: As above 2,520 + Bonus 240 2,760 38.33%
2. To Government: Taxes 770 10.69%
3. To Providers of Capital: Interest on 80 1.11%
Debentures
4. To Shareholders
(a) Depreciation 1,520
Page 38 of 328
Paper1: Advanced Financial Reporting [RTP 2017 June]

(b) Retained Profits (given 2,310 – Incentive 2,070 3,590 48.87%


240)
Total 7,200 100.00%

Solution to Q No. 18

NRs. m
Contract revenue recognised in the period 1,014.0
Contract costs incurred and recognised profits (less recognised losses) to 1,202.0
date (W)
Advances received 65.0
Gross amount due from customers for contract work: asset (W) 254.3
Gross amount due to customers for contract work: liability (W) (15.6)

Workings
V W X Y Z Total
Particulars NRs. m NRs. m NRs. m NRs. m NRs. m NRs. m
Contract costs incurred 85.8 437.6 386.1 195 85.8 1,190.30
Recognised profits less recognised
losses 27.3 54.6 23.4 -70.2 -23.4 11.7
113.1 492.2 409.5 124.8 62.4 1,202.00
Less: progress billings to date -78 -405.6 -296.4 -140.4 -42.9 -963.3
Due from customers 35.1 86.6 113.1 19.5 254.3
Due to customers -15.6 -15.6

Solution to Q No. 19

Information about business segments: JYCE Group

Office Office Computer


furniture stationery products Other Consolidated
Revenue: NRs.m NRs.m NRs.m Division Elimination NRs. m
External sales 800 500 400 100 1800
Intersegment sales 200 150 80 0 -430 1800
Total revenue 1000 650 480 100 -430
Result:
Segment result 180 90 72 18 360
Unallocated expenses -50
Finance costs -60
Share of profit of associates 10 10
Income tax expense -70
Profit for the period 190
Other information:
Segment assets 440 170 96 20 726
Page 39 of 328
Paper1: Advanced Financial Reporting [RTP 2017 June]
Office Office Computer
furniture stationery products Other Consolidated
NRs.m NRs.m NRs.m Division Elimination NRs. m
Investment in associate 70 70
Unallocated asset 4
Consolidated total assets 800
Segment liabilities 75 54 12 3 144
Unallocated liabilities 6
Consolidated total liabilities 150

Workings

Segment result NRs.m


Gross profit 600
Other income 60
Distribution costs -200
Administrative expenses -100
Net profit 360
Allocated 180
Office furniture 90
Office stationery 72
Computer products 18
Other divisions 360

Office Office
furniture stationery NRs. Computer products
Segment assets NRs. m m NRs. m
Property, plant, and equipment 300 100 80
Goodwill 60 30 10
Current assets 80 40 6
440 170 96
Segment liabilities
Noncurrent 30 21 4
Current 45 33 8
75 54 12
Page 40 of 328
Paper1: Advanced Financial Reporting [RTP 2017 June]

Solution to Q No. 20

The unrealized profit of NRs. 1 million will have to be eliminated from the consolidated income
statement and from the consolidated balance sheet in group inventory. The sale of the inventory
is a taxable event, and it causes a change in the tax base of the inventory. The carrying amount in
the consolidated financial statements of the inventory will be NRs. 10 million, but the tax base is
NRs. 11 million. This gives rise to a deferred tax asset of NRs. 1 million at the tax rate of 30%,
which is NRs. 300,000 (assuming that both the parent and subsidiary are resident in the same tax
jurisdiction)

Solution to Q No. 21

a. Economic Value Added (EVA) is primarily a benchmark to measure earnings efficiency. EVA
as a residual income measure of financial performance is simply the operating profit after tax
less a charge for the capital employed, equity as well as debt, used in the business.
Mathematically EVA= OPBT  Tax  (TCE × COC)
Where:
OPBT = Opening Profit Before Tax
TCE = Total Capital Employed
COC = Cost of Control
Because EVA includes both profit and loss as well as balance sheet efficiency as well as the
opportunity cost of investor capital - it is better linked to changes in shareholders wealth and is
superior to traditional financial measures such as PAT or percentage of return measures such as
ROCE or ROE.
EVA, additionally, is a tool for management to focus on the impact of their decisions in
increasing shareholders wealth. These include both strategic decisions such as what investments
to make, which business to exit, what financing structure is optimal; as well as operational
decisions involving trade-offs between profit and asset efficiency such as whether to make
inhouse or outsource, repair or replace an equipment, whether to make short or long production
runs etc.
Most importantly the real key to increasing shareholders wealth is to integrate EVA framework
in four key areas, viz., to measure business performance, to guide managerial decision making,
to align managerial incentives with the shareholders' interests and to improve the financial and
business literacy throughout the organisation.
To better align managers interests with shareholders' - the EVA framework needs to be
holistically applied in an integrated approach - simply measuring EVA is not enough; it must
also become the basis of key management decisions as well as be linked to senior management's
variable compensation.
However, EVA as a strategic tool has the following limitations:
1. Not easy to use; too complicated for small businesses.
2. Recommends inexpensive debts in order to reduce the cost of capital.
3. A passive tool, measures past performance

b. Corporate Social Reporting is the information communique with respect to discharge of social
responsibilities of corporate entity. The transition in accounting function from historical cost
Page 41 of 328
Paper1: Advanced Financial Reporting [RTP 2017 June]

based profitability accounting to social responsibility accounting is a good fit to the present-day
data requirement of the ―Users of accounts‖.
The content of Corporate Social Report is essentially based on the social objectives,
namely Net Income Contribution, Human Resource Contribution, Public Contribution,
Environmental Contribution and Product or Service Contribution.
Considering the major socio-economic problems of the country, eight major heads can be
identified for social reporting purpose:
(i) Employment Opportunities;
(ii) Foreign Exchange Transactions;
(iii) Energy Conservation;
(iv) Research and Development;
(v) Contribution to Government Exchequer;
(vi) Social Projects;
(vii) Environmental Control;
(viii) Consumerism.
Initially, it is difficult to express social costs incurred by a corporate enterprise and social
benefits generated in money terms. Until suitable methologies are available for conversion
of social cost-benefit in money terms, it is desirable to begin with descriptive social report.
Further research is necessary in this area either to improve heads of corporate social reporting
in the context of dynamic socio-economic environment.

c. Qualitative characteristics are the attributes that make the information provided in the financial
statements useful to the users. The four principal qualitative characteristics are: (i)
Understandability, (ii) Relevance, (iii) Reliability and (iv) Comparability.
(i) Understandability: An essential, quality of the information provided in the financial
statement is that it is readily understandable by the users. For this purpose, users are deemed
to have reasonable knowledge of business and economic activities. However, information
about complex matters should be included in the financial statements which is relevant to the
users of accounts for their economic decision making although this may be too difficult for
certain users to understand.
(ii) Relevance: To be useful, information must be relevant to the decision making needs of all
the users. Information has the quality of relevance when it influences the economic decisions
of users by helping them to evaluate past, present or future events or confirming, or
correcting their past evaluations.
Relevance of an information is affected by its nature and materiality. In some cases, the
nature of information alone is sufficient to determine its relevance. In other cases, both the
nature and materiality are important:
(iii) Reliability: To be useful, information must also be reliable. Information has the
quality of reliability when it is free from material error and bias and can be depended upon
by users to represent faithfully that which, it either purports to represent or could reasonably
be expected to represent.
Reliability of the financial statement information is dependent on faithful representation,
substance over form, neutrality, prudence, and completeness. If information is to represent
faithfully the transactions and other events, it is necessary that they are accounted for and
presented in accordance with their substance and economic reality and not merely by their
legal form. To be reliable, the information contained in financial statement must be neutral
Page 42 of 328
Paper1: Advanced Financial Reporting [RTP 2017 June]

i.e. free from bias. Financial statements are not neutral if, by the selection or presentation of
information, they influence the making of a decision or judgement in order to achieve a pre-
determined result or outcome. Prudence is the inclusion of a degree of caution in the exercise
of the judgements needed in making the estimates required under conditions of uncertainty.
To be reliable, information in financial statements must also be complete within the bounds
of materiality and cost. An omission can cause information to be false or misleading and thus
unreliable and deficient in terms of its relevance.
(iv) Comparability: Users must be able to compare the financial statements of an
enterprise through time in order to identify trends in its financial position and performance.
An important implication of this qualitative characteristic is that users should be informed of
the accounting policies employed in the preparation of the financial statements, any changes
in those policies and the effects of such changes.
Page 43 of 328
Paper1: Advanced Financial Reporting [RTP 2017 December]
Questions

1. Danfe Corporation owns buildings with a cost of CU 200,000 and estimated useful life of five years.
Accordingly, depreciation of CU 40,000 per year is anticipated. After two years, Danfe obtains
market information suggesting that a current fair value of the buildings is CU 300,000 and decided to
write the buildings up to a fair value of CU 300,000. There are two approaches to apply the
revaluation model in NAS 16: the asset and accumulated depreciation can be ―grossed up‖ to reflect
the new fair value information, or the asset can be restated on a ―net‖ basis.

Required:
Show the journal entries in the books of Danfe Corporation under both ―grossed up‖ & ―net‖
approaches to revaluation.

2. Salaam distributed property with a carrying amount of CU 10,000 to its shareholder as a dividend
during the current year. The property had a fair market value of CU 17,000 at the date of the transfer.
Pass Journal Entries in the books of Salaam

3. Assume the following:


1. On January 1, 20XX, Scopio Corp. contracted with Leo Company to construct a building for CU
20,000,000 on land that Scopio had purchased years earlier.
2. Scopio Corp. was to make five payments in 20XX, with the last payment scheduled for the date of
completion.
3. The building was completed December 31, 20XX.
4. Scopio Corp. made the following payments during 20XX:
January 1,20XX CU 2,000,000
March 31, 20XX CU 4,000,000
June 30, 20XX CU 6,100,000
September 30, 20XX CU 4,400,000
December 31, 20XX CU 3,500,000
CU 20,000,000
5. Scopio Corp. had the following debt outstanding at December 31, 20XX:
a. A 12%, 4-year note dated 1/1/20XX-3 with interest compounded quarterly. Both principal and
interest due 12/31/20XX+1 (relates specifically to building project) CU 8,500,000
b. A 10%, 10-year note dated 13/31/20XX-8 with simple interest and interest payable annually on
December 31 CU 6,000,000
c. A 12%, 5-year note dated 13/31/20XX-6 with simple interest and interest payable annually on
December 31 CU 7,000,000
Required:
Calculate the amount of interest to be capitalized &/or expensed:
4. An entity is developing a new product. Costs incurred by the R&D department in 20XX-1 on
the ―research phase‖ amounted to CU 200,000. In 20XX, technical and commercial feasibility of the
product was established. Costs incurred in 20XX were CU 20,000 personnel costs and CU 15,000
legal fees to register the patent. In 20XX, the entity incurred CU 30,000 to successfully defend a legal
suit to protect the patent. How would the entity account for these costs?

5. A patent right is acquired July 1, 20XX-1, for CU 250,000; while it has a legal life of 15 years, due to
rapidly changing technology, management estimates a useful life of only five years. Straight-line
amortization will be used. At January 1, 20XX, management is uncertain that the process can actually
be made economically feasible, and decides to write down the patent to an estimated market value of
CU 75,000. Amortization will be taken over three years from that point. On January 1, 20XX+2,
having perfected the related production process, the asset is now appraised at a depreciated
replacement cost of CU 300,000. Furthermore, the estimated useful life is now believed to be six more
years. Pass Journal entries to reflect these events.
Page 44 of 328
Paper1: Advanced Financial Reporting [RTP 2017 December]
6.
a. X has a 70% holding in Y and a 20% holding in Z. Y, in its turn, has a 35% holding in Z. Are either Z
or Y subsidiary of X?

b. Vehicle manufacturer X is the parent company of investment entity B. B has holdings in various
companies that are not active in the automotive sector. Whereas X is not an investment entity pursuant
to IFRS 10, B is classed as an investment entity pursuant to IFRS 10. For the purposes of this example
it should be assumed that both X and B must prepare consolidated financial statements. How will the
consolidation be done?

7. Regency Corporation acquired 40% of Alpha Company‘s shares on January 2, 20XX, but that the
price paid was CU 140,000. Alpha Company‘s assets and liabilities at that date had the following
book and fair values:

Book value Fair value


Cash CU 10,000 CU 10,000
Accounts receivable (net) 40,000 40,000
Inventories (FIFO cost) 80,000 90,000
Land 50,000 40,000
Plant and equipment (net of accumulated depreciation) 140,000 220,000
Total assets CU 320,000 CU 400,000
Liabilities (70,000) (70,000)
Net assets (shareholders’ equity) CU 250,000 CU 330,000

Assume that Alpha‘s plant and equipment have 10 years of useful life remaining and that Alpha
depreciates its property, plant, and equipment on a straight-line basis
a. How will Regency Corporation account for these investments under equity method of accounting?
b. Assume that Alpha sold the land in the year 2034 for CU 80,000. How will this be reflected in the
books of Regency?

8. Assume that Beta Co., that owns 25% of Lamda Co., sold to Lamda an item of property, plant, and
equipment having a five-year remaining life, at a gain of CU 100,000. Beta Co. expects to remain in
the 34% marginal tax bracket. The sale occurred at the end of 20XX-1; Lamda Co. will use straight-
line depreciation to amortize the asset over the years 20XX through 20XX+4.
Pass journal Entries relating to above transactions over years 20XX through 20XX+4

9. Winter Corp. is encouraged to relocate to Springville Township on July 1, 20XX, by an economic


stimulus package that includes a CU 3,000,000 loan due in equal annual instalments (inclusive of
interest) through 2025. The local government provides this loan at a below-market rate of 3%, which
differs markedly from Winter‘s own marginal borrowing rate of 6.5%.
Pass Journal Entries to demonstrate accounting treatment of above as per NFRS.

10. During 20XX-2 Trident Corp invests CU 1,000,000 in an item of plant, which has an anticipated
useful life of five years. Depreciation is recognized on a straight line basis. In the year of acquisition,
Trident Corp receives a government grant of CU 250,000 towards the purchase of this plant, which is
conditional on certain employment targets being achieved within the next three years (i.e. to the end
of 20XX).At the end of 20XX, it is evident that the employment targets will not be achieved and
therefore the criteria attached to the receipt of this grant has been failed. Present accounting treatment
under the two methods of presentation of the grant.

11. XYZ Inc. is a manufacturer of specialized equipment. Many of its customers do not have the
necessary funds or financing available for outright purchase. Because of this, XYZ offers a leasing
alternative. The data relative to a typical lease are as follows:
Page 45 of 328
Paper1: Advanced Financial Reporting [RTP 2017 December]
1. The non-cancelable fixed portion of the lease term is five years. The lessor has the option to
renew the lease for an additional three years at the same rental. The estimated useful life of the
asset is 10 years. The lessee guarantees a residual value of CU 40,000 at the end of five years,
but the guarantee lapses if the full three-year renewal period is exercised.
2. The lessor is to receive equal annual payments over the term of the lease. The leased property
reverts back to the lessor on termination of the lease.
3. The lease is initiated on January 1, 20XX. Payments are due on December 31 for the duration of
the lease term.
4. The cost of the equipment to XYZ Inc. is CU 100,000. The lessor incurs cost associated with
the inception of the lease in the amount of CU 2,500.
5. The selling price of the equipment for an outright purchase is CU 150,000.
6. The equipment is expected to have a residual value of CU 15,000 at the end of five years and
CU 10,000 at the end of eight years.
7. The lessor desires a return of 12% (the implicit rate).
Required:
Calculate Minimum Lease Payments
a. Accounting treatment for the lease in the books of lessor at the time of providing the asset on
lease
b. Accounting entry on December 31, 20XX+1 to record lease rental received

12. Needy company issued 5,000 convertible bonds on January 1, 20XX, which are due December 31,
20XX+3. Followings are other relevant details.
1. 500 convertible Needy Company bonds are acquired by Investor Corp. on January 1, 20XX.
The bonds are due December 31, 20XX+3.
2. The purchase price is par (CU 1,000 per bond); total cost is thus CU 500,000.
3. Interest is due in arrears, semi-annually, at a nominal rate of 5%.
4. Each bond is convertible into 150 ordinary shares of the issuer.
5. At purchase date, similar, non-convertible debt issued by borrowers having the same credit
rating as Needy Company yield 8%.
6. At purchase date, Needy Company common shares are trading at CU 5, and dividends over
the next 4 years are expected to be CU 0.20 per share per year.
7. The relevant risk-free rate on 4-year obligations is 4%
8. The historic variability of Needy Company‘s share price can be indicated by a standard
deviation of annual returns of 25%
In accordance with NAS 32,
a. Provide accounting entries for issuance of bonds in the books of the issuer Needy Company
b. Provide accounting entries for purchase of bonds in the books of Investor Corp.
c. How will Investor Corp account for the financial asset if the value of derivative cannot be
determined?

13. Assume that Debtor Corp. owes Friendly Bank CU 90,000 on a 5% interest-bearing non-amortizing
note payable in five years, plus accrued and unpaid interest, due immediately, of CU 4,500. Friendly
Bank agrees to a restructuring to assist Debtor Corp., which is suffering losses and is threatening to
declare bankruptcy. The interest rate is reduced to 4%, the principal is reduced to CU 72,500, and the
accrued interest is forgiven outright. Future payments will be on normal terms. Given the debtor‘s
current condition, the market rate of interest for its debt would actually be 12%.
Prepare accounting entries in the books of Debtor Corp. for this restructuring in accordance with NAS
39.

14. Assume that Raphael Corporation acquires the following equity instruments for investment
purposes during 20XX:
Security description Acquisition cost Fair value at year-end
1,000 shares Belarus Steel common stock CU 34,500 CU 37,000
2,000 shares Wimbledon pfd. ―A‖ share 125,000 109,500
1,000 shares Hillcrest common stock 74,250 88,750
Page 46 of 328
Paper1: Advanced Financial Reporting [RTP 2017 December]

Assume that, at the respective dates of acquisition, management of Raphael Corporation designated
the Belarus Steel and Hillcrest common stock investments as being for trading purposes, while the
Wimbledon preferred shares were designated as having been purchased for long-term investment
purposes (and will thus be categorized as available-for-sale rather than trading).

In accordance with NAS 39,


a. Provide the entries to record the purchases
b. Provide the entries to record for fair value adjustments
c. Assume that in January 20XX new information comes to Raphael Corporation management
regarding the viability of Wimbledon Corp. Based on this information, it is determined that the
decline in Wimbledon preferred share is probably not a temporary one, but rather is an
impairment of the asset as that term is used in NAS 39. Provide accounting entries to account for
the impairment.
d. Assume that in March 20XX new information comes to management‘s attention, which suggests
that the decline in Wimbledon preferred had indeed been only a temporary decline and in fact, the
value of Wimbledon now rises to CU 112,000

15. On July 1, 20XX, Orange Corp. borrows CU 5 million with a fixed maturity (no prepayment option)
of June 30, 20XX+4, carrying interest at the US prime interest rate + 1/2%. Interest payments are due
semi-annually; the entire principal is due at maturity. At the same date, Orange Corp. enters into a
―plain-vanilla-type‖ swap arrangement, calling for fixed payments at 8% and the receipt of prime +
1/2%, on a notional amount of CU 5 million. At that date prime is 7.5%, and there is no premium due
on the swap arrangement since the fixed and variable payments are equal.

Assume that in fact the hedge proves to be highly effective. Also, assume that the prime rate over the
four-year term of the loan, as of each interest payment date, is as follows, along with the fair value of
the remaining term of the interest swap at those dates:
Date Prime rate (%) Fair value of swap∗
(CU )
December 31, 20XX 6.5 (150,051)
June 30, 20XX+1 6.0 (196,580)
December 31, 20XX+1 6.5 (111,296)
June 30, 20XX+2 7.0 (45,374)
December 31, 20XX+2 7.5 0
June 30, 20XX+3 8.0 23,576
December 31, 20XX+3 8.5 24,038
June 30, 20XX+4 8.0 0
∗Fair values are determined as the present values of future cash flows resulting from expected interest
rate differentials, based on current prime rate, discounted at 8%.

Regarding the fair values presented in the foregoing table, it should be assumed that the fair values of
the swap contract are precisely equal to the present value, at each valuation date (assumed to be the
interest payment dates), of the differential future cash flows resulting from utilization of the swap.
Future variable interest rates (prime + 1/2%) are assumed to be the same as the existing rates at each
valuation. The discount rate, 8%, is assumed to be constant over time.

Required:
Prepare semi-annual accounting entries in the books of Orange Corp.

16. Assume that Kirloski Company has a CU 180,000 operating loss carryforward as of December 31,
20XX-1, scheduled to expire at the end of the following year. Taxable temporary differences of CU
240,000 exist that are expected to reverse in approximately equal amounts of CU 80,000 in 20XX,
20XX+1 and 20XX+2. Kirloski Company estimates that taxable income for 20XX (exclusive of the
Page 47 of 328
Paper1: Advanced Financial Reporting [RTP 2017 December]
reversal of existing temporary differences and the operating loss carry forward) will be CU
20,000.Kirloski Company expects to implement a qualifying tax planning strategy that will accelerate
the total of CU 240,000 of taxable temporary differences to 20XX. Expenses to implement the
strategy are estimated to approximate CU 30,000. The applicable expected tax rate is 40%.
How will this affect Deferred Tax accounted for in the books of Kirloski?

17. Parent Company owns 30% of the outstanding ordinary shares of an Associate Company and 70% of
the ordinary shares of a Subsidiary Company. Additional data for the year 20XX are as follows:
Associate Company Subsidiary Company
Net income CU 50,000 CU 100,000
Dividends paid 20,000 60,000

Discuss the tax effects of the stated events.

18. The following is a group structure of Entity A ―The Parent.‖


Entity A has a controlling interest in Entity B and C. Further the company has significant influence
over an Associate (Entity D) whereas one of the subsidiaries (Entity C) has significant influence over
another Associate (Entity E).
Entity A controls a Joint Venture (JV) together with a company called Capital Investors Inc.
Analyze the disclosures considerations with respect to related parties.

19. Dakar Corporation encounters the following expense situations as part of its quarterly reporting:
• Its largest customer, Festive Fabrics, has placed firm orders for the year that will result in sales of
CU 1,500,000 in the first quarter, CU 2,000,000 in the second quarter, CU 750,000 in the third quarter
and CU 1,650,000 in the fourth quarter.Dakar gives Festive Fabrics a 5%rebate if Festive Fabrics
buys at least CU 5 million of goods each year. Festive Fabrics exceeded the CU 5 million goal in the
preceding year and was expected to do so again in the current year.
• It incurs CU 24,000 of trade show fees in the first quarter for a trade show that will occur in the third
quarter.
• It pays CU 64,000 in advance in the second quarter for a series of advertisements that will run
through the third and fourth quarters.
• It receives a CU 32,000 property tax bill in the second quarter that applies to the following 12
months.
• It incurs annual factory air filter replacement costs of CU 6,000 in the first quarter.
• Its management team is entitled to a year-end bonus of CU 120,000 if it meets a sales target of CU
40 million, prior to any sales rebates, with the bonus dropping by CU 10,000 for every million dollars
of sales not achieved.
How does Dakar present these situations in the quarterly financial statements for the year?

20. ABC Inc. presented its most recent financial statements under the national GAAP through 20XX-1. It
adopted IFRS from 20XX and is required to prepare an opening IFRS statement of financial position
as at January 1, 20XX-1. In preparing the IFRS opening statement of financial position, ABC Inc.
noted the following:

Under its previous GAAP, ABC Inc. sold certain financial receivables as well as trade receivables for
the amount of CU 250,000 to special-purpose entities (SPEs) that are not consolidated although they
conduct activities on behalf of the Group. In addition, ABC Inc. was using the last- in first-out (LIFO)
method to account for certain inventories, and, consequently, reported the carrying value of inventory
reduced by CU 150,000, as compared to the value under the FIFO method. Furthermore, it had not
discounted, to present value, long-term provisions for warranty of CU 100,000 although the effect of
discounting would be material (CU 10,000). Finally, all research and development costs of CU
500,000 (of which total CU 300,000 relates to research costs) for the invention of new products were
expensed when incurred.

Required:
Page 48 of 328
Paper1: Advanced Financial Reporting [RTP 2017 December]
State the adjustments that would be required in the opening statement of financial position under IFRS
1
21. The following is the Statement Of Financial Position of N Ltd. as on 31st March, 2002:

Liabilities Rs. Assets Rs.


400000 Equity shares of Rs. 10 Goodwill 400,000
each fully paid 4,000,000 Building 2,400,000
13.5% Redeemable preference shares of Rs. 100 Machinery 2,200,000
each fully paid 2,000,000 Furniture 1,000,000
General Reserve 1,600,000 Vehicles 1,800,000
Profit and Loss Account 320,000 Investments 1,600,000
Bank Loan (Secured against PPE) 1,200,000 Stock 1,100,000
Bills Payable 600,000 Debtors 1,800,000
Creditors 3,100,000 Bank Balance 320,000
Preliminary Expenses 200,000
Total 12,820,000 Total 12,820,000

Further information:
(i) Return on capital employed is 20% in similar businesses.
(ii) PPE are worth 30% more than book value. Stock is overvalued by Rs. 1,00,000, Debtors are to
be reduced by Rs. 20,000. Trade investments, which constitute 10% of the total investments
are to be valued at 10% below cost.
(iii) Trade investments were purchased on 1.4.2001. 50% of non-Trade Investments were
purchased on 1.4.2000 and the rest on 1.4.1999. Non-Trade Investments yielded 15% return
on cost.
(iv) In 1999-2000 new machinery costing Rs. 2,00,000 was purchased, but wrongly charged to
revenue. This amount should be adjusted taking depreciation at 10% on reducing value
method.
(v) In 2000-2001 furniture with a book value of Rs. 1,00,000 was sold for Rs. 60,000.
(vi) For calculating goodwill two years purchase of super profits based on simple average
profits of last four years are to be considered. Profits of last four years are as under:
(vii) 1998-1999 Rs. 16,00,000, 1999-2000 Rs. 18,00,000, 2000-2001 Rs. 21,00,000, 2001- 2002
Rs. 22,00,000.
(viii) Additional depreciation provision at the rate of 10% on the additional value of Plant and
Machinery alone may be considered for arriving at average profit.
Find out the intrinsic value of the equity share
Income-tax and Dividend tax are not to be considered.

22. The primary objective of accounting and auditing regulatory bodies is to ensure that financial
statements present a true and fair view of the financial performance, position and cash flows of an
entity.
i. Define the term ‗true and fair view‘.
ii.Describe the role played by each of the following regulatory bodies:
a.The International Accounting Standards Board (‗NASB‘);
b.The IFRS Interpretation Committee;
c.The IFRS Advisory Council

23. The draft statements of financial position of Hulk Co, Molehill Co and Pimple Co as at 31 May 20X5
are as follows.
Page 49 of 328
Paper1: Advanced Financial Reporting [RTP 2017 December]
Particulars Hulk Co Molehill Pimple Co
Co
$ $ $
Non-current assets 90,000 60,000 60,000
Tangible assets
Investments in subsidiaries(cost)
Shares in Molehill Co 90,000
Shares in Pimple Co 25,000 42,000
Current assets 40,000 50,000 40,000
Total 245,000 152,000 100,000
Equity and liabilities
Equity
Ordinary shares $1 100,000 50,000 50,000
Share premium 50,000 20,000
Retained earnings 45,000 32,000 25,000
Non-current liabilities
12% loan 10,000
Current liabilities
Payables 50,000 40,000 25,000
Total 245,000 152,000 100,000

(a) Hulk Co acquired 60% of the shares in Molehill on 1 January 20X3 when the balance on that
company's retained earnings was $8,000 (credit) and there was no share premium account.
(b) Hulk acquired 20% of the shares of Pimple Co and Molehill acquired 60% of the shares of Pimple
Co on 1 January 20X4 when that company's retained earnings stood at $15,000.
(c) There has been no payment of dividends by either Molehill or Pimple since they became
subsidiaries.
(d) There was no impairment of goodwill.
(e) It is the group‘s policy to measure the non-controlling interest at acquisition at its proportionate
share of the fair value of the subsidiary‘s net assets.
Required
Prepare the consolidated statement of financial position of Hulk Co as at 31 May 20X5
Page 50 of 328
Paper1: Advanced Financial Reporting [RTP 2017 December]

Answers
Solution to Q. No. 1
For both illustrations, the net carrying amount (carrying amount or depreciated cost) immediately
prior to the revaluation is CU 120,000 [CU 200,000 – (2 × CU 40,000)]. The net upward revaluation
is given by the difference between fair value and net carrying amount, or CU 300,000 – CU 120,000 =
CU 180,000.

Option 1(a)
Applying the ―gross up‖ approach, since the fair value after two years of the five- year useful life have
already elapsed is found to be CU 300,000, the gross fair value (gross carrying amount) calculated
proportionally is 5/3 × CU 300,000 = CU 500,000. In order to have the net carrying amount equal to
the fair value after two years, the balance in accumulated depreciation needs to be CU 200,000.
Consequently, the buildings and accumulated depreciation accounts need to be restated upward as
follows: buildings up CU 300,000 (CU 500,000 – CU 200,000) and accumulated depreciation CU
120,000 (CU 200,000 – CU 80,000). Alternatively, this revaluation could be accomplished by
restating the buildings account and the accumulated depreciation account so that the ratio of net
carrying amount to gross carrying amount is 60% (CU 120,000/CU 200,000) and the net carrying
amount is CU 300,000. New gross carrying amount is calculated CU 300,000/.60 = x; x = CU
500,000.

The following journal entry and table illustrate the restatement of the accounts:
Buildings 300,000
Accumulated depreciation 120,000
Other comprehensive income—gain on revaluation 180,000
Original cost Revaluation Total %
Gross carrying amount CU 200,000 +CU 300,000 = CU 100
500,000
Accumulated depreciation 80,000 + 120,000 = 200,000 40
Net carrying amount CU 120,000 + CU 180,000 = CU 60
300,000

After the revaluation, the carrying amount of the buildings is CU 300,000 (= CU 500,000–200,000)
and the ratio of net carrying amount to gross carrying amount is 60% (= CU 300,000/CU 500,000).
This method is often used when an asset is revalued by means of applying an index to determine its
depreciated replacement cost.

Option 1(b).
Applying the ―gross up‖ approach where the gross fair value had separately been valued at CU
450,000 then both the Building and Accumulated depreciation entry would be reduced by CU 50,000
from the example above.
Buildings 250,000
Accumulated depreciation 70,000
Other comprehensive income—gain on revaluation 180,000

Option 2.
Applying the ―netting‖ approach, Danfe would eliminate accumulated depreciation of CU 80,000 and
then increase the building account by CU 180,000 so the net carrying amount is CU 300,000 (= CU
200,000 – CU 80,000 + CU 180,000):
Accumulated depreciation 80,000
Buildings 80,000
Buildings 180,000
Other comprehensive income—gain on revaluation 180,000
Page 51 of 328
Paper1: Advanced Financial Reporting [RTP 2017 December]
This method is often used for buildings. In terms of total assets reported in the statement of financial
position, option 2 has exactly the same effect as option 1.
Solution to Q. No. 2
The transaction is to be valued at the fair market value of the property transferred, and any gain or
loss on the transaction is to be recognized. Thus, Salaam should recognize a gain of CU 7,000 (= CU
17,000 – CU 10,000) in the determination of the current period‘s profit or loss. The entry to record the
transaction would be as follows:
Dividend paid 17,000
Property 10,000
Gain on transfer of property 7,000

Solution to Q. No. 3
Average Accumulated Expenditures

Date Expenditure Capitalization period∗ Average accumulated expenditures


1/1/20XX CU 2,000,000 2/12 CU 2,000,000
3/31/20XX CU 4,000,000 9/12 CU 3,000,000
6/30/20XX CU 6,100,000 6/12 CU 3,050,000
9/30/20XX CU 4,400,000 3/12 CU 1,100,000
12/31/20XX CU 3,500,000 0/12 --
CU 20,000,000 CU 9,150,000

∗The number of months between the date when expenditures were made and the date on which
interest capitalization stops (December 31, 20XX).

Potential Interest Cost to Be Capitalized


(CU 8,500,000 × 1.12551)∗ - CU 8,500,000 = CU 1,066,840
650,000 × 0.1109∗∗= 72,020
CU 9,150,000 CU 1,138,860

∗The principal, CU 8,500,000, is multiplied by the factor for the future amount of CU 1 for 4 periods
at 3% to determine the amount of principal and interest due in 20XX.
∗∗ Weighted-average interest rate
Principal Interest
10%, 10-year note CU 6,000,000 CU 600,000
12%, 5-year note CU 7,000,000 CU 840,000
CU 13,000,000 CU 1,440,000

Total interest CU 1,440, 000/Total principal CU 13,000,000 = 11:08%

The actual interest is


12%, 4-year note [(CU 8,500,000 × 1.12551) CU 8,500,000] = CU 1,066,840
10%, 10-year note (CU 6,000,000 × 10%) = CU 600,000
12%, 5-year note (CU 7,000,000 × 12%) = CU 840,000
Total interest CU 2,506,840

The interest cost to be capitalized is the lesser of CU 1,138,860 (avoidable interest) or CU 2,506,840
(actual interest). The remaining CU 1,367,980 (= CU 2,506,840 CU 1,138,860) must be expensed.
Page 52 of 328
Paper1: Advanced Financial Reporting [RTP 2017 December]
Solution to Q. No. 4
• Research and development costs incurred in 20XX-1, amounting to CU 200,000, should be
expensed, as they do not meet the recognition criteria for intangible assets. The costs do not result
in an identifiable asset capable of generating future economic benefits.
• Personnel and legal costs incurred in 20XX, amounting to CU 35,000, would be capitalized as
patents. The company has established technical and commercial feasibility of the product, as well
as obtained control over the use of the asset. The standard specifically prohibits the reinstatement
of costs previously recognized as an expense. Thus CU 200,000, recognized as an expense in the
previous financial statements, cannot be reinstated and capitalized.
• Legal costs of CU 30,000 incurred in 20XX to defend the entity in a patent lawsuit should be
expensed. These could be considered as expenses incurred to maintain the asset at its originally
assessed standard of performance, would not meet the recognition criteria under NAS 38.
• Alternatively, if the entity were to lose the patent lawsuit, then the useful life and the recoverable
amount of the intangible asset would be in question. The entity would be required to provide for
any impairment loss, and in all probability, even to fully write off the intangible asset. What is
required must be determined by the facts of the specific situation.

Solution to Q. No. 5
7/1/20XX-1 Patent 250,000
Cash, etc. 250,000
12/31/20XX-1 Amortization expense 25,000
Patent 25,000
1/1/20XX Loss from asset impairment 150,000
Patent 150,000
12/31/20XX Amortization expense 25,000
Patent 25,000
12/31/20XX+1 Amortization expense 25,000
Patent 25,000
1/1/20XX+2 Patent 275,000
Gain on asset value recovery 100,000
Other comprehensive income 175,000

The entry at year-end


20XX-1 is to record amortization based on original cost, since there had been no revaluations through
that time; only a half-year amortization is provided [(CU 250,000/5) × 1/2]. On January 1, 20XX, the
impairment is recorded by writing down the asset to the estimated value of CU 75,000, which
necessitates a CU 150,000 charge against profit (carrying amount, CU 225,000, less fair value, CU
75,000).

In 20XX and 20XX+1, amortization must be provided on the new lower value recorded at the
beginning of 20XX; furthermore, since the new estimated life was three years from January 20XX,
annual amortization will be CU 25,000.

As of January 1, 20XX+2, the carrying amount of the patent is CU 25,000; had the January 20XX
revaluation not beenmade, the carrying amount would have been CU 125,000 (CU 250,000 original
cost, less two-and-one-half years‘ amortization versus an original estimated life of five years). The
new appraised value is CU 300,000, which will fully recover the earlier write-down and add even
more asset value than the originally recognized cost. Under the guidance of NAS 38, the recovery of
CU 100,000 that had been charged to expense should be recognized as profit; the excess will be
recognized in other comprehensive income and increases the revaluation surplus for the asset in
equity.

Solution to Q. No. 6(a)


X controls Y and thus also Z, because X directly and indirectly holds 55% of its shares (voting rights).
Page 53 of 328
Paper1: Advanced Financial Reporting [RTP 2017 December]
Y‘s holding in Z must be allocated in full to X, as X controls Y. Y and Z must consequently be
included in X‘s consolidated financial statements.
X‘s share of Y‘s profit or loss or changes in equity is 44.5% (20% + (70% of 35%)). X consequently
allocates 55.5% of its subsidiary Y to non-controlling interests.

Solution to Q. No. 6 (b)


Since B is an investment entity, it must report companies in which it holds a controlling interest at fair
value through profit or loss. X, conversely, must fully consolidate all its subsidiaries, including B.

Solution to Q. No. 7

The first order of business is the calculation of the differential, as follows:


Regency‘s cost for 40% of Alpha‘s ordinary share CU 140,000
Book value of 40% of Alpha‘s net assets (CU 250,000 × 40%) (100,000)
Total differential CU 40,000

Next, the CU 40,000 is allocated to those individual assets and liabilities for which fair value differs
from book value. In the example, the differential is allocated to inventories, land, and plant and
equipment, as follows:
Item Book Value Fair Value Difference debit 40% of difference
(credit) debit (credit)
Inventories CU 80,000 CU 90,000 CU 10,000 CU 4,000
Land 50,000 40,000 (10,000) (4,000)
Plant and equipment 140,000 220,000 80,000 32,000
Differential allocated CU 32,000

The difference between the allocated differential of CU 32,000 and the total differential of CU 40,000
is essentially identical to goodwill of CU 8,000. As shown by the following computation, goodwill
represents the excess of the cost of the investment over the fair value of the net assets acquired.
Regency‘s cost for 40% of Alpha‘s ordinary share CU 140,000
40% of Alpha‘s net assets (CU 330,000 × 40%) (132,000)
Excess of cost over fair value (goodwill) CU 8,000

At this point it is important to note that the allocation of the differential is not recorded formally by
either Regency Corporation or Alpha Company. Furthermore, Regency does not remove the
differential from the investment account and allocate it to the respective assets, since the use of the
equity method does not involve the recording of individual assets and liabilities. Regency leaves the
differential of CU 40,000 in the investment account, as part of the balance of CU 140,000 at January
2, 20XX. Accordingly, information pertaining to the allocation of the differential is maintained by the
investor, but this information is outside the formal accounting system, which is comprised of journal
entries and account balances.

Under the provisions of IFRS 3, Regency may not amortize the unallocated differential, which is akin
to goodwill, but must consider its possible impairment whenever NAS 39 indicates that the
investment may be impaired. Regency would prepare the following amortization schedule

Amortization
Item Differential Useful life 20XX-1 20XX 20XX+1
debit (credit)
Inventories (FIFO) CU 4,000 Sold in 20XX-1 CU 4,000 CU – CU –
Land (4,000) Indefinite – – –
Plant and equipment (net) 32,000 10 years 3,200 3,200 3,200
Page 54 of 328
Paper1: Advanced Financial Reporting [RTP 2017 December]
Goodwill 8,000 N/A – – –
Totals CU 40,000 CU 7,200 CU 3,200 CU 3,200

Note that the entire differential allocated to inventories is amortized in 20XX because the cost flow
assumption used by Alpha is FIFO. If Alpha had been using weighted-average costing instead of
FIFO, amortization might have been computed on a different basis. Note also that the differential
allocated to Alpha‘s land is not amortized, because land is not a depreciable asset.
Goodwill likewise is not subject to amortization.
The amortization of the differential, to the extent required under IFRS, is recorded formally in the
accounting system of Regency Corporation. Recording the amortization adjusts the equity in Alpha‘s
income that Regency recorded based on Alpha‘s statement of comprehensive income.
Alpha‘s income must be adjusted because it is based on Alpha‘s book values, not on the cost that
Regency incurred to acquire Alpha. Regency would make the following entries in 20XX, assuming
that Alpha reported profit of CU 30,000 and paid cash dividends of CU 10,000:

1. Investment in Alpha 12,000


Equity in Alpha income (CU 30,000 × 40%) 12,000
2. Equity in Alpha income (amortization of differential) 7,200
Investment in Alpha 7,200
3. Cash 4,000
Investment in Alpha (CU 10,000 × 40%) 4,000

The balance in the investment account on Regency‘s records at the end of 20XX is CU 140,800 [= CU
140,000 + CU 12,000 – (CU 7,200 + CU 4,000)], and Alpha‘s shareholders‘ equity, as shown
previously, is CU 270,000. The investment account balance of CU 140,000 is not equal to 40% of CU
270,000.
However, this difference can easily be explained, as follows:
Balance in investment account at December 31, 20XX CU 140,800
40% of Alpha‘s net assets at December 31, 20XX 108,000
Difference at December 31, 20XX CU 32,800
Differential at January 2, 20XX CU 40,000
Differential amortized during 20XX (7,200)
Unamortized differential at December 31, 20XX CU 32,800

As the years go by, the balance in the investment account will come closer and closer to representing
40% of the book value of Alpha‘s net assets. After 20 years, the remaining difference between these
two amounts would be attributed to the original differential allocated to land (a CU 4,000 credit) and
the amount similar to goodwill (CU 8,000), unless written off due to impairment.
This CU 4,000 difference on the land would remain until Alpha sold it.

Since Alpha‘s cost for the land was CU 50,000, it would report a gain of CU 30,000, of which CU
12,000 (= CU 30,000 × 40%) would be recorded by Regency, when it records its 40% share of
Alpha‘s reported profit, ignoring income taxes. However, from Regency‘s viewpoint, the gain on sale
of land should have been CU 40,000 (CU 80,000 CU 40,000) because the cost of the land from
Regency‘s perspective was CU 40,000 at January 2, 20XX.
Therefore, besides the CU 12,000 share of the gain recorded above, Regency should record an
additional CU 4,000 gain [(= CU 40,000 – CU 30,000) × 40%] by debiting the investment account
and crediting the equity in Alpha income account. This CU 4,000 debit to the investment account will
negate the CU 4,000 differential allocated to land on January 2, 20XX, since the original differential
was a credit (the fair value of the land was CU 10,000 less than its book value).

Solution to Q. No. 8

20XX-1
Page 55 of 328
Paper1: Advanced Financial Reporting [RTP 2017 December]
1. Gain on sale of property, plant and equipment 25,000
Deferred gain 25,000
To defer the unrealized portion of the gain.
2. Deferred tax benefit 8,500
Income tax expense 8,500
Tax effect of gain deferral.

Alternatively, the 20XX-1 events could have been reported by this single entry.
Equity in Winter income 16,500
Investment in Winter Co. 16,500

20XX through 20XX+4 (each year):


1. Deferred gain 5,000
Gain on sale of property, plant and equipment 5,000
To amortize deferred gain.
2. Income tax expense 1,700
Deferred tax benefit 1,700
Tax effect of gain realization.

The alternative treatment would be:


Investment in Winter Co. 3,300
Equity in Winter income 3,300

The tax currently paid by Beta Co. (34%× CU 25,000 taxable gain on the transaction) is recorded as a
deferred tax benefit in 20XX-1 since taxes will not be due on the book gain recognized in the years
20XX through 20XX+4. Under provisions of NAS 12, deferred tax benefits should be recorded to
reflect the tax effects of all deductible temporary differences. Unless Beta Co. could demonstrate that
future taxable amounts arising from existing temporary differences exist, this deferred tax benefit
might be offset by an equivalent valuation allowance in Beta Co.‘s statement of financial position at
year-end 20XX-1, because of the doubt that it will ever be realized. Thus, the deferred tax benefit
might not be recognizable, net of the valuation allowance, for financial reporting purposes unless
other temporary differences not specified in the example provided future taxable amounts to offset the
net deductible effect of the deferred gain.

Solution to Q. No. 9
The present value of the annual payments (CU 351,000 each), discounted at 6.5%, is only CU
2,528,251. Accordingly, the receipt of the loan on July 1, 20XX, is recorded by the following journal
entry:
Cash 3,000,000
Loan payable 2,528,251
Income—government grants 471,749

The discount on the loan payable is amortized over the 10-year term, such that an effective rate of
6.5% on the loan balance will be reported as interest expense in Maytag‘s income statements. If the
grant was unconditional, it would be taken into income immediately, as suggested by the above
journal entry. However, if Maytag has ongoing obligations (such as to remain as an employer in the
community throughout the term of the loan), then it should be amortized to income (on a straight-line
basis) over the term of the obligation.

Solution to Q. No. 10
Method A: Grant shown as deferred income
Grant received and credited in 20XX-2 to deferred income CU 250,000
Recognized in profit or loss 20XX-2 to 20XX (3 × CU 50,000) (CU 150,000)
Deferred income balance at end of 20XX before repayment of grant CU 100,000
Page 56 of 328
Paper1: Advanced Financial Reporting [RTP 2017 December]
Total repayment of grant (cr bank) CU 250,000
Repayment debited to deferred income balance (dr deferred income) (CU 100,000)
Balance of repayment recognized in profit or loss (dr profit or loss) CU 150,000

Method B: Grant deducted from cost of asset


Cost of plant CU 1,000,000
Less grant received (CU 250,000)
Net cost of equipment CU 750,000
Depreciation expense recognized 20XX-2 to 20XX (3 × CU 150,000) (CU 450,000)
Carrying amount of plant at end of 20XX before repayment of grant CU 300,000
Add back grant repayable CU 250,000
CU 550,000
Cumulative additional depreciation to be recognized in profit or loss for CU 150,000
20XX-2 to 20XX (3 × CU 50,000)
Carrying amount of equipment at end of 20XX after repayment of grant CU 400,000

Plant Dr. 250,000


Depreciation Dr. (P/L) 150,000
Bank Cr. 250,000
Accumulated Depreciation 150,000

Solution to Q. No. 11
The first step is to calculate the annual payment due to the lessor. Recall that the present value (PV) of
the minimum lease payments is equal to the selling price adjusted for the present value of the residual
amount. The present value is to be computed using the implicit interest rate and the lease term. In this
case, the implicit rate is given as 12% and the lease term is eight years (which includes the fixed non-
cancelable portion plus the renewal period, since the lessee guarantee terms make renewal virtually
inevitable). Thus, the structure of the computation would be as follows:

Normal selling price - PV of residual value = PV of minimum lease payment


Or, in this case,
CU 150,000 - (0.40388∗× CU 10,000) = 4.96764∗∗× Minimum lease payment
CU 145,961.20 ÷ 4.96764 = Minimum lease payment
CU 29,382.40 = Minimum lease payment
∗0.40388 is the present value of an amount of CU 1 due in eight periods at a 12% interest rate.
∗∗4.96764 is the present value of an annuity of CU 1 for eight periods at a 12% interest rate.

Prior to examining the accounting implications of a lease, we must determine the lease classification.
In this example, the lease term is eight years (discussed above) while the estimated useful life of the
asset is 10 years; thus this lease qualifies as something other than an operating lease. Note that the
lease also meets the FMV versus PV criterion because the PV of the minimum lease payments of CU
145,961.20, which is 97% of the FMV [CU 150,000], could be considered to be equal to substantially
all of the fair value of the leased asset. Now it must be determined if this is a sales-type or direct
financing lease. To do this, examine the FMV or selling price of the asset and compare it to the cost.
Because the two are not equal, we can determine this to be a sales-type lease. Next, obtain the figures
necessary to record the entry on the books of the lessor. The gross investment is the total minimum
lease payments plus the unguaranteed residual value, or CU 29; 382.40 × 8)+ CU 10,000 =
245,059:20

The cost of goods sold is the historical cost of the inventory (CU 100,000) plus any initial direct costs
(CU 2,500) less the PV of the unguaranteed residual value (CU 10,000 × 0.40388). Thus, the cost of
goods sold amount is CU 98,461.20 (CU 100,000 + CU 2,500 - CU 4,038.80). Note that the initial
direct costs will require a credit entry to some account, usually accounts payable or cash. The
inventory account is credited for the carrying value of the asset, in this case CU 100,000. The adjusted
Page 57 of 328
Paper1: Advanced Financial Reporting [RTP 2017 December]
selling price is equal to the PV of the minimum payments, or CU 145,961.20. Finally, the unearned
finance income is equal to the gross investment (i.e., lease receivable) less the present value of the
components making up the gross investment (the minimum lease payment of CU 29,382.40 and the
unguaranteed residual of CU 10,000). The present value of these items is CU 150,000 [(CU 29,382.40
× 4.96764) + (CU 10,000 × 0.40388)]. Therefore, the entry necessary to record the lease is:

Lease receivable 245,059.20


Cost of goods sold 98,461.20
Inventory 100,000.00
Sales 145,961.20
Unearned finance income 95,059.20
Accounts payable (initial direct costs) 2,500.00

According to NAS 17, interest is recognized on a basis such that a constant periodic rate of return is
earned over the term of the lease. This will require setting up an amortization schedule as illustrated
below.
Date or year ended Payment Interest Principal Balance
January 1, 20XX CU 150,000.00
December 31, 20XX+1 CU 29,382.40 CU 18,000.00 CU 11,382.40 CU 138,617.00
December 31, 20XX+2 CU 29,382.40 CU 16,634.11 CU 12,748.29 CU 125,869.31
December 31, 20XX+3 CU 29,382.40 CU 15,104.32 CU 14,278.08 CU 111,591.23
December 31, 20XX+4 CU 29,382.40 CU 13,390.95 CU 15,991.45 CU 95,599.78
December 31, 20XX+5 CU 29,382.40 CU 11,471.97 CU 17,910.43 CU 77,689.35
December 31, 20XX+6 CU 29,382.40 CU 9,322.72 CU 20,059.68 CU 57,629.67
December 31, 20XX+7 CU 29,382.40 CU 6,915.56 CU 22,466.84 CU 35,162.83
December 31, 20XX+8 CU 29,382.40 CU 4,219.57 CU 25,162.83 CU 10,000.00
Total CU CU 95,059.20 CU 140,000.0
235,059.20

The entries below illustrate the proper treatment to record the receipt of the lease payment and the
amortization of the unearned finance income in the year ended December 31, 20XX+1.
Cash 29,382.40
Lease receivable 29,382.40
Unearned finance income 18,000.00
Interest revenue 18,000.00
Page 58 of 328
Paper1: Advanced Financial Reporting [RTP 2017 December]
Solution to Q. No. 12
a. Under the provisions of revised NAS 32, the issuer of compound financial instruments must assign
full fair value to the portion that is to be classified as a liability, with only the residual value being
allocated to the equity component. The computation for the above fact situation would be as follows:
1. Use the reference discount rate, 8%, to compute the market value of straight debt carrying a
5% yield:
PV of CU 5,000,000 due in 4 years, discounted at 8% CU 3,653,451
PV of semi-annual payments of CU 125,000 for 8 periods, discounted at 8% 841,593
Total CU 4,495,044
2. Compute the amount allocable to the conversion feature:
Total proceeds from issuance of compound instrument CU 5,000,000
Value allocable to debt 4,495,044
Residual value allocable to equity component CU 504,956

Thus, Needy Company received CU 4,495,044 in consideration of the actual debt being issued, plus a
further CU 504,956 for the conversion feature, which is a call option on the underlying ordinary share
of the issuer. The entry to record this would be:
Cash 5,000,000 Dr
Discount on bonds payable 504,956 Dr
Bonds payable 5,000,000 Cr
Paid-in capital—bond conversion option 504,956 Cr
The bond discount would be amortized as additional interest over the term of the debt.

b. In terms of NAS 32, the fair value of the conversion feature should be determined, if possible, and
assigned to that embedded derivative. In this example, the popular Black-Scholes-Merton model will
be used (but other approaches are also acceptable).
1. Compute the standard deviation of proportionate changes in the fair value of the asset
underlying the option multiplied by the square root of the time to expiration of the option.
0 25 X √4 =0.25 X 2= 0.50

2. Compute the ratio of the fair value of the asset underlying the option to the present value of
the option exercise price.
a. Since the expected dividend per share is CU 0.20 per year, the present value of this stream over 4
years would (at the risk-free rate) be CU 0.726.
b. The shares are trading at CU 5.00.
c. Therefore, the value of the underlying optioned asset, stripped of the stream of dividends that a
holder of an unexercised option would obviously not receive, is CU 5:00 - .726 = CU 4.274 per share
d. The implicit exercise price is CU 1,000 ÷ 150 shares = CU 6.667 per share. This must be
discounted at the risk-free rate, 4%, over 4 years, assuming that conversion takes place at the
expiration of the conversion period, as follows:
CU 6:667 ÷ 1.044= 6:667 ÷ 1.170 = CU 5:699

e. Therefore, the ratio of the underlying asset, CU 4.274, to the present value of the exercise price, CU
5.699, is .750.

3. Reference must now be made to a call option valuation table to assign a fair value to these
two computed amounts (the standard deviation of proportionate changes in the fair value of the asset
underlying the option multiplied by the square root of the time to expiration of the option, .50, and the
ratio of the fair value of the asset underlying the option to the present value of the option exercise
price, .750). For this, assume that the table value is 13.44% (meaning that the fair value of the option
is 13.44%) of the fair value of the underlying asset.

4. The valuation of the conversion option, then, is given as .1344 X CU 4:274 per share X 150
shares/bond X 500 bonds= CU 43; 082
Page 59 of 328
Paper1: Advanced Financial Reporting [RTP 2017 December]
5. Since the fair value of the options (CU 43,082) has been determined, this is assigned to the
conversion option. The difference between the cost of the hybrid investment, CU 500,000, and the
amount allocated to the conversion feature, CU 43,082, or CU 456,918, should be attributed to the
debt instrument.

6. The discount on the debt should be amortized, using the effective yield method, over the
projected four-year holding period. The effective yield, taking into account the semi-annual interest
payments to be received, will be about 7.54%.

c. If, for some reason, the value of the derivative (the conversion feature, in this case) could not be
ascertained, the fair value of the debt portion would be computed, and the residual allocated to the
derivative. (Same as solution a)

Solution to Q. No. 13
Whether there is recognition of a gain on the restructuring depends on the 10% threshold. The
relevant discount rate to be used to compare the present values of the old and the new debt obligations
is 5%. The present value of the old debt is simply the principal amount, CU 90,000, plus the interest
due at present, CU 4,500, for a total of CU 94,500.
The present value of the replacement debt is the discounted present value of the reduced principal and
the reduced future interest payments; the forgiven interest does not affect this. The new principal, CU
72,500, discounted at 5%, equals CU 56,806. The stream of future interest payments (CU 72,500 ×
.04 = CU 2,900 annually in arrears), discounted at 5%, equals CU 12,555.
The total present value, therefore, is CU 69,361, which is about 27% below the present value of the
old debt obligation. Thus, the 10% threshold is exceeded, and a gain will be recognized at the date of
the restructuring.
However, given Debtor‘s current condition, the market rate of interest for its debt would actually be
12%, and since the new obligation must be recorded at fair value, this must be computed. The present
value of the reduced principal, CU 72,500, discounted at 12%, has a present value of CU 41,138. The
stream of future interest payments (CU 72,500 × .04 = CU 2,900 annually, in arrears), discounted at
12%, has a present value of CU 10,454. The total obligation thus has a fair value of CU 51,592.

The entry to record this event would be:


Debt obligation (old) payable 90,000
Interest payable 4,500
Discount on debt obligation (new) 20,908
Debt obligation (new) payable 72,500
Gain on debt restructuring 42,908

Note that the new debt obligation is recorded at a net of CU 51,592, not at the face value of CU
72,500. The difference, CU 20,908, is a discount to be amortized to interest expense over the next five
years, in order to reflect the actual market rate of 12%, rather than the nominal 4% being charged.
Amortization should be accomplished on the effective yield method

Solution to Q. No. 14
a. The entries to record the purchases are as follows:
Investment in equity instruments—held-for-trading 108,750
Cash 108,750
Investment in equity instruments—available-for-sale 125,000
Cash 125,000
b. At year-end, both portfolios are adjusted to fair value; the decline in Wimbledon preferred share,
series A, is judged to be a temporary market fluctuation because there is no objective evidence of
impairment. The entries to adjust the investment accounts at December 31, 20XX are as follows:

Investment in equity instruments—held-for-trading 17,000


Gain on holding equity instruments 17,000
Page 60 of 328
Paper1: Advanced Financial Reporting [RTP 2017 December]
Unrealized loss on equity instruments (OCI)—available-for-sale 15,500
Investment in equity instruments—available-for-sale 15,500

Thus, the change in value of the portfolio of trading financial assets is recognized in profit or loss,
whereas the change in the value of the available-for-sale financial assets is reflected in other
comprehensive income and accumulated in equity.

c. The standard prescribes that such a decline be reflected in profit or loss. The share‘s fair value has
remained at the amount last reported, CU 109,500, but this value is no longer viewed as being
only a market fluctuation. Accordingly, the entry to recognize the fact of the investment‘s
permanent impairment is as follows:
Impairment loss on holding equity instruments 15,500
Unrealized loss on equity instruments—available-for-sale
15,500
(other comprehensive income)

d. It would not be permitted under NAS 39 to reverse the impairment loss that had been included in
profit or loss. The carrying value after the recognition of the impairment was CU 109,500, and the
current period increase to CU 112,000 will have to be accounted for as an increase to be reflected
in other comprehensive income, rather than in profit or loss. The entry required to reflect this is:

Investment in equity instruments—available-for-sale 2,500


Unrealized gain on equity instruments—available-for-sale 2,500
(other comprehensive income)

Solution to Q. No. 15
December 31, 20XX
Interest expense 175,000
Accrued interest (or cash) 175,000
To accrue or pay interest on the debt at the variable rate of prime + 1/2% (7.0%)
Interest expense 25,000
Accrued interest (or cash) 25,000
To record net settle-up on swap arrangement [8.0 -7.0%]
Other comprehensive income 150,051
Obligation under swap contract 150,051
To record the fair value of the swap contract as of this date (a net liability because fixed rate payable
is below expected variable rate based on current prime rate)

June 30, 20XX+1


Interest expense 162,500
Accrued interest (or cash) 162,500
To accrue or pay interest on the debt at the variable rate of prime + 1/2% (6.5%)
Interest expense 37,500
Accrued interest (or cash) 37,500
To record net settle-up on swap arrangement [8.0 - 6.5%]
Other comprehensive income 46,529
Obligation under swap contract 46,529
To record the fair value of the swap contract as of this date (increase in obligation because of
further decline in prime rate)

December 31, 20XX+1


Interest expense 175,000
Accrued interest (or cash) 175,000
To accrue or pay interest on the debt at the variable rate of prime + 1/2% (7.0%)
Interest expense 25,000
Page 61 of 328
Paper1: Advanced Financial Reporting [RTP 2017 December]
Accrued interest (or cash) 25,000
To record net settle-up on swap arrangement [8.0 - 7.0%]
Obligation under swap contract 85,284
Other comprehensive income 85,284
To record the fair value of the swap contract as of this date (decrease in obligation due to increase in
prime rate)
June 30, 20XX+2
Interest expense 187,500
Accrued interest (or cash) 187,500
To accrue or pay interest on the debt at the variable rate of prime + 1/2% (7.5%)
Interest expense 12,500
Accrued interest (or cash) 12,500
To record net settle-up on swap arrangement [8.0 - 7.5%]
Obligation under swap contract 65,922
Other comprehensive income 65,922
To record the fair value of the swap contract as of this date (further increase in prime rate reduces fair
value of derivative)
December 31, 20XX+2
Interest expense 200,000
Accrued interest (or cash) 200,000
To accrue or pay interest on the debt at the variable rate of prime + 1/2% (8.0%)
Interest expense 0
Accrued interest (or cash) 0
To record net settle-up on swap arrangement [8.0 -8.0%]
Obligation under swap contract 45,374
Other comprehensive income 45,374
To record the fair value of the swap contract as of this date (further increase in prime rate eliminates
fair value of the derivative)
June 30, 20XX+3
Interest expense 212,500
Accrued interest (or cash) 212,500
To accrue or pay interest on the debt at the variable rate of prime + 1/2% (8.5%)
Accrued interest (or cash) 12,500
Interest expense 12,500
To record net settle-up on swap arrangement [8.0 - 8.5%]
Receivable under swap contract 23,576
Other comprehensive income 23,576
To record the fair value of the swap contract as of this date (increase in prime rate creates net asset
position for derivative)
December 31, 20XX+3
Interest expense 225,000
Accrued interest (or cash) 225,000
To accrue or pay interest on the debt at the variable rate of prime + 1/2% (9.0%)
Accrued interest (or cash) 25,000
Interest expense 25,000
To record net settle-up on swap arrangement [8.0 - 9.0%]
Receivable under swap contract 462
Other comprehensive income 462
To record the fair value of the swap contract as of this date (increase in asset value due to further rise
in prime rate)
June 30, 20XX+4
Interest expense 212,500
Accrued interest (or cash) 212,500
To accrue or pay interest on the debt at the variable rate of prime + 1/2% (8.5%)
Accrued interest (or cash) 12,500
Page 62 of 328
Paper1: Advanced Financial Reporting [RTP 2017 December]
Interest expense 12,500
To record net settle-up on swap arrangement [8.0 - 8.5%]
Other comprehensive income 24,038
Receivable under swap contract 24,038
To record the fair value of the swap contract as of this date (value declines to zero as expiration date
approaches)

Solution to Q. No. 16
In the absence of the tax planning strategy, CU 100,000 of the operating loss carryforward could be
realized in 20XX based on estimated taxable income of CU 20,000 plus CU 80,000 of the reversal of
taxable temporary differences. Thus, CU 80,000 would expire unused at the end of 20XX and the net
amount of the deferred tax asset at 12/31/XX would be recognized at CU 40,000, computed as CU
72,000 (= CU 180,000 × 40%) minus the valuation allowance of CU 32,000 (CU 80,000 × 40%).

However, by implementing the tax planning strategy, the deferred tax asset is calculated as follows:
Taxable income for 20XX
Expected amount without reversal of taxable temporary differences 20,000
Reversal of taxable temporary differences due to tax planning strategy (net of cost) 210,000
230,000
Operating loss to be carried forward (180,000)
Operating loss expiring unused at 12/31/XX CU 0
The deferred tax asset to be recorded at 12/31/15 is CU 54,000. This is computed as follows:
Full benefit of tax loss carryforward CU 180,000 × 40% = CU 72,000
Less: Net-of-tax effect of anticipated expenses related to implementation of the strategy CU 30,000 -
(CU 30,000 × 40%) = 18,000
Net CU 54,000
Kirloski Company will also recognize a deferred tax liability of CU 96,000 at the end of 20XX (40%
of the taxable temporary differences of CU 240,000).

Solution to Q. No. 17
Investment in associate company. The investment in the associate company will be equity
accounted. The equity income capitalized will be after the dividend received. The investments in the
associate will thus increase with CU 9,000 (30% × (CU 50,000 CU 20,000)). Deferred tax needs to
be created on the increase of the investment of CU 9,000. The increase in the carrying amount could
be recovered through dividends or through the ultimate sale of the associate. Dividend income might
be taxed at a different rate than the capital gains on the sale of the associate. Assume that only 20% of
the dividend is subject to tax of 34%and the capital gains tax rate is also 34%. Based on recovery
through dividends the deferred tax will be CU 612 (20% × 34% × CU 9,000). Based on the recovery
through sale the deferred tax will be CU 3,060 (34% × CU 9,000).

Investment in subsidiary company. Normally an investment in a subsidiary company will be


recorded at cost in the records of the parent company. No deferred tax will therefore be recognized.
However, if the option is followed to fair value the investment, deferred tax must be created using the
appropriate rate of recovery of the investment, unless the exception to the general rule applies.

However, in the consolidated financial statements the investment in the subsidiary will be replaced by
the assets and liabilities. Therefore any deferred tax created on the investment in the subsidiary
company in the parents‘ own financial statements should also be reversed.

Solution to Q. No. 18
In the Entity A‘s separate financial statements, all the Subsidiaries, Associates and the Joint Ventures
(JV) are all related parties. The joint venturer, Capital Investors Inc., doesn‘t qualify to be disclosed as
a related party.

In Entity B‘s financial statements, The Parent, Entity C, the Associates and the JV are related parties.
Page 63 of 328
Paper1: Advanced Financial Reporting [RTP 2017 December]

In Entity C‘s separate financial statements, The Parent, Entity B, the Associates and the JV are related
parties.

In the financial statements of Associates D and E, The Parent and Subsidiaries and the JV are related
parties. Associates D and E are not related to each other though.
Page 64 of 328
Paper1: Advanced Financial Reporting [RTP 2017 December]
Solution to Q. No. 19
Quarter 1 Quarter 2 Quarter 3 Quarter 4 Full year
Sales CU CU CU CU CU
10,000,000 8,500,000 7,200,000 11,800,000 37,500,000
Deduction from sales (75,000)1 (100,000) (37,500) (82,500) (295,000)
Marketing expense 24,0002 24,000
Advertising expense 32,0003 32,000 64,000
Property tax expense 8,0004 8,000 8,000 24,000
Maintenance expense 1,5005 1,500 1,500 1,500 6,000
Bonus expense 30,0006 25,500 21,600 17,900 95,000

1 The sales rebate is based on 5% of the actual sales to the customer in the quarter when the sale is
incurred. The actual payment back to the customer does not occur until the end of the year, when the
CU 5 million goal is definitively reached. Since the firm orders for the full year exceed the threshold
for rebates, the obligation is deemed probable and must be recorded.
2 The CU 24,000 trade show payment is initially recorded as a prepaid expense and then charged to
marketing expense when the trade show occurs.
3 The CU 64,000 advertising payment is initially recorded as a prepaid expense and then charged to
advertising expense when the advertisements run.
4 The CU 32,000 property tax payment is initially recorded as a prepaid expense and then charged to
property tax expense on a straight-line basis over the next four quarters.
5 The CU 6,000 air filter replacement payment is initially recorded as a prepaid expense and then
charged to maintenance expense over the one-year life of the air filters.
6 The management bonus is recognized in proportion to the amount of revenue recognized in each
quarter. Once it becomes apparent that the full sales target will not be reached, the bonus accrual
should be adjusted downward. In this case, the downward adjustment is assumed to be in the fourth
quarter, since past history and seasonality factors made non-achievement of the full goal unlikely until
fourth quarter results were known. (Note: with other fact patterns, quarterly accruals may have
differed.)

Solution to Q. No. 20
In order to prepare the opening IFRS statement of financial position at January 1, 20XX-1, ABC Inc.
would need to make the following adjustments to its statement of financial position at December 31,
20XX-2, presented under its previous GAAP:
1. IFRS 10 requires ABC Inc. to consolidate a SPE where it is deemed to control it. Indicators of
control include the SPE conducting activities on behalf of the Group and/or the Group holding the
majority of the risks and rewards of the SPE. Thus, SPEs should be consolidated and CU 250,000 of
receivables is recognized under IFRS;
2. NAS 2 prohibits the use of LIFO. Consequently, the Group adopted the FIFO method and had
to increase inventory by CU 150,000 under IFRS;
3. NAS 37 states that long-term provisions must be discounted to their present value if the effect
from discounting is material. As a result, the Group adjusted the amount of provisions for warranty by
CU 10,000, the effect from discounting;
4. NAS 38 allows that development costs are capitalized as intangible assets if the technical and
economic feasibility of a project can be demonstrated. Thus, CU 200,000 incurred on development
costs should be capitalized as an intangible asset under IFRS.

Solution to Q. No. 21
1. Calculation of Goodwill
(i) Capital employed
Fixed Assets Amount Amount
Building 2,400,000.00
Machinery (Rs. 2200000 + Rs. 145800) 2,345,800.00
Page 65 of 328
Paper1: Advanced Financial Reporting [RTP 2017 December]
Furniture 1,000,000.00
Vehicles 1,800,000.00
7,545,800.00
Add: 30% increase 2,263,740.00
9,809,540.00
Trade investments (Rs.1600000 × 10% × 90%) 144,000.00
Debtors (Rs. 1800000 – Rs. 20000) 1,780,000.00
Stock (Rs. 1100000 – Rs. 100000) 1,000,000.00
Bank balance 320,000.00 13,053,540.00
Less: Outside liabilities
Bank Loan 1,200,000.00
Bills payable 600,000.00
Creditors 3,100,000.00 4,900,000.00
Capital employed 8,153,540.00

(ii) Future maintainable profit


Calculation of adjusted profit 1998-99 1999-2000 2000-01 2001-02
Profit given 1,600,000 1,800,000 2,100,000 2,200,000
Add: Capital expenditure of machinery charged to
revenue 200,000
Loss on sale of furniture 40,000
1,600,000 2,000,000 2,140,000 2,200,000
Less: Depreciation on machinery 20,000 18,000 16,200
Income from non- trade investments 108,000 216,000 216,000
Reduction in value of stock 100,000
Bad debts 20,000
Adjusted Profits 1,600,000 1,872,000 1,906,000 1,847,800

Total adjusted profit for four years (1998-1999 to 2001-2002) 7,225,800


Average profit (Rs. 7225800/4) 1,806,450
Less: Depreciation at 10% on additional value of machinery
(2200000 + 145800) × 30/100 i.e. Rs. 703740) 70,374
Adjusted average profit 1,736,076

(iii) Normal Profit = 20% on capital employed i.e. 20% on Rs. 8,153,540 = Rs.1,630,708
(iv) Super profit = Expected profit – normal profit = Rs. 1,736,076 – Rs. 1,630,708 = Rs.
105,368

(v) Goodwill
At 2 years‘ purchase of super profit = Rs. 105368 × 2 = Rs. 210,736

2. Net assets available to equity


shareholders

Goodwill as calculated in 1(v) above 210,736


Sundry fixed assets 9,809,540
Trade and Non-trade investments 1,584,000
Debtors 1,780,000
Page 66 of 328
Paper1: Advanced Financial Reporting [RTP 2017 December]
Stock 1,000,000
Bank balance 320,000
14,704,276
Less: Outside liabilities
Bank loan 1,200,000
Bills payable 600,000
Creditors 3,100,000 4,900,000
Preference share capital 2,000,000
Net assets for equity shareholders 7,804,276

3. Valuation of Equity Shares


Value of equity share = (Net Assets available to equity shareholders)/(Number of equity shares) = Rs.
19.51

Note:
1. Depreciation on the overall increased value of assets (worth 30% more than book value) has not
been considered. Depreciation on the additional value of only plantand machinery has been
considered taking depreciation at 10% on reducing value method while calculating average adjusted
profit.
2. Loss on sale of furniture has been taken as non-recurring or extraordinary item.
3. It has been assumed that preference dividend has been paid till date.

Solution to Q. No. 22
(a) i.True and fair view
The term true and fair has never been defined by the courts however it would appear that in
order for a set of account s to give a true and fair view of the financial performance, position
and cash flows of an entity they must comply with both International Financial Reporting
Standards (IFRS‘s) and Company Law. In very rare circumstances entities may chose not to
comply with the provisions of accounting standards. This occurs where compliance with the
standard together
with additional disclosure would not result in a fair presentation and is referred to as the true
and fair override.
Financial reporting standards tend to deal with how the various figures in the financial
statements should be calculated while the Company Law tends to focus on how the
information should be presented to users. This is how both the standards and law work
together to ensure that the information presented in financial statements is true and fair and
therefore can be relied upon by users of financial information as the basis for making
decisions regarding the entity.
Page 67 of 328
Paper1: Advanced Financial Reporting [RTP 2017 December]
NASB
The NASB has responsibility in two main areas:
-
Issuing new International Financial Reporting Standards (IFRS). An IFRS will sometimes
contain more than one permitted accounting treatment for a transaction or event. One of the
permitted accounting treatments will usually be designated the most appropriate treatment.
The other permitted treatments are classified as allowed alternative treatments.

Approving interpretations developed by the IFRS Interpretations Committee. The NASB has
16 members drawn from around the world. The NASB has full control over developing and
setting its own agenda, the IFRS Foundation considers this agenda but does not have the
power of determination.

The IFRS Interpretation Committee


The IFRS Interpretation Committee is comprised of 14 members and is responsible for the
following:
- Interpreting the application of IFRSs and providing timely guidance on financial
reporting issues not specifically addressed in IFRSs or NASs.
- Undertaking other tasks as requested by the NASB.
- Publishing Draft Interpretations for public comment and considering comments made
within a reasonable period before finalising an Interpretation.
- Reporting to the NASB and obtaining NASB approval for final Interpretations.

IFRS Advisory Council


The IFRS Advisory Council (‗the Council‘) provides a forum for participation by
organisations and individuals interested in international financial reporting. There are
between 30 and 40 members of the Council including representatives from national standard
setters.
The role envisaged for national standard setters in the future is for them to become part of the
Council that will feed into the setting of new standards. The main objectives of the Council
are as follows:

To advise the NASB on the technical agenda and to prioritise the NASBs work
To inform the NASB of the views of the organisations and individuals on the Council on
major standard setting projects
To give other advice to the NASB
Page 68 of 328
Paper1: Advanced Financial Reporting [RTP 2017 December]
Solution to Q. No. 23

The direct non-controlling interest in Molehill Co is 40%


The direct non-controlling interest in Pimple Co is 20%
The indirect non-controlling interest in Pimple Co is (40% of 60%) 24%
The total non-controlling interest in Pimple Co is 44%
The group share of Molehill Co is 60% and of Pimple Co is (100 – 44)% = 56%

Workings
1 Goodwill
Goodwill
Hulk in Molehill Molehill in Pimple Hulk in Pimple
$ $ $ $ $ $
Consideration transferred 90,000 (60% × 42,000) 25,200 25,000
Fair value at NA acquired
Share capital 50,000 50,000 50,000
Retained earnings 8,000 15,000 15,000
58,000 65,000 65,000
Group share 60% 36% 20%
(34,800) (23,400) (13,000)
55,200 1,800 12,000
Goodwill Total = $ 69,000
Page 69 of 328
Paper1: Advanced Financial Reporting [RTP 2017 December]
2 Non-controlling interests

Molehill Pimple
$ $
Net assets per question 102,000 75,000
Investment in Pimple (42,000) –
60,000 75,000
NCI Proportion × 40% × 44%
$24,000 $33,000
Total NCI = $57,000
3 Retained earnings

Particulars Hulk Molehill Pimple


$ $ $
Per question 45,000 32,000 25,000
Pre-acquisition profits (8,000) (15,000)
Post-acquisition retained earnings 24,000 10,000
Group share:
In Molehill ($24,000 × 60%) 14,400
In Pimple ($10,000 × 56%) 5,600
Group retained earnings 65,000

4 Share premium account

$
Hulk Co 50,000
Molehill Co: all post-acquisition ($20,000 × 60%) 12,000
62,000

HULK CO
CONSOLIDATED STATEMENT OF FINANCIAL POSITION AS AT 31 MAY 20X5

$ $
Assets
Non-current assets
Tangible assets 210,000
Goodwill (W1) 69,000
279,000
Current assets 130,000
Total Assets 409,000
Equity and liabilities
Equity
Ordinary shares $1 100,000
Share premium (W4) 62,000
Retained earnings (W3) 65,000
Shareholders' funds 227,000
Non-controlling interests (W2) 57,000
284,000
Non-current liabilities
12% loan 10,000
294,000
Page 70 of 328
Paper1: Advanced Financial Reporting [RTP 2017 December]
Current liabilities
Payables 115,000
Total Liabilities 409,000
Page 71 of 328
Paper1: Advanced Financial Reporting [RTP 2018 June]
Questions
Cash and Cash Equivalents
1. Mr. Suresh is working as an accountant in a Manufacturing Company and is preparing a statement of
cash flows. As he is new to the field seeks advice about whether the following items can be included
within 'cash and cash equivalents'.
• An overdraft of NRs.400,000.
• A high interest deposit account has an amount of NRs. 700,000 in the name of the company. The
company must give 30 days' notice in order to access this money, which is held with the intention
of meeting working capital shortages.
• An investment in the ordinary shares of XYZ Co. having par value of NRs. 300,000 The shares
are listed and therefore could be sold immediately. The shares have a fair value of NRs. 10
00,000.

Required: Advise the accountant whether the above items qualify as 'cash and cash
equivalents'.

2. Calculate the required cash flows in each of the following scenarios:

a)
2017 2018
NRs. NRs.
Non-controlling interest 840 440

The group statement of profit or loss and other comprehensive income reported total
comprehensive income attributable to the non-controlling interest of NRs.500.

Required: How much was the cash dividend paid to the non-controlling interest?

b)
2017 2018
NRs. NRs.
Investment in Associates 500 200

The group statement of profit or loss reported ‗share of profit of associates' of NRs.750.
Required: How much was the cash dividend received by the group?

c)
2017 2018
NRs. NRs.
Investment in Associates 3,200 600

The group statement of profit or loss reported ‗share of profit of associates‘ of NRs.4,000. In
addition, the associate revalued its non-current assets during the period. The group share of this
gain is NRs.500.
Required: How much was the cash dividend received by the group?

d)
Page 72 of 328
Paper1: Advanced Financial Reporting [RTP 2018 June]
2017 2018
NRs. NRs.
Property, Plant & Equipment 500 150

During the year depreciation charged was NRs.50, and the group acquired a subsidiary which
held PPE of NRs.200 at the acquisition date.

Required: How much cash was spent on property, plant and equipment in the period?

Financial Instruments
3. When does debt seem to be equity?

Provisions, Contingent Liabilities, Contingent Assets


4. Skycity, a public limited company, has produced draft consolidated financial statements as at 31
Ashad. The group accountant has asked your advice on several matters. These issues are set out
below and have not been dealt with in the draft group financial statements:

a) Skycity has buildings under an operating lease. A requirement of the operating lease for the
corporate offices is that the asset is returned in good condition. The operating lease was signed in
the current year and lasts for six years. Skycity intends to refurbish the building in six years‘ time
at a cost of NRs6m in order to meet the requirements of the lease. This amount includes the cost
of renovating the exterior of the building and is based on current price levels. Currently, there is
evidence that due to severe and exceptional weather damage the company will have to spend
NRs1.2m in the next year on having the exterior of the building renovated. The company feels
that this expenditure will reduce the refurbishment cost at the end of the lease by an equivalent
amount. There is no provision for the above expenditure in the financial statements.

b) An 80% owned subsidiary company, Unicity, has a leasehold property (historical cost Cu.8m,
acquired at the year start). It has been modified to include a sports facility for the employees.
Under the terms of the lease, the warehouse must be restored to its original state when the lease
expires in 10 years‘ time or earlier termination. The present value of the costs of reinstatement are
likely to be NRs2m measured at the year start and the directors wish to provide for NRs200,000
per annum for 10 years. The lease was signed and operated from the current year start and the
modifications occurred immediately after. The directors estimate that the lease has a recoverable
value of NRs9.5m at 31 Ashad year-end and have not provided for any of the above amounts.

c) Additionally, Skycity owns buildings at a carrying value of NRs20m, which will require repair
expenditure of approximately NRs6m over the next five years. No provision has been made for
this amount in the financial statements and depreciation is charged on leasehold buildings at 10%
per annum and on owned buildings at 5% per annum, on the straight-line basis.

d) Unicity has developed a database during the year to 31 Ashad and it is included in intangible non-
current assets at a cost of NRs3m. The asset comprises the internal and external costs of
developing the database. The cost of such intangible assets is amortised over five years and one
year‘s amortisation has been charged. The database is used to produce a technical accounting
manual, which is used by the whole group and sold to other parties. Net revenue of NRs2m is
expected from sales of the manual over its four-year life. It has quickly become a market leader in
this field. Any costs of maintaining the database and the technical manual are written off as
incurred. The technical manual requires substantial revision every four years. Therefore, Unicity
is considering providing for the cost of revision.
Page 73 of 328
Paper1: Advanced Financial Reporting [RTP 2018 June]

e) Skycity purchased a wholly owned subsidiary company, Velocity, on 1 Shrawan, at the prior year
start. The vendors commenced a legal action on 31 Kartik during the current year over the
amount of the purchase consideration, which was based on the performance of the subsidiary. An
amount had been paid to the vendors and included in the calculation of goodwill but the vendors
disputed the amount of this payment. The court made a decision on 31 Ashad at the current year-
end that requires Skycity to pay an additional Cu.8m to the vendors within three months. The
directors do not know how to treat the additional purchase consideration and have not accounted
for the item.

Required
(a) Discuss the recognition criteria for the recognition of a provision (NAS 37).
(b) Discuss how the above five issues should be dealt with in the group financial statements of
Skycity.

5. One of the matters addressed in NFRS 9 – Financial Instruments is the initial and subsequent
measurement of financial assets. NFRS 9 requires that financial assets are initially measured at their
fair value at the date of initial recognition. However, subsequent measurement of financial assets
depends on their classification for which NFRS 9 identifies three possible alternatives.

Required:
Explain the three classifications which NFRS 9 identifies for financial assets and the basis of
measurement which is appropriate for each classification. You should also identify any exceptions to
the normal classifications which may apply in specific circumstances.

Group Financial Statements


6. City Ltd. has held a 70% investment in Village Ltd. for two years. Goodwill has been calculated
using the full goodwill method. There have been no goodwill impairments to date.

City disposes of all of its shares in Village. The following information has been provided:
$
Cost of investment 2,000
Village – Fair value of net assets at acquisition 1,900
Village – Fair value of the non-controlling interest at acquisition 800
Sales proceeds 3,000
Village – Net assets at disposal 2,400

Required: Calculate the profit or loss on disposal in:

(a) City's individual financial statements


(b) City‘s consolidated financial statements.

7. Statements of financial position for three entities at the reporting date are as follows:

YAM BEA YEA


NRs000 NRs000 NRs000
Assets 1,000 800 500
Investment in BEA 326 – –
Investment in YEA 165 – –
Page 74 of 328
Paper1: Advanced Financial Reporting [RTP 2018 June]
––––– ––––– –––––
Total assets 1,491 800 500
––––– ––––– –––––
Equity Ordinary share capital (NRs1) 500 200 100
Retained earnings 391 100 200
––––– ––––– –––––
891 300 300
Liabilities 600 500 200
––––– ––––– –––––
Total equity and liabilities 1,491 800 500
––––– ––––– –––––

YAM acquired 80% of BEA when BEA's retained earnings were NRs25,000, paying cash
consideration of NRs300,000. It is group policy to measure NCI at fair value at the date of
acquisition. The fair value of the NCI holding in BEA at acquisition was NRs65,000. At the reporting
date, YAM purchased an additional 8% of BEA's equity shares for cash consideration of NRs26,000.
This amount has been debited to YAM's investment in BEA. YAM acquired 75% of YEA when
YEA's retained earnings were NRs60,000, paying cash consideration of NRs200,000. The fair value
of the NCI holding in YEA at the date of acquisition was NRs50,000. At the reporting date, YAM
sold 10% of the equity shares of YEA for NRs35,000. The cash proceeds have been credited to
YAM's investment in YEA.

Required:

Prepare the consolidated statement of financial position of the YAM group.

8. The following draft statements of financial position relate to Riddy, Siddy, and Ganesh, all public
limited companies, as at 31 December 2017:

Riddy Siddy Ganesh


NRs m NRs m Dinars m
Assets
Non-current assets:
Property, plant and equipment 250 120 360
Investment in Siddy 98 – –
Investment in Ganesh 30 – –
Financial assets 10 5 148

Current assets 22 17 120


–––– –––– ––––
Total assets 410 142 62.8
–––– –––– ––––

Equity and liabilities:


Equity shares 60 40 209
Other components of equity 30 10 –
Retained earnings 120 80 299
–––– –––– ––––
Total equity 210 130 508
Non-current liabilities 90 5 48
Page 75 of 328
Paper1: Advanced Financial Reporting [RTP 2018 June]
Current liabilities 110 7 72
–––– –––– ––––
Total equity and liabilities 410 142 628
–––– –––– ––––

The following information needs to be taken account of in the preparation of the group financial
statements of Riddy:

(a) Riddy acquired 70% of the equity shares of Siddy on 1 January 2016 when Siddy‘s other reserves
were NRs10 million and retained earnings were NRs60 million. The fair value of the net assets
of Siddy was NRs120 million at the date of acquisition. Riddy acquired 60% of the equity shares
of Ganesh for 330 million dinars on 1 January 2016 when Ganesh‘s retained earnings were 220
million dinars. The fair value of the net assets of Ganesh on 1 January 2016 was 495 million
dinars. The excess of the fair value over the net assets of Siddy and Ganesh is due to an increase
in the value of non-depreciable land. There have been no issues of ordinary shares since
acquisition and goodwill on acquisition is not impaired for either Siddy or Ganesh. Goodwill is to
be calculated on the proportion of net assets basis i.e. without allocating any goodwill to the non-
controlling interest.

(b) Ganesh is located in a foreign country and imports its raw materials at a price which is normally
denominated in dollars. The product is sold locally at selling prices denominated in dinars, and
determined by local competition. All selling and operating expenses are incurred locally and paid
in dinars. Distribution of profits is determined by the parent company, Riddy. Ganesh has
financed part of its operations through a NRs4 million loan from Siddy which was raised on 1
January 2017. This is included in the financial assets of Siddy and the non-current liabilities of
Ganesh. Ganesh‘s management have a considerable degree of authority and autonomy in carrying
out the operations of Ganesh and other than the loan from Siddy, are not dependent upon group
companies for finance.

(c) Riddy has a building which it purchased on 1 January 2017 for 40 million dinars and which is
located overseas. The building is carried at cost and has been depreciated on the straight-line
basis over its useful life of 20 years. At 31 December 2017, as a result of an impairment review,
the recoverable amount of the building was estimated to be 3.6 million dinars.

(d) Riddy has a long-term loan of NRs10 million which is owed to a third party bank. At 31
December 2017, Riddy decided that it would repay the loan early on 1 January 2018 and formally
agreed this repayment with the bank prior to the year end. The agreement sets out that there will
be an early repayment penalty of NRs1 million.

(e) The directors of Riddy announced on 1 January 2017 that a bonus of NRs6 million would be paid
to the employees of Riddy if they achieved a certain target production level by 31 December
2017. The bonus is to be paid partly in cash and partly in share options. Half of the bonus will be
paid in cash on 30 June 2018 whether or not the employees are still working for Riddy. The other
half will be given in share options on the same date, provided that the employee is still in service
on 30 June 2018. The exercise price and number of options will be fixed by management on 30
June 2018. The target production was met and management expect 10% of employees to leave
between 31 December 2017 and 30 June 2018. No entry has been made in the financial
statements of Riddy.
Page 76 of 328
Paper1: Advanced Financial Reporting [RTP 2018 June]
(f) Riddy operates a defined benefit pension plan. On 1 January 2017, Riddy improved the pension
entitlement. This improvement applied to all prior years‘ service of the employees. As a result,
the present value of the defined benefit obligation on 1 January 2017 increased by NRs4 million.
Riddy had not accounted for the improvement in the pension plan.

(g) Riddy is considering selling its subsidiary, Siddy. Just prior to the year end, Siddy sold inventory
to Riddy at a price of NRs6 million. The carrying value of the inventory in the financial records
of Siddy was NRs2 million. The cash was received before the year end, and as a result the bank
overdraft of Siddy was virtually eliminated at 31 December 2017. After the year end the
transaction was reversed and it was agreed that this type of transaction would be carried out again
when the interim financial statements were produced for Siddy, if the company had not been sold
by that date.

(h) All financial assets, with the exception of the loan by Siddy to Ganesh, referred to in note (ii)
above are correctly stated at fair value at 31 December 2017.

(i) The following exchange rates are relevant to the preparation of the group financial statements:

Dinars to NRs
1 January 2016 11
1 January 2017 10
31 December 2017 12
Average for year to 31 December 2017 10.5

Required:
(a) Prepare a consolidated statement of financial position of the Riddy Group at 31
December 2017 in accordance with Nepal Financial Reporting Standards.

Intangible Assets
9. Mr. Rajaram, an accountant in a film making company, Picture Production P. Ltd, was confused
about the recording of transactions relating to a new film ―Sagarmatha‖. You are requested to advise
him considering the nature of following transactions:

a) The company has started to shoot a film from 01.05.2074 and expected to release on 01.01.2075.
b) As per the accounting records, following costs/payments are incurred/made by the end of Poush
End 2074 and film is ready for release. However, the company wanted to release the film only on
the occasion of New Year.
NRs. In Thousand

Direct Labour (Actors, Film Crew, Security) 4,000


Production Costs (Editing, Visual Effects) 2,000
Production Overheads (Studio Rents, Customs, Catering) 2,500
Administrative Cost (Insurance, Travels) 1,000
Interest Cost 500
Loan Repayment 1,000

Note:
Page 77 of 328
Paper1: Advanced Financial Reporting [RTP 2018 June]
Loan outstanding as on Poush end 2074 is NRs. 5,000 thousand. Expected additional interest cost
and expected additional loan repayment was of NRs. 800 thousand and NRs. 1500 Million
respectively till release date.

c) The company has decided to take control of the film ―Sagarmatha‖ and decided to distribute
under the company‘s banner only.

Required:
Suggest the accountant how the costs can be recorded in the financial statements as at 01.01.2075 (i.e.
film release date) and why? Show the extracts of financial statements relating to film cost.

Biological Assets
10. As at 31 December 20X1, a plantation consists of 100 Pinus Radiata trees that were planted 10 years
earlier. Pinus Radiata takes 30 years to mature, and will ultimately be processed into building material
for houses or furniture. The enterprise‘s weighted average cost of capital is 6% p.a.

Only mature trees have established fair values by reference to a quoted price in an active market. The
fair value (inclusive of current transport costs to get 100 logs to market) for a mature tree of the same
grade as in the plantation is:

 As at 31 December 20X1: 171


 As at 31 December 20X2: 165

Required:
a. Calculate the fair value of Biological Assets as at 31 December 20X1 and 31 December 20X2.
b. Calculate the net gains or losses to be charged during the year 20X2 and reconcile the net gains or
losses so arrived with the gains or losses arising due to:

o The effects of change in market price, and


o The physical change (growth) of the trees in the plantation.

Reorganization/ Restructuring
11. X, a public limited company, owns 100% of companies Y and Z which are both public limited
companies. The X group operates in the telecommunications industry and the directors are
considering two different plans to restructure the group. The directors feel that the current group
structure is not serving the best interests of the shareholders and wish to explore possible alternative
group structures.

The statements of financial position of X and its subsidiaries Y and Z at 31 May 20X7 are as follows:

X Y Z
NRsm NRsm NRsm
Property, plant and equipment 600 200 45
Cost of investment in Y 60
Cost of investment in Z 70
Net current assets 160 100 20
890 300 65

Share capital – ordinary shares of NRs1 120 60 40


Retained earnings 770 240 25
Page 78 of 328
Paper1: Advanced Financial Reporting [RTP 2018 June]
890 300 65

X acquired the investment in Z on 1 June 20X1 when the company's retained earnings balance was
NRs20 million. The fair value of the net assets of Z on 1 June 20X1 was NRs60 million. Company Y
was incorporated by X and has always been a 100% owned subsidiary. The fair value of the net assets
of Y at 31 May 20X7 is NRs310 million and of Z is NRs80 million. The fair values of the net current
assets of both Y and Z are approximately the same as their book values.

The directors are unsure as to the impact or implications that the following plans are likely to have on
the individual accounts of the companies and the group accounts.

Local companies legislation requires that the amount at which share capital is recorded is dictated by
the nominal value of the shares issued and if the value of the consideration received exceeds that
amount, the excess is recorded in the share premium account. Shares cannot be issued at a discount.
In the case of a share for share exchange, the value of the consideration can be deemed to be the book
value of the investment exchanged.

The two different plans to restructure the group are as follows:


Plan 1
Y is to purchase the whole of X's investment in Z. The purchase consideration would be 50 million
NRs1 ordinary shares of Y.

Plan 2
The same scenario as Plan 1, but the purchase consideration would be a cash amount of NRs75
million.

Required
Discuss the key considerations and the accounting implications of the above plans for the X group.
Your answer should show the potential impact on the individual accounts of X, Y and Z and the
group accounts after each plan has been implemented.

Employee Benefits
12. At 1 January 20X7 the fair value of the assets of a defined benefit plan were valued at NRs 1,100,000
and the present value of the defined benefit obligation was NRs 1,250,000. On 31 December 20X7,
the plan received contributions from the employer of CU 490,000 and paid out benefits of NRs
190,000.

The current service cost for the year was NRs 360,000 and a discount rate of 6% is to be applied to
the net liability/(asset).
After these transactions, the fair value of the plan's assets at 31 December 20X7 was NRs1.5m. The
present value of the defined benefit obligation was NRs 1,553,600.

Required
Calculate the gains or losses on re-measurement through OCI and the return on plan assets and
illustrate how this pension plan will be treated in the statement of profit or loss and other
comprehensive income and statement of financial position for the year ended 31 December 20X7.
Page 79 of 328
Paper1: Advanced Financial Reporting [RTP 2018 June]
Foreign Exchanges
13. The following transactions relate to Britney for the year ended 31 December 20X7.

(a) Britney purchased 6,000 kg of materials on December 20X7 to use in their production process.
The supplier is located in Erehwon where the currency is the Won. The goods cost 300,000 Wons
and have not been paid for at the year end. The relevant exchange rates are:

1 December USNRs1 = 20 Wons


31 December USNRs1 = 16 Wons

Show how this transaction will be included in the financial statements at 31 December 20X7.

(b) Britney's finance manager does not understand the difference between functional and presentation
currencies. Britney's local currency is the USNRs. They are a wholly owned autonomous
subsidiary of a large corporation based in Europe who reports group results in Euros.

Define functional and presentation currencies in relation to Britney and in its parent.

Value Added Statements


14. The following information is available of a concern; calculate E.V.A.:

Debt capital 12% NRs. 2,000 crores

Equity capital NRs. 500 crores

Reserve and surplus NRs. 7,500 crores


Capital employed NRs. 10,000 crores

Risk-free rate 9%

Beta factor 1.05

Market rate of return 19%

Equity (market) risk premium 10%

Operating profit after tax NRs.2,100 crores

Tax rate 30%

15. Write Short Notes on:


a. What differentiates Profit or Loss from Other Comprehensive Income
b. Methods of valuation of intangible assets

Share Based Payments


Page 80 of 328
Paper1: Advanced Financial Reporting [RTP 2018 June]
16. Entity A prepares financial statements to 31 December each year. On 1 January 2012 A granted 300
options to 400 employees. The options vest on 31 December 2014 provided the relevant employees
remain in employment with A throughout the three-year period.

On 1 January 2012 the fair value of each share option was NRs2.40. The fair value increased to
NRs2.50 by 31 December 2012, to NRs2.70 on 31 December 2013, and was NRs2.75 on 31
December 2014.

On 1 January 2012 the directors of entity A estimated that 340 employees would remain in
employment throughout the three-year period ending on 31 December 2014. This estimate was re-
computed to 350 employees on 31 December 2012 and 360 employees on 31 December 2013. The
actual number of employees who remained over the three-year period was 365 employees.

Required:
Show the impact on the financial statements of entity A for each of the three years:

17. Entity B prepares financial statements to 30 November each year. On 1 December 2012 entity B
granted share options to a group of 200 employees. The options will vest on 30 November 2015
provided the employees remain in employment over the three-year period ending 30 November 2015.
The number of options that will vest for each eligible employee will depend on the cumulative growth
in revenue for the three-year period ending 30 November 2015:

 250 options if the cumulative growth in revenue is more than 20% but not more than 25%.
 300 options if the cumulative growth in revenue is more than 25% but not more than 30%.
 350 options if the cumulative growth in revenue is more than 30%.

The directors of entity B made the following estimates regarding the vesting conditions at relevant
dates:

Cumulative
Number of growth in
Date eligible revenue in the
employees three-year
period

1 December 2012 180 22%

30 November 2013 183 24%

30 November 2014 185 28%

The fair value of a share option on 1 December 2012 was NRs3.60.

Required:
Show the impact of the share-based payment arrangement on the financial statements for the years
ended 30 November 2013 and 2014:
Page 81 of 328
Paper1: Advanced Financial Reporting [RTP 2018 June]
18. On 1 December 2012, MoonSun acquired a trademark, Golfo, for a line of golf clothing for NRs.3
million. Initially,MoonSun expected to continue marketing and receiving cash flows from the Golfo
product-line indefinitely. However, because of the difficulty in determining its useful life, MoonSun
decided to amortise the trademark over a 10-year life, using the straight-line method. In December
2015, a competitor unexpectedly revealed a technological breakthrough which is expected to result in
a product which, when launched, will significantly reduce the demand for the Golfo product-line. The
demand for the Golfo product-line is expected to remain high
until May 2018, when the competitor is expected to launch its new product. At 30 November 2016,
the end of the financial year, MoonSun assessed the recoverable amount of the trademark at
NRs.500,000 and intends to continue manufacturing Golfo products until 31 May 2018. The directors
of MoonSun require advice as to how to deal with the trademark in the financial statements for the
year ended 30 November 2016.

Lease
19. Revolta is an entity which prepares financial statements to 30 September each year. The financial
statements for the year ended 30 September 2017 are shortly to be authorised for issue. The following
event is relevant to these financial statements:

On 1 October 2015, Revolta purchased an asset for NRs.20 million. The estimated useful life of the
asset was 10 years, with an estimated residual value of zero. Revolta immediately leased the asset to
ADATA. The lease term was 10 years and the annual rental, payable in advance by ADATA, was
NRs.2,787,000. Revolta incurred direct costs of NRs.200,000 in arranging the lease. The lease
contained no early termination clauses and responsibility for repairs and maintenance of the asset rest
with ADATA for the duration of the lease. The directors of Revolta correctly computed the annual
rate of interest implicit in the lease as 8%. At an annual discount rate of 8% the present value of
NRs.1 receivable at the start of years 1–10 is NRs.7·247.

20. Write Short Notes on:

a. Current and non-current assets


b. NAS 16 Property, Plant and Equipment and NAS 38 Intangible Assets.

21. What are insurance contracts? List down the examples that are insurance contracts and that are not
insurance contracts.
Page 82 of 328
Paper1: Advanced Financial Reporting [RTP 2018 June]
Answers

Cash and Cash Equivalents


1. To qualify as a cash equivalent, an item must be readily convertible to cash and have an insignificant
risk of a change in value. Furthermore, it should be held for the purpose of meeting short-term cash
commitments.

 Overdrafts
Bank overdrafts are an integral part of most company's cash management. They are therefore
generally treated as a component of cash.

 Balance in Deposit Account


The balance of NRs. 700,000 in a high interest deposit account is readily available (only 30 days'
notice is required to access it). This money is also held to meet short-term needs. Assuming that
there is not a significant penalty for accessing this money, it should be included within cash
equivalents.

 Investments in Shares
The shares are not a cash equivalent. Shares are investments rather than a way of meeting short-
term cash requirements. Moreover, there is a significant risk that the value of the shares will
change. Any cash spent on shares in the period should be shown within cash flows from investing
activities.

2.

a) Non Controlling Interest NRs.

Balance B/forward 440


Total Comprehensive Income 500
Dividend paid (B.F) (100)
Balance C/forward 840

b) Investment in Associates NRs.

Balance B/forward 200


Profit or Loss 750
Dividend Received (B.F) (450)
Balance C/forward 500

c) Investment in Associates NRs.

Balance B/forward 600


Profit or Loss 4,000
Revaluation 500
Dividend Received (B.F) (1,900)
Balance C/forward 3,200

d) Property, Plant & Equipment NRs.

Balance B/forward 150


Page 83 of 328
Paper1: Advanced Financial Reporting [RTP 2018 June]
New Subsidiary 200
Depreciation (50)
Additions (B.F) 200
Balance C/forward 500

Financial Instruments
3. Many financial instruments have both features of debt and equity that this can lead to inconsistency of
reporting. It is not easy always to distinguish the debt and equity in an entity‘s statement of financial
position.

The key feature of debt is that the issuer is obliged to deliver either cash or another financial asset to
the holder. In contrast, equity is any contract that evidences a residual interest in the entity‘s assets
after deducting all of its liabilities. Thus, A financial instrument is an equity instrument only if the
instrument includes no contractual obligation to deliver cash or another financial asset to another
entity, and if the instrument will or may be settled in the issuer's own equity instruments.

For example,
 a bond that requires the issuer to make interest payments and redeem the bond for cash is
classified as debt.
 ordinary shares, where all the payments are at the discretion of the issuer, are classified as equity
of the issuer.
 preference shares required to be redeemed on a fixed date, or on the occurrence of an event that is
certain to occur, should be classified as debt.
 preference shares required to be converted into a fixed number of ordinary shares on a fixed date,
or on the occurrence of an event that is certain to occur, should be classified as equity.

A contract is not an equity instrument solely because it may result in the receipt or delivery of the
entity‘s own equity instruments. The classification of this type of contract is dependent on whether
there is variability in either the number of equity shares delivered or variability in the amount of cash
or financial assets received. A contract that will be settled by the entity receiving or delivering a fixed
number of its own equity instruments in exchange for a fixed amount of cash, or another financial
asset, is an equity instrument.

However, if there is any variability in the amount of cash or own equity instruments that will be
delivered or received, then such a contract is a financial asset or liability as applicable.

Other factors that may result in an instrument being classified as debt are:
 is redemption at the option of the instrument holder?
 is there a limited life to the instrument?
 is redemption triggered by a future uncertain event that is beyond the control of both the holder
and issuer of the instrument?
 are dividends non-discretionary?

Similarly, other factors that may result in the instrument being classified as equity are whether the
shares are non-redeemable, whether there is no liquidation date or where the dividends are
discretionary.

Some instruments are structured to contain elements of both a liability and equity in a single
instrument. Such instruments – for example, bonds that are convertible into a fixed number of equity
Page 84 of 328
Paper1: Advanced Financial Reporting [RTP 2018 June]
shares and carry interest – are accounted for as separate liability and equity components. 'Split
accounting' is used to measure the liability and the equity components upon initial recognition of the
instrument. This method allocates the fair value of the consideration for the compound instrument
into its liability and equity components.

Provisions, Contingent Liabilities, Contingent Assets


4.
(a) Provision

There are following recognition criteria for recognition of provision.

Reasonably reliable estimate -It must be possible to make a reasonably reliable estimate of the
outflow that will result from the obligation before a provision is permitted.

Obligation -There must be a present legal or constructive obligation at the year-end before a
provision is permitted.

Transfer -There must be an expectation that economic benefit will flow out in the future as a
result of the obligation.

(b) Five Issues

a) Operating lease
Actually, it is irrelevant whether the above is operating or finance lease in the context of
analysing related provisions. Either type of lease creates an obligation.

Present obligation
Here, Skycity does have a present obligation for the damage done during the tenure (NRs1.2m)
but not for the damage that might be done in the future (maybe NRs.4.8m).

Conclusion
So Skycity should provideNRs1.2m and should recognize the charge to the SoPL.

b) Unicity
First we must eye this problem form the perspective of the subsidiary. It is the subsidiary that will
be putting through the double entry. Then we can look at the effect on the group.

The key term in this paragraph is ‗modify‘. We can see that Unicity already has made the
modifications and therefore has a present obligation as a result of this past obliging event. So a
provision is required for the cost of restoration. The provision is required at the point of
modification. The modification occurred at the year start.

However, the restoration will not take place until the end of the lease; so the time value of money
must be considered. But we need to be careful here, as the NRs2m is already discounted.

Double entry
So the year start double entry is:

Dr Non-Current Asset NRs2m


Cr Provision NRs2m
Page 85 of 328
Paper1: Advanced Financial Reporting [RTP 2018 June]

Non-current asset
In fact, the above non-current asset entry goes on top of the Historical Cost:
Historical Cost NRs8m
Restoration NRs2m
_____
Initial cost NRs10m

Depreciation
Then, of course, the above is depreciated over its life, which is 10 years.
Depreciation double entry

This is the same every year:


Dr SoPL NRs1m
Cr NCA NRs1m

Unwinding
Assume the discounting rate is 10% as it is not given.

Unwinding double entry


We need to unwind every year, but, the first year journal would be as follows:
Dr SoPL NRs0.2m
Cr Prov (NRs2m)(10%) NRs0.2m

Impairment test
There is no impairment as the carrying value of the asset at the year-end (NRs9m) is less than the
recoverable value (NRs9.5m).

Group effect
The above double entry will be accommodated by the sub. However, because this is a partially
owned sub, the group/non-controlling interest effect will be 80%/20%.

c) Repairs and depreciation

There appears to be no obligation for the repairs. Just an intent to repair sometime in the future.

So there can be no provision for the repairs.


Buildings should be depreciated over their useful lives regardless of being owned or otherwise.

d) Intangible
An intangible is recognised if it is purchased. Also, development is recognised if it is recoverable.

So it appears to be reasonable to capitalize and depreciate the asset. However, it is advisable that
Skycity to adjust the life down to four years, as that appears to be more realistic. Also, there is no
obligation to revise. So no provision is possible.

e) Goodwill
Clearly, if Skycity had predicted the extra NRs8m, it would have put it in the consideration and
the acquisition goodwill would have been higher. But the only way to adjust last year‘s goodwill
is via a prior period adjustment (PPA) restatement. This is only permissible if the NRs8m is a
material error. But it is a change in an estimate. So the NRs8m will simply have to be charged to
Page 86 of 328
Paper1: Advanced Financial Reporting [RTP 2018 June]
the SoPL. NFRS 3 supports this view, by giving a 12 months ‘ limit on dealing with goodwill
after acquisition.

5. The classification and measurement of financial assets is largely based on:

The business model for managing the asset – specifically whether or not the objective is to hold the
financial asset in order to collect the contractual cash flows.

Whether or not the contractual cash flows are solely payments of principal and interest on the
principal amount outstanding.

Where the business model for managing the asset is to hold the financial asset in order to collect the
contractual cash flows and the contractual cash flows are solely payments of principal and interest on
the principal amount outstanding, then the financial asset is normally measured at amortized cost.

Where the business model for managing the asset is to both hold the financial asset in order to collect
the contractual cash flows and to sell the financial asset and the contractual cash flows are solely
payments of principal and interest on the principal amount outstanding, then the financial asset is
normally measured at fair value through other comprehensive income. Interest income on such assets
is recognised in the same way as if the asset were measured at amortized cost.

In other circumstances, financial assets are normally measured at fair value through profit or loss.

Notwithstanding the above, where equity investments are not held for trading, an entity may make an
irrevocable election to measure such investments at fair value through other comprehensive income.

Finally an entity may, at initial recognition, irrevocably designate a financial asset as measured at fair
value through profit or loss if to do so eliminates or significantly reduces an accounting mismatch.

Group Financial Statements


6.

(a) City's individual financial statements


NRs
Sales proceeds 3,000
Cost of shares sold (2,000)
––––––
Profit on disposal 1,000
––––––

(b) Consolidated financial statements


NRs NRs
Sales proceeds 3,000
Interest in subsidiary disposed of:
Net assets at disposal 2,400
Goodwill at disposal (W1) 900
Carrying amount of NCI at disposal (W2) (950)
––––– (2,350)
–––––
Profit on disposal 650
–––––
Page 87 of 328
Paper1: Advanced Financial Reporting [RTP 2018 June]

(W1) Goodwill
NRs
Consideration 2,000
FV of NCI at acquisition 800
––––––
2,800
FV of net assets at acquisition (1,900)
––––––
Goodwill 900
––––––

(W2) NCI at disposal date


NRs
NCI at acquisition 800
NCI % of post acquisition net assets
(30% × (NRs2,400 – NRs1,900)) 150
–––––
950
–––––
7.
Consolidated statement of financial position
NRs000
Assets (NRs1,000 + NRs800 + NRs500) 2,300
Goodwill (NRs140 + NRs90) (W3) 230
–––––
Total assets 2,530
–––––
Equity
Ordinary share capital (NRs1) 500
Retained earnings (W5) 556
Other components of equity (NRs6 – NRs4) (W6, W7) 2
–––––
1,058
Non-controlling interests (NRs48 + NRs124) (W4) 172
–––––
1,230
Liabilities (NRs600 + NRs500 + NRs200) 1,300
–––––
Total equity and liabilities 2,530
–––––

Workings

(W1) Group structure

YAM

BEA YEA
Page 88 of 328
Paper1: Advanced Financial Reporting [RTP 2018 June]

Initial holding 80% Initial holding 75%


Acquisition 8% Disposal (10%)
Reporting Date 88% Reporting Date 65%

(W2) Net Assets of subsidiaries

BEA Acquisition date Reporting date


NRs000 NRs000
Share capital 200 200
Retained earnings 25 100
–––– ––––
225 300
–––– ––––

YEA Acquisition date Reporting date


NRs000 NRs000
Share capital 100 100
Retained earnings 60 200
–––– ––––
160 300
–––– ––––

(W3) Goodwill

BEA NRs000
Consideration 300
FV of NCI at acquisition 65
Fair value of net assets at acquisition (W2) (225)
––––
140
––––

YEA NRs000
Consideration 200
FV of NCI at acquisition 50
Fair value of net assets at acquisition (W2) (160)
––––
90
––––

(W4) Non-controlling interest

BEA NRs000
NCI at acquisition (W3) 65
NCI% × post acquisition net assets (20% × NRs75 (W2)) 15
Page 89 of 328
Paper1: Advanced Financial Reporting [RTP 2018 June]
––––
NCI before control to control adjustment 80 Decrease in NCI (W6) (32)
––––
48
––––

YEA NRs000
NCI at acquisition (W3) 50
NCI% × post acquisition net assets (25% × NRs140 (W2)) 35
––––
NCI before control to control adjustment 85
Increase in NCI (W7) 39
––––
124
––––

(W5) Retained earnings

NRs000
YAM's retained earnings 391
YAM's % of BEA's post acquisition retained earnings (80% × NRs75 (W2)) 60
YAM's % of YEA's post acquisition retained earnings (75% × NRs140 (W2)) 105
––––
556
––––

(W6) Control to control adjustment – BEA

NRs000
Cash paid 26 Cr
Decrease in NCI (8/20 × NRs80 (W4)) (32) Dr
––––
Increase to other components of equity (6) Cr
––––

(W7) Control to control adjustment – YEA

NRs000
Cash received 35 Dr
Increase in NCI (10% × (NRs300 (W2) + NRs90 (W3)) (39) Cr
––––
Decrease to other components of equity (4) Dr
––––
Page 90 of 328
Paper1: Advanced Financial Reporting [RTP 2018 June]
8.
Riddy Group

Consolidated Statement of Financial Position


As at 31 December 2017

NRs m
Assets
Non-current assets
Goodwill (W3) 16.8
Property, plant and equipment (W9) 414.7
Financial assets at fair value (W14) 23.3
––––– 454.8
Current assets (W15) 51.0
–––––
Total assets 505.8
–––––

NRs m
Equity and liabilities
Equity shares 60.0
Other components of equity (W6) 31.8
Retained earnings (W5) 121.7
–––––
Total shareholders‘ equity 213.5
Non-controlling interests (W4) 59.6
–––––
Total equity 273.1
Non-current liabilities (W16) 89.7
Current liabilities (W17) 143.0
–––––
505.8
–––––

Workings: (W1) Group structure

Riddy 70% Siddy

Riddy 60% Ganesh

(W2a) Net assets – Siddy

Acq’n date Rep date


NRs m NRs m
Share capital 40 40
Retained earnings 60 80
Other Equity 10 10
FVA – Land 10 10
PURP (W13) (4)
–––– ––––
Page 91 of 328
Paper1: Advanced Financial Reporting [RTP 2018 June]
120 136
–––– ––––

(W2b) Net assets– Ganesh

Acq’n date Rep date


Dinar(m) Dinar(m)
Equity capital 209 209
Pre-acq‘n retained earnings 220 220
FVA – Land 66 66
Post acq‘n RE (299 – 220) 79
Loan – FX loss (W8) (8)
–––– ––––
495 566
–––– ––––

(W3a) Goodwill – Siddy

NRs m
Cost of investment 98
NCI at acquisition (30% × NRs120m) (W2a) 36
––––
134
Less: FV of net assets at acquisition (W2a) (120)
––––
Goodwill 14
––––

(W3b) Goodwill and retranslation schedule – Ganesh

Dinar (m)
Cost of investment 330
NCI at acquisition (40% × 495m dinar (W2b)) 198
––––
528
FV of net assets at acquisition (W2b) (495)
––––
Goodwill at acquisition – no impairment 33
Translated at closing rate of 12 = NRs2.8m

The exchange gain or loss on retranslation of goodwill at the reporting date must also be identified as
follows:
Dinar(m) Rate NRs m
Goodwill at acquisition 33 11 3.0
FX gain (loss) on retranslation (W5) Bal fig (0.2)
–––– ––––
Goodwill at reporting date 33 12 2.8
Page 92 of 328
Paper1: Advanced Financial Reporting [RTP 2018 June]
–––– ––––

As goodwill was calculated using the proportionate basis, all of the exchange gain or loss on
retranslation is allocated to the parent in (W5).

(W4) Non-controlling Interest

NRs m
Siddy: NCI at acquisition (30% × NRs 120m) (W2a) 36.0
NCI % of post-acquisition net assets (30% ×(NRs136m – NRs120m)) (W2a)) 4.8
Ganesh: NCI at acquisition (D198m/11) (W3) 18.0
NCI % of post-acquisition net assets (40% × NRs2.2m) (W7) 0.8
––––
59.6
––––

(W5) Group reserves

NRs m
Riddy 120.0
Siddy: (70% × (136 – 120) (W2a) 11.2
Ganesh (60% × 2.2) (W7) 1.3
Riddy – building impaired (W9) (0.8)
Riddy – penalty re early loan repayment (W10) (1.0)
Riddy – cash bonus (W11) (3.0)
Riddy – share options (W11) (1.8)
Riddy – past service cost – pension costs (W12) (4.0)
Exchange loss on retranslation of goodwill (W3b) (0.2)
–––––
121.7
–––––

(W6) Other components of equity

NRs m
Riddy 30.0
Riddy – equity re share options (W10) 1.8
––––
31.8
––––

(W7) Net asset translation

Dinar(m) Rate NRs m


Net assets at acquisition 495 11 45.0
Profit since acquisition 71 Bal fig 2.2
–––– ––––
Net assets at reporting date 566 12 47.2
Page 93 of 328
Paper1: Advanced Financial Reporting [RTP 2018 June]
–––– ––––

Profit earned during the year is translated at the average rate for that year. However, the D71m profit
has been earned over two years. Without knowing how much profit is earned in each year, and the
average rate for each year, it is not possible to split out the post-acquisition profit from the post-
acquisition foreign exchange differences arising on the translation of the profit.

Therefore, the NRs2.2m balance above represents both post-acquisition profits and the foreign
exchange on net assets and profit. The group‘s share of this is taken to group reserves.

(W8) Ganesh – Exchange loss on loan from Siddy

Loans between subsidiaries cannot be treated as part of the holding company‘s net investment in a
foreign subsidiary (NAS 21). Ganesh will recognise an exchange difference on the loan from Siddy in
profit or loss and the exchange difference will flow through to the consolidated statement of profit or
loss and will not be reclassified as a separate component of equity.

Dinar (m)
Loan at 1 January 2017 NRs4m at 10 dinars 40
Loan at 31 December 2017 NRs4m at 12 dinars 48
–––
Exchange loss 8
–––

The loan of NRs4 million should be eliminated on consolidation from both financial assets (W14) and
non-current liabilities (W16).

(W9) Tangible assets (including building impaired)

NRs m NRs m
Riddy 250.0
Siddy (120 + 10(FVA) (W2a)) 130.0
Ganesh ((360 + 66(FVA) (W2b))/12) 35.5
––––– 415.5
Building – impairment loss
1 January 2017 cost 40m dinar @ 10 4.0
Depreciation (20 years) (0.2)
–––––
3.8
31 December 2017 36m dinar @12 3.0 (0.8)
––––– –––––
414.7
–––––

(W10) Early repayment of loan

As Riddy entered into an agreement to repay the debt early plus a penalty, it should adjust the
carrying value of the financial liability to reflect actual and revised estimated cash flows (NFRS 9).
Therefore, the carrying amount of the loan liability should be increased by NRs1 million and be
transferred to current liabilities.
Page 94 of 328
Paper1: Advanced Financial Reporting [RTP 2018 June]
(W11) Cash bonus and share options to employees of Riddy

A liability of NRs3 million should be accrued for the bonus to be paid in cash to the employees of
Riddy. The management should also recognise an expense of (12/18 × 90% × NRs3 million) NRs1.8
million, with a corresponding increase in equity. The terms of the share options have not been fixed
and, therefore, the grant date becomes 30 June 2018 as this is the date that the terms and conditions
will be fixed. However, NFRS 2 requires the entity to recognise the services when received and,
therefore, adjustment is required to the financial statements. Once the terms are fixed, the fair value
can be calculated and any adjustments made.

NRs m
Dr Expense – in retained earnings (W5) 4.8
Cr Equity (W6) 1.8
Cr Current liabilities (W16) 3.0

(W12) Defined benefit plan – past service cost

A past service cost of NRs4 million should be recognised immediately as, part of the service cost
component. Thus the following entries will be required to account for the past service costs.

Dr Retained earnings (W5) NRs4.0m


Cr Non-current liabilities (defined benefit obligation) (W16) NRs4.0m

(W13) Accounting for sale of inventory (see part (c))

The transaction should not be shown as a sale. Inventory should be reinstated at NRs2 million instead
of NRs6 million and a decrease in retained earnings of NRs4 million should occur in the accounting
records of Siddy.

Cr Inventory (W15) NRs4m


Dr Retained earnings of Siddy (W2a) NRs4m

The cash position should be reversed also by increasing Riddy‘s cash balance by NRs6m (W15) and
also increasing Siddy‘s overdraft by NRs6m (W17).

(W14) Financial assets at fair value

NRs m
Riddy 10.0
Siddy 5.0
Ganesh (148m dinar @ 12) 12.3
Elimination of loan from Siddy to Ganesh (W16) (4.0)
––––
23.3
––––

(W15) Current assets


Page 95 of 328
Paper1: Advanced Financial Reporting [RTP 2018 June]

NRs m
Riddy 22.0
Siddy 17.0
Ganesh (120m dinar @ 12) 10.0
Inventory adjustment (W13) (4.0)
Cash reinstated re window dressing transaction (W13) 6.0
–––––
51.0
–––––

(W16) Non-current liabilities

NRs m
Riddy 90.0
Siddy 5.0
Ganesh (48 + 8 (W8) m dinar @ 12) 4.7
–––– 99.7

Increase carrying amount of loan liability (W10) 1.0


Eliminate of loan from Siddy to Ganesh (W14) (4.0)
Past service cost (W12) 4.0
Loan & penalty reclassified to current liabs (W10)(W17) (11.0)
–––––
89.7
–––––

(W17) Current liabilities

NRs m
Riddy 110.0
Siddy 7.0
Ganesh (72m dinar @ 12) 6.0
Cash bonus to Riddy employees (W11) 3.0
Cash reinstated re window dressing transaction (W13) 6.0
Loan & penalty reclassified from N-C liabs (W10)(W16) 11.0
–––––
143.0
–––––

Intangible Assets
9.

Extracts of Statement of Financial Position


As at 31.03.2075
NRs. In thousand
Assets
Non Current Assets
Intangible Assets (W1) 10,000
Page 96 of 328
Paper1: Advanced Financial Reporting [RTP 2018 June]

Liability
Loan Liability (5000 – 1500) 3,500

Extracts of Statement of Profit or Loss


For the period up to 01.01.2075
NRs. In thousand
Interest Expenses 800

(W1) Intangible Assets:

a) Since the management has intended to take to control of the movie and do not want to either sale
in the ordinary course of business or the film was not made under the contract. It was the sole
development (production) of a company and it will have control over the right to use the film.
Therefore, the film will be an identifiable non-monetary assets without physical substance,
controlled by the company and it is expected that future economic benefits can be obtained from
it.

Following directly attributable cost needs to be included as Intangible Assets:

NRs. In thousand
Direct Labour (Actors, Film Crew, Security) 4,000
Production Costs (Editing, Visual Effects) 2,000
Production Overheads (Studio Rents, Customs, Catering) 2,500
Administrative Cost (Insurance, Travels) 1,000
Interest Cost 500
–––––
Intangible Assets 10,000
–––––

Interest cost can be capitalized up to the date of ready for use i.e. till the date when substantially
all the activities relating to production of the film has been completed.

Biological Assets
10.
a. Calculation of fair value of Biological Assets as at 31 December 20X1 and 31 December 20X2:

 As at 31 December 20X1,
the mature plantation would have been valued at 17,100.

 As at 31 December 20X2,
the mature plantation would have been valued at 16,500.
Page 97 of 328
Paper1: Advanced Financial Reporting [RTP 2018 June]
Assuming immaterial cash flow between now and the point of harvest, the fair value (and therefore
the amount reported as an asset on the statement of financial position) of the plantation is estimated as
follows:

 As at 31 December 20X1: 17,100 ÷ [(1+6%)^20] = 5,332.


 As at 31 December 20X2: 16,500 ÷ [(1+6%)^19] = 5,453.

b. Calculating GAINS AND LOSSES


At initial recognition, the fair value (less estimated point-of-sale costs) of a biological asset is
reported as a gain or loss in the statement of profit or loss. A loss may arise on initial recognition
when the estimated point-of-sale costs exceed the fair value of the asset in its present state.

The change in fair value (less estimated point-of-sale costs) of a biological asset between two year
end dates is reported as a gain or loss in the statement or profit or loss.

A gain or loss arising on initial recognition of agricultural produce at fair value less estimated point-
of-sale costs is included in net profit or loss for the period in which it arises.

In the given case, the difference in fair value of the plantation between the two year end dates is 121
(5,453 – 5,332) and this will be reported as a gain in the statement or profit or loss (regardless of the
fact that it has not yet been realized).

The aggregate gain of 121 is attributed to two factors:

2. The effects of change in market price, and


3. The physical change (growth) of the trees in the plantation.

The aggregate gain is analyzed as follows:


1. The price change, which represents, at the biological asset‘s state as at the previous year end date:

the value of the biological asset at prices prevailing as at the current year end date
less the value of the biological asset at prices prevailing as at the previous year end date:
= (16,500 ÷ [(1+6%)^20]) – (17,100 ÷ [(1+6%)^20]) = 5,145 – 5,332 = 187 (loss).

2. The physical change, which represents, at current prices:


the value of the biological asset in its state as at the current year end date
less the value of the biological asset in its state as at the previous year end date:
= (16,500 ÷ [(1+6%)^19]) – (16,500 ÷ [(1+6%)^20]) = 5,453 – 5,145 = 308 (gain).

Thus, net gain for the year is 121 (i.e. 308-187).

Reorganization/ Restructuring

11. The impact of each of the proposed structures is discussed below.

Plan 1: share for share exchange


If the purchase consideration is in the form of shares, then a share premium account will need to be
set up in the books of Y. This share premium account must comprise the minimum premium value,
Page 98 of 328
Paper1: Advanced Financial Reporting [RTP 2018 June]
which is the excess of the book value of the investment over the nominal value of the shares issued:
NRs70m - NRs50m = NRs20m.

The impact on the individual company accounts and on the group accounts is as follows:

Note X Y Z Group
NRsm NRsm NRsm NRsm
Property, plant and equipment 600 200 45 845
Goodwill 10
Cost of investment in Y 1 130
Cost of investment in Z 2 70
Net current assets 160 100 20 280
890 370 65 1,135

Note X Y Z Group
NRsm NRsm NRsm NRsm
Share capital 3 120 110 40 120
Share premium 4 20
Retained earnings 5 770 240 25 1,015
890 370 65 1,135

Notes
1. Cost of investment in Y
This is increased by the total value of the shares issued: NRs50m + NRs20m = NRs70m.

2. Cost of investment in Z
Transferred to Y. The book value of the investment is preserved.

3. Share capital
Y's share capital is increased by the nominal value of the shares issued, NRs50m.

4. Share premium
This is as discussed above.

5. Retained earnings
Goodwill arising on the purchase of Z is NRs10m (NRs70m - (NRs40m + NRs20m)). The group
retained earnings are calculated as follows.

X Y Z
NRsm NRsm NRsm
Per question 770 240 25
Retained earnings at acquisition – (20)
770 240 5
Share of post-acquisition retained earnings of Y (240 X 100%) 240
Share of post-acquisition retained earnings of Z (5 X 100%) 5
1,015

Plan 2: cash purchase


Page 99 of 328
Paper1: Advanced Financial Reporting [RTP 2018 June]
The group accounts are not affected by the change as the reorganisation is internal. It has no impact
on the group as a single entity. If the purchase consideration is in the form of cash, a gain or loss on
the sale of Z will arise in the books of X. This does not count as a distribution as the cash price of
NRs75m is not in excess of the fair value of the net assets of Z, NRs80m. The effect on the accounts
would be as follows:

Note X Y Z Group
NRsm NRsm NRsm NRsm
Property, plant and equipment 600 200 45 845
Goodwill 10
Cost of investment in Y 60
Cost of investment in Z 1 75
Net current assets 2 235 25 20 280
895 300 65 1,135
Share capital 120 60 40 120
Retained earnings 3 775 240 25 1,015
895 300 65 1,135

Notes
1. Cost of investment in Z
This is the cash consideration of NRs75m.

2. Net current assets


X's cash increases by NRs75m and Y's cash decreases by NRs75m.

3. Retained earnings
X's retained earnings have been increased by NRs5m, being the profit on the sale of the
investment in Z. This is eliminated on consolidation as it is an intragroup transaction. The
consolidated retained earnings are calculated in exactly the same way as in the share for share
exchange.

Summary and conclusion


There are advantages and disadvantages to each of the two plans. Before we could make a
recommendation we would need more information about why the group wishes to restructure.
Neither plan changes the group financial statements. From an internal point of view it results in a
closer relationship between Y and Z. This may be advantageous if Y and Z are close geographically
or in terms of similarity of business activities. Alternatively, it might be advantageous for tax reasons.

Employee Benefits
12. It is always useful to set up a working reconciling the assets and obligation:

Assets Obligation
NRs NRs

Fair value/present value at 1/1/X2 1,100,000 1,250,000


Interest (1,100,000 × 6%)/(1,250,000 × 6%) 66,000 75,000
Page 100 of 328
Paper1: Advanced Financial Reporting [RTP 2018 June]
Current service cost 360,000

Contributions received 490,000


Benefits paid (190,000) (190,000)

Return on plan assets excluding amounts in net interest 34,000 -


(balancing figure) (OCI) (re-measurement)
Loss on re-measurement (balancing figure) (OCI) - 58,600

Total 1,500,000 1,553,600

The following accounting treatment is required.

(a) In the statement of profit or loss and other comprehensive income, the following amounts will be
recognised

In profit or loss:

Current service cost 360,000


Net interest on net defined benefit liability (75,000 – 66,000) 9,000

In other comprehensive income (34,000 – 58,000) 24,000

(b) In the statement of financial position, the net defined benefit liability of NRs 53,600 (1,553,600 –
1,500,000) will be recognised.

Foreign Exchanges
13.
(a) Britney must recognise the purchase of goods at the exchange rate in place at the date of the
transaction.

Therefore:
300,000 Wons/20 = NRs15,000
DR Purchases NRs15,000
CR Trade payables NRs15,000

At the year end, the supplier has not been paid, so the liability is still outstanding. It must be
translated at the closing rate at the year end and any exchange gains or losses recognised in the
statement of profit or loss.

The liability at 31 December 20X7 is:


3000,000 Wons/16 = NRs18,750

It has increased and Britney must recognise an exchange loss of NRs3,750 (18,750 – 15,000).
DR Statement of profit or loss NRs3,750
Page 101 of 328
Paper1: Advanced Financial Reporting [RTP 2018 June]
CR Trade payables NRs3,750

(b) Functional currency is the currency of the primary economic environment in which the entity
operates. Determining an entity's functional currency involves looking at the currency that
influences sales prices and costs. Additionally, if an entity raises finance in its home currency,
that is likely to be its functional currency.

In Britney's case, it operates in NRs, which is the functional currency. The presentation currency is
the currency in which the financial statements are presented. Britney may well prepare financial
statements in their functional currency (NRs), but the parent company reports in Euros, so Britney's
results will have to be translated into Euros so that they can be consolidated. The group presentation
currency is the Euro.

Value Added Statements


14. E.V.A. = NOPAT – COCE
NOPAT = Net Operating Profit after Tax
COCE = Cost of Capital Employed
COCE = Weighted Average Cost Of Capital X Average Capital Employed

Debt Capital = NRs.2,000 crores


Equity capital = 500 + 7,500 NRs.8,000 crores
Capital employed = 2,000+8,000 = NRs.10,000 crores
Debt to capital employed = 2000/10000 = 0.20
Equity to capital employed = 1 - 0.20 = 0.8

Debt cost before Tax =12%


Less: Tax (30% of 12%) =3.60%
Debt cost after Tax =8.40%

According to Capital Asset Pricing Model (CAPM)


Cost of Equity Capital = Risk Free Rate + Beta (Market Rate – Risk Free Rate)
= 9 + 1.05 X (19-9)
= 9 + 1.05 X 10
=19.50%
WACC= Equity to CE x Cost of Equity capital + Debt to CE x Cost of debt
= 0.8 x 19.5% + 0.20 x 8.40%
= 17.28%
COCE = WACC x Capital employed
= 17.28% x 10,000 crores = 1,728 crores

E.V.A. = NOPAT – COCE


= NRs. 2,100 – NRs. 1,728
= NRs. 372 crores

15.
a. What differentiates Profit or Loss from Other Comprehensive Income

The purpose of the statement of profit or loss and other comprehensive income (OCI) is to show an
entity‘s financial performance in a way that is useful to a wide range of users so that they may attempt
Page 102 of 328
Paper1: Advanced Financial Reporting [RTP 2018 June]
to assess the future net cash inflows of an entity. The statement should be classified and aggregated in
a manner that makes it understandable and comparable. IFRS currently requires that the statement be
presented as either one statement, being a combined statement of profit or loss and other
comprehensive income or two statements, being the statement of profit or loss and the statement of
profit or loss and other comprehensive income. An entity has to show separately in OCI, those items
which would be reclassified (recycled) to profit or loss and those items which would never be
reclassified (recycled) to profit or loss. The related tax effects have to be allocated to these sections.

Profit or loss includes all items of income or expense (including reclassification adjustments) except
those items of income or expense that are recognised in OCI as required or permitted by IFRS.
Reclassification adjustments are amounts recycled to profit or loss in the current period that were
recognised in OCI in the current or previous periods. An example of items recognised in OCI that
may be reclassified to profit or loss are foreign currency gains on the disposal of a foreign operation
and realised gains or losses on cash flow hedges. Those items that may not be reclassified are changes
in a revaluation surplus under NAS 16, Property, Plant and Equipment, and actuarial gains and losses
on a defined benefit plan under NAS 19, Employee Benefits.

However, there is a general lack of agreement about which items should be presented in profit or loss
and in OCI. The interaction between profit or loss and OCI is unclear, especially the notion of
reclassification and when or which OCI items should be reclassified. A common misunderstanding is
that the distinction is based upon realised versus unrealised gains. This lack of a consistent basis for
determining how items should be presented has led to an inconsistent use of OCI in NFRS. It may be
difficult to deal with OCI on a conceptual level since the NASB are finding it difficult to find a sound
conceptual basis. However, there is urgent need for some guidance around this issue.

b. Methods of valuation of intangible assets

Valuation of intangible assets is a complex exercise, as the non-physical form of intangible


assets poses the difficulty of identifying the future economic benefits that the enterprise can expect
to derive from them. There are three main approaches for valuing intangible assets:

Cost approach:
In cost approach, historical expenditure incurred in developing the asset is aggregated. Cost is
measured by purchase price, where the asset has been acquired recently.

Market value approach:


In comparable market value approach, intangible assets are valued with reference to
transactions involving similar assets that have cropped up recently in similar markets. This
approach is possible when there is an active market in which arm‘s length transactions have occurred
recently involving comparable intangible assets and adequate information of terms of transactions is
available.

Economic value approach:


This approach is based on the cash flows or earnings attributable to those assets and the
capitalization thereof, at an appropriate discount rate or multiple. Some of the key parameters
used in this approach are projected revenues, projected earnings, discount rate, rate of return
etc. The information required can be derived from either internal sources, external sources or both.
Page 103 of 328
Paper1: Advanced Financial Reporting [RTP 2018 June]
Under this approach, the valuator has to identify cash flows or earnings directly associated
with the intangible assets like the cash flows arising from the exploitation of a patent or copyright,
licensing of an intangible asset etc. This approach can be put to practice only if cash flows arising
from the intangible assets are identifiable from the management accounts and budgets, forecasts
or plans of the company. In most situations of valuation of intangible assets, the economic based
approach is used, because of the uniqueness of intangible assets and the lack of comparable market
data for the use of market value approach.

Share Based Payments

16. Year ended 31 December 2012


The estimated total cost:
=300 (number of options per employee) x 350 (the estimated number of employees in whom the
options will vest) x NRs2.40 (the fair value of each option at the grant date)
= NRs252,000.
This cost is recognised over the three-year vesting period so the cost to date is NRs84,000
(NRs252,000 x 1/3). This amount is debited to employment expenses and credited to equity.

Year ended 31 December 2013


The estimated total cost
=300 x 360 (note this estimate is updated) x NRs2.40.
= NRs259,200.
This cost is recognised over the three-year vesting period so the cost to date is NRs172,800
(NRs259,200 x 2/3).
This is the cumulative amount that is included in equity. Therefore the additional employment
expense charge for 2013 is NRs88,800 (NRs172,800 – NRs84,000).

Year ended 31 December 2014


The actual total cost is 300 x 365 x NRs2.40 = NRs262,800.
The vesting period is now complete so this is the total cost recognised.
Therefore the additional employment expense charge for 2014 is NRs90,000 (NRs262,800 –
NRs172,800).

The effect of the transactions on the three years can be summarised in the following table:

Year ended 31 Charge to employment Cumulative balance


December expenses in equity

2012 84,000 84,000

2013 88,800 172,700

2014 90,000 262,800

17. Year ended 30 November 2013


Page 104 of 328
Paper1: Advanced Financial Reporting [RTP 2018 June]
Based on latest estimates 250 options will vest for 183 employees. Since this is the first year of a
three-year vesting period the charge to profit or loss and the credit to equity will be NRs54,900 (250 x
183 x NRs3.60 x 1/3).

Year ended 30 November 2014


Based on latest estimates 300 options will vest for 185 employees. Since this is the second year of a
three-year vesting period the cumulative credit to equity will be NRs133,200 (300 x 185 x NRs3.60 x
2/3). The charge to profit or loss for this period will be NRs78,300 (NRs133,200 – NRs54,900).

18. NAS 38 Intangible Assets states that the cost less residual value of an intangible asset with a finite
useful life should be amortised on a systematic basis over that life, that the amortisation method
should reflect the pattern of benefits and that it should be reviewed at least annually. The amortisation
method should be reviewed at least annually and, if the pattern of consumption of benefits has
changed, the amortisation method should be changed prospectively as a change in estimate under
NAS 8 Accounting Policies, Changes in Accounting Estimates and Errors. Expected future
reductions in sales could be indicative of a higher rate of consumption of the future economic benefits
embodied in an asset. Hence, the trademark would be amortised over a 2·5-year period until May
2018.

NAS 36 states that an entity should assess at the end of each reporting period whether there is any
indication that an asset may be impaired. If any such indication exists, the entity should estimate the
recoverable amount of the asset. Irrespective of whether there is any indication of impairment, an
entity shall also test an intangible asset with an indefinite useful life or an intangible asset not yet
available for use for impairment annually by comparing its carrying amount with its recoverable
amount. This impairment test may be performed at any time during an annual period, provided it is
performed at the same time every year. Thus, MoonSun should test the trademark for impairment.

For the year ended 30 November 2016


Dr Profit or loss (operating expenses) – amortisation of trademark NRs.840,000
Cr Intangible asset (trademark) – accumulated amortisation NRs.840,000

To recognise the annual amortisation of the trademark during the period.

At 30 November 2016
Dr Profit or loss (operating expenses) – impairment of trademark NRs.760,000
Cr Intangible asset (finite life trademark) – impairment NRs.760,000
To recognise the impairment loss for the trademark.
Workings
Cost of trademark NRs.3m ÷ 10 years useful life = NRs.300,000 amortisation per year. Therefore the
carrying amount at 1 December 2015 is (NRs.3m cost less (NRs.300,000 amortisation per year x 3
years since acquisition)) = NRs.2·1m.

The useful life of the trademark is reduced to 2·5 years and therefore this amount has to be amortised
over this period.
NRs.2·1 m ÷ 2·5 years remaining useful life = NRs.840,000 per year

Therefore the carrying amount at 30 November 2016 is NRs.3m cost less NRs.900,000 less
NRs.840,000 = NRs.1·26 million.
Page 105 of 328
Paper1: Advanced Financial Reporting [RTP 2018 June]
The recoverable amount is NRs.500,000, so the impairment loss is NRs.760,000.

Lease
19. All numbers in NRs.‘000 unless otherwise stated

The lease of the asset by Revolta to ADATA would be regarded as a finance lease because the risks
and rewards of ownership have been transferred to ADATA. Evidence of this includes the lease is for
the whole of the life of the asset and ADATA being responsible for repairs and maintenance. Since
the lease is a finance lease and Revolta is the lessor, Revolta will recognise a financial asset – the ‗net
investment in finance leases‘. The amount recognised will be the present value of the minimum lease
payments which will be 2,787 x 7·247 which (subject to rounding) equals

The impact of the lease on the financial statements for the year ended 30 September 2017 can best be
seen by preparing a profile of the net investment in the lease for the first three years of the lease and
shown below:

Year to Balance Rental Balance Finance Balance


30 September b/fwd in period income c/fwd
2016 20,200 (2,787) 17,413 1,393 18,806
2017 18,806 (2,787) 16,019 1,282 17,301
2018 17,301 (2,787) 14,514

During the year ended 30 September 2017, Revolta will recognise income from finance leases of
1,282. The net investment on 30 September 2017 will be 17,301.

Of the closing net investment of 17,301, 2,787 will be shown as a current asset and 14,514 as a
non-current asset.

20.
a. Current and non-current assets

NAS 1 distinguishes between current and non-current assets by identifying the meaning of the
term ‗current asset‘.
An asset is classified as current when the entity:
– Expects to realise the asset, or intends to sell or consume it, in its normal operating cycle.
– Holds the asset primarily for the purpose of trading.
– Expects to realise the asset within 12 months after the reporting period.
– Has cash or a cash equivalent which is not subject to an exchange restriction.
An entity classifies all other assets as non-current.

b. NAS 16 property, plant and equipment and NAS 38 intangible assets.

NAS 16 defines property, plant and equipment as tangible items which are held for use in the
production or supply of goods and services, for rental to others, or for administrative purposes
and are expected to be used for more than one period.

NAS 38 defines intangible assets as identifiable, non-monetary assets without physical


substance.
Page 106 of 328
Paper1: Advanced Financial Reporting [RTP 2018 June]

21. Insurance contract as a "contract under which one party (the insurer) accepts significant insurance
risk from another party (the policyholder) by agreeing to compensate the policyholder if a specified
uncertain future event (the insured event) adversely affects the policyholder."

The following are examples of contracts that are insurance contracts, if the transfer of insurance risk
is significant:

 Insurance against theft or damage to property


 Insurance against product liability, professional liability, civil liability or legal expenses
 Life insurance and prepaid funeral expenses
 Life-contingent annuities and pensions
 Disability and medical cover
 Surety bonds, fidelity bonds, performance bonds and bid bonds
 Credit insurance that provides for specified payments to be made to reimburse the holder for a
loss it incurs because a specified debtor fails to make payment when due
 Product warranties (other than those issued directly by a manufacturer, dealer or retailer)
 Title insurance
 Travel assistance
 Catastrophe bonds that provide for reduced payments of principal, interest or both if a specified
event adversely affects the issuer of the bond
 Insurance swaps and other contracts that require a payment based on changes in climatic,
geological or other physical variables that are specific to a party to the contract
 Reinsurance contracts.

The following are examples of items that are not insurance contracts:
 Investment contracts that have the legal form of an insurance contract but do not expose the
insurer to significant risk
 Contracts that pass all significant insurance risk back to the policyholder
 Self-insurance i.e. retaining a risk that could have been covered by insurance
 Gambling contracts
 Derivatives that expose one party to financial risk but not insurance risk
 A credit-related guarantee
 Product warranties issued directly by a manufacturer, dealer or retailer
 Financial guarantee contracts accounted for under NAS 39 Financial Instruments: Recognition
and Measurement.
Page 107 of 328
Paper1: Advanced Financial Reporting [RTP 2018 December]
Question No 1:
(a) Kiddy is a public limited entity. It designs and manufactures children‘s toys. It has a
reporting date of 31 December 20X7 and prepares its financial statements in accordance with
International Financial Reporting Standards. The directors require advice about the following
situations.

(i) Kiddy sells NRs. 10,000 gift cards. These can be used when purchasing any of
Kiddy‘s products through its website. The gift cards expire after 12 months. Based on
significant past experience, Kiddy estimates that its customers will redeem 70% of
the value of the gift card and that 30% of the value will expire unused. Kiddy has no
requirement to remit any unused funds to the customer when the gift card expires
unused.

The directors are unsure about how the gift cards should be accounted for.

(ii) Kiddy‘s best-selling range of toys is called Scarimon. In 20X6 Color, another listed
company, entered into a contract with Kiddy for the rights to use Scarimon characters
and imagery in a monthly comic book. The contract terms state that Color must pay
Kiddy a royalty fee for every issue of the comic book which is sold. Before signing
the contract, Kiddy determined that Color had a strong credit rating. Throughout
20X6, Color provided Kiddy with monthly sales figures and paid all amounts due in
the agreed-upon period. At the beginning of 20X7, Color experienced cash flow
problems. These were expected to be short term. Color made nominal payments to
Kiddy in relation to comic sales for the first half of the year. At the beginning of July
20X7, Color lost access to credit facilities and several major customers. Color
continued to sell Scarimon comics online and through specialist retailers but made no
further payments to Kiddy.

The directors are unsure how to deal with the above issues in the financial statements
for the year ended 31 December 20X7.

Required:
Advise the accountant on the matters set out above with reference to International
Financial Reporting Standards.

(b) As a result of rising property prices, Kiddy purchased five buildings during the current period
in order to benefit from further capital appreciation. Kiddy has never owned an investment
property before. In accordance with NAS 40 Investment Property, the directors are aware
that they can measure the buildings using either the fair value model or the cost model.
However, they are concerned about the impact that this choice will have on the analysis of
Kiddy‘s financial performance, position and cash flows by current and potential investors.

Required:
Discuss the potential impact which this choice in accounting policy will have on
investors‟ analysis of Kiddy‟s financial statements. Your answer should refer to key
financial performance ratios.
Page 108 of 328
Paper1: Advanced Financial Reporting [RTP 2018 December]

Question No 2:
―Italian Foods‖ is a public limited company which produces a range of luxury Italian food
products which are sold to restaurants, shops and supermarkets. It prepares its financial
statements in accordance with Nepal Financial Reporting Standards. The directors of ―Italian
Foods‖ receive a cash bonus each year if reported profits for the period exceed a pre-
determined target. ―Italian Foods‖ has performed in excess of targets in the year ended 31
December 20X7. Forecasts for 20X8 are, however, pessimistic due to economic uncertainty
and stagnant nationwide wage growth.

Provisions
A new accountant has recently started work at ―Italian Foods‖. She noticed that the
provisions balance as at 31 December 20X7 is significantly higher than in the prior year. She
made enquiries of the finance director, who explained that the increase was due to substantial
changes in food safety and hygiene laws which become effective during 20X8. As a result,
―Italian Foods‖ must retrain a large proportion of its workforce. This retraining has yet to
occur, so a provision has been recognized for the estimated cost of $2 million. The finance
director then told the accountant that such enquiries were a waste of time and would not be
looked at favorably when deciding on her future pay rises and bonuses.

Wheat contract
―Italian Foods‖ purchases significant quantities of wheat for use in its bread and pasta
products. These are high-value products on which ―Italian Foods‖ records significant profit
margins. Nonetheless, the price of wheat is volatile and so, on 1 November 20X7, ―Italian
Foods‖ entered into a contract with a supplier to purchase 500,000 bushels of wheat in June
20X8 for $5 a bushel. The contract can be settled net in cash. ―Italian Foods‖ has entered into
similar contracts in the past and has always taken delivery of the wheat. By 31 December
20X7 the price of wheat had fallen. The finance director recorded a derivative liability of
$0·5 million on the statement of financial position and a loss of $0·5 million in the statement
of profit or loss. Wheat prices may rise again before June 20X8. The accountant is unsure if
the current accounting treatment is correct but feels uncomfortable approaching the finance
director again.

Required:
Discuss the ethical and accounting implications of the above situations from the
perspective of the accountant.
Page 109 of 328
Paper1: Advanced Financial Reporting [RTP 2018 December]
Question No 3:
You are the financial controller of Megha Group, a listed entity which prepares consolidated
financial statements in accordance with Nepal Financial Reporting Standards (NFRS). The
chief executive officer (CEO) of Megha Group has reviewed the draft consolidated financial
statements of the Megha Group group and of a number of the key subsidiary companies for
the year ended 31 March 2018. None of the subsidiaries are listed entities but all prepare
their financial statements in accordance with NFRS. The CEO has sent you an email with the
following queries:

Query One
I notice that the disclosures relating to operating segments in the consolidated financial
statements appear to be based on the geographical location of the customers of the group. I
am the non-executive director of another large listed entity and the segment disclosures in
their consolidated financial statements are based on the type of products sold. Also some of
our larger subsidiaries have customers located in more than one geographical region, yet they
provide no segment disclosures whatsoever in their individual financial statements. I would
like to see segment disclosures given in the individual subsidiary accounts as well. I really
don‘t understand these inconsistencies given that all these financial statements have been
prepared using NFRS. Please explain the reasons for these apparent inconsistencies.

Query Two
When reading the accounting policies note in the consolidated financial statements I notice
that we measure all of our freehold properties using a fair value model but that we measure
our plant and equipment using a cost model. I further notice that both of these asset types are
shown in the ‗property, plant and equipment‘ figure which is a single component of non-
current assets in the consolidated statement of financial position. It makes no sense to me that
assets which are shown as property, plant and equipment are measured inconsistently. If it‘s
OK to measure different parts of property, plant and equipment using two different
measurement models, why not use the fair value model for the more readily accessible
properties and use the cost model for the properties in remote locations to save on time and
cost?

Query Three
When I read the disclosure note relating to intangible non-current assets in the consolidated
financial statements, I notice that this figure includes brand names associated with
subsidiaries which we‘ve acquired in recent years. However, the brand names which are
associated directly with products sold by Megha Group (the parent entity) are not included
within the non-current assets figure. This is another inconsistency that I don‘t understand.
Please explain how this practice can be in line with NFRS requirements. One final question:
would I be right in thinking that, as with property, plant and equipment, we can use the fair
value model to measure intangible assets?

Required:
Provide answers to the three queries raised by the chief executive officer. Your answers
should refer to relevant provisions of Nepal Financial Reporting Standards.
Page 110 of 328
Paper1: Advanced Financial Reporting [RTP 2018 December]
Question No 4:
a. The current medium term strategic plan of Damak Municipal Office (DMO) has overall
objective of improving the performance of the Office. One critical strategy towards
attainment of this goal is adoption of new public management strategy to increase
participation of private sector in the provision of public services without losing sight of
the differences between public sector and private sector. In fact, some of these
differences are so fundamental that they cannot be washed away any time soon.

The Chief Executive suggested outsourcing as a key strategy in improving the delivery of
public services at local level through the private sector. DMO is currently bedevilled with
poor revenue mobilization, lack of proper data on the office‘s activities, and poor
infrastructure provision. Other supporting activities like cleaning and security are not
well performed or performed at very high cost by internal staff. These issues have been
tabled at the first strategy meeting convened by the Chief Executive.

Required:
i) Describe THREE fundamental differences between public sector and private sector
entities that DMO should take cognizance of in pursuance of the new public
management strategy.
ii) Explain the term ‘outsourcing’ in public sector context and advance TWO arguments
for the use of outsourcing by DMO in its operations.
iii) Explain THREE factors that the management of DMO should consider in making the
decision to outsource some of its functions.

b. You have received an official email from your Director which reads:

“Hello Accountant,

Hope you are doing well. We have closed from a workshop organised by the Financial
Controller General‘s Office on public financial management not long ago and the discussion
was all about adoption of accrual accounting in the public sector. It was emphasised that
migration from cash basis to accrual basis is necessary to improve financial reporting and
transparency in the public sector. You know I have little knowledge in Accountancy so I was
completely lost in the discussions and I wished you had attended the workshop with me.

Another issue discussed was commitment accounting. We were made to understand that
commitment accounting strengthens public financial management and therefore departments
must ensure that every expenditure is committed in accordance with the appropriation prior
to spending.

Please could you help me with some information on these issues?


Thank you,

Director‖.

Required:
Page 111 of 328
Paper1: Advanced Financial Reporting [RTP 2018 December]
1. Explain to the Director THREE differences between accrual accounting and cash
accounting.
2. Identify THREE justifications for adopting accrual accounting in the public sector
3. Explain the term commitment accounting and illustrate how it could strengthen public
financial management

Question No 5:
Background
Himal is a public limited company which has investments in a number of other entities. All of
these entities prepare their financial statements in accordance with International Financial
Reporting Standards. Extracts from the draft individual statements of profit or loss for Himal,
Pahad and Terai for the year ended 30 September 20X6 are presented below.

Himal Pahad Terai


$m $m $m
Profit/(loss) before taxation (45 ) 67 154
Taxation 9 (15 ) (31 )
––– ––– ––––
Profit/(loss) for the period (36 ) 52 123
––– ––– ––––

Acquisition of 80% of Pahad


Himal purchased 80% of the ordinary shares of Pahad on 1 October 20X5. Cash consideration of
$150 million has been included when calculating goodwill in the consolidated financial
statements. The purchase agreement specified that a further cash payment of $32 million
becomes payable on 1 October 20X7 but no entries have been posted in the consolidated
financial statements in respect of this. A discount rate of 5% should be used.

In the goodwill calculation, the fair value of Pahad‘s identifiable net assets was deemed to be
$170 million. Of this, $30 million related to Pahad‘s non-depreciable land. However, on 31
December 20X5, a survey was received which revealed that the fair value of this land was
actually only $20 million as at the acquisition date. No adjustments have been made to the
goodwill calculation in respect of the results of the survey. The non-controlling interest at
acquisition was measured using the proportionate method as $34 million ($170m x 20%).

As at 30 September 20X6, the recoverable amount of Pahad was calculated as $250 million. No
impairment has been calculated or accounted for in the consolidated financial statements.

Disposal of 20% holding in Terai


On 1 October 20X4, Himal purchased 60% of the ordinary shares of Terai. At this date, the fair
value of Terai‘s identifiable net assets was $510 million. The non-controlling interest at
acquisition was measured at its fair value of $215 million. Goodwill arising on the acquisition of
Terai was $50 million and had not been impaired prior to the disposal date. On 1 April 20X6,
Himal disposed of a 20% holding in the shares of Terai for cash consideration of $140 million.
At this date, the net assets of Terai, excluding goodwill, were carried in the consolidated
Page 112 of 328
Paper1: Advanced Financial Reporting [RTP 2018 December]
financial statements at $590 million. From 1 April 20X6, Himal has the ability to appoint two of
the six members of Terai‘s board of directors. The fair value of Himal‘s 40% shareholding was
$300 million at that date.

Issue of convertible bond


On 1 October 20X5, Himal issued a convertible bond at par value of $20 million and has
recorded it as a non-current liability. The bond is redeemable for cash on 30 September 20X7 at
par. Bondholders can instead opt for conversion in the form of a fixed number of shares. Interest
on the bond is payable at a rate of 4% a year in arrears. The interest paid in the year has been
presented in finance costs. The interest rate on similar debt without a conversion option is 10%.

Discount factors
Year Discount rate 5% Discount rate 10%
1 0·952 0·909
2 0·907 0·826

Required:
(i) In respect of the investment in Pahad, explain, with suitable calculations, how goodwill
should have been calculated, and show the adjustments which need to be made to the
consolidated financial statements for this as well as any implications of the recoverable
amount calculated at 30 September 20X6.
(ii) Discuss, with suitable calculations, how the investment in Terai should be dealt with in the
consolidated financial statements for the year ended 30 September 20X6.
(iii) Discuss, with suitable calculations, how the convertible bond should be dealt with in the
consolidated financial statements for the year ended 30 September 20X6, showing any
adjustments required.

Question No 6:
KaKhaGa is an entity which prepares financial statements to 31 March each year. The functional
currency of KaKhaGa is the dollar ($). The following event has occurred which are relevant to
the year ended 31 March 2018:

On 1 February 2018, KaKhaGa purchased some inventory from a supplier whose functional
currency was the dinar. The total purchase price was 3·6 million dinars. The terms of the
purchase were that KaKhaGa would pay for the goods in two instalments. The first instalment
payment of 1,260,000 dinars was due on 15 March 2018 and the second payment of 2,340,000
dinars on 30 April 2018. Both payments were made on the due dates. KaKhaGa did not
undertake any activities to hedge its currency exposure arising under this transaction. KaKhaGa
sold 60% of this inventory prior to 31 March 2018 for a total sales price of $480,000. All sales
proceeds were receivable in $. After 31 March 2018,

KaKhaGa sold the remaining inventory for sales proceeds which were in excess of their cost.

Relevant exchange rates are as follows:


– 1 February 2018 – 6·0 dinars to $1.
Page 113 of 328
Paper1: Advanced Financial Reporting [RTP 2018 December]
– 15 March 2018 – 6·3 dinars to $1.
– 31 March 2018 – 6·4 dinars to $1. (11 marks)

Required:
Explain and show how the two events would be reported in the financial statements of
KaKhaGa for the year ended31 March 2018.

Question No 7:

1. Please detail the non-compliances that you observe in the financial statements as follows
of M/S Nepal Ltd.

Nepal Ltd.
Balance Sheet
32 Ashadh 2075
In '000
Particulars Amount Amount
Sources of funds:
Share Capital:
Authorized: 500
Issued: Equity shares of Rs. 10 each fully paid
up 500
Reserves and surplus:
Capital reserves 40
Revenue reserves 700
Surplus 10 750
Owners‘ funds 1,250
Loan funds 250
Total 1,500
Fund employed in:
Fixed assets:
Cost 1,200
Less: Depreciation 400 800
Net current assets:
Current assets 2,000
Less: Current liabilities 1,300 700
Total 1,500

Additional Notes:
a. Fixed assets (Cost) includes NRs. 100 with respect to expenses incurred on account of
development of accounting software. Assume the expense has qualified the recognition
criteria for Intangible assets
Page 114 of 328
Paper1: Advanced Financial Reporting [RTP 2018 December]
b. Fixed assets (Cost) includes NRs. 200 relating to a building that has purchased during the
current year & is held for capital appreciation purpose. In addition, an amount of NRs. 20
incurred for legal expenses on account of acquisition of the said property has been
charged to the Income Statement.

c. Current assets include deferred tax assets of NRs. 200

d. A sundry debtor relating to foreign trade included in the current assets is NRs. 200
(Equivalent to $2 at the recognition date). Closing rate was $1= NRs. 90

Question No 8;

Earnings Per Share


From the following information compute Diluted Earnings Per Share (DEPS).
- Net profit for the year Rs.1,800,000
- Weighted Average no. of equity shares outstanding during the year 2068-69 is 75,000
- Average fair value of one equity share during the year: Rs. 200.
- Weighted average no. of the shares under the options during the year is 15,000.
- Exercise price per share under option during the year is Rs. 150.

Question No 9:
Define Government Assistance & state the disclosure requirements with respect to Government
Grants

Question No 10
a. Arc owns 100% of the ordinary share capital of Bend and Curve. All ordinary shares of all
three entities are listed on a recognized exchange. The groups operates in the engineering
industry, and are currently struggling to survive in challenging economic conditions. Curve
has made losses for the last three years and its liquidity is poor. The view of the directors is
that Curve needs some cash investment. The directors have decided to put forward a
restructuring plan as at 30 June 20X1. Under this plan:

(1) Bend is to purchase the whole of Arc‘s investment in Curve. The purchase consideration
is to be $105 million payable in cash to Arc and this amount will then be loaned on a
long-term unsecured basis to Curve; and
(2) Bend will purchase land and buildings with a carrying amount of $15 million from Curve
for a total purchase consideration of $25 million. The land and buildings has a mortgage
outstanding on it of $8 million. The total purchase consideration of $25 million comprises
both ten million $1 nominal value non-voting shares issued by Bend to Curve and the $4
million mortgage liability which Bend will assume; and
(3) Curve had also entered into a lease obligation on 1 July 20X0 for an asset with a useful
economic life of six years. The present value of the lease payments at that date was $3
million, and the implicit rate of interest associated with the lease obligation was 10.2%.
Page 115 of 328
Paper1: Advanced Financial Reporting [RTP 2018 December]
The lease required that annual payments in arrears of $700,000 must be made. No entries
had been made in respect of the lease in the draft financial statements of Curve; and
(4) A dividend of $25 million will be paid from Bend to Arc to reduce the accumulated
reserves of Bend.

The draft statements of financial position of Arc and its subsidiaries at 30 June 20X1 are
summarised below:

Arc Bend Curve


$m $m $m
Non-current assets:
Tangible non-current assets 500 200 55
Cost of investment in Bend 150
Cost of investment in Curve 95
Current assets 125 145 25
870 345 80

Equity and liabilities


Ordinary share capital 100 100 35
Share premium 8
Retained earnings 720 230 5
820 330 48

Non-current liabilities:
Long-term loan 5 12

Current liabilities:
Trade payables 45 15 20
870 345 80
As a result of the restructuring, some of Bend‘s employees will be made redundant. Based upon
a detailed plan, the costs of redundancy will be spread over three years with $2.08 million being
payable in one year‘s time, $3.245 million payable in two years' time and $53.375 million in
three years‘ time. The market yield of high quality corporate bonds is 4%. The directors of Arc
consider that, based upon quantification of relevant and reliable data at 30 June 20X1, it will
incur additional restructuring obligations amounting to $3 million.

Required:
(i) Prepare the individual entity statements of financial position after the proposed restructuring
plan.
(ii) Discuss the key implications of the proposed plans, in particular whether the financial
position of each company has been improved as a result of the reorganisation

b. State the assets that are not covered by NAS 38.

Question No 11:
Page 116 of 328
Paper1: Advanced Financial Reporting [RTP 2018 December]
William is a public limited company and would like advice in relation to the following
transactions.

(a) William sold a building for its fair value of $5 million to a finance company on 1 June 2011
when its carrying amount was $3.5 million. The same building was leased back from the
finance company for a period of 5 years when the remaining useful life was 25 years. The
lease rentals for the period are $440,000 payable annually in arrears. The interest rate implicit
in the lease is 7%. The present value of the annual lease payments is $1.8 million. William
wishes to know how to account for the above transaction for the year ended 31 May 2012.

(b) William operates a defined benefit scheme for its employees. The scheme was revised on 1
June 2011. This resulted in the benefits being enhanced for some members of the plan and,
because benefits do not vest for these members for five years, William wishes to spread the
increased cost over that period. During the current year, several highly paid employees left
William to work for other companies.

William transferred plan assets with a fair value of $0.4 million to the employees‘ new
pension schemes and in return extinguished its obligation to these employees with respect to
pension benefits. The actuary estimates that the present value of the defined benefit
obligation eliminated by this transaction was $0.3 million. William requires advice on how to
account for the above scheme under IAS 19 Employee Benefits

Required:
Discuss, with suitable computations, the advice that should be given to William in
accounting for the above events.

Question No 12:
William is a public limited company and would like advice in relation to the following
transactions.

(a) On 1 June 2009, William granted 500 share appreciation rights to each of its 20 managers.
All of the rights vest after two years‘ service and they can be exercised during the following
two years up to 31 May 2013. The fair value of the right at the grant date was $20. It was
thought that three managers would leave over the initial two-year period and they did so. The
fair value of each right was as follows: Year Fair value at year end $ 31 May 2010 23 31
May 2011 14 31 May 2012 24 On 31 May 2012, seven managers exercised their rights when
the intrinsic value of the right was $21. William wishes to know what the liability and
expense will be at 31 May 2012.

(b) William acquired another entity, Chrissy, on 1 May 2012. At the time of the acquisition,
Chrissy was being sued as there is an alleged mis-selling case potentially implicating the
entity. The claimants are suing for damages of $10 million. William estimates that the fair
value of any contingent liability is $4 million and feels that it is more likely than not that no
outflow of funds will occur. William wishes to know how to account for this potential
Page 117 of 328
Paper1: Advanced Financial Reporting [RTP 2018 December]
liability in Chrissy‘s entity financial statements and whether the treatment would be the same
in the consolidated financial statements.

Required: Discuss, with suitable computations, the advice that should be given to William
in accounting for the above events.

Question No 13;
Lockfine, a public limited company, operates in the fishing industry and has recently made the
transition to International Financial Reporting Standards. Lockfine‘s reporting date is 30 April
2015.

(a) In the NFRS opening statement of financial position at 1 May 2013, Lockfine elected to
measure its fishing fleet at fair value and use that fair value as deemed cost in accordance
with NFRS 1 First Time Adoption of International Financial Reporting Standards. The
fair value was an estimate based on valuations provided by two independent selling
agents, both of whom provided a range of values within which the valuation might be
considered acceptable. Lockfine calculated fair value at the average of the highest
amounts in the two ranges provided. One of the agents‘ valuations was not supported by
any description of the method adopted or the assumptions underlying the calculation.
Valuations were principally based on discussions with various potential buyers. Lockfine
wished to know the principles behind the use of deemed cost and whether agents‘
estimates were a reliable form of evidence on which to base the fair value calculation of
tangible assets to be then adopted as deemed cost.

(b) Lockfine was unsure as to whether it could elect to apply NFRS 3 Business Combinations
retrospectively to past business combinations on a selective basis, because there was no
purchase price allocation available for certain business combinations in its opening NFRS
statement of financial position. As a result of a major business combination, fishing rights
of that combination were included as part of goodwill. The rights could not be recognized
as a separately identifiable intangible asset at acquisition under the local GAAP because a
reliable value was unobtainable for the rights. The fishing rights operated for a specified
period of time. On transition from local GAAP to International Financial Reporting
Standards, the fishing rights were included in goodwill and not separately identified
because they did not meet the qualifying criteria set out in NFRS 1, even though it was
known that the fishing rights had a finite life and would be fully impaired or amortized
over the period specified by the rights. Lockfine wished to amortize the fishing rights
over their useful life and calculate any impairment of goodwill as two separate
calculations.

Question No 14;
The difference between debt and equity in an entity‘s statement of financial position is not easily
distinguishable for preparers of financial statements. Some financial instruments may have both
features, which can lead to inconsistency of reporting. The International Accounting Standards
Board (the Board) has agreed that greater clarity may be required in its definitions of assets and
liabilities for debt instruments. It is thought that defining the nature of liabilities would help the
Page 118 of 328
Paper1: Advanced Financial Reporting [RTP 2018 December]
Board‘s thinking on the difference between financial instruments classified as equity and
liabilities.

Required:
(i) Discuss the rules that should be applied when deciding if a financial instrument should be
classified as debt or equity. Note: Examples should be given to illustrate your answer.
(ii) Explain why it is important for entities to correctly classify a financial instrument as debt or
equity in the financial statements.

Question No 15:
The directors of Avco, a public limited company, are reviewing the financial statements of two
entities which are acquisition targets, Cavor and Lidan. They have asked for clarification on the
treatment of the following financial instruments within the financial statements of the entities.

Cavor has two classes of shares: A and B shares. A shares are Cavor‘s ordinary shares and are
correctly classed as equity. B shares are not mandatorily redeemable shares but contain a call
option allowing Cavor to repurchase them. Dividends are payable on the B shares if, and only if,
dividends have been paid on the A ordinary shares. The terms of the B shares are such that
dividends are payable at a rate equal to that of the A ordinary shares. Additionally, Cavor has
also issued share options which give the counterparty rights to buy a fixed number of its B shares
for a fixed amount of $10 million. The contract can be settled only by the issuance of shares for
cash by Cavor.

Lidan has in issue two classes of shares: A shares and B shares. A shares are correctly classified
as equity. Two million B shares of nominal value of $1 each are in issue. The B shares are
redeemable in two years‘ time at the option of Lidan. Lidan has a choice as to the method of
redemption of the B shares. It may either redeem the B shares for cash at their nominal value or
it may issue one million A shares in settlement. A shares are currently valued at $10 per share.
The lowest price for Lidan‘s A shares since its formation has been $5 per share.

Required:
Discuss whether the above arrangements regarding the B shares of each of Cavor and Lidan
should be treated as liabilities or equity in the financial statements of the respective issuing
companies.

Question No 16:
The International Accounting Standards Board (IASB) is undertaking a broad-based initiative to
explore how disclosures in IFRS financial reporting can be improved. The Disclosure Initiative is
made up of a number of implementation and research projects. The IASB has decided that the
project should include a discussion on whether the definition of materiality should be changed
and whether IAS 1 Presentation of Financial Statements should include additional guidance
which clarifies the key characteristics of materiality. Materiality is a matter which has been
debated extensively in the context of many forms of reporting, including the International
Integrated Reporting Framework. There are difficulties in applying the concept of materiality in
Page 119 of 328
Paper1: Advanced Financial Reporting [RTP 2018 December]
practice when preparing the financial statements and it is thought that these difficulties
contribute to a disclosure problem, namely, that there is both too much irrelevant information in
financial statements and not enough relevant information. Further, the IASB has published for
public comment an Exposure Draft of proposed amendments to IAS 7 Statement of Cash Flows.
The proposal responds to requests from investors for improved disclosures about an entity‘s
financing activities and its cash and cash equivalents balances.

Required:
(i) Discuss the current definition of materiality and how the current application of the concept of
materiality may be leading to a reduction in the clarity and understandability of financial
statements.
(ii) Discuss how the concepts of materiality would be used in applying the International
Integrated Reporting Framework.

Question No 17:
Write short notes on:
a) Financial Comptroller General
b) Nepal Public Sector Accounting Standards

Question No 18:
What is temporary difference and what are the examples of temporary differences?

Question No 19:
What are insurance contracts? List down the examples that are insurance contracts and that are
not insurance contracts.

Question No 20:
Related party relationships are a common feature of commercial life. The objective of NAS 24:
Related Party Disclosures is to ensure that financial statements contain the necessary disclosures
to make users aware of the possibility that financial statements may have been affected by the
existence of related parties.

Required:
Explain Three importance of disclosing related party relationships and transactions in financial
statements.

Question No 21:
Explain with justification, whether each of the following could most likely be classified as a
discontinued operation under NFRS 5: Non-current Assets Held for Sale and Discontinued
Operations in this year's financial statements:

i) A reportable operating segment that met the definition of held for sale after the year end, but
before the financial statements were authorized for issue.

ii) A reportable operating segment that was closed down during the financial year. The assets of
the segment were broken up and used in other divisions of the company.
Page 120 of 328
Paper1: Advanced Financial Reporting [RTP 2018 December]

iii) A division of a business, classified as held for sale, that was correctly treated as a
discontinued operation in last year's financial statements, but which has not been sold by this
year-end due to the sale being referred to the National Insurance Commission, which
regulates the Insurance industry. The commission is not expected to report its findings until 6
months after this year end.
Page 121 of 328
Paper1: Advanced Financial Reporting [RTP 2018 December]

Suggested Answer Hint


Answer No 1:

(a)

(i) Gift cards


IFRS 15 Revenue from Contracts with Customers says that revenue should be recognized
when or as a performance obligation is satisfied by transferring the promised good or service
to the customer. When a customer buys a gift card they are pre-paying for a product.
Revenue cannot be recognized because the entity has not yet transferred control over an asset
and so has not satisfied a performance obligation. As such, cash received in respect of gift
cards should be initially recognized as a contract liability.

IFRS 15 refers to a customer‘s unexercised rights as breakage. The guidance for variable
consideration is followed when estimating breakage. In other words, the expected breakage is
included in the transaction price if it is highly probable that a significant reversal in the
amount of cumulative revenue recognized will not occur once the uncertainty is subsequently
resolved. This means that if the company is unable to reliably estimate the breakage amount,
then revenue for the unused portion of the gift card is recognized when the likelihood of the
customer exercising their remaining rights becomes remote. However, if an entity is able to
reliably estimate the breakage amount, then it recognizes the expected breakage amount as
revenue in proportion to the pattern of rights exercised by the customer.

In relation to Kiddy, it appears that the amount of breakage can be reliably determined and so
this should be recognized in revenue as the gift card is redeemed. For every Re 1 redeemed,
Kiddy should recognize Re 1·43 ($1 x 100/70) in revenue.

(ii) Royalty
According to IFRS 15, an entity should only account for revenue from a contract with a
customer when it meets the following criteria:

– The contract has been approved;


– Rights regarding goods and services can be identified;
– Payment terms can be identified;
– It is probable the seller will collect the consideration it is entitled to.

At inception of the agreement, Kiddy and Colour entered an explicit contract which specified
payment terms and conditions. Moreover, Colour had a strong credit rating and so payment
was probable. As such, it would seem that the above criteria were met. IFRS 15 says that
revenue from a usage-based royalty should be recognized as the usage occurs.

Whether a contract with a customer meets the above criteria is only reassessed if there is a
significant change in facts and circumstances. In July 20X7, Colour lost major customers and
sources of finance. As such, it was no longer probable that Kiddy would collect the
Page 122 of 328
Paper1: Advanced Financial Reporting [RTP 2018 December]
consideration it was entitled to. From July 20X7, no further revenue from the contract should
be recognized.

According to IFRS 9 Financial Instruments, non-payment is an indicator that the outstanding


receivables are credit impaired. A loss allowance should be recognized equivalent to the
difference between the gross carrying amount of the receivables and the present value of the
expected future cash flows receivable from Colour. Any increase or decrease in the loss
allowance is charged to profit or loss.

(b) Investment properties


In accordance with NAS 40 Investment Properties, the buildings should be initially measured
at cost.

If the cost model is applied, then the buildings will be recognized at cost less accumulated
depreciation and impairment losses. If the fair value model is applied, then the buildings will
be remeasured to fair value at each reporting date. Gains and losses on remeasurement are
recognized in the statement of profit or loss. No depreciation is charged.

Statement of financial position


Assuming that property prices rise, the fair value model will lead to an increase in reported
assets on the statement of financial position. In contrast, investment property measured using
the cost model is depreciated, which reduces its carrying amount. This means that the fair
value model may make Kiddy appear more asset-rich. Some stakeholders may place
importance on an entity‘s asset base, as it can be used as security for obtaining new finance.
However, reporting higher assets can sometimes be perceived negatively. For example, asset
turnover ratios will deteriorate, and so Kiddy may appear less efficient.

If assets increase, then equity also increases. As such, the fair value model may lead to Kiddy
reporting a more optimistic gearing ratio. This may reduce the perception of risk,
encouraging further investment.

Statement of profit or loss


In times of rising prices, the use of the fair value model will lead to gains being reported in
the statement of profit or loss. This will increase profits for the period. In contrast, the
depreciation charged under the cost model will reduce profits for the period. Therefore,
earnings per share, a key stock market and investor ratio, is likely to be higher if the fair
value model is adopted. However, it should be noted that fair values are volatile. In some
years, fair value gains may be much larger than in other years.

If property prices decline, then the fair value model will result in losses. As such, reported
profits are subject to more volatility if the fair value model is adopted. This may increase
stakeholders‘ perception of risk. In contrast, the depreciation expense recorded in accordance
with the cost model will be much more predictable, meaning that investors will be better able
to predict Kiddy‘s future results.
Page 123 of 328
Paper1: Advanced Financial Reporting [RTP 2018 December]
Disclosure
It should be noted that entities using the cost model for investment properties are required to
disclose the fair value. Such disclosures enable better comparisons to be drawn between
entities which account for investment property under different models.

Answer No 2:
Provision
NAS 37 Provisions, Contingent Liabilities and Contingent Assets states that a provision should
only be recognized if:

– there is a present obligation from a past event,


– an outflow of economic resources is probable, and
– the obligation can be measured reliably.

No provision should be recognized because ―Italian Foods‖ does not have an obligation to incur
the training costs. The expenditure could be avoided by changing the nature of ―Italian Foods‘
operations and so it has no present obligation for the future expenditure. The provision should be
derecognized. This will reduce liabilities by $2 million and increase profits by the same amount.

Contract
NFRS 9 Financial Instruments applies to contracts to buy or sell a non-financial item which are
settled net in cash. Such contracts are usually accounted for as derivatives. However, contracts
which are for an entity‘s ‗own use‘ of a non-financial asset are exempt from the requirements of
NFRS 9. The contract will qualify as ‗own use‘ because ―Italian Foods‖ always takes delivery of
the wheat. This means that it falls outside NFRS 9 and so the recognition of a derivative is
incorrect.

The contract is an executory contract. Executory contracts are not initially recognized in the
financial statements unless they are onerous, in which case a provision is required. This
particular contract is unlikely to be onerous because wheat prices may rise again. Moreover, the
finished goods which the wheat forms a part of will be sold at a profit. As such, no provision is
required. The contract will therefore remain unrecognized until ―Italian Foods‖ takes delivery of
the wheat.

The derivative liability should be derecognized, meaning that profits will increase by $0·5
million.

Ethical implications
The users of Italian Foods‘s financial statements, such as banks and shareholders, trust
accountants and rely on them to faithfully represent the effects of a company‘s transactions. NAS
1 Presentation of Financial Statements makes it clear that this will be obtained when accounting
standards are correctly applied.

Both of the errors made by ―Italian Foods‖ overstate liabilities and understate profits. It is
possible that these are unintentional errors. However, incentives exist to depart from particular
NFRS and NAS standards: most notably the bonus scheme. The bonus target in 20X7 has been
Page 124 of 328
Paper1: Advanced Financial Reporting [RTP 2018 December]
exceeded, and so the finance director may be attempting to shift ‗excess‘ profits into the next
year in order to increase the chance of meeting 20X8‘s bonus target. In this respect, the finance
director has a clear self-interest threat to objectivity and may be in breach of Code of Ethics and
Conduct.

The accountant is correct to challenge the finance director and has an ethical responsibility to do
so. Despite the fact that the finance director is acting in an intimidating manner, the accountant
should explain the technical issues to the director. If the director refuses to comply with
accounting standards, then it would be appropriate to discuss the matter with other directors and
to seek professional advice. Legal advice should be considered if necessary. The accountant
should keep a record of conversations and actions. Resignation should be considered if the
matters cannot be satisfactorily resolved.

Answer No 3:
Query One
The relevant NFRS which deals with operating segments is NFRS 8 – Operating Segments. The
definition of an operating segment in NFRS 8 is based around an entity‘s business model, which
could be different from entity to entity and the disclosures focus on the information which
management believes is important when running the business.

NFRS 8 defines an operating segment as a component of an entity:


– Which engages in business activities from which it may earn revenues and incur expenses,
and
– Whose operating results are regularly reviewed by the chief operating decision maker, and
– For which discrete financial information is available.

The ‗chief operating decision maker‘ is a role rather than a title or it is a function and not
necessarily a person. The role/function is defined around who monitors performance and
allocates resources of the operating segments.

NFRS 8 is only compulsory for listed entities. If we wanted to include information regarding the
operating segments of individual subsidiaries, then we could as NFRS 8 requires judgement in its
application. However, the information in the individual financial statements would either need to
comply with NFRS 8 in all respects or the information cannot be described as ‗segment
information‘.

Query Two
IAS 16 – Property, Plant and Equipment (PPE) – allows (but does not require) entities to revalue
its PPE to fair value. However, it requires that the measurement model used (cost or fair value)
for PPE should be consistent on a class by class basis.

A class of PPE is a grouping of assets of a similar nature and use in an entity‘s operations. Based
on this definition, it is likely that property (or ‗land and buildings‘) would form one distinct class
of PPE and that plant and equipment would form another class. Therefore it is perfectly
consistent with NFRS for property to be measured under the revaluation (fair value) model and
Page 125 of 328
Paper1: Advanced Financial Reporting [RTP 2018 December]
plant and equipment to be measured under the cost model. However, it would be inappropriate
to ‗cherry pick‘ or apply a ‗mixed measurement model‘ to property (or land and buildings) based
simply on its geographical location. This prevents entities only revaluing items which have
increased in value and leaving other items at their (depreciated) cost.

If we do use the fair value model, then we need to make sure we revalue with sufficient
regularity to ensure that the carrying amount of the revalued asset is a true reflection of its
current value.

Query Three
Under the provisions of NAS 38 – Intangible Assets – the ability to recognise an intangible asset
depends on how the potential asset arose. From the perspective of the Megha Group, brand
names generated by Megha Group are internally generated. The recognition criteria for such
potential assets are very stringent and only costs associated with the development phase of an
identifiable research and development project would satisfy them. This explains why the Megha
Group brand names are not recognised.

In contrast, intangible items which relate to an acquired subsidiary which exist at the date of
acquisition are acquired as part of a business combination and for such assets the recognition
criteria are different. Provided the fair value of such an intangible can be reliably measured at the
date of acquisition, it is recognised in the consolidated statement of financial position based on
its fair value at the date of acquisition. The use of the fair value model for intangible non-current
assets is restricted to those assets which are traded in an active market. This is relatively
uncommon in the case of intangibles. It is most unlikely that brand names would be traded in
such a market, so the fair value model is unlikely to be available here.

Answer No 4:
a.
i) In the new public management strategy cognizance should be made of the following
differences between the public sector and the private sector:

 Objectives are different.


Objective of public sector entities is to deliver public goods and services to all citizens in
order to maximize welfare of the citizenry. However, the principal objective of the private
sector entities is to make profit on the goods and services they produce and sold in the
market.

 Means of funding is different.


Public sector entities are financed largely from the public resources resulting from taxes,
levies and other public moneys whereas private sector is financed through capital
contribution of owners such as shares. Therefore, public entities are collectively owned by
the citizens without equity interest but equity interest is the basis of private ownership.

 Accountability structures are different.


Page 126 of 328
Paper1: Advanced Financial Reporting [RTP 2018 December]
Public sector entities are accountable to citizens through parliament but private entities are
accountable to the shareholders through the board of directors.

 The nature of goods and services they offer is different.


Public sector is engaged in the provision of public goods and services which are non-
excludable and nondivisible. Private sector is involved in the provision of private goods that
are rivalry, divisible and discriminatory. Thus, public sector operates in a monopolistic
market while private sector operates in a competitive market.

 The regulatory environment is different.


Public sector entities are established by a specific enactment of Parliament whereas private
sector entities are governed by general commercial enactments and other industry specific
enactments.

ii) Outsourcing refers to the practice where a public sector entity contracts out some of its
supporting or non-essential services to the private vendor to perform for an agreed
consideration. In outsourcing arrangement, the risk is retained by the contracting entity,
which is the public sector entity.

Some argument for outsourcing are that:


 Outsourcing leads to improvement in the delivery of public services through the private
sector due to assumed greater efficiency and effectiveness in private sector.
 Outsourcing will enable DMO to focus on its core functions and activities since the
supporting services will be taken over by a private vendor.
 Outsourcing serves as a medium through which DMO could involve private sector in the
delivery of public services.
 Outsourcing will help DMO to serve cost as it is often cheaper to outsource a service than to
provide it internally

iii) Factors that the management of DMO should consider in making the decision to
outsource some of its functions.
 Legal requirement and government policy.
DMO must consider the enabling law and government policy carefully to determine whether
outsourcing is permitted.
 Cost savings.
DMO will evaluate the decision to outsource in terms of cost by comparing the cost of
internal provision of the service to the cost of outsourcing. Most often, outsourcing is
supported in terms of cost advantage.
 Internal capabilities.
The internal expertise and capabilities of DMO should be considered to determine whether
the existing capabilities are adequate to perform the service internally. For example, it will be
unacceptable for an organisation with very strong human resource function to outsource
recruitment and related function.
 Availability of vendors.
The availability and willingness of private vendors to execute the function is an important
determinant of successful outsourcing.
Page 127 of 328
Paper1: Advanced Financial Reporting [RTP 2018 December]
 Avoidance of conflict of interest.
The management should ensure that all issues of conflict of interest is considered in the
award of outsourcing contract.

b.
i) There are differences in accrual accounting and cash accounting and some of the differences
are that:
 In accrual accounting comprehensive set of financial statement are prepared to measure
financial performance, position and cash condition of the entity. However, cash accounting
reports mainly on cash condition of the entity by emplacing receipt and payment information.
 Under accrual accounting, non-financial assets are capitalised and depreciated over their
useful life span but under cash accounting cost of non-financial assets are written off in the
year of acquisition or construction, hence no depreciation is charged.
 Under cash accounting all obligations of government are disclosure on the statement of
financial position but in the cash accounting system such information is not disclosed on the
financial statement until they are paid.
 Under accrual accounting allowances are permitted for receivables but no such allowances
are accounted for under cash accounting.
 Under accrual accounting revenues are reported when they are earned and expenditures when
incurred but under cash accounting revenues are recognised only when received and
expenditures when actually paid for.

ii) Justifications for adopting accrual accounting are that:


 It provides superior measure of performance in terms of cost of service, efficiency and
effectiveness in service delivery.
 It promotes accountability and transparency in public financial management through
increasing disclosure of all assets and liabilities.
 It provides comprehensive financial information that support decision making and control.
 It ensures general improvement in the quality of financial reporting in the public sector.

iii) Commitment accounting is a technique of accounting in which expenditures are recorded


when decision is made by management to spend on an activity or item. It is a process by
which appropriations are encumbered against future expenditure decisions of management/

Commitment accounting strengthens public financial management in the following ways:


 It ensures that departments do not over spend their appropriation without further
authorisations.
 It also ensures that spending is done within the ambit of a vote hence misapplication
funds may be reduced or curtailed.
 It promotes effective planning of expenditure within the available resources and ensure
that disbursements are synchronised with commitment.

Answer No 5:

(i) Deferred consideration


Page 128 of 328
Paper1: Advanced Financial Reporting [RTP 2018 December]
When calculating goodwill, NFRS 3 Business Combinations states that purchase consideration
should be measured at fair value. For deferred cash consideration, this will be the present value
of the cash flows. This amounts to $29 million ($32m x 0·907). Goodwill arising on acquisition
should be increased by $29 million and a corresponding liability should be recognised:

Dr Goodwill $29 million


Cr Liability $29 million

Interest of $1·5 million ($29m x 5%) should be recorded. This is charged to the statement of
profit or loss and increases the carrying amount of the liability:
Dr Finance costs $1·5 million
Cr Liability $1·5 million

Property, plant and equipment (PPE)


During the measurement period NFRS 3 states that adjustments should be made retrospectively
if new information is determined about the value of consideration transferred, the subsidiary‘s
identifiable net assets, or the non-controlling interest. The measurement period ends no later than
12 months after the acquisition date.

The survey detailed that Pahad‘s PPE was overvalued by $10 million as at the acquisition date. It
was received four months after the acquisition date and so this revised valuation was received
during the measurement period. As such, goodwill at acquisition should be recalculated. As at
the acquisition date, the carrying amount of PPE should be reduced by $10 million and the
carrying amount of goodwill increased by $10 million:

Dr Goodwill $10 million


Cr PPE $10 million

NCI
The NCI at acquisition was valued at $34 million but it should have been valued at $32 million
(($170m – $10m PPE adjustment) x 20%). Both NCI at acquisition and goodwill at acquisition
should be reduced by $2 million:

Dr NCI $2 million
Cr Goodwill $2 million

Goodwill
Goodwill arising on the acquisition of Pahad should have been calculated as follows:
$m
Fair value of consideration ($150m + $29m) 179
NCI at acquisition 32
Fair value of identifiable net assets acquired (160 )
––––
Goodwill at acquisition 51
––––
Goodwill impairment
Page 129 of 328
Paper1: Advanced Financial Reporting [RTP 2018 December]
According to NAS 36 Impairment of Assets, a cash generating unit to which goodwill is allocated
should be tested for impairment annually by comparing its carrying amount to its recoverable
amount. As goodwill has been calculated using the proportionate method, then this must be
grossed up to include the goodwill attributable to the NCI.
$m $m
Goodwill 51
Notional NCI ($51m x 20/80) 12·8
––––––
Total notional goodwill 63·8

Net assets at reporting date:


Fair value at start of period 160
Profit for period 52
––––––
212
––––––
Total carrying amount of assets 275·8
Recoverable amount (250·0)
––––––
Impairment 25·8
––––––

The impairment is allocated against the total notional goodwill. The NCI share of the goodwill
has not been recognized in the consolidated financial statements and so the NCI share of the
impairment is also not recognised. The impairment charged to profit or loss is therefore $20·6
million ($25·8m x 80%) and this expense is all attributable to the equity holders of the parent
company.

Dr Operating expenses $20·6 million


Cr Goodwill $20·6 million

The carrying amount of the goodwill relating to Pahad at the reporting date will be $30·4 million
($51m acquisition – $20·6m impairment).

(ii) Terai
The share sale results in Himal losing control over Terai. The goodwill, net assets and NCI of
Terai must be derecognized from the consolidated statement of financial position. The difference
between the proceeds from the disposal (including the fair value of the shares retained) and these
amounts will give rise to a $47 million profit on disposal. This is calculated as follows:
$m $m
Proceeds 140
Fair value of remaining interest 300
––––
440
Goodwill at disposal (50 )
Net assets at disposal (590 )
Page 130 of 328
Paper1: Advanced Financial Reporting [RTP 2018 December]
NCI:
At acquisition 215
NCI % of post acquisition profit (40% x ($590m – $510m)) 32
––––
NCI at disposal 247
––––
Profit on disposal 47
––––

After the share sale, Himal owns 40% of Terai‘s shares and has the ability to appoint two of the
six members of Terai‘s board of directors. NAS 28 Investments in Associates and Joint Ventures
states that an associate is an entity over which an investor has significant influence. Significant
influence is presumed when the investor has a shareholding of between 20 and 50%.
Representation on the board of directors provides further evidence that significant influence
exists.

Therefore, the remaining 40% shareholding in Terai should be accounted for as an associate. It
will be initially recognized at its fair value of $300 million and accounted for using the equity
method. This means that the group recognises its share of the associate‘s profit after tax, which
equates to $24·6 million ($123m x 6/12 x 40%). As at the reporting date, the associate will be
carried at $324·6 million ($300m + $24·6m) in the consolidated statement of financial position.

(iii) Convertible bond


Himal has issued a compound instrument because the bond has characteristics of both a financial
liability (an obligation to repay cash) and equity (an obligation to issue a fixed number of
Himal‘s own shares). NAS 32 Financial Instruments:

Presentation specifies that compound instruments must be split into:


– a liability component (the obligation to repay cash);
– an equity component (the obligation to issue a fixed number of shares).

The split of the liability component and the equity component at issue date is calculated as
follows:
– the liability component is the present value of the cash repayments, discounted using the
market rate on non‑convertible bonds;
– the equity component is the difference between the cash received and the liability component
at the issue date.

The initial carrying amount of the liability should have been measured at $17·9 million,
calculated as follows:
Date Cash flow Discount rate Present value
$m $m
30 September 20X6 0·8 0·909 0·73
30 September 20X7 20·8 0·826 17·18
––––––
17·91
Page 131 of 328
Paper1: Advanced Financial Reporting [RTP 2018 December]
––––––
The equity component should have been initially measured at $2·1 million ($20m – $17·9m).
The adjustment required is:

Dr Non-current liabilities $2·1m


Cr Equity $2·1m

The equity component remains unchanged. After initial recognition, the liability is measured at
amortised cost, as follows:

1 October 20X5 Finance charge (10%) Cash paid 30 September 20X6


$m $m $m $m
17·9 1·8 (0·8) 18·9

The finance cost recorded for the year was $0·8 million and so must be increased by $1·0 million
($1·8m – $0·8m).
Dr Finance costs $1·0m
Cr Non-current liabilities $1·0m
The liability has a carrying amount of $18·9 million as at the reporting date.

Answer No 6:

Under the principles of NAS 21 – The Effects of Changes in Foreign Exchange Rates – the
purchase of inventory on 1 February 2018 would be recorded using the spot rate of exchange on
that date. Therefore KaKhaGa would recognise a purchase and an associated payable of
$600,000 (3·6 million dinars/6).

KaKhaGa would recognise revenue of $480,000 in the statement of profit or loss because goods
to the value of $480,000 were sold prior to 31 March 2018.

KaKhaGa would recognise $360,000 ($600,000 x 60%) in cost of sales because the revenue of
$480,000 is recognised.

The closing inventory of goods purchased from the foreign supplier would be $240,000
($600,000 – $360,000) and would be recognised as a current asset. This would not be re-
translated since inventory is a non-monetary asset.

The payment of 1,260,000 dinars on 15 March 2018 would be recorded using the spot rate of
exchange on that date, therefore the payment would be recorded at $200,000 (1,260,000
dinars/$6·3).

The closing payable of 2,340,000 dinars (3,600,000 dinars – 1,260,000 dinars) is a monetary
item, therefore would be translated at the rate of exchange in force at the year end (6·4 dinars to
$1). Therefore the closing payable (recorded in current liabilities) would be $365,625 (2,340,000
dinars/$6·4).
Page 132 of 328
Paper1: Advanced Financial Reporting [RTP 2018 December]
The difference between the initially recognised payable ($600,000) and the subsequently
recognized payment ($200,000) is $400,000. Since the closing payable is $365,625 (see above),
KaKhaGa has made an exchange gain of $34,375 ($400,000 – $365,625). This gain is recognised
in the statement of profit or loss, either under other income category or as a reduction in cost of
sales.

Answer No 7:
Following are the discrepancies that are noted in the financial statements of Nepal Ltd.
a. Currently, the statement is termed as ―Statement of financial Position‖ rather than ―Balance
Sheet‖.
b. The reporting currency is not disclosed in the financial statement.
c. Presentation of assets as Net Current Assets is not in line with the framework for presentation
of financial statements which restricts setting off Assets & Liabilities with each other
d. The term ―Fixed Asset‖ used in the financial statement is not correct rather the appropriate
term to be used shall be ―Property, Plant & Equipment‖.
e. Amount spent with regards to development of software that has satisfied recognition criteria
of NAS 38 shall be presented separately as ―Intangible Assets‖ rather than grouping it under
PPE.
f. Assets held for capital appreciation purpose shall be classified as ―Investment Property‖
rather than PPE. In addition, the cost of NRs. 20 incurred for acquisition of the property shall
be included in the cost of the property as per NAS 40.
g. Deferred tax shall be shown as a separate asset as per the provisions of NAS 12. Grouping it
under current asset is not correct.
h. As per NAS 21, the amount of sundry debtors to be recognized in the financial statement is
NRs. 180 (NRs. 90 X 2$). The difference shall be charged as foreign exchange loss in the
Statement of Profit or Loss.

Answer No 8:
Working note 1: Calculation of Basic Earnings Per Share
Basic EPS: Earnings for Equity Share holders/ W.avg. no of shares
=1,800,000/75,000= Rs.24.00 Per share

Calculation of Diluted Earnings per share:


- Identify Potential equity share: (ESOP)
= ESOP- (ESOP× Exercise price/Fair Value)
= 15,000-(15,000×150/200)
= 3,750 Shares
- Incremental EPS: = 0/3,750=0 per share
- Test of Dilution:
Particulars Numerator De-numerator Ratio
BEPS-COO 1,800,000 75,000 24.00
+ ESOP - 3,750
1,800,000 78,750 22.86

- Diluted Earnings Per Share= (1,800,000/78,750)=Rs.22.86 Per share.


Page 133 of 328
Paper1: Advanced Financial Reporting [RTP 2018 December]
Answer No 9:
Government assistance is action by government designed to provide an economic benefit specific
to an entity or range of entity‘s qualifying under certain criteria. Government assistance for the
purpose of this Standard does not include benefits provided only indirectly through action
affecting general trading conditions, such as the provision of infrastructure in development areas
or the imposition of trading constraints on competitors.

The following matters shall be disclosed:


(a) the accounting policy adopted for government grants, including the methods of presentation
adopted in the financial statements;
(b) the nature and extent of government grants recognised in the financial statements and an
indication of other forms of government assistance from which the entity has directly
benefited; and
(c) unfulfilled conditions and other contingencies attaching to government assistance that has
been recognized.

Answer No 10:

a.

Arc – restatement
Initial Adjusts Notes Final
$m $m $m
Non-current assets:
Tangible non-current assets 500 500
Cost of investment in Bend 150 150
Cost of investment in Curve 95 (95) (1)
Loan to Curve 105 (2) 105
Current assets 125 105 (1) 150
(105) (2)
25 (3)
––––– ––––– –––––
870 35 905
––––– ––––– –––––

Equity and liabilities:


Ordinary share capital 100 100
Retained earnings 720 10 (1) 755
25 (3)
––––– ––––– –––––
870 35 855

Non-current liabilities:
Long-term loan 5 5
Current liabilities:
Trade payables 45 45
Page 134 of 328
Paper1: Advanced Financial Reporting [RTP 2018 December]
––––– ––––– –––––
870 35 905
––––– ––––– –––––

Notes: Notes:
(1) Disposal of investment in Curve for $105m, resulting in a profit of $10m.
(2) Long-term loan made to Curve.
(3) Dividend due from Bend.

Bend restatement
Initial Adjusts Notes Final
$m $m $m
Non-current assets:
Tangible non-current assets 200 25 (3) 225
Cost of investment in Curve 105 (1) 105
Current assets 145 (105) (1) 15
(25) (2)
––––– ––––– –––––
345 – 345
––––– ––––– –––––

Equity and liabilities:


Ordinary share capital 100 10 (3) 110
Share premium 11 (3) 11
Retained earnings 230 (25) (2) 205
––––– ––––– –––––
330 (4) 326

Non-current liabilities:
Long-term loan 4 (3) 4
Current liabilities:
Trade payables 15 15
––––– ––––– –––––
345 – 345
––––– ––––– –––––

Notes
(1) Purchase of investment in Curve for $105m.
(2) Dividend due to Arc.
(3) Purchase of land and buildings from Curve – comprising:
$m
Non-voting shares of $1 each 10
Share premium (bal fig) 11
Mortgage liability taken over 4
–––––
25
Page 135 of 328
Paper1: Advanced Financial Reporting [RTP 2018 December]
–––––

3 – Lease obligation as follows:


Bal b/fwd Int @ 10.2% Cash paid Bal c/fwd
$000 $000 $000 $000
Y/end 30/06/X1 3,000 306 (700) 2,606
Y/end 30/06/X2 2,606 266 (700) 2,172
Current liability element = $2,606,000 – $2,172,000 = $434,000

a) Curve – restatement
Initial Adjusts Notes Final
$m $m $m
Non-current assets:
Tangible non-current assets 55 (15.0) (2) 40.0
Lease assets 3.0 (3) 2.5
(0.5) (3)
Cost of investment in Bend 21.0 21.0
Current assets 25 105.0 (1) 129.3
(0.7) (3)
––––– ––––– –––––
80 112.8 192.8
––––– ––––– –––––

Equity and liabilities:


Ordinary share capital 35 35.0
Share premium 8 8.0
Retained earnings 5 10.0 (2) 14.2
(0.5) (3)
(0.3) (3)
––––– ––––– –––––
48 9.2 57.2

Non-current liabilities:
Long-term loan 12 (4.0) 8.0
Loan from Arc 105.0 (1) 105.0
Lease obligation 3.0 (3) 2.2
0.3 (3)
(0.7) (3)
(0.4) (3)
Current liabilities: Lease obligation (0.4) (3) 0.4
Trade payables 20 20.0
––––– ––––– –––––
80 112.8 192.8
––––– ––––– –––––

Notes:
Page 136 of 328
Paper1: Advanced Financial Reporting [RTP 2018 December]
1 – Loan from Arc of $105m.
2 – Sale of land and buildings to Bend as follows:

$m
Disposal proceeds (Mort tfr at + shares at FV $21m) 25
CV of land and buildings 15
–––
Profit on disposal 10
–––

(b) The plan has no impact on the group financial statements as all of the internal transactions
will be eliminated on consolidation but does affect the individual accounts of the companies.
The reconstruction only masks the problem facing Curve. It does not solve or alter the
business risk currently being faced by the group.

A further issue is that such a reorganisation may result in further costs and expenses being
incurred. Note that any proposed provision for restructuring must meet the requirements of
NAS 37 Provisions, Contingent Liabilities and Contingent Assets before it can be included in
the financial statements. A constructive obligation will arise if there is a detailed formal plan
produced and a valid expectation in those affected that the plan will be carried out. This is
normally crystallised at the point when there is communication by the company with those
who are expected to be affected by the plan.

The transactions outlined in the plans are essentially under common control and must be
viewed in this light. This plan overcomes the short-term cash flow problem of Curve and
results in an increase in the accumulated reserves. The plan does show the financial
statements of the individual entities in a better light except for the significant increase in
long-term loans in Curve‘s statement of financial position. The profit on the sale of the land
from Curve to Bend will be eliminated on consolidation. In the financial statements of Curve,
the investment in Bend should be accounted for under NFRS 9. There is now cash available
for Curve and this may make the plan attractive. However, the dividend from Bend to Arc
will reduce the accumulated reserves of Bend but if paid in cash will reduce the current assets
of Bend to a critical level.

The purchase consideration relating to Curve may be a transaction at an overvalue in order to


secure the financial stability of the former entity. A range of values are possible which are
current value, carrying amount or possibly at zero value depending on the purpose of the
reorganisation. Another question which arises is whether the sale of Curve gives rise to a
realised profit. Further, there may be a question as to whether Bend has effectively made a
distribution. This may arise where the purchase consideration was well in excess of the fair
value of Curve. An alternative to a cash purchase would be a share exchange. In this case,
local legislation would need to be reviewed in order to determine the requirements for the
setting up of any share premium account.

b. Certain assets are not covered by the NAS 38, including


Page 137 of 328
Paper1: Advanced Financial Reporting [RTP 2018 December]

• Inventories
• Assets arising from construction contracts
• Deferred tax assets
• Assets arising from employee benefits
• Financial assets
• Investment property carried at fair value
• Biological assets carried at fair value
• Assets arising from insurance contracts
• Assets that classified as held for sale

Answer No 11

(a) Sale and leaseback


Sale and leaseback transactions are accounted for under NAS 17 Leases. If the transfer of
the asset qualifies as a sale then the seller-lessee must measure the right- of-use asset at
the proportion of the previous carrying amount that relates to the rights it retains. This
means that the seller-lessee will recognise a profit or loss based only on the rights
transferred to the buyer-lessor. If the transfer does not qualify as a sale then the seller-
lessee continues to recognise the underlying asset and will recognise a financial liability
equal to the proceeds received.

It would seem that the transfer does represent a sale because William is only leasing the
asset back for a fraction of its remaining useful life and so the buyer-lessor seems to have
obtained control of the underlying asset.

William must initially measure the right-of-use asset at $1.26 million (($1.8m/$5m) ×
$3.5 million).

The lease liability will be initially measured at the present value of the lease payments,
which is $1.80 million.

The entry required is as follows:

Dr Cash $5.00m
Dr Right of use asset $1.26m
Cr Building $3.50m
Cr Lease liability $1.80m
Cr Profit or loss (bal. fig.) $0.96m

The right-of-use asset is depreciated over the five year lease term, because it is shorter
than the remaining useful economic life. This gives a charge of $0.25 million ($1.26m/5).

Dr Depreciation (P/L) $0.25m


Cr Right-of-use asset $0.25m
Page 138 of 328
Paper1: Advanced Financial Reporting [RTP 2018 December]
The carrying amount of the right-of-use asset will be reduced to $1.01 million ($1.26m –
$0.25m). Interest is charged on the liability of $0.13 million ($1.8m × 7%).

Dr P/L $0.13m
Cr Liability $0.13m

The cash payment reduces the liability: Dr Liability $0.44m Cr Cash $0.44m The liability
has a carrying amount of $1.49 million at the reporting date ($1.8m + $0.13m – $0.44m).

(b) Defined benefit plans


NAS 19 Employee Benefits says that the expense in profit or loss for the defined benefit
plan will include:
• the net interest component
• the service cost component.

Net interest component


The net interest component is calculated by applying the discount rate at the start of the
year to the net defined benefit asset or liability at the start of the year. The discount rate
used is a high-quality corporate bond rate where there is a deep market in such bonds,
and a government bond rate in other markets.

The service cost component


The service cost component is charged to profit or loss. It comprises:
• the current service cost
• any past service cost
• any gain or loss on settlement or curtailment.

Past-service costs are recognised as part of the service cost component in the period of a
plan amendment. The plan benefits which were enhanced on 1 June 2011 would have to
be immediately recognised and the unvested benefits would not be spread over five years
from that date.

The transaction with the former employees will produce a loss on settlement. This will be
the difference between the assets transferred and the present value of the obligation
extinguished. Therefore a loss of $0.1m ($0.4m – $0.3m) will be charged to profit or loss
as part of the service cost component.

Recognition of remeasurements
Remeasurements gains and losses are recognised immediately in ‗other comprehensive
income‘ (OCI). Remeasurements recognised in OCI cannot be recycled through profit or
loss in subsequent periods.

Presentation The benefit cost will be split between (i) the cost of benefits accrued in the
current period (service cost) and benefit changes (past-service cost, settlements and
curtailments); and (ii) finance expense or income. This analysis can be in the statement of
profit or loss or in the notes.
Page 139 of 328
Paper1: Advanced Financial Reporting [RTP 2018 December]

Answer No 12:

a) Expenses in respect of cash-settled share-based payment transactions should be recognised


over the period during which goods are received or services are rendered, and measured at
the fair value of the liability. The fair value of the liability should be remeasured at each
reporting date until settled. Changes in fair value are recognised in profit or loss.

The fair value of each share appreciation right (SAR) is made up of an intrinsic value and its
time value. The time value reflects the fact that the holders of each SAR have the right to
participate in future gains. Although the scheme vested in the previous year, there is a
liability to pay cash to the SAR holders who have not yet exercised their rights.

This liability must be remeasured at the year-end giving rise to an additional expense:
Liability 31 May 2011 (17 × 500 × $14) $119,000
Cash paid (7 × 500 × $21) ($73,500)
Expense (bal. fig.) $74,500
––––––––
Liability 31 May 2012 (10 × 500 × $24) $120,000
––––––––

Therefore the expense for the year is $74,500 and the liability at the year end is $120,000.

b) NAS 37 Provisions, Contingent Liabilities and Contingent Assets describes contingent


liabilities in two ways.
 Firstly, as reliably possible obligations whose existence will be confirmed only on the
occurrence or non-occurrence of uncertain future events outside the entity‘s control, and
 Secondly, as present obligations that are not recognised because:
– it is not probable that an outflow of economic benefits will be required to settle the
obligation, or
– the amount cannot be measured reliably.

Treatment in the individual financial statements


In Chrissy‘s financial statements contingent liabilities are not recognised but are disclosed
and described in the notes to the financial statements. The disclosure should include an
estimate of their potential financial effect and uncertainties relating to the amount or timing
of any outflow, unless the possibility of settlement is remote.

Treatment in the consolidated financial statements


In a business combination, a contingent liability is recognised at the acquisition date if it
meets the definition of a liability and if it can be measured. This means William would
recognise a liability of $4 million in the consolidated accounts. This will increase the
goodwill arising on the acquisition of Chrissy.
Page 140 of 328
Paper1: Advanced Financial Reporting [RTP 2018 December]
Answer No 13:

(a) According to NFRS 1 First Time Adoption of International Financial Reporting Standards,
assets carried at cost (e.g. property, plant and equipment) may be measured at their fair value
at the date of the opening NFRS statement of financial position. Fair value becomes the
‗deemed cost‘ going forward. Deemed cost is an amount used as a surrogate for cost or
depreciated cost at a given date.

NFRS 13 Fair Value Measurement defines the fair value of an asset as ‗the price that would
be received to sell an asset or paid to transfer a liability in an orderly transaction
between market participants at the measurement date‘ (NFRS 13). Fair value is a
market-based measurement, not an entity-specific measurement.

In NFRS 13, fair value measurements are categorised into a three-level hierarchy based on
the type of inputs and are not based on a valuation method. This is as follows:
• Level 1 inputs are „unadjusted quoted prices in active markets for items identical to
the asset being measured‟.
• Level 2 inputs are ‗inputs other than quoted prices in active markets included within
Level 1 that are directly or indirectly observable‟.
• Level 3 inputs are unobservable inputs that are usually determined based on
management‘s assumptions.

The selling agents‘ estimates provided very little information about the valuation methods
and underlying assumptions. It may be that the valuations are comprised of level 2 inputs but,
dependent on the level of adjustment made to observable prices, they may be based on level
3 inputs. Level 3 inputs should not be used if level 1 or 2 inputs are available. It is therefore
vital to understand more clearly the valuation methods adopted.

It may not be prudent to value the boats at the average of the higher end of the range of
values, particularly if these values involve a large degree of judgement.

Therefore, Lockfine was not in breach of NFRS 1 but it is unclear whether fair value has
been determined in accordance with NFRS 13.

(b) In accordance with NFRS 1, an entity which, during the transition process to Nepal Financial
Reporting Standards, decides to retrospectively apply NFRS 3 Business Combinations to a
certain business combination must apply that decision consistently to all business
combinations occurring between the date on which it decides to adopt NFRS 3 and the date
of transition. The decision to apply NFRS 3 cannot be made selectively.

The entity must consider all similar transactions carried out in that period; and when
allocating values to the various assets (including intangibles) and liabilities of the entity
acquired in a business combination to which NFRS 3 is applied, an entity must necessarily
have documentation to support its purchase price allocation. If there is no such basis,
alternative or intuitive methods of price allocation cannot be used unless they are based on
the strict application of the standards.
Page 141 of 328
Paper1: Advanced Financial Reporting [RTP 2018 December]

NAS 38 Intangible Assets requires an entity to recognise an intangible asset, whether


purchased or self-created (at cost) if, and only if:
• it is probable that the future economic benefits that are attributable to the asset will flow
to the entity; and
• the cost of the asset can be measured reliably.

Lockfine was unable to obtain a reliable value for the fishing rights, and thus it was not
possible to them to be separately recognised. The rights have therefore been correctly
subsumed within goodwill.

The goodwill arising on acquisition should be accounted for in accordance with NAS 36
which requires an annual impairment test.

Answer No 14:

(i) To determine whether a financial instrument should be classified as debt or equity, NAS 32
Financial Instruments: Presentation uses principles-based definitions of a financial liability
(debt) and of equity.

The key feature of debt is that the issuer is obliged to deliver either cash or another financial
asset to the holder. The contractual obligation may arise from a requirement to repay
principal or interest or dividends. For example, a bond which requires the issuer to make
interest payments and redeem the bond for cash is classified as debt.

In contrast, equity is any contract which evidences a residual interest in the entity‘s assets
after deducting all of its liabilities. A financial instrument is normally an equity instrument if
the instrument includes no contractual obligation to deliver cash or another financial asset to
another entity. For example, ordinary shares, where all the payments are at the discretion of
the issuer, are classified as equity of the issuer.

However, a contract may involve the receipt or delivery of the entity‘s own equity
instruments. The classification of this type of contract as debt or equity is dependent on
whether there is variability in either the number of equity shares delivered or variability in
the amount of cash or financial assets received. A contract which will be settled by the entity
receiving or delivering a fixed number of its own equity instruments in exchange for a fixed
amount of cash or another financial asset is an equity instrument. However, if there is any
variability in the amount of cash or own equity instruments which will be delivered or
received, then such a contract is a financial asset or liability as applicable.

For example, where a contract requires the entity to deliver as many of the entity‘s own
equity instruments as are equal in value to a certain amount of cash, the holder of the contract
would be indifferent as to whether it received cash or shares to the value of that amount.
Thus this contract would be treated as debt.
Page 142 of 328
Paper1: Advanced Financial Reporting [RTP 2018 December]
(ii) Reporting a financial instrument as debt rather than equity would lead to a deterioration in
the gearing ratio. This may make the company appear more risky to potential investors or
lenders, potentially creating problems when raising further finance.

Liability classification normally results in the servicing of the finance being treated as
interest and charged to profit or loss. Finance costs in profit or loss will reduce earnings per
share, which may lower investor confidence in the entity. Lower profits may also lead to a
breach of loan covenants, potentially triggering the need to repay borrowings.

Equity classification may avoid these impacts, but could be perceived negatively if seen to be
diluting existing equity interests.

Answer No 15:

Cavor
The B shares of Cavor should be classified as equity as there is no contractual obligation to pay
the dividends or to call the instrument. Dividends can only be paid on the B shares if dividends
have been declared on the A shares. However, there is no contractual obligation to declare A
share dividends.

The classification of the B share options in Cavor is dependent on whether there is variability in
either the number of equity shares delivered or variability in the amount of cash or financial
assets received. As the contract will be settled by the entity issuing a fixed number of its own
equity instruments in exchange for a fixed amount of cash, then the share options are classified
as an equity instrument.

Lidan
An obligation to settle in cash can be established indirectly through the terms and conditions of
the financial instrument.

In this case, the value of the own share settlement alternative substantially exceeds that of the
cash settlement option, meaning that the entity is implicitly obliged to redeem the option for a
cash amount of $1 per share. The B shares of Lidan will therefore be classified as a liability.

Answer No 16:
(i) Information is material if omitting it or misstating it could influence decisions which users
make on the basis of financial information about a specific reporting entity. Materiality is an
entity-specific aspect of relevance, based on the nature and/or magnitude of the items to
which it relates in the context of the entity‘s financial report. It is therefore difficult to
specify a uniform quantitative threshold for materiality or predetermine what could be
material in a particular situation.

Materiality should ensure that relevant information is not omitted or mis-stated and it should
help filter out obscure information which is not useful to users of financial statements.
Page 143 of 328
Paper1: Advanced Financial Reporting [RTP 2018 December]
The Conceptual Framework describes materiality as an application of relevance by a
particular entity. When an entity is assessing materiality, it is assessing whether the
information is relevant to the readers of its own financial statements. Information relevant for
one entity might not be as relevant for another entity. IAS 1 Presentation of Financial
Statements says that an entity need not provide a specific disclosure required by IFRS if the
information is not material and that the application of IFRSs, with additional disclosure when
necessary, is presumed to result in financial statements which achieve a fair presentation. In
other words, material information must be disclosed irrespective of whether there is an
explicit disclosure requirement.

Although preparers may understand the concept of materiality, they may be less certain about
how it should be applied. Preparers may be reluctant to filter out information which is not
relevant to users as auditors and regulators may challenge their reasons for the omissions.
The way in which some IFRSs are drafted suggests that their specific requirements override
the general statement in IAS 1 that an entity need not provide information which is not
material. Standards are often complied with rigidly which leads to ‗boiler plate‘ disclosures.
Regulators are not keen to encourage the use of judgement but rather wish compliance with
IFRS. If the concept of materiality was applied successfully, then immaterial information
would be removed and the performance and the position of the entity would be more visible.
Investors require better, more relevant, material information. The time and resources
available in a reporting cycle may mean that many preparers are not capable or willing to
make a materiality judgement. Regulators are often unwilling to apply a principle-based
view, and are often quick to raise a query when a disclosure has been removed or not
included.

Accounting policy disclosures often repeat information which was contained in IFRS and are
not entity-specific. Information outside of financial statements cannot be policed and often
key information is given in corporate presentations or outside the financial statements, which
reinforces the perception that financial statements are a compliance document.

(ii) Integrated reporting (IR) takes a broader view of business reporting, emphasising the need
for entities to provide information to help investors assess the sustainability of their business
model. IR is a process which results in communication, through the integrated report, about
value creation over time. An integrated report is a concise communication about how an
organisation‘s strategy, governance, performance and prospects lead to the creation of value
over the short, medium and long term. The materiality definition for IR purposes would
consider that material matters are those which are of such relevance and importance that they
could substantively influence the assessments of the intended report users. In the case of IR,
relevant matters are those which affect or have the potential to affect the organisation‘s
ability to create value over time. For financial reporting purposes, the nature or extent of an
omission or misstatement in the organisation‘s financial statements determines relevance.
Matters which are considered material for financial reporting purposes, or for other forms of
reporting, may also be material for IR purposes if they are of such relevance and importance
that they could change the assessments of providers of financial capital with regard to the
organisation‘s ability to create value. Another feature of materiality for IR purposes is that
the definition emphasises the involvement of senior management and those charged with
Page 144 of 328
Paper1: Advanced Financial Reporting [RTP 2018 December]
governance in the materiality determination process in order for the organisation to determine
how best to disclose its value creation development in a meaningful and transparent way.

Answer No 17:

a) Financial Comptroller General


Financial Comptroller General Office (FCGO) is the main government agency responsible
for the treasury operation of Government of Nepal. This office is under the Ministry of
Finance and is headed by Financial Comptroller General who is a special class officer of
Government of Nepal .

The organizational set up of FCGO consists of 3 divisions and 15 sections. The field level
offices under FCGO are spread all over 77 districts of the country. In each district there is a
District Treasury Controller Office (DTCO) which is involved in releasing budgets to
government offices, budgetory controls and reporting.

FCGO is responsible for overseeing all government expenditure against budget, tracking
revenue collection and other receipts and preparation of consolidated financial statements of
the government. Its responsibility also includes ensuring maintenance of basic accounts
keeping of the government through the accounting personnel and staff recruited and
administered by it. Its functions also cover conducting of the internal audit of revenue and
expenditure of the government. Other important responsibility of FCGO include ensure
timely repayment of internal and external debts, investing in the loan and equity of public
enterprises and maintaining the records related to these financial transactions. It also
manages the distribution of pension to retired government employees and maintains
necessary records thereof.

b) Nepal Public Sector Accounting Standards


Accounting Standards Board (ASB) is entrusted by the Government of Nepal with the
responsibility to develop accounting standards for public sector. The ASB has developed
Nepal Public Sector Accounting Standards (NPSAS) for Financial Reporting under Cash
Basis of Accounting in line with the International Public Sector Accounting Standards
prepared by International Public Sector Accounting Standards Board (IPSASB), an
independent standard-setting body within the International Federation of Accountants
(IFAC).

Structure of the Standards This Standard comprises two parts:


 Part 1 is mandatory. It sets out the requirements which are applicable to all entities
preparing general purpose financial statements under the cash basis of accounting. It
defines the cash basis of accounting, establishes requirements for the disclosure of
information in the financial statements and supporting notes, and deals with a number of
specific reporting issues. The requirements in this part of the Standard must be complied
with by entities which claim to be reporting in accordance with the Nepal Public Sector
Accounting Standards Financial Reporting Under The Cash Basis of Accounting.
Page 145 of 328
Paper1: Advanced Financial Reporting [RTP 2018 December]

 Part 2 is not mandatory. It identifies additional accounting policies and disclosures that an
entity is encouraged to adopt to enhance its financial accountability and the transparency
of its financial statements. It includes explanations of alternative methods for presenting
certain information.

Answer No 18:

A temporary difference is the difference between the carrying amount of an asset or liability
and its tax base.

Examples of temporary differences include (but are not restricted to):


 Tax deductions for the cost of non-current assets that have a different pattern to the write-
off of the asset in the financial statements.
 Different rate of depreciation as per tax and as per accounts
 Pension liabilities that are accrued in the financial statements, but are allowed for tax only
when the contributions are made to the pension fund at a later date.
 Assets are revalued upwards in the financial statements, but no adjustment is made for tax
purposes.
 Foreign exchange gain or loss on revaluation of monetary assets and liabilities as at
reporting date whereas not accounted for tax purpose unless realized.

Answer No 19:

Insurance contract as a "contract under which one party (the insurer) accepts significant
insurance risk from another party (the policyholder) by agreeing to compensate the policyholder
if a specified uncertain future event (the insured event) adversely affects the policyholder."

The following are examples of contracts that are insurance contracts, if the transfer of insurance
risk is significant:

 Insurance against theft or damage to property


 Insurance against product liability, professional liability, civil liability or legal expenses
 Life insurance and prepaid funeral expenses
 Life-contingent annuities and pensions
 Disability and medical cover
 Surety bonds, fidelity bonds, performance bonds and bid bonds
 Credit insurance that provides for specified payments to be made to reimburse the holder
for a loss it incurs because a specified debtor fails to make payment when due
 Product warranties (other than those issued directly by a manufacturer, dealer or retailer)
 Title insurance
 Travel assistance
 Catastrophe bonds that provide for reduced payments of principal, interest or both if a
specified event adversely affects the issuer of the bond
Page 146 of 328
Paper1: Advanced Financial Reporting [RTP 2018 December]
 Insurance swaps and other contracts that require a payment based on changes in climatic,
geological or other physical variables that are specific to a party to the contract
 Reinsurance contracts.

The following are examples of items that are not insurance contracts:
 Investment contracts that have the legal form of an insurance contract but do not expose
the insurer to significant risk
 Contracts that pass all significant insurance risk back to the policyholder
 Self-insurance i.e. retaining a risk that could have been covered by insurance
 Gambling contracts
 Derivatives that expose one party to financial risk but not insurance risk
 A credit-related guarantee
 Product warranties issued directly by a manufacturer, dealer or retailer
 Financial guarantee contracts accounted for under IAS 39 Financial Instruments:
Recognition and Measurement.

Answer No 20:

Importance of related party disclosures


 Investors invest in a business on the assumption that it aims to maximize its own profits for
the benefit of its own shareholders. This means that all transactions have been negotiated at
arm's length between willing and informed parties. The existence of related parties may
encourage directors to make decisions for the benefit of another entity at the expense of their
own shareholders. This can be done actively by selling goods and services cheaply to related
parties, or by buying in goods and services at an above market price. It can also happen when
directors chose not to compete with a related party, or offer guarantees or collateral for other
party's loans.
 Disclosure is particularly important when a business is being sold. It may receive a lot of
custom, supplies, services or general help and advice from family or group companies. When
the company is sold these benefits may be withdrawn.
 Related party transactions are not illegal, nor are they necessarily a bad thing. However
shareholders and potential investors need to be informed of material related party
transactions in order to make informed investment and stewardship decisions.

Answer No 21:

i) Classification as held for sale is a non-adjusting event after the reporting period (NAS 10).
Therefore the definition of a discontinued operation is not met as the assets are neither
discontinued in the period nor classified as held for sale at the year end.

ii) A reportable operating segment closed down during the year is likely to meet the
definition of a discontinued operation because it represents a major part of the entity's
business and it was disposed of during the entity's accounting period.
Page 147 of 328
Paper1: Advanced Financial Reporting [RTP 2018 December]

iii) For an operation not yet sold or abandoned to meet the definition of a discontinued operation,
it must meet the NFRS 5 held for sale criteria. All of these criteria were met at the
previous year end, however at the current year end the operation was not sold within 12
months of classification and the period is expected to be extended well past this. However,
where the period is extended by an event beyond the entity's control such as this NFRS 5
permits classification as held for sale (and therefore treatment as a discontinued operation) to
continue provided the other criteria are still met, which appears to be the case here.
Page 148 of 328
Paper1: Advanced Financial Reporting [RTP 2019 June]

REVISION QUESTIONS
Question No. 1
(a) Explain the term “Conceptual Framework” in relation to Nepal Financial Reporting Standards
(NFRS).
(b) Define assets and liabilities.
(c) The Accounting Standards Board’s framework for the preparation of financial statements
requires that entities should comply with certain accounting concepts and underlying
assumptions which include:
(i) Substance over form;
(ii) Materiality;
(iii) Comparability; and
(iv) Going concern.
Explain briefly the meaning of these concepts.
(d) Discuss the information needs of the following users of a company’s financial statements:
(i) Lenders;
(ii) Suppliers;
(iii) Customers ;
(iv) Employees; and
(v) Government and its agencies.

Question No. 2
Marrie starts in business on 1 January Year 1. Marrie’s sole shareholder contributed capital of Rs.
1,000. Marrie purchased one item of inventory for Rs. 1,000 and sold that inventory for cash of Rs.
1,400. At the end of Year 1 the replacement cost of the same item of inventory is Rs. 1,100.
General inflation during the year was 7%.

Required
Calculate the profit for the year and set out a summary statement of financial position as of 31
December Year 1 under the following capital maintenance concepts.
(a) Physical capital maintenance
(b) Financial capital maintenance
i. Historical cost accounting
ii. Constant purchasing power accounting

Question No. 3
Nozal Limited is engaged in the manufacturing of specialized spare parts for automobile
assemblers. During the year 2018, the company has undertaken the following transactions with its
related parties:
I. Sales of Rs. 500 million were made to its only subsidiary M/s Aago Motors Limited (AML).
Being the subsidiary, a special discount of Rs. 25 million was allowed to AML.
II. AML returned spare parts worth Rs. 5.5 million.
III. Raw materials of Rs. 5 million were purchased from Lal Enterprises, which is owned by the
wife of the CFO of Nozal Limited.
IV. Equipment worth Rs. 3 million was purchased from Auto Limited (AL). The wife of the
Production Director of the company is a director in AL.
V. The company awarded a contract for supply of two machines amounting to Rs. 7 million
per machine to an associated company.
Page 149 of 328
Paper1: Advanced Financial Reporting [RTP 2019 June]
VI. In 2016, an advance of Rs. 2 million was given to the Chief Executive of the company.
During the year 2018, he repaid of Rs.0.3 million. The balance outstanding as on
December 31, 2018 was Rs. 1,100,000.

Required
Prepare a note for inclusion in the company’s financial statements in accordance with the
requirement of NAS 24: Related Party Disclosures.

Question Number 4
During the year ended June 30, 2018, Banjhyang Limited (BL) has carried out several transactions
with the following individuals/entities:
I. AK Associates provides information technology services to BL. One of the directors of BL is
also the partner in AK Associates.
II. SS Bank Limited is the main lender. By virtue of an agreement it has appointed a nominee
director on the Board of BL.
III. Mr. Zoe who supplies raw materials to BL, is the brother of the Chief Executive Officer of
the company.
IV. JB Limited is the distributor of BL’s products and has exclusive distribution rights for the
province 3.
V. Mr. Tee is the General Manager-Marketing of BL and is responsible for all major decisions
made in respect of sales prices and discounts.
VI. BL’s gratuity fund is administered by the Trustees appointed by the company.
VII. MM Limited is the leading supplier of BL and supplies 60% of BL’s raw materials.
VIII. Ms. Lee who conducted various training programmes for the employees of the company,
is the wife of BL’s Chief Executive Officer.

Required
Comment as to whether the above individuals/entities are ‘related parties’ of the company or not.
Support your arguments with references from International Accounting Standards.

Question Number 5
SG Industries Ltd has recently acquired four large subsidiaries. These subsidiaries manufacture
products which are of different lines from those of the parent company. The parent company
manufactures plastics and related products whereas the subsidiaries manufacture the following:
Product Location
Subsidiary 1 Textiles Kakarvitta
Subsidiary 2 Car Products Lumbini
Subsidiary 3 Fashion garments Palpa
Subsidiary 4 Furniture items Morang
The directors have purchased these subsidiaries in order to diversify their product base but do not
have any knowledge of the information required in the financial statements regarding these
subsidiaries other than the statutory requirements.

Required
(a) Explain to the directors the purpose of segmental reporting of financial information.
(b) Explain to the directors the criteria which should be used to identify the separate
reportable segments. (You should illustrate your answer by reference to the above
information)
Page 150 of 328
Paper1: Advanced Financial Reporting [RTP 2019 June]
(c) Critically evaluate NFRS 8, Operating segments, setting out any problems with the
standard.

Question Number 6
MML is engaged in manufacturing of spare parts for motor car assemblers. The audited financial
statements for the year ended December 31, 2017 disclosed that the profit and retained earnings
were Rs. 21 million and Rs. 89 million respectively. The draft financial statements for the year
show a profit of Rs. 15 million. However, following adjustments are required to be made:

(i) The management of the company has decided to change the method for valuation of raw
materials from FIFO to weighted average. The value of inventory under each method is
as follows:
in Rs. million
FIFO Weighted
Average
December 31, 2016 37.0 35.5
December 31, 2017 42.3 44.5
December 31, 2018 58.4 54.4

(ii) In 2017, the company purchased a plant for Rs. 100 million. Depreciation on plant was
recorded at Rs. 25 million instead of Rs. 10 million. This error was discovered after the
publication of financial statements for the year ended December 31, 2017. The error
is considered to be material.

Required
Produce an extract showing the movement in retained earnings, as would appear in the statement
of changes in equity for the year ended December 31, 2018.

Question Number 7
a. NEX Ltd owns and operates radio stations. The main revenue stream is advertising
revenue. Contracts are signed with various businesses for the sale of airtime. The account
executives obtain these contracts and are compensated through a 5% commission on the
total contract price for each new contract signed. Executive B has obtained a new two-
year advertising contract with Company. Total contract costs related to this contract are
as follows:
In Rs.
Legal fees contract drafting 10,000
Commission paid to the account executive 7,500
Meals and entertainment incurred during the sales process 1,750
Creative Director’s time salary allocation of Creative Director to 1,500
develop on-air ad
Actors amounts paid to external actors to record the on-air ad 750
Total Costs 21,500

Required

Discuss whether to capitalize or expense out each cost component?


Page 151 of 328
Paper1: Advanced Financial Reporting [RTP 2019 June]
b. A customer enters into a contract with a heavy duty machine manufacturer for the
purchase of a tractor for Rs.10 million. All pieces of equipment sold by the manufacturer
come with a one-year standard warranty that specifies the equipment will comply with
the agreed-upon specifications and will operate as promised for a one-year period from
the date of purchase.

In signing this contract, the customer also requested to purchase an additional


Rs.200,000 two-year warranty commencing after the expiry of the standard one year
warranty.

Required
Explain how would you treat the warranty?

Question Number 8
On July 1, 2015, Queen Steel Limited (QSL) signed an agreement with People Construction Limited
for construction of a factory building at a cost of Rs. 100 million. It was agreed that the factory
would be ready for use from January 1, 2017. The terms of payments were agreed as under:

(i) 10% advance payment would be made on signing of the agreement. The advance
paid would be adjusted at 10% of the quarterly progress bills.
(ii) 5% retention money would also be deducted from the progress bills. Retention
money will be refunded one year after completion of the factory building.
(iii) Progress bills will be raised on last day of each quarter and settled on 15th of the
next month.

The under mentioned progress bills were received and settled by QSL as per the agreement:

On April 30, 2016 an invoice of Rs. 1.5 million was raised by the contractor for damages sustained
at the site, on account of rains. After negotiations, QSL finally agreed to make additional payment
of Rs. 1.0 million to compensate the contractor. The amount was paid on May 15, 2016. It is
expected that 75% of the payment would be recovered from the insurance company.

The cost of the project has been financed through the following sources:
(i) Issue of right shares amounting to Rs. 15 million, on September 1, 2015. The
company has been following a policy of paying dividend of 20% for the past many
years.
(ii) Bank loan of Rs. 25 million obtained on December 1, 2015. The loan carries a
markup of 13% per annum. The principal is repayable in 5 half yearly equal
installments of Rs. 5 million each along with the interest, commencing from May
31, 2016. Loan processing charges of Rs.0.5 million were deducted by the bank at
the time of disbursement of loan. Surplus funds, when available, were invested in
short term deposits at 8% per annum.
(iii) Cash withdrawals from the existing running finance facility provided by a bank.
Average running finance balance for the year was Rs. 60 million. Markup charged
by the bank for the year was Rs. 9 million.

Required
Page 152 of 328
Paper1: Advanced Financial Reporting [RTP 2019 June]
Compute cost of capital work in progress for the factory building as of June 30, 2016 in accordance
with the requirements of relevant IFRSs.

(Borrowing costs calculations should be based on number of months)

Question Number 9
The assistant financial controller of the ABC Associates Ltd group has identified the matters below
which she believes may indicate impairment of one or more assets:

(a) ABC Associates Ltd owns and operates an item of plant that cost Rs. 640,000 and had
accumulated depreciation of Rs. 400,000 at 1 October 2015. It is being depreciated at
121⁄2% on cost. On 1 April 2016 (exactly half way through the year) the plant was
damaged when a factory vehicle collided into it. Due to the unavailability of replacement
parts, it is not possible to repair the plant, but it still operates, albeit at a reduced capacity. It
is also expected that as a result of the damage the remaining life of the plant from the date
of the damage will be only two years. Based on its reduced capacity, the estimated present
value of the plant in use is Rs. 150,000. The plant has a current disposal value of Rs. 20,000
(which will be nil in two years’ time), but ABC Associates Ltd has been offered a trade-in
value of Rs. 180,000 against a replacement machine which has a cost of Rs. 1 million (there
would be no disposal costs for the replaced plant). ABC Associates Ltd is reluctant to replace
the plant as it is worried about the long-term demand for the product produced by the
plant. The trade-in value is only available if the plant is replaced.

Required
Prepare extracts from the statement of financial position and statement of profit or loss of ABC
Associates Ltd in respect of the plant for the year ended 30 September 2016.

Your answer should explain how you arrived at your figures.

(b) On 1 April 2015 ABC Ltd acquired 100% of the share capital of XYZ Limited, whose only
activity is the extraction and sale of spa water. XYZ Limited had been profitable since its
acquisition, but bad publicity resulting from several consumers becoming ill due to a
contamination of the spa water supply in April 2016 has led to unexpected losses in the last
six months. The carrying amounts of XYZ Limited’s assets at 30 September 2016 are:
Rs.000
Brand (XYZ Spring – see below) 7,000
Land containing spa 12,000
Purifying and bottling plant 8,000
Inventories 5,000
32,000

The source of the contamination was found and it has now ceased.

The company originally sold the bottled water under the brand name of ‘XYZ Spring’, but because
of the contamination it has re-branded its bottled water as ‘Refresh’. After a large advertising
campaign, sales are now starting to recover and are approaching previous levels. The value of the
brand in the balance sheet is the depreciated amount of the original brand name of ‘XYZ Spring’.
Page 153 of 328
Paper1: Advanced Financial Reporting [RTP 2019 June]

The directors have acknowledged that Rs. 1.5 million will have to be spent in the first three
months of the next accounting period to upgrade the purifying and bottling plant.

Inventories contain some old ‘XYZ Spring’ bottled water at a cost of Rs. 2 million; the remaining
inventories are labelled with the new brand ‘Refresh’. Samples of all the bottled water have been
tested by the health authority and have been passed as fit to sell. The old bottled water will have
to be relabeled at a cost of Rs. 250,000, but is then expected to be sold at the normal selling price
of (normal) cost plus 50%.

Based on the estimated future cash flows, the directors have estimated that the value in use of
XYZ Limited at 30 September 2016, calculated according to the guidance in NAS 36, is Rs. 20
million. There is no reliable estimate of the fair value less costs to sell of XYZ Limited.

Required
Calculate the amounts at which the assets of XYZ Limited should appear in the consolidated
statement of financial position of ABC Associates Ltd at 30 September 2016. Your answer should
explain how you arrived at your figures.

Question Number 10
Prime is a listed company with a year end of 31 December. It operates two businesses, the first is
the rental of luxury yachts and the second is a chain of luxury holiday villas in Europe. The
directors have requested your advice on the following matters.

Holiday villas
Prime’s policy is to carry the holiday villas at their re-valued amount, which, based on the most
recent valuation in 20X0, was Rs. 20m (historical cost was Rs. 10m). Prime is unsure how
frequently a revaluation of such properties is required and so has instructed a surveyor to provide
an up-to-date valuation as at 31 December Year 4. This valuation has provided the following
information:

Rs. Million
Replacement cost 17
Value in use 28
Open market value 25

One of the villas has received very few bookings over the past two years and so a decision was
reached to exclude it from the Year 5 brochure. It is currently up for sale. The villa has a carrying
value of Rs. 1.25m. Its value in use is only Rs.0.85m and its expected market value is Rs. 1m,
before expected agents and solicitor’s fees of Rs. 50,000. The directors are unsure as to the
accounting treatment of this villa. A number of potential buyers have expressed an interest in the
property, and it is hoped that a deal will be negotiated in the first few months of Year 5.

Prime’s accounting policy is to not charge depreciation on the villas. It’s justification is that the
villas are maintained to a high standard and have useful lives of at least 50 years.

Head Office
Page 154 of 328
Paper1: Advanced Financial Reporting [RTP 2019 June]
Over the past two years, Prime has built its own head office. Construction began on 1 October
Year 2 and finished on 1 June Year 4, although minor modifications meant that the company did
not relocate until 1 September Year 4.

The site cost Rs. 1m and the costs of construction were a further Rs. 8m. Prime took out a two
year loan of Rs. 5m on 1 October Year 2, at an interest rate of 9% per annum, to help fund the
work. In order to encourage businesses to operate in areas of high unemployment, the
government offered a Rs. 1.5m grant towards the cost of construction. The terms of settlement
were that payment would only be made upon completion of the building once a government
inspection had taken place. This inspection had not taken place by the year end, but Prime is
confident that the grant will be received shortly after the year end.

The company intends to use the head office for the next 50 years and, as for the villas, does not
intend to depreciate the land or buildings.

Yachts
Prime has spent the past year designing a new range of luxury yachts. Work was completed on 1
April Year 4 at a cost of Rs. 20m. During the construction, the economy took a downturn and the
company now believes that the market value of the yachts is only Rs. 17m, although the value in
use is estimated to be Rs. 18m. The engines of the yachts have a three year life, the interior has a
two year life, and the remainder should have a life of 15 years. The engine cost is believed to
represent 15% of the total cost of manufacture and the interior approximately 25%.

Required
Explain the accounting issues relating to the villas, head office and yachts, referring to relevant
NFRS guidance. Numerical information, where possible, relating to the 31 December Year 4
financial statements should be provided.

Question Number 11
Himal is a diverse group with many subsidiaries. The group is proud of its reputation as a ‘caring’
organisation and has adopted various ethical policies towards its employees and the wider
community in which it operates. As part of its Annual Report, the group publishes details of its
environmental policies, which include setting performance targets for activities such as recycling,
controlling emissions of noxious substances and limiting use of non-renewable resources.

The finance director is reviewing the accounting treatment of various items prior to finalising the
accounts for the year ended 31 March 20X4. All items are material in the context of the accounts
as a whole. The accounts are due to be approved by the directors on 30 June 20X4.

Closure of factory
On 15 February 20X4, the board of Himal decided to close down a large factory in Gorkha. The
board is trying to draw up a plan to manage the effects of the reorganisation, and it is envisaged
that production will be transferred to other factories. The factory will be closed on 31 August
20X4, but at 31 March 20X4 this decision had not yet been announced to the employees or to any
other interested parties. Costs of the reorganisation have been estimated at Rs. 45 million

Relocation of subsidiary
Page 155 of 328
Paper1: Advanced Financial Reporting [RTP 2019 June]
During December 20X3, one of the subsidiary companies moved from Pokhara to Chitwan in order
to take advantage of government development grants. Its main premises in Pokhara are held
under an operating lease, which runs until 31 March 20X9. Annual rentals under the lease are Rs.
10 million. The company is unable to cancel the lease, but it has let some of the premises to a
charitable organisation at a nominal rent. The company is attempting to rent the remainder of the
premises at a commercial rent, but the directors have been advised that the chances of achieving
this are less than 50%.

Legal claim
During the year to 31 March 20X4, a customer started legal proceedings against the group,
claiming that one of the food products that it manufactures had caused several members of his
family to become seriously ill. The group’s lawyers have advised that this action will probably not
succeed.

Environmental impact of overseas subsidiary


The group has an overseas subsidiary that is involved in mining precious metals. These activities
cause significant damage to the environment, including deforestation. The company expects to
abandon the mine in eight years’ time. The mine is situated in a country where there is no
environmental legislation obliging companies to rectify environmental damage and it is very
unlikely that any such legislation will be enacted within the next eight years. It has been estimated
that the cost of cleaning the site and re-planting the trees will be Rs. 25 million if the re-planting
was successful at the first attempt, but it will probably be necessary to make a further attempt,
which will increase the cost by a further Rs. 5 million.

Required
Explain how each of the items above should be treated in the consolidated financial statements
for the year ended 31 March 20X4.

Question Number 12
Om is a private limited company and has two 100% owned subsidiaries, Namah and Sibaya, both
themselves private limited companies. Om acquired Sibaya on 1 January 20X2 for Rs. 5 million
when the fair value of the net assets was Rs. 4 million, and the tax base of the net assets was Rs.
3.5 million. The acquisition of Sibaya and Namah was part of a business strategy whereby Om
would build up the value of the group over a three-year period and then list its share capital on
the Stock Exchange.

(a) The following details relate to the acquisition of Sibaya, which manufactures electronic
goods:

(i) Part of the purchase price has been allocated to intangible assets because it relates to the
acquisition of a database of key customers of Sibaya. The recognition and measurement
criteria for an intangible asset under NFRS 3 Business Combinations and NAS 38 Intangible
Assets do not appear to have been met but the directors feel that the intangible asset of
Rs. 500,000 will be allowed for tax purposes and have computed the tax provision
accordingly. However, the tax authorities could possibly challenge this opinion.
(ii) Sibaya has sold goods worth Rs. 3 million to Om since acquisition and made a profit of Rs.
1 million on the transaction. The inventory of these goods recorded in Om ’s statement of
financial position at the year ending 31 May 20X2 was Rs. 1.8 million.
Page 156 of 328
Paper1: Advanced Financial Reporting [RTP 2019 June]
(iii) The retained earnings of Sibaya at acquisition were Rs. 2 million. The directors of Om have
decided that, during the three years leading up to the date that they intend to list the
shares of the company, they will realise earnings through future dividend payments from
the subsidiary amounting to Rs. 500,000 per year. Tax is payable on any remittance of
dividends and no dividends have been declared for the current year.

(b) Namah was acquired on 1 June 20X1 and is a company which undertakes various projects
ranging from debt factoring to investing in property and commodities. The following
details relate to Namah for the year ending 31 May 20X2:

(i) Namah has a portfolio of readily marketable government securities which are held as
current assets. These investments are stated at market value in the statement of financial
position with any gain or loss taken to profit or loss. These gains and losses are taxed
when the investments are sold. Currently the accumulated unrealised gains are Rs. 4
million.
(ii) Namah has calculated that it requires a general allowance of Rs. 2 million against its total
loan portfolio. Tax relief is available when the specific loan is written off. Management
feel that this part of the business will expand and thus the amount of the general
provision will increase.
(iii) When Om acquired Namah it had unused tax losses brought forward. At 1 June 20X1, it
appeared that Namah would have sufficient taxable profit to realise the deferred tax asset
created by these losses but subsequent events have proven that the future taxable profit
will not be sufficient to realise all of the unused tax loss.

Impairment of goodwill is not allowed as a deduction in determining taxable profit.

Required
Write a note suitable for presentation to the partner of an accounting firm setting out the
deferred tax implications of the above information for the Om Group of companies.

Question Number 13
On January 1, 2018, Karuwa Limited (KL) acquired 375 million ordinary shares and 40 million
preference shares in Gurans Limited (GL) whose general reserve and retained earnings on the date
of acquisition, stood at Rs. 200 million and Rs. 1,000 million respectively.

The following balances were extracted from the records of KL and its subsidiary on December 31,
2018:
KL GL
Dr Cr Dr Cr
Page 157 of 328
Paper1: Advanced Financial Reporting [RTP 2019 June]

Following relevant information is available:


(i) At the date of acquisition, the fair value of buildings, included in property, plant
and equipment of GL was assessed at Rs. 1,000 million above its carrying value. All
other identifiable assets and liabilities were considered to be fairly valued. GL
provides for depreciation on buildings at 10% per annum on the straight line
basis.
(ii) GL purchased the TFCs in KL on January 1, 2018.
(iii) The non-controlling interests are measured at their proportionate share of the
GL’s identifiable net assets.
(iv) There is no impairment in the value of goodwill since its acquisition.
(v) There are no components of other comprehensive income.

Required
Prepare the following in accordance with the requirements of Nepal Financial Reporting
Standards:
Page 158 of 328
Paper1: Advanced Financial Reporting [RTP 2019 June]
(a) Consolidated statement of financial position as at December 31, 2018.
(b) Consolidated statement of comprehensive income for the year ended December 31,
2018.
(c) Consolidated statement of retained earnings for the year ended December 31,
2018.
Note:
Ignore deferred tax and corresponding figures.
Notes to the above statements are not required. However, show workings wherever it is
necessary.

Question Number 14
The profit and loss account of Yard Ltd and its subsidiary More Limited for the year ended 31
December 2016 are as follows:

Yard Ltd. More Ltd.


Rs.’000 Rs.’000
Revenue 312,500 125,000
Cost of Sales (125,000) (50,000)
Gross Profit 187,500 75,000
Distribution Cost (25,000) (10,000)
Administrative Expenses (20,000) (8,000)
Operating Profit 142,500 57,000
Investment Income 7,950 -
Debenture Interest (47,500) (15,000)
Profit on ordinary activities before taxation 102,950 42,000
Taxation on ordinary activities (35,000) (17,500)
Profit on ordinary activities after taxation 67,950 24,500
Dividends:
Preference (13,750) (4,375)
Ordinary (20,000) (5,250)
Retained profits: 34,200 14,875
Retained profits: 01.01.2016 66,750 19,500
Retained Profits: 31.12.2016 100,950 34,375

Additional information:
(1) Included in the revenue of More Limited is Rs. 12.5million in respect of sales to Yard Ltd,
giving More Limited a profit of 25% on cost. These are sales of components that More
Limited has been supplying to Yard Ltd on a regular basis for a number of years. The
Page 159 of 328
Paper1: Advanced Financial Reporting [RTP 2019 June]
amounts included in the inventories of Yard Ltd in respect of goods purchased from
More Limited at the beginning and end of the year were as follows:

Inventories of components in Yard Ltd’s books


Date Rs.’000
31.12.2016 2,000
31.12.2015 1,500

(2) Some years ago, Yard Ltd bought 50 million ordinary shares in More Limited at a cost of
Rs. 67million. On the same date, Yard Ltd bought 25% of the debentures of More Limited
at par.

At the date of Yard Ltd’s investment in More Limited, the statement of financial position of More
limited showed:

Rs.’000
Ordinary share capital 62,500
Preference share capital 43,750
Profit and Loss account 12,500
118,750
The goodwill acquired by Yard Ltd in More Limited had been written off fully in December 2016 as
a result of impairment losses.

Required
Prepare the consolidated profit and loss account of Yard Ltd for the year. Assume that investment
income is dealt with by Yard Ltd on an accrual basis.

Question Number 15
1. Which of the following do not come within the definition of a share-based payment under
NFRS 2?
A. employee share purchase plans
B. employee share option plans
C. share appreciation rights
D. a rights issue that includes some shareholder employees

2. A company issues fully paid shares to 500 employees on 31 July 20X8. Shares issued to
employees normally have vesting conditions attached to them and vest over a three-year
period, at the end of which the employees have to be in the company’s employment. These
shares have been given to the employees because of the performance of the company during
the year. The shares have a market value of Rs. 2m on 31 July 20X8 and an average fair value
for the year of Rs. 3m. It is anticipated that in three years’ time there will be 400 employees
at the company.

What amount would be expensed to profit or loss for the above share issue?
A. Rs 3m
B. Rs 2m
C. Rs 1m
D. Rs 666,667
Page 160 of 328
Paper1: Advanced Financial Reporting [RTP 2019 June]

3. A company grants 750 share options to each of its six directors on 1 May 20X7. The options
vest on 30 April 20X9. The fair value of each option on 1 May 20X7 is Rs 15 and their intrinsic
value is Rs 10 per share. It is anticipated that all of the share options will vest on 30 April
20X9. What will be the accounting entry in the financial statements for the year ended 30
April 20X8?
A. Increase equity Rs 33,750; increase in expense in profit or loss Rs 33,750
B. Increase equity Rs 22,500; increase in expense in profit or loss Rs 22,500
C. Increase liability Rs 67,500; increase in expense profit or loss Rs 67,500
D. Increase liability Rs 45,000; increase in current assets Rs 45,000

4. A public limited company has granted 700 share appreciation rights (SARs) to each of its 400
employees on 1 January 20X6. The rights are due to vest on 31 December 20X8 with payment
being made on 31 December 20X9. During 20X6, 50 employees leave, and it is anticipated
that a further 50 employees will leave during the vesting period. Fair values of the SARs are as
follows:
Rs
1 January 20X6 15
31 December 20X6 18
31 December 20X7 20
What liability will be recorded on 31 December 20X6 for the share appreciation rights?
A. Rs 1,260,000
B. Rs 1,680,000
C. Rs 2,520,000
D. Rs 3,780,000

Question Number 16
a. A company issued share options on 1 June 20X6 to pay for the purchase of inventory. The
inventory is eventually sold on 31 December 20X8. The value of the inventory on 1 June
20X6 was Rs 6m and this value was unchanged up to the date of sale. The sale proceeds
were Rs 8m. The shares issued have a market value of Rs 6.3m.

How will this transaction be dealt with in the financial statements?

b. A company grants 2,000 share options to each of its three directors on 1 January 20X6,
subject to the directors being employed on 31 December 20X8. The options vest on 31
December 20X8. The fair value of each option on 1 January 20X6 is Rs 10, and it is
anticipated that on 1 January 20X6 all of the share options will vest on 30 December 20X8.
The options will only vest if the company’s share price reaches Rs 14 per share.

The share price at 31 December 20X6 is Rs 8 and it is not anticipated that it will rise over
the next two years. It is anticipated that on 31 December 20X6 only two directors will be
employed on 31 December 20X8.

How will the share options be treated in the financial statements for the year ended 31
December 20X6?
Page 161 of 328
Paper1: Advanced Financial Reporting [RTP 2019 June]
c. Jay, a public limited company, has granted 300 share appreciation rights to each of its 500
employees on 1 July 20X5. The management feel that as at 31 July 20X6, the year end of
Jay, 80% of the awards will vest on 31 July 20X7. The fair value of each share appreciation
right on 31 July 20X6 is Rs 15.

What is the fair value of the liability to be recorded in the financial statements for the year
ended 31 July 20X6?

d. A company operates in a country where it receives a tax deduction equal to the intrinsic
value of the share options at the exercise date. The company grants share options to its
employees with a fair value of Rs 4.8m at the grant date. The company receives a tax
allowance based on the intrinsic value of the options which is Rs 4.2m. The tax rate
applicable to the company is 30% and the share options vest in three years’ time.

Question Number 17
Write short notes on:
a. Nepal Public Sector Accounting Standards
b. Accounting Standards Board

Question Number 18
How do you differentiate Small and Medium-sized Entities (SMEs) with Micro Enterprises?

Question Number 19
Following the request of the commercial banks of Nepal through Nepal Banker’s Association with
endorsement from Nepal Rastra Bank, Accounting Standards Board, Nepal has initiated the carve-
out proceeding. Please elaborate which are the 3 carve-outs among others, resolved by
Accounting Standards Board.

Question Number 20
Coal Goal Ltd. is a coal mining company and sells its coal on the spot and futures markets. On the
spot market, the commodity is traded for immediate delivery and, on the forward market, the
commodity is traded for future delivery. The inventory is divided into different grades of coal. One
of the categories included in inventories at 30 November 20X6 is coal with a low carbon content
which is of a low quality. Coal Goal Ltd. will not process this low quality coal until all of the other
coal has been extracted from the mine, which is likely to be in three years’ time. Based on market
information, Coal Goal Ltd. has calculated that the three-year forecast price of coal will be 20%
lower than the current spot price.

The directors of Coal Goal Ltd. would like advice on two matters:
(i) whether the Conceptual Framework affects the valuation of inventories;
(ii) how to calculate the net realisable value of the coal inventory, including the low quality
coal.
Page 162 of 328
Paper1: Advanced Financial Reporting [RTP 2019 June]

SUGGESTED ANSWERS HINTS


Answer Number 1
(a)
Conceptual framework is a constitution, a coherent system of interrelated objectives and
fundamentals that can lead to consistent standards and prescribes the nature, functions and limits
of financial accounting and financial statements.
It enables certain critical issues to be addressed.
It also facilitates the development of accounting standards and Generally Accepted Accounting
Practice (GAAP), in accordance with the principles and underlying assumptions of the concepts.
It also promotes consistency in the application of accounting principles and policies. Without
conceptual framework there will not be a common definition of the elements of financial
statements (asset, liabilities, income and expenses). For example, in line with the framework for
preparation and presentation of financial statements gives a precise definition of assets and
liabilities and no expenditure on these could be recognized unless such expenditure meets the
definition of the framework.

(b)
Assets
Assets are resources controlled by an entity as a result of past events and from which future
economic benefits are expected to flow to the entity

Liabilities
Liabilities are present obligations of the entity arising from past events, the settlement of which,
are expected to result in an outflow from the entity of resources embodying economic benefit.

(c)
Substance over form
Substance over form refers to the impact which a transaction or event has on the reporting entity,
and this determines the accounting treatment. Substance over form indicates that transactions
and events should be reported in the financial statement in accordance with their economic
reality or their commercial intent. In applying the substance over form, NFRS framework requires
the entity to adopt the statement of financial position liability method.
This is done by looking at the impact of the event or transaction on assets or liabilities on the
reporting date.

Materiality
An item is considered material if its inclusion, omission or misstatement will have a fundamental
effect on the financial statements as a whole and affect the economic decision of the user. The
requirement of the framework, is that items which are material in nature to an entity should be
accorded separate recognition, presentation and disclosure, while those that are immaterial
(small and separate unimportant) should be aggregated or added up.

Comparability
This implies that the financial statements of a given year should have relative figure of the past
period or periods. This helps to evaluate the performance of the entity and trend analysis over
Page 163 of 328
Paper1: Advanced Financial Reporting [RTP 2019 June]
time. It also helps to assess the comparability on consistent application of accounting policies over
time.

Going Concern
The going concern concept assumes that the entity will be in operational existence in the
foreseeable future time, and that it has no intention to scale down its operation significantly.
Except otherwise stated, financial statements are usually prepared on the going concern concept.
If, however, the entity can no longer continue as a going concern, then, subsequent financial
statement should be prepared on a break- up basis that is, the assets should be stated at fair value
less cost to point of sales (net realizable value) rather than at cost less accumulated depreciation.

(d).
Lenders
The lenders are concerned with the ability of the company to pay the finance cost on the
borrowed fund and pay the loan when due. They are also interested in the availability of assets to
secure loans.

Suppliers
The suppliers are interested in information that indicates that their debts can be paid by the entity
and that the entity will continue as a going concern in order to ensure continued patronage.

Customers
The customers are interested in information relating to the entity’s continued existence, especially
for those that depend on the entity to meet their daily needs.

Employees
The employees are concerned with their job security and the company’s ability to be profitable, in
order to guarantee the payment of their salaries in the future.

Government and their agencies


Government and their agencies are interested in information relating to taxes, regulations,
resource allocation and evaluation of government policies on businesses.

Answer No. 2
Calculation of Profit for the Year 1:
Physical
capital Financial capital maintenance
maintenance
Profit for the year Constant
Historical
purchasing
cost
power
accounting
accounting
Rs. Rs. Rs.
Sales 1,400 1,400 1,400
Cost of sales (1,000) (1,000) (1,000)
Inflation
adjustment
Page 164 of 328
Paper1: Advanced Financial Reporting [RTP 2019 June]
- Specific
(1,100 – 1,000) (100) - -
- General
(1,000 X 7%) - - (70)
Profit 300 400 330

Statement of Financial Position as of 31 December Year 1


Cash at bank 1,400 1,400 1,400
Share capital 1,000
(1,000 + 100) 1,100*
(1,000 + 70) 1070*
Reserves 300 400 330
1,400 1,400 1,400
Share capital at the year-end is restated under the physical capital maintenance concept for an
increase in specific price changes and under constant purchasing power accounting for general
price changes. This is the other side of the entry to the inflation adjustments in the statement of
profit or loss.

Answer No. 3
The related parties comprise a subsidiary, an associated undertaking/an entity having significant
influence, director and key management personnel. Aggregate transactions with related parties
are as follows:
Rs. In million
Entity having Key
Subsidiary Associate significant Director Management
influence Personnel
Transaction
s
Sales 500
Sales
25
Discount
Sales Return 5.5
Raw
Material 5
Purchase
Purchase of
3
Equipment
Purchase of
14
Machinery

Balances
At the
Beginning of 1.4
the year
Repaid
during the 0.3
year
Page 165 of 328
Paper1: Advanced Financial Reporting [RTP 2019 June]
At the end
1.1
of the year

(i) Sales discount represents a special discount which is not usually allowed to other
customers.
(ii) All transactions with related parties have been carried out on commercial terms and
conditions.

Answer Number 4
(i) AK Associates will not be treated as related party merely on the ground that both entities
have a director in common. However, if it can be proved that an entity has some influence
on other entity; they will be treated as related parties.
(ii) Provider of finance is not necessarily a related party. However, SS Bank has power to
appoint its nominee director in the Board and therefore enjoys significant influence; it will
be treated as related party.
(iii) Mr. Zoe will not be treated as related party unless it can be proved that he has significant
influence over the CEO. Further, NAS-24 does not explicitly include ‘Brother’ in the
definition of close family member of an individual.
(iv) A distributor with whom an entity transacts a significant volume of business will not be
treated as related party, merely by virtue of the resulting economic dependence.
(v) Since Mr. Tee is the key management personnel of the company, he will be treated as
related party.
(vi) A post-employment benefit plan for the benefits of the employee is treated as related
party.
(vii) A supplier with whom an entity transacts a significant volume of business cannot be
termed a related party, merely by virtue of the resulting economic dependence.
(viii) Ms. Lee will be treated as related party as she is a close family member of CEO, and hence
in a position to influence her husband.

Answer Number 5
(a) The purposes of segmental information are:
(i) to provide users of financial statements with sufficient details for them to be able to
appreciate the different rates of profitability, different opportunities for growth and
different degrees of risk that apply to an entity’s classes of business and various
geographical locations.
(ii) to appreciate more thoroughly the results and financial position of the entity by
permitting a better understanding of the entity’s past performance and thus a better
assessment of its future prospects.
(iii) to create awareness of the impact that changes in significant components of a
business may have on the business as a whole.

(b) NFRS 8 defines an operating segment as a component of an entity:


 that engages in business activities from which it may earn revenues and incur
expenses (including revenues and expenses relating to transactions with other
components of the same entity).
Page 166 of 328
Paper1: Advanced Financial Reporting [RTP 2019 June]
 whose operating results are regularly reviewed by the entity’s chief operating
decision-maker to make decisions about resources to be allocated to the segment
and assess its performance.
 for which discrete financial information is available.

In order to identify the separate reportable segments, the following criteria should be
adopted;
 The reported revenue of the segment in the company, including both sales to
external customers and inter-segment sales, is ten percent or more of the
combined revenue of its four operating segments.
 The assets of the segment in the company are ten percent or more of the
combined assets of its four operating segments.
 The reported profit or loss of the segment in the company should be ten percent
or more of the greater, in absolute amount, of:
 The combined reported profit of all its operating segments that did not
report a loss, and
 The combined reported loss of all operating segments that reported
losses.

(c) NFRS 8 lays down some very broad and inclusive criteria for reporting segments. Unlike
earlier attempts to define segments in more quantitative terms, segments are defined
largely in terms of the breakdown and analysis used by management. This is, potentially, a
very powerful method of ensuring that preparers provide useful segmental information.

There will still be problems in deciding which segments to report, if only because
management may still attempt to reduce the amount of commercially sensitive
information that they produce.
The growing use of executive information systems and data management within
businesses makes it easier to generate reports on an ad hoc basis. It would be relatively
easy to provide management with a very basic set of internal reports and analyses and
leave the individual managers to prepare their own more detailed information using the
interrogation software provided by the system.

If such analyses become routine then they would be reportable under NFRS 8, but that
would be very difficult to check and audit.

There are problems in the measurement of segmental performance if the segments trade
with each other. Disclosure of details of inter-segment pricing policy is often considered to
be detrimental to the good of a company. There is little guidance on the policy for transfer
pricing.

Different internal reporting structures could lead to inconsistent and incompatible


segmental reports, even from companies in the same industry.
Page 167 of 328
Paper1: Advanced Financial Reporting [RTP 2019 June]
Answer Number 6
MML
Statement of changes in equity
For the year ended December 31, 2018
Rs. In Million
Retained
Particulars
Earnings
Balance at December 31, 2016 as previously reported
68.00
(Rs. 89m – Rs. 21m)
Effect of change in accounting policy
(1.50)
(Rs. 37m - Rs. 35.5m)
Balance at December 31, 2016 – restated 66.50
Profit for the year ended December 31, 2017 - restated (W1) 39.70
Balance at December 31, 2017 – restated 106.20
Profit for the year ended December 31, 2018 (W2) 8.80
Balance at December 31, 2018 115.00

Rs. In Million
W1: Profit for the year ended December 31, 2017 (as restated)
Particulars Amount
Profit as previously reported 21.00
Incorrect recording of depreciation
15.00
(Rs. 25 million – Rs. 10 million)

Reversal of FIFO method (5.30)


Opening inventory 37.00
Closing inventory (42.30)

Application of weighted average method 9.00


Opening inventory (35.50)
Closing inventory 44.50
Profit for the year ended December 31, 2017 – restated 39.70

W2: Adjusted profit for year ended December 31, 2018


Particulars Amount
Profit as per draft financial statements 15.00
Adjustment in Opening Inventory (2.20)
FIFO 42.30
Weighted average (44.50)

Adjustment in Closing Inventory (4.00)


FIFO (58.40)
Weighted average 54.40
Adjusted profit 8.80
Page 168 of 328
Paper1: Advanced Financial Reporting [RTP 2019 June]
Answer Number 7
The legal fees and commission will be considered an “incremental cost of obtaining a contract”
because these costs were only incurred as a result of obtaining the contract. The company expects
to recover these costs through the future advertising revenue that will be earned under the
contract. Had the contract not been obtained, these costs would not have been incurred. If the
legal fees had been incurred prior to obtaining the contract, they would not be capitalized.

The meals and entertainment costs are not eligible to be capitalized because they would have
been incurred regardless of whether the contract had been obtained.

The costs related to the Director’s time and the costs associated with hiring actors are direct
labour costs associated with providing the advertising services and are considered to be costs
directly related to the contract and are not covered by any other standard. These costs also meet
the required criteria for capitalization, because they:

 are specifically identified through time-sheet allocations and invoices (i.e., relate directly
to the contract)
 provide a resource (e.g., Director and Actor) used to satisfy the performance obligation
 will be recovered through contract revenue earned. Once these costs are capitalized, they
will be amortized and assessed for impairment in accordance with IFRS 15.

b. There are two warranties in this contract:


i. assurance-type warranty — for first year after purchase
ii. service-type warranty — for two years after expiry of the initial standard warranty

Given that the customer has the option of purchasing the additional warranty separately,
this is a service-type warranty and is accounted for as a separate performance obligation.
Deferred revenue of Rs. 200,000 is recognized (as opposed to revenue being recorded)
until the performance obligation is satisfied.

The assurance-type warranty is accounted for in accordance with NAS 37 Provisions,


Contingent Liabilities and Contingent Assets.

Answer Number 8
Calculation of Capital work in progress – Factory building
In Rs.’ 000
Particulars Amount
Progress invoices received from the contractor 75,000
(30,000+20,000+10,000+15,000)
(Rain damages paid would be chargeable to profit and loss
account/ insurance claim)
Borrowing costs to be capitalised:
Loan processing charges 500.00
Interest on bank loan (W1) 1,841.67
Interest on running finance (W2) 2,730.00
Interest income from surplus loan amount (W4) (395.00)
Page 169 of 328
Paper1: Advanced Financial Reporting [RTP 2019 June]
Capital work in progress – June 30, 2016 79,676.67

W1: Interest on bank loan:

Rs.000
Interest amount Months Outstanding Interest at
From To loan 13%
amount
01.12.2015 31.05.2016 6 25,000 1,625.00
01.06.2016 31.06.2016 1 20,000 216.67
1,841.67

W2: Interest on running finance:


Rs. 000
Payments from Interest
Paymen at 15%
ts net of Months per
Payment Invoice deducti Right Bank Running o/s up to annum
date Description amount ons issue Loan finance 30.06.16 (W3)
Advanced
01.07.15 10,000 10,000 10,000 12.00 1,500
payment
st
1 progress
15.10.15 30,000 25,500 15,000 10,500 8.50 1,116
bill
nd
2 progress
15.01.16 20,000 17,000 17,000 - - -
bill
rd
3 progress
15.04.16 10,000 8,500 7,500 1,000 2.50 31
bill
Loan
31.05.16 1,625 1,625 1.00 20
interest
Loan
31.05.16 5,000 5,000 1.00 63
installment
15,000 *24,000 28,125 2,730

*Loan amount of Rs. 25,000,000 less processing charges of Rs. 500,000


W3: Average rate of interest for running finance facility
(9,000/60,000) 15%

W4: Interest income from surplus loan amounts:


Rs. 000
Interest Income Surplus Interest
From To Months Loan Income at
Amounts 8%
01.12.15 15.01.16 1.5 24,500 (245)
16.01.16 15.04.16 3 7,500 (150)
(395)
Answer Number 9
(a) Impairment of plant
The plant had a carrying amount of Rs. 240,000 on 1 October 2015. The accident that may have
caused impairment occurred on 1 April 2016 and an impairment test would be done at this date.
Page 170 of 328
Paper1: Advanced Financial Reporting [RTP 2019 June]
The depreciation on the plant from 1 October 2015 to 1 April 2016 would be Rs. 40,000 (640,000 x
121/2% x 6/12) giving a carrying amount of Rs. 200,000 at the date of impairment. An impairment
test requires the plant’s carrying amount to be compared with its recoverable amount. The
recoverable amount of the plant is the higher of its value in use of Rs. 150,000 or its fair value less
costs to sell. If ABC Associates Ltd trades in the plant it would receive Rs. 180,000 by way of a part
exchange, but this is conditional on buying new plant which ABC Associates Ltd is reluctant to do.
A more realistic amount of the fair value of the plant is its current disposal value of only Rs.
20,000. Thus the recoverable amount would be its value in use of Rs. 150,000 giving an
impairment loss of Rs. 50,000 (Rs. 200,000 – Rs. 150,000).

The remaining effect on income would be that a depreciation charge for the last six months of the
year would be required. As the damage has reduced the remaining life to only two years (from the
date of the impairment) the remaining depreciation would be Rs. 37,500 (Rs. 150,000/ 2 years x
6/12).Thus extracts from the financial statements for the year ended 30 September 2016 would
be:
Statement of financial position
Non-current assets Rs.
Plant (150,000 – 37,500) 112,500

Statement of profit or loss


Plant depreciation (40,000 + 37,500) 77,500
Plant impairment loss 50,000

(b) Purchase of XYZ


There are a number of issues relating to the carrying amount of the assets of XYZ Limited that
have to be considered. It appears the value of the brand is based on the original purchase of the
‘XYZ Spring’ brand. The company no longer uses this brand name; it has been renamed ‘Refresh’.
Thus it would appear the purchased brand of ‘XYZ Spring’ is now worthless. XYZ Limited cannot
transfer the value of the old brand to the new brand, because this would be the recognition of an
internally developed intangible asset and the brand of ‘Refresh’ does not appear to meet the
recognition criteria in NAS 38. Thus prior to the allocation of the impairment loss the value of the
brand should be written off as it no longer exists.

The inventories are valued at cost and contain Rs. 2 million worth of old bottled water (XYZ Spring)
that can be sold, but will have to be relabeled at a cost of Rs. 250,000. However, as the expected
selling price of these bottles will be Rs. 3 million (Rs. 2 million x 150%), their net realizable value is
Rs. 2,750,000. Thus it is correct to carry them at cost i.e. they are not impaired. The future
expenditure on the plant is a matter for the following year’s financial statements.

Applying this, the revised carrying amount of the net assets of XYZ Limited’s cash-generating unit
(CGU) would be Rs. 25 million (Rs. 32 million – Rs. 7 million re the brand). The CGU has a
recoverable amount of Rs. 20 million, thus there is an impairment loss of Rs. 5 million. This would
be applied first to goodwill (of which there is none) then to the remaining assets pro rata.
However under NAS2 the inventories should not be reduced as their net realizable value is in
excess of their cost. This would give revised carrying amounts at 30 September 2016 of:

Brand nil
Land containing spa: 12,000 – [(12,000/20,000) X 5,000] 9,000
Page 171 of 328
Paper1: Advanced Financial Reporting [RTP 2019 June]
Purifying and bottling plant:
8,000 – [(8,000/20,000) X 5,000] 6,000
Inventories 5,000
20,000
Answer Number 10

Holiday villas
NAS 16 allows property, plant and equipment to be re-valued or left at historical cost. Revaluation
should be based on the fair value (the open market value in an arm’s length transaction).
Revaluation is not required every year, but must be conducted when it is believed that the fair
value differs materially from the carrying value.

The method of accounting for the villa that is to be sold is covered by NFRS 5 which requires that
where, at the end of a reporting period, an asset is held for sale it should be reclassified, re-
measured and no longer depreciated. An asset is only classified as held for sale where the
following conditions are all met:
 The asset is available for sale in its present condition.
 The sale is believed to be highly probable:
 Appropriate level of management is committed to the sale;
 There is an active program underway to find a buyer;
 The asset is marketed at a realistic price.
 Completion of sale expected within 12 months of classification.

From the limited information provided it appears that these conditions have been met and
therefore, under the rules of NFRS 5, the villa should be re-measured to the lower of its carrying
value and its fair value minus costs to sell.

Therefore, the villas should be valued at 31 December Year 4 as follows:

Fair Value Carrying Value


Rs. Rs.
All villas 25.00 20.00
Property held for sale (1.00) (1.25)
Properties to be retained 24.00 18.75

The villas to be retained should be re-valued to Rs. 24m, resulting in an increase in the revaluation
reserve of Rs. 5.25m (24-18.75).

The villa to be sold should be written down from its carrying value to its fair value minus costs to
sell of Rs.0.95m (Rs. 1m – 50,000). This impairment of Rs. 300,000 (1.25m – 0.95m) will be charged
against the revaluation reserve for this asset. If there is insufficient revaluation reserve, then the
write down must be charged to profit or loss.

The villa held for sale must be re-classified from ‘Non-current assets’ to ‘Current assets’ as a
separate line item.

Depreciation should not be charged when an asset has been classified as held for sale. However,
the other villas should be depreciated. NAS 16 states that expenditure on repairs and
Page 172 of 328
Paper1: Advanced Financial Reporting [RTP 2019 June]
maintenance does not remove the need to depreciate an asset. The villas have a finite useful life
and therefore must be depreciated. If the residual value of these assets is greater than the
carrying value then the depreciation charge will be zero. It is not acceptable therefore to have a
policy of non-depreciation on such assets, and a prior year adjustment should be made to correct
the error if the error is material.

Head office
The head office should be recorded under property, plant and equipment at cost. NAS 23 requires
that borrowing costs should be capitalized as part of the cost of an asset if they are directly
attributable to the acquisition, construction or production of a ‘qualifying asset’. A qualifying asset
is an asset that necessarily takes a long period of time to get ready for its intended use or sale.

In this situation the company is therefore required to capitalize the borrowing costs as part of the
asset cost. Capitalization must cease when the asset is substantially complete. Construction
finished on 31 May Year 4 and, although minor modifications continued for a further three
months, the standard states that minor modifications indicate that the asset is substantially
complete.

Cost at 30 June Year 4: Rs.000 Rs.000


Land 1,000
Building: Construction cost 8,000
Interest 9% X 5 million X (20/12) years
(1 October Year 2 to 1 June Year 4) 750
8,750
9,750

Prime is to receive a government grant. NAS 20 requires that the grant be recognised when there
is reasonable assurance that the entity will meet any conditions and receive the grant. As the
grant has not been received, a receivable will be recorded under current assets. The credit can be
treated in one of two ways:

Option 1: Record as deferred income and release to profit or loss over the useful life of the asset

Option 2: Deduct the grant from the carrying amount of the asset.

If the second option is taken, the asset will be carried at Rs. 8.25m rather than at
Rs. 9.75m. The effect on profit or loss will be the same in both cases.

Land should not normally be depreciated, because land has an indefinite useful life in most
situations. However, as buildings have a limited useful life, a residual value must be allocated to
the building and the depreciable amount must then be written off over the 50 year useful life.
Depreciation will be charged in Year 4 for the four months from 1 September to 31 December.

The estimates of residual value and useful life must be revised each year and the depreciation
amended prospectively.
Page 173 of 328
Paper1: Advanced Financial Reporting [RTP 2019 June]
Yachts
It is important to note that the yachts are held for rental purposes, so they are non-current assets,
not inventory.

The yachts cost Rs. 20m to build, but the recoverable amount on completion (higher of value in
use and net selling price) is only Rs. 18m, and so the assets must be initially recognised at their
recoverable amount. The impairment write down of Rs. 2m will be charged to profit or loss in Year
4 in accordance with NAS 36.
Cost Recoverable amount
Rs. M Rs.m
Engines (15%) 3 2.7
Interior (25%) 5 4.5
Remainder (60%) 12 10.8
20 18

NAS 16 requires that each part of the asset that has a cost that is significant in relation to the total
cost must be depreciated separately. Therefore, in the financial year the depreciation charge will
be as follows:
Rs. m
Engines Rs. 2.7m X 1/3 X 9/12 0.675
Interior Rs. 4.5m X 1/2 X 9/12 1.688
Remainder Rs. 10.8, X 1/5 X 9/12 1.620
Charge to Profit or Loss in Year 4 3,983

Answer Number 11
Introduction
All four scenarios relate to the rules of NAS 37 Provisions, contingent liabilities and contingent
assets. In each scenario, the key issue is whether or not a provision should be recognised.

Under NAS 37, a provision should only be recognised when three conditions are met:
 there is a present obligation as a result of a past event; and
 it is probable that a transfer of economic benefits will be required to settle the
obligation; and
 a reliable estimate can be made of the amount of the obligation.

Factory closure
As the factory closure changes the way in which the business is conducted (it involves the
relocation of business activities from one part of the country to another) it appears to fall within
the NAS 37 definition of a restructuring.

The key issue here is whether the group has an obligation at the end of the reporting period to
incur expenditure in connection with the restructuring. There is clearly no legal obligation, but
there may be a constructive obligation. NAS 37 states that a constructive obligation only exists if
the group has created valid expectations in other parties such as employees, customers and
suppliers that the restructuring will actually be carried out. As the group is still in the process of
drawing up a formal plan for the restructuring and no announcements have been made to any of
Page 174 of 328
Paper1: Advanced Financial Reporting [RTP 2019 June]
the parties affected, there cannot be an obligation to restructure. A board decision alone is not
sufficient. Therefore no provision should be made.

If the group starts to implement the restructuring or makes announcements to those affected
after the end of the reporting period but before the accounts are approved by the directors it may
be necessary to disclose the details in the financial statements as a non-adjusting post event after
the reporting period in accordance with NAS 10. This will be the case if the restructuring is of such
importance that non-disclosure would affect the ability of the users of the financial statements to
reach a proper understanding of the group’s financial position.

Operating lease
The lease contract appears to be an ‘onerous contract’ as defined by NAS 37 as the unavoidable
costs of meeting the obligations under it exceed the economic benefits expected to be received
from it.

Because the enterprise has signed the lease contract there is a clear legal obligation and the
enterprise will have to transfer economic benefits (pay the lease rentals) in settlement. Therefore,
the group should recognize a provision for the net present value of the remaining lease payments.

In principle, a corresponding asset may be recognized in relation to the future rentals expected to
be received, if these receipts are virtually certain. The current arrangement with the charity
generates only nominal rental income and so the asset is unlikely to be material enough to
warrant recognition. The chances of renting the premises at a commercial rent are less than 50%
and so no further potential rent receivable may be taken into account as the outcome is not
virtually certain and so recognition would not be prudent.

The financial statements should disclose the carrying amount of the onerous lease provision at the
end of the reporting period, a description of the nature of the obligation and the expected timing
of the lease payments. Disclosure should also be made of the contingent assets where the amount
of any expected rentals receivable from sub-letting are material and the likelihood is believed
probable.

Legal proceedings
It is unlikely that the group has a present obligation to compensate the customer; therefore no
provision should be recognized. However, there is a contingent liability. Unless the possibility of a
transfer of economic benefits is remote, the financial statements should disclose a brief
description of the nature of the contingent liability, an estimate of its financial effect and an
indication of the uncertainties relating to the amount or timing of any outflow.

Environmental damage
It is clear that there is no legal obligation to rectify the damage. However, through its published
policies, the group has created expectations on the part of those affected that it will take action to
do so. There is, therefore, a constructive obligation to rectify the damage and a transfer of
economic benefits is probable.

The group must recognize a provision for the best estimate of the cost. As the most likely outcome
is that more than one attempt at re-planting will be needed, the full amount of Rs. 30 million
should be provided. The expenditure will take place sometime in the future, and so the provision
Page 175 of 328
Paper1: Advanced Financial Reporting [RTP 2019 June]
should be discounted at a pre- tax rate that reflects current market assessments of the time value
of money and the risks specific to the liability.

The financial statements should disclose the carrying amount at the end of the reporting period, a
description of the nature of the obligation and the expected timing of the expenditure. The
financial statements should also give an indication of the uncertainties about the amount and
timing of the expenditure.

Answer Number 12
Note for presentation to partner
Subject: Deferred Taxation
The calculation and presentation of deferred tax is considered by NAS 12 Income taxes. A
company is required to provide deferred tax on all material temporary differences using the full
provision method. Temporary differences arise because there is a difference in timing between
transactions being reflected in the financial statements and the item being taxed.

In light of the recent acquisitions of Namaha and Sibaya, deferred tax must be considered for the
group accounts. Additional tax issues arise at the group level that will not have been reflected in
the individual entity’s accounts and these points are outlined below.

Once the temporary differences have been identified, deferred tax must be provided at the tax
rate expected to be effective at the date when the tax is settled. Given this rate is not known
when the differences arise, a provision is made using the rates enacted at that time and the
estimate is then confirmed as tax changes arise.

a. Sibaya
The acquisition of Sibaya mid-year gives rise to a number of issues:

1. Intangible asset
There is some concern that the acquisition of the database of key customers may
not be allowed for tax purposes but it has nevertheless been included in the tax
calculation on the assumption that a deduction will be allowed by the tax
authorities. If this deduction is not allowed, then an additional tax payment will need
to be made to the authorities, hence it would be prudent to recognise a liability for
this amount (probably classified as current taxation, rather than deferred taxation).

2. Inter-company sales
When goods are sold between group members, the profits made are seen as
unrealised in the group accounts until the items are sold outside of the group.
However, the tax authorities tax the individual entities, not the group, and so the
profit will be subject to tax at the time of the inter-company sale. The unrealised
profit represents the temporary difference on which deferred tax must be provided.
The goods were sold at a margin of 33 1⁄3%. Goods sold for Rs. 1.8m remain in
inventory at the year end, and hence the unrealised profit, and therefore temporary
difference, is Rs.0.6m.

3. Unremitted profits
Page 176 of 328
Paper1: Advanced Financial Reporting [RTP 2019 June]
Un-remitted earnings represent a temporary difference in the group accounts. This
is because the tax base is the cost of the investment, yet in the consolidated
accounts, this cost is uplifted by the post- acquisition un-remitted profits. NAS 12
requires a provision to be made on this timing difference unless the parent controls
the timing of the reversal and it is probable that the difference will not reverse for
the foreseeable future. The payment of dividends is under the control of Om, but we
understand that the intention is to realise these un- remitted earnings through
future dividends and hence a provision must be made.

b. Namah
The acquisition of Namah at the start of the year brings further deferred tax issues in the group
accounts as outlined below.

1. Fair value through the profit or loss investments


The fair value adjustment has been reflected in the financial statements, yet the gain
is not taxed until the investments are sold. Hence the fair value adjustment of Rs. 4m
gives rise to a temporary difference upon which deferred tax must be provided. As
the gain has been taken to profit or loss rather than other comprehensive income or
reserves, the deferred tax must also be expensed to profit or loss.

2. General allowance
The allowance against the loan portfolio has reduced the carrying value of the loans
but the tax relief is not available until the loan is written off, and hence a temporary
difference is created on the provision. As the tax relief will reduce future tax charges,
the temporary difference of Rs. 2m gives rise to a deferred tax asset. The temporary
difference is accounted for even though there is no expectation that the difference
will reverse in the immediate future.

However, a deferred tax asset can only be reflected to the extent that it is probable
that there will be future taxable profits against which the temporary difference can
be relieved.

3. Unrelieved tax losses


When Namah was acquired, it had unused tax losses brought forward which could, in
principle, give rise to a deferred tax asset. However, it can only be recognised to the
extent that it is believed that the loss can be recovered. Given your belief that there
will not be sufficient future profits, the deferred tax can only be partially recognised.
If the fortunes of Namah change in the future, the deferred tax asset should then be
recognised, leading to a compensating amendment to goodwill.

Answer Number 13
a. Karuwa Ltd.
Consolidated Statement of Financial Position
As at 31 Dec 2018
Rupees
In million
ASSETS
Page 177 of 328
Paper1: Advanced Financial Reporting [RTP 2019 June]
Non-current assets

Property, Plant & Equipment (W-1) 14,800

Goodwill (W – 2) 100

14,900

Current assets (1,069 + 1,316) 2,385

17,285

EQUITY AND LIABILITIES

Equity

Share capital 6,800

General reserve (W-5) 1,975

Retained earnings 3,844

12,619

Non-controlling interest (W-8) 2,420

Total equity 15,039

Non-current liabilities

14% Term finance certificates (2,250-1,500) 750

Current liabilities

Account payable (445+190) 635

Dividend payable (W-3) 861

17,285

b. Karuwa Ltd.

Consolidated Statement of Profit or Loss & Other Comprehensive Income


Page 178 of 328
Paper1: Advanced Financial Reporting [RTP 2019 June]
For the year ended 31 Dec 2018

Rupees
In million
Profit before tax and interest (W-
4,315
4)

Interest Expense (315-210) (105)

Profit before tax 4,210

Taxation expense (650+474) (1,124)

Profit for the period 3,086

Other comprehensive income -

Total comprehensive income 3,086

Attributable to:

Equity holders of parent Balancing 2,894

Non-controlling interest (W-


192
7)

3,086

c. Karuwa Ltd.

Consolidated statement of retained earnings


for the year ended December 31, 2018

Particulars Rupees

in
Million

Balance as at 01 Jan, 2018 (W-6) 1,700

Total comprehensive income for the year 2,894

Dividends (750)

Balance as at 31 Dec 2018 3,844


Page 179 of 328
Paper1: Advanced Financial Reporting [RTP 2019 June]
Answer Number 14
Yard Ltd.
Consolidated Statement of Profit or Loss
For the year ended 31 Dec 2016

NOTE Rs.’000

Revenue 1 425,000

Cost of sales 2 (162,600)

Gross profit 262,400

Distribution costs (35,000)

Impairment losses 4 (7,000)

Administrative expenses (28,000)

Operating profit 192,400

Investment income 5 3,750

Debenture interest 6 (58,750)

Profit before tax 137,400

Tax (52,500)

Profit after Tax 84,900

Non-controlling interests 7 (8,380)

Group profit 76,520

Preference dividends (13,750)

Ordinary dividends (20,000)

Retained Profit 42,770

Retained profit 01 January 2016 8 72,350

Retained profit 31 December 2016 115,120

Workings:

1 Revenue Rs.’000
Page 180 of 328
Paper1: Advanced Financial Reporting [RTP 2019 June]
Yard 312,500

More 125,000

437,500

Less; inter-company group 12,500


sales

425,000

2 Cost of Sales:

Yard 125,000

More 50,000

Inter-company sales (12,500)

Unrealized profits (3) 100

162,600

3 Provision for unrealized


profits on inventory

400

(300)

Charged to profit or loss 100

4 Goodwill on acquisition

Cost of investment 67,000

Net assets acquired:

- On ordinary share capital 62,500


Page 181 of 328
Paper1: Advanced Financial Reporting [RTP 2019 June]
- Profit or Loss 12,500

75,000

Percentage acquired 50/62.5


X 100= 80%
(60,000)
80% of Rs. 75,000

Goodwill (Written-off in 7,000


2016)

5 Investment income:

As per draft account 7,950

Inter-company dividend 80% (4,200)


X Rs. 5,250

Investment income 3,750

6 Debenture interest

Parent’s 47,500

Subsidiary 15,000

Inter-company amount 25% X (3,750)


Rs. 15,000

58,750

7 Non-controlling interest:

Profit after tax 24,500

Less: Preference dividend (4,375) 4,375

20,125
X 20% 4,025

8,400

Non-controlling interest share


of unrealized profit (20% of
Rs. 100) (20)
Page 182 of 328
Paper1: Advanced Financial Reporting [RTP 2019 June]
8,380

8 Retained Profit b/f

Parent’s 66,750

Subsidiary 19,500

Pre-acquisition (12,500)

Parent’s share 7,000 X 5,600


80%

72,350

Answer Number 15
1. (D).
2. (B). Rs 2m.
The issue of fully paid shares is deemed to relate to past service and should be expensed
to profit or loss at 31 July 20X8.
3. (A). 750 x 6 (directors) x Rs 15 / 2 years = Rs 33,750
4. (A). 700 x (400 – 100) x Rs 18 x 1/3 = Rs 1,260,000

Answer Number 16
a. NFRS 2 states that the fair value of the goods and services received should be used to
value the share options unless the fair value of the goods cannot be measured reliably.
Thus equity would be increased by Rs 6m and inventory increased by Rs 6m. The inventory
value will be expensed on sale.

b. The market-based condition (i.e. the increase in the share price) can be ignored for the
purpose of the calculation. However the employment condition must be taken into
account. The options will be treated as follows:

2,000 options x 2 directors x Rs 10 x 1 year / 3 years = Rs 13,333

Equity will be increased by this amount and an expense shown in profit or loss for the year
ended 31 December 20X6.

c. 300 rights x 500 employees x 80% x Rs 15 x 1 year / 2 years = Rs 900,000

d. A deferred tax asset would be recognised of:

Rs 4.2m @ 30% tax rate x 1 year / 3 years = Rs 420,000


Page 183 of 328
Paper1: Advanced Financial Reporting [RTP 2019 June]
The deferred tax will only be recognised if there are sufficient future taxable profits
available.

Answer Number 17
a) Nepal Public Sector Accounting Standards
Accounting Standards Board (ASB) is entrusted by the Government of Nepal with the responsibility
to develop accounting standards for public sector. The ASB has developed Nepal Public Sector
Accounting Standards (NPSAS) for Financial Reporting under Cash Basis of Accounting in line with
the International Public Sector Accounting Standards prepared by International Public Sector
Accounting Standards Board (IPSASB), an independent standard-setting body within the
International Federation of Accountants (IFAC).

Structure of the Standards This Standard comprises two parts:


 Part 1 is mandatory. It sets out the requirements which are applicable to all entities
preparing general purpose financial statements under the cash basis of accounting. It
defines the cash basis of accounting, establishes requirements for the disclosure of
information in the financial statements and supporting notes, and deals with a number of
specific reporting issues. The requirements in this part of the Standard must be complied
with by entities which claim to be reporting in accordance with the Nepal Public Sector
Accounting Standards Financial Reporting Under The Cash Basis of Accounting.
 Part 2 is not mandatory. It identifies additional accounting policies and disclosures that an
entity is encouraged to adopt to enhance its financial accountability and the transparency
of its financial statements. It includes explanations of alternative methods for presenting
certain information.

b) Accounting Standards Board


The Accounting Standards Board (ASB) is an independent statutory body with the responsibility to
set and issue accounting standards for preparation and presentation of financials statements in
Nepal. The Government of Nepal established ASB in March 2003 with an amendment to the
Institute of Chartered Accountants of Nepal Act 1997 incorporating the provision for its
establishment and operation. The ASB is Primarily responsible for setting accounting and financial
reporting standards for business enterprises in line with the International Financial Reporting
Standards (IFRSs). Since 2007, ASB has also been entrusted by Nepal Government with the
responsibility to develop accounting standards for public sector in line with the International
Public Sector Accounting Standards (IPSASs).

The ASB consists of 13 members comprising a Chairman appointed by the Government of Nepal
from Fellow Chartered Accountants (FCA) and other members are representative of Ministry of
Finance, representative of Office of the Auditor General, representative of Financial Comptroller
General Office, Company Registrar of Office of the Company Registrar, Director General of Inland
Revenue Department, Chairman of Securities Board of Nepal & Five Chartered Accountants (CA) &
One Registered Auditor (RA) nominated by Government of Nepal on recommendation of Institute
of Chartered Accountants of Nepal (ICAN).
Page 184 of 328
Paper1: Advanced Financial Reporting [RTP 2019 June]

Answer Number 18
Accounting Standards Board, Nepal has approved the Nepal Financial Reporting Standard for Small
and Medium-sized Entities (NFRS for SMEs) 2017 on 12 April 2017 (2073 Chaitra 30) and sent it to
the Institute of Chartered Accountants of Nepal (ICAN) on 13 April 2017 for the pronouncement.
Subsequently the ICAN has pronounced the NFRS for SMEs effective from 2076 Shrawan 1 (17 July
2019).

According to the Standards, Small and Medium-sized Entities (SMEs) are entities that:
 Do not have public accountability; and
 Publish general purpose financial statements for external users

Criteria for assessing for the entities having public accountability among others are as below:
1. Borrowings from banks, financial institutions, or public funds or from entities holding
assets in fiduciary capacity of NRs 500 million or more;
2. Balance Sheet total (without offsetting current liabilities with current assets) of NRs 1000
million or more;
3. Employing more than 300 employees including workers in an average or more;
4. Annual Turnover of NRs 1000 million or more;
5. Holding assets in fiduciary capacity in excess of NRs 500 million (includes security brokers
handling demat account, micro finance and cooperatives);

An entity which attains at least 1 of these limits in 2 consecutive years shall be deemed to be an
entity having economic significance to qualify as an entity with Public Accountability and once
qualified, must fall below all of these limits for 2 consecutive years to cease to qualify.

Similarly, micro entities are those entities with the following thresholds (all):
 Annual Turnover of NRs 100 million or less
 Borrowings from banks or financial institutions or public funds or from entities holding
assets in fiduciary capacity of NRs 50 million or less
 Balance Sheet Total of NRs 100 million (without off-setting current liabilities in current
assets) or less
 Holding assets in fiduciary capacity of NRs 50 million or less (includes security brokers
handling demat account, micro finance and cooperatives.

An entity must meet all of these limits in 2 consecutive years to qualify as a micro-entity and once
qualified, must exceed at least 1 of these limits for 2 consecutive years to cease to qualify.
Micro Entities are not specifically required to follow NFRS or NFRS for SMEs but can prepare
financial statements in accordance with the Accounting Guidelines for Micro Entities on
consistent basis.
Page 185 of 328
Paper1: Advanced Financial Reporting [RTP 2019 June]
Answer Number 19
Meeting of the Accounting Standards Board, Nepal held on September 13, 2018 (Bhadra 28, 2075)
resolved the following 7 Carve-outs in NFRS with Alternative Treatment and effective Period of
Carve-outs. The carve-outs proceeding was initiated by Accounting Standards Board, Nepal
following the request of the Commercial Banks of Nepal through Nepal Bankers Association which
was endorsed by the meeting of NFRS Implementation Committee of Nepal Rastra Bank held on
Bhadra 07, 2075 and was recommended to Accounting Standards Board, Nepal for its
consideration and necessary action.

Among others, following are the 3 carve-outs that have been considered based on the specific
request of the Banks and Financial Institutions regulated by Nepal Rastra Bank; however, other
entities may also use these carve-outs with necessary disclosures.

Carve-out : Alternative Period of


NFRS Reference Existing Provisions
Treatment Carve-out
A parent shall
NFRS 10 :
prepare A parent shall prepare
Consolidated Financial
consolidated consolidated financial
Statements
financial statements statements using uniform
FY 2017-18 FY
using uniform accounting policies for
Preparation of 2018-19 FY
accounting policies like transactions and
consolidated financial 2019-20
for like transactions other events in similar
statement using
and other events in circumstances unless it is
uniform accounting
similar impracticable to do so.
policies
circumstances.
Lease payments under an
operating lease shall be
recognised as an expense
on a straight-line basis
Lease payments over the lease term
under an operating unless either:
lease shall be (a) Another systematic
recognised as an basis is more
NAF 17 : Lease expense on a representative of the
straight-line basis time pattern of the user’s
Operating lease in the over the lease term benefit even if the 2017-18
financial statements of unless another payments to the lessors
Lessees systematic basis is are not on that basis; or
more representative (b) The payments to the
of the time pattern lessor are structured to
of the user’s benefit increase in line with
expected general
inflation to compensate
for the lessor’s expected
inflationary cost
increases. If payments to
Page 186 of 328
Paper1: Advanced Financial Reporting [RTP 2019 June]
Carve-out : Alternative Period of
NFRS Reference Existing Provisions
Treatment Carve-out
the lessor vary because
of factors other than
general inflation, then
this condition is not met.
If an entity’s interim
financial report is
If an entity’s interim described as complying
NAS 34 : Interim financial report is with NFRSs, it must
Financial Reporting described as comply with all of the
complying with requirements of this
Accounting for NFRSs, it must Standard. Paragraph 19
FY 2017-18 FY
Investment in comply with all of requires certain
2018-19
Associates as per the requirements of disclosures in that
equity method using this Standard. regard. However, an
uniform accounting Paragraph 19 entity shall not required
policies requires certain to restate its
disclosures in that corresponding previous
regard. interim period balance if
it is impracticable to
restate.

Answer Number 20
a. The Framework acknowledges a variety of measurement bases including historical cost,
current cost, net realisable value (NRV) and present value. It refers to NRV as a settlement
value which will be determined by a future transaction. Thus in order to determine NRV, the
directors would need to refer to NAS 2 Inventories for the definition and NAS 10 Events after
the Reporting Date. The directors should consider any adjusting events which provide
evidence of conditions which existed at the end of the reporting period in order to
determine NRV.

NAS 2 defines NRV as the estimated selling price in the ordinary course of business less the
costs of completion and costs of sale. In this case, the NRV will be determined on the basis of
conditions which existed at the date of the statement of financial position. NFRS 13 Fair Value
Measurement does not apply to NAS 2 as regards NRV even though the measurement
method is very similar. Any future price movements will be considered if they provide
information about the conditions at the date of the statement of financial position but
normally these movements would reflect changes in the market conditions after that date
and therefore would not affect the calculation of NRV. The NRV will be based upon the most
reliable estimate of the amounts which will be realised for the coal. The year-end spot price
will provide good evidence of the realisable value of the inventories and where the company
has an executory contract to sell coal at a future date, then the use of the forward contract
price may be appropriate. However, if the contract is not executory but is a financial
Page 187 of 328
Paper1: Advanced Financial Reporting [RTP 2019 June]
instrument under NFRS 9 Financial Instruments or an onerous contract recognised as a
provision under NAS 37 Provisions, Contingent Liabilities and Contingent Assets, it is unlikely
to be used to calculate NRV.

b) Coal Goal Ltd. should calculate the NRV of the low carbon coal using the forecast market
price based upon when the inventory is expected to be processed and realised. Future
changes in the forecast market price or the processing and sale of the low carbon coal may
result in adjustments to the NRV. As these adjustments are changes in estimates, NAS 8
Accounting Policies, Changes in Accounting Estimates and Errors will apply with the result
that such gains and losses will be recognised in the statement of profit or loss in the period in
which they arise.
Page 188 of 328
Paper1: Advanced Financial Reporting [RTP 2019 December]
Revision Questions:
1. Investment Property (Numerical) - Financial Reporting Standards
Delta is an entity which is engaged in the construction industry and prepares financial statements
to 30 September each year. The financial statements for the year ended 30 September 2015 are
shortly to be authorized for issue. The following events are relevant to these financial statements:

a) On 1 October 2000, Delta purchased a large property for $20 million and immediately began
to lease the property to Epsilon on an operating lease. Annual rentals were $2 million.
b) On 30 September 2014, the fair value of the property was $26 million. Under the terms of the
lease, Epsilon was able to cancel the lease by giving six months’ notice in writing to Delta.
c) Epsilon gave this notice on 30 September 2014 and vacated the property on 31 March 2015.
On 31 March 2015, the fair value of the property was $29 million.
d) On 1 April 2015, Delta immediately began to convert the property into ten separate flats of
equal size which Delta intended to sell in the ordinary course of its business.
e) Delta spent a total of $6 million on this conversion project between 31 March 2015 and 30
September 2015.
f) The project was incomplete at 30 September 2015 and the directors of Delta estimate that
they need to spend a further $4 million to complete the project, after which each flat could be
sold for $5 million.
g) Delta uses the fair value model to measure property whenever permitted by International
Financial Reporting Standards.

Required

Explain and show how the events would be reported in the financial statements of Delta for the
year ended 30 September 2015.

2. Earnings Per Share (Simple)- Financial Reporting Standards


The profit after tax earned by AAZ Limited during the year ended December 31, 2016 amounted
to Rs. 127.83 million. The weighted average number of shares outstanding during the year were
85.22 million.

Details of potential ordinary shares as at December 31, 2016 are as follows:


a) The company had issued debentures which are convertible into 3 million ordinary shares. The
debenture holders can exercise the option on December 31, 2018. If the debentures are not
converted into ordinary shares they shall be redeemed on December 31, 2018. The interest on
debentures for the year 2016 amounted to Rs. 7.5 million.
b) Preference shares issued in 2013 are convertible into 4 million ordinary shares at the option of
the preference shareholders. The conversion option is exercisable on December 31, 2020. The
dividend paid on preference shares during the year 2016 amounted to Rs. 2.45 million.
c) The company has issued options carrying the right to acquire 1.5 million ordinary shares of
the company on or after December 31, 2016 at a strike price of Rs. 9.90 per share. During the
year 2016, the average market price of the shares was Rs. 11 per share.
d) The company is subject to income tax at the rate of 30%.

Required
(a) Compute basic and diluted earnings per share.
(b) Prepare a note for inclusion in the company’s financial statements for the year ended
December 31, 2016 in accordance with the requirements of International Accounting
Standards.
Page 189 of 328
Paper1: Advanced Financial Reporting [RTP 2019 December]
3. Earnings Per Share (Complex)- Financial Reporting Standards
The following information pertains to ABC Limited, in respect of year ended March 31, 2016.
Rs. in ‘000
Consolidated profit for the year (including non-controlling interest) 15,000
Profit attributable to non-controlling interest 2,000
Dividend paid during the year to ordinary shareholders 4,000
Dividend paid on 10% Cumulative preference shares for the year 2015 2,000
Dividend paid on 10% Cumulative preference shares for the year 2016 2,000
Dividend declared on 12% Non-cumulative preference shares for the year 2016 2,400

(i) The company had 10 million ordinary shares at March 31, 2015.
(ii) The cumulative preference shares were issued at the time of inception of the company.
(iii) The 12% non-cumulative preference shares are convertible into ordinary shares, on or
before December 31, 2017 at a premium of Rs. 2 per share. The conversion rights are not
adjusted for subsequent bonus issues.
(iv) 0.50 million non-cumulative preference shares were converted into ordinary shares on
July 1, 2015.
(v) The dividend declared on the non-cumulative preference shares, as referred above, was
paid in April 2016.
(vi) 1.20 million right shares of Rs. 10 each were issued at a premium of Rs. 1.50 per share on
October 1, 2015. The market price on the date of issue was Rs. 12.50 per share.
(vii) 20% bonus shares were issued on January 1, 2016.
(viii) Due to insufficient profit no dividend was declared during the year ended March 31,
2015.
(ix) The average market price for the year ended March 31, 2016 was Rs. 15 per share.

Required
Compute the basic and diluted earnings per share and prepare a note for inclusion in the consolidated
financial statements for the year ended March 31, 2016.

4. Share Based Payment (Logical)- Financial Reporting Standards


NFRS 2 requires an entity to recognise share-based payment transactions in its financial
statements. These include transactions with the employees or other parties where they are to be
settled in cash, other assets or equity instruments of the entity. The NFRS identifies three types of
share-based payment transaction and sets out the measurement principles and specific
requirements for each.

Required
(i) Suggest why there was a need for a standard in this area.
(ii) Identify and briefly explain the three types of share based payments recognised by NFRS 2.

5. Intangible Assets, Provisions, PPE (Comprehensive Logic) - Financial Reporting Standards

Brooklyn is a bio-technology company performing research for pharmaceutical companies. The


finance director has contacted your financial consulting company to arrange a meeting to discuss
issues relevant to the preparation of the financial statements for the year to 30th June 2016. Your
initial telephone conversation has provided the necessary background information.

a. On 1st August 2015 Brooklyn began investigating a new bio-process. On 1st September 2016,
the new process was widely supported by the scientific community and the feasibility project
was approved. A grant was then obtained relating to future work. Several pharmaceutical
companies have expressed an interest in buying the ‘know how’ when the project completes
Page 190 of 328
Paper1: Advanced Financial Reporting [RTP 2019 December]
in June 2017. The nominal ledger account set up for the project shows that the expenditure
incurred between 1st August 2015 and 30th June 2016 was Rs. 300,000 per month.

b. In August 2016, an employee lodged a legal claim against the company for damage to his
health as a result of working for the company for the two years through to 31st March 2015
when he had to retire due to ill health. He has argued that his health deteriorated as a result of
the stress from his position in the organisation. Brooklyn has denied the claim and has
appointed an employment lawyer to assist with contesting the case. The lawyer has advised
that there is a 25% chance that the claim will be rejected, 50% chance that the damages will
be Rs. 600,000 and 25% chance of Rs. 1 million. The company has an insurance policy that
will pay 10% of any damages to the company. The lawyer has said that the case could take
until 30th June 2019 to resolve. The present value of the estimated damages discounted at 8%
is Rs. 476,280 and Rs. 793,800 respectively.

c. Brooklyn owns several buildings, which include an administrative office in the centre of
London. The company has revalued these on a regular basis every five years and the next
valuation is due on 30th June 2018. Property prices have increased since the last review and
particularly for the London premises. The cost of engaging a professionally qualified valuer is
very expensive and so to reduce costs the finance director is proposing that the property
manager, who is a professionally qualified valuer, should value the London property and that
the increase in value should be included in the financial statements. The finance director is of
the opinion that the property prices may fall next year.

Required
Prepare notes for your meeting with the finance director which explain and justify the
accounting treatment of these issues, preparing calculations where appropriate and identifying
matters on which your require further information.

6. Consolidation, First Time Adoption and share based payment (Comprehensive Logic) -
Financial Reporting Standards

You are the financial controller of Omega, a listed entity which prepares consolidated financial
statements in accordance with International Financial Reporting Standards (IFRS). One of your
assistants, a trainee accountant, is involved in the preparation of the consolidated financial
statements for the year ended 31 March 2017. She is also involved in the preparation of the
individual financial statements for the entities in the group. She has sent you an email with the
following queries:

Dear Sir / Madam,

While preparing financial statements of the companies I am working in, I have face the following
issues. Please suggest me the way forward.

Query One
On 1 April 2016 we acquired a new subsidiary. This subsidiary has always prepared its financial
statements in $ but has used IFRS for the first time this year. Previously, they have used local
standards. This means that the comparative figures (they present comparatives for one year only),
taken from last year’s financial statements, will be based on local standards not IFRS. How do I
make sure we are comparing like with like in the current year individual financial statements of
the subsidiary? Please just give me the general procedure, rather than dealing with any specialised
exemptions.
Page 191 of 328
Paper1: Advanced Financial Reporting [RTP 2019 December]
Query Two
I notice that on 1 April 2016 we lent $50 million to a key supplier. The loan has an annual rate of
interest of 5%, with interest of $2·5 million payable on 31 March each year in arrears. The loan is
repayable on 31 March 2026 but I believe that if interest rates change, we might consider
assigning the loan to a third party. As it turns out, interest rates have fallen since 1 April 2016 and
the fair value of the loan asset at 31 March 2017 was $52 million. I have been told that this loan
asset should be measured at ‘fair value through other comprehensive income’. Why is this? I
thought loan assets were measured at amortised cost. If the loan asset is measured at fair value
through other comprehensive income, does the interest income get recorded in other
comprehensive income rather than profit or loss?

Query Three
I’m not sure whether we need to make any entries in respect of the equity settled share-based
payment scheme we started on 1 April 2016. I believe we granted options to 1,000 employees to
purchase 100 shares in Omega for a fixed price. The options vest on 31 March 2021 subject to
two conditions. The first vesting condition is that the employees remain employed by Omega
throughout the five-year period up to the date of vesting. Best estimates are that 900 of the 1,000
will stay for that period – only 25 left in the year ended 31 March 2017. The other condition is
that the Omega share price on 31 March 2021 should be at least $10. The share price on 31 March
2017 was only $8·50 so it doesn’t look like this condition is satisfied yet. I’ve also noticed that
the fair value of one share option was $1 on 1 April 2016, rising to $1·05 on 31 March 2017. Do
we need any accounting entries and, if so, what should they be?

Required: Provide answers to the three queries raised by the trainee accountant. Your answers
should refer to relevant provisions of International Financial Reporting Standards.

Sincerely,
Dipesh Lama
Account Trainee

7. Lease, Going Concern, Joint Arrangement (Comprehensive Logic with Numerical) -


Financial Reporting Standards

Delta is an entity which prepares financial statements to 31 March each year. Each year, the
financial statements are authorised for issue on 25 May. The following events have occurred
which are relevant to the year ended 31 March 2017:

Event (a)
On 1 April 2016, Delta purchased an asset for $771,000 and immediately leased this asset to
entity X. The lease term was for five years and the lease rental, receivable annually in arrears on
31 March, was $200,000. Delta incurred direct costs of $20,000 in arranging this lease. The
annual rate of interest implicit in this lease was 10%. Under the terms of the lease, entity X is
responsible for insuring the asset and for carrying out any necessary repairs and maintenance of
the asset. At a discount rate of 10% per annum the present value of $1 receivable annually in
arrears for five years is $3·80.

Event (b)
On 1 April 2016, Delta entered into a joint arrangement with entity Y to jointly operate a delivery
depot. Entity Y is located, and has major customers in, the same geographical region as Delta.
Delta and entity Y each made the following payments in respect of the arrangement on 1 April
2016:
• $25 million each to purchase a joint 25-year leasehold interest in a depot which was close to
both Delta and entity Y’s business premises. This depot was to act as headquarters for the
delivery vehicles (see below).
Page 192 of 328
Paper1: Advanced Financial Reporting [RTP 2019 December]
• $7·5 million each to purchase a fleet of delivery vehicles. The vehicles have an expected
useful life of five years, with no expected residual value.

Delta and entity Y agreed to jointly use the delivery vehicles to deliver products to their
customers, and to share the operating costs of the depot equally. Any delivery charges to
customers were levied by Delta and entity Y directly at the discretion of the individual entities.
During the year ended 31 March 2017, the total cash cost of operating the depot was $8 million.
This was paid equally by Delta and entity Y. In the year ended 31 March 2017, Delta charged its
customers a total of $2 million in delivery charges.

Event (c)
On 31 March 2017, Delta was owed $10m by entity Z. The amount was due for payment by 30
April 2017. Entity Z has been a customer for many years and has an excellent payment record. At
31 March 2017, there was no reason to suppose that entity Z would fail to pay the $10m owed to
Delta by 30 April 2017. By 20 April 2017, entity Z’s going concern status was in considerable
doubt.

Required:
Explain and state (where possible by quantifying amounts) how the three events would be
reported in the financial statements of Delta for the year ended 31 March 2017.

8. Income Taxes (Tax expenses and Deferred Tax – Simple Numerical) - Financial Reporting
Standards

Given below is the statement of profit or loss and other comprehensive income of Shakir
Industries for the year ended December 31, 2016:
FY 2016
Rs. m
Sales 143.00
Cost of goods sold (96.60)
Gross profit 46.40
Operating expenses (28.70)
Operating profit 17.70
Other income 3.40
Profit before interest and tax 21.10
Financial charges (5.30)
Profit before tax 15.80

Following information is available:


(i) Operating expenses include an amount of Rs.0.7 million paid as penalty to SECP on
noncompliance of certain requirements of the Companies Act, 2017.
(ii) During the year, the company made a provision of Rs. 2.4 million for gratuity. The actual
payment on account of gratuity to outgoing members was Rs. 1.6 million.
(iii) Lease payments made during the year amounted to Rs.0.65 million which include
financial charges of Rs.0.15 million. As at December 31, 2016, obligations against assets
subject to finance lease stood at Rs. 1.2 million. The movement in assets held under
finance lease is as follows:
Rs. m
Opening balance – 01/01/2016 2.50
Depreciation for the year (0.7)
Closing balance – 31/12/2016 1.80
Page 193 of 328
Paper1: Advanced Financial Reporting [RTP 2019 December]
(iv) The details of owned fixed assets are as follows:
Accounting Tax
Rs. m Rs. m
Opening balance – 01/01/2016 12.50 10.20
Purchased during the year 5.3 5.3
Depreciation for the year (1.10) (1.65)
Closing balance – 31/12/2016 16.70 13.85

(v) Capital work-in-progress as on December 31, 2016 include financial charges of Rs. 2.3
million which have been capitalised in accordance with NAS-23 Borrowing Costs.
However, the entire financial charges are admissible, under the Income Tax Act.
(vi) Deferred tax liability and provision for gratuity as at January 1, 2016 was Rs.0.55 million
and Rs.0.7 million respectively.
(vii) Applicable income tax rate is 35%.
Required: Based on the available information, compute the current and deferred tax expenses for
the year ended December 31, 2016.

9. Income Taxes (Tax expenses and Deferred Tax – Simple Numerical) - Financial Reporting
Standards

The following statement of financial position relates to Model Town Group, a public limited
company at 30 June 2016:
Rs.000
Assets:
Non-current assets:
Property, plant, and equipment 10,000
Goodwill 6,000
Other intangible assets 5,000
Financial assets (cost) 9,000
a. Total 30,000
Trade receivables 7,000
Other receivables 4,600
Cash and cash equivalents 6,700
b. Total 18,300
c. Total (a+b) 48,300

Equity and liabilities


Share capital 9,000
Other reserves 4,500
Retained earnings 9,130
a. Total equity 22,630

Non-current liabilities
Long term borrowings 10,000
Deferred tax liability 3,600
Employee benefit liability 4,000
b. Total non-current liabilities 17,600
Current tax liability 3,070
Trade and other payables 5,000
c. Total current liabilities 8,070
Total liabilities (b+c) 25,670
Total equity and liabilities (a+b+c) 48,300
Page 194 of 328
Paper1: Advanced Financial Reporting [RTP 2019 December]
The following information is relevant to the above statement of financial position:

(i) The financial assets are investments in equity. Model Town has made an irrevocable
election to recognise gains and losses on these assets in other comprehensive income.
However, they are shown in the above statement of financial position at their cost on
1 July 2015. The market value of the assets is Rs. 10.5 million on 30 June 2016.
Taxation is payable on the sale of the assets.

(ii) The stated interest rate for the long term borrowing is 8 per cent. The loan of Rs. 10
million represents a convertible bond which has a liability component of Rs. 9.6
million and an equity component of Rs.0.4 million. The bond was issued on 30 June
2016.

(iii) The tax bases of the assets and liabilities are the same as their carrying amounts in the
statement of financial position at 30 June 2016 except for the following:
(a)
Rs.000
Property, plant, and equipment 2,400
Trade receivables 7,500
Other receivables 5,000
Employee benefits 5,000
(b) Other intangible assets were development costs which were all allowed for tax
purposes when the cost was incurred in 2015.

(c) Trade and other payables include an accrual for compensation to be paid to
employees. This amounts to Rs. 1 million and is allowed for taxation when paid.

(iv) Goodwill is not allowable for tax purposes in this jurisdiction.

(v) Assume taxation is payable at 30%.

Required
Calculate the provision for deferred tax at 30 June 2016 after any necessary adjustments to the
financial statements showing how the provision for deferred taxation would be dealt with in the
financial statements. (Assume that any adjustments do not affect current tax. You should briefly
discuss the adjustments required to calculate the provision for deferred tax).

10. Write down the differences between a Principles Based System and a Rules Based System of
developing accounting standards?

11. Share Based payments (Numerical) - Financial Reporting Standards

a. Kappa prepares financial statements to 31 March each year. The following share-based
payment arrangements were in force during the year ended 31 March 2015:
(i) On 1 April 2013, Kappa granted options to 500 employees to subscribe for 400 shares
each in Kappa on 31 March 2017, providing the employees still worked for Kappa at
that time. On 1 April 2013, the fair value of each option was $1·50. In the year ended
31 March 2014, ten of these employees left Kappa and at 31 March 2014, Kappa
expected that 20 more would leave in the three-year period from 1 April 2014 to 31
March 2017.
Kappa’s results for the year ended 31 March 2014 were below expectations and at 31
March 2014 the fair value of each option had fallen to 25 cents. Therefore, on 1 April
Page 195 of 328
Paper1: Advanced Financial Reporting [RTP 2019 December]
2014 Kappa amended the exercise price of the original options. This amendment
caused the fair value of these options to rise from 25 cents to $1·45.

During the year ended 31 March 2015, five of the employees left and at 31 March
2015, Kappa expected that ten more would leave in the two-year period from 1 April
2015 to 31 March 2017. The results of Kappa for the year ended 31 March 2015 were
much improved and at 31 March 2015, the fair value of a re-priced option was $1·60.

(ii) On 1 April 2013, Kappa granted share appreciation rights to 50 senior employees.
The number of rights to which each employee becomes entitled depends on the
cumulative profit of Kappa for the three years ended 31 March 2016:
 1,000 rights per employee are awarded if the cumulative profit for the three-year
period is below $500,000.
 1,500 rights per employee are awarded if the cumulative profit for the three-year
period is between $500,000 and $1 million.
 2,000 rights per employee are awarded if the cumulative profit for the three-year
period exceeds $1 million.

On 1 April 2013, Kappa expected that the cumulative profits for the three-year period would be
$800,000. After the disappointing financial results for the year ended 31 March 2014, this
estimate was revised at that time to $450,000. However, given the improvement in results for the
year ended 31 March 2015, the estimate was revised again at 31 March 2015 to $1,100,000.

On 1 April 2013, the fair value of one share appreciation right was $1·10. This estimate was
revised to $0·90 at 31 March 2014 and to $1·20 at 31 March 2015. All the senior employees are
expected to remain employed by Kappa for the relevant three-year period. The rights are
exercisable on 30 June 2016.

Show how and where transactions would be reported in the financial statements of Kappa for the
year ended 31 March 2015.

12. Calculation of Distributable Profit – Banking Industry


BBL Bank Limited has earned profit of Rs. 150 Million. The profit earned has been calculated
using NFRS. The profit includes Rs. 10 million foreign exchange revaluation amount. The Bank
has also issued Rs. 50 million Debenture 3 years back to be redeemed in next 2 years. The bank
has booked Rs.316 million as Interest Income out of which Rs. 300 was received in cash. The
bank also has Rs. 25 million of Non-Banking Assets, Deferred tax assets of Rs. 15 million and
Actuarial Losses of Rs. 5 Million.
You are required to prepare statement of distributable profit as per NRB Directive.

13. Foreign Exchange (Numerical) - Financial Reporting Standards


DND Limited is a listed company, having its operations within Pakistan. During the year ended
December 31, 2016, the company contracted to purchase plants and machineries from a US
Company. The terms and conditions thereof, are given below:
(i) Total cost of contract = US$ 100,000.
(ii) Payment to be made in accordance with the following schedule:
Payment dates Amount Payable
on Signing contract July 01, 2016 USD 20,000
on shipment* Sep 30, 2016 USD 50,000
After Instalation & Test Jan 31, 2017 USD 30,000
*(risk and rewards of ownership are transferred on shipment)
Page 196 of 328
Paper1: Advanced Financial Reporting [RTP 2019 December]
The contract went through in accordance with the schedule and the company made all the
payments on time. The following exchange rates are available:

July 01, 2016 US 1 = Rs. 60.50


Sep 30, 2016 US 1 = Rs. 61.00
Dec 31, 2016 US 1 = Rs. 61.20
Jan 31, 2017 US 1 = Rs. 61.50
Required
Prepare journals to show how the above contract should be accounted for under NAS 21.

14. Concept of Various entities for applying NFRS


Ram has invested in several companies ranging from Small retail to big listed companies. He as a
investor, has received varieties of financial statements. Some state of being compiled with NFRS
some doesn't. You as a CA finalist clarify Mr. Ram about varieties of organizations and
applicability of NFRS.

15. Financial Instruments (Concept) - Financial Reporting Standards


On 1 July 2016, Passila Ltd, issued 20,000 8% debentures at Rs. 97.50. The security is
redeemable in five years’ time. The interest on the debentures is payable bi-annually on 30 June
and 31 December.

On 31 December 2016, the Company’s year-end date, the debentures were quoted on the Karachi
Stock Exchange for Rs. 96.00. The company accountant has suggested each of the following as
possible valuation basis for reporting the debentures liability on the statement of financial position
as at 31 December 2016:

(i) Face value of the debentures.


(ii) Face value of the debenture plus interest payment for five years.
(iii) Market value on the statement of financial position as at the year end.

Required
a) Determine the face value of the debentures and the proceeds accruing to the company.
b) Determine the amount and explain the nature of the differences between the face value and
the market value of the debentures on 1 July, 2016.
c) Distinguish between nominal and effective rate of interest.
d) Determine the nominal interest payable on the debentures for the year ended 31 December
2016.
e) State arguments for or against each of the suggested alternatives for reporting the debentures
liability on the statement of financial position as at 31 December 2016.

16. Related Party (Concept) - Financial Reporting Standards

Discuss whether the following events would require disclosure in the financial statements of the
RP Group, a public limited company, under NAS 24 Related party disclosures. The RP Group,
merchant bankers, has a number of subsidiaries, associates and joint ventures in its group
structure. During the financial year to 31 October 20X9 the following events occurred.

(a) The company agreed to finance a management buyout of a group company, AB, a limited
company. In addition to providing loan finance, the company has retained a 25% equity holding
in the company and has a main board director on the board of AB. RP received management fees,
interest payments and dividends from AB.
Page 197 of 328
Paper1: Advanced Financial Reporting [RTP 2019 December]

(b) On 1 July 20X9, RP sold a wholly owned subsidiary, X, a limited company, to Z, a public
limited company. During the year RP supplied X with secondhand office equipment and X leased
its factory from RP. The transactions were all contracted for at market rates.

(c) The retirement benefit scheme of the group is managed by another merchant bank. An
investment manager of the group retirement benefit scheme is also a non-executive director of the
RP Group and received an annual fee for his services of $25,000 which is not material in the
group context. The company pays $16m per annum into the scheme and occasionally transfers
assets into the scheme. In 20X9, property, plant and equipment of $10m were transferred into the
scheme and a recharge of administrative costs of $3m was made.

17. Change in Estimates, Policies and Error (Concept) - Financial Reporting Standards
Duncan Company has previously written off any expenditure on borrowing costs in the
period in which it was incurred.
The company has appointed new auditors this year. They have expressed the view that the
previous recognition of borrowing costs in the statement of profit or loss was in error. The
company has decided to correct the error retrospectively in accordance with NAS 8.
The financial statements for 2015 and the 2016 draft financial statements, both reflecting
the old policy, show the following:

Statements of changes in Equity (extract) ('000)


2015 2016
Retained Earnings Retained Earnings
Opening Balance 222,500 23,950
Profit after tax for the period 3200 4712
Dividends Paid (1750) (2500)
Closing Balance 23950 26162

Borrowing costs written off were Rs. 500,000 in 2015 and Rs. 600,000 in 2016. The
directors have calculated that borrowing costs, net of depreciation which should have
been included in property, plant and equipment had the correct policy been applied, are as
follows.
Rs.000
At 30 December 2014 400
At 31 December 2015 450
At 31 December 2016 180
Had the correct policy been in force depreciation of Rs. 450,000 would have been
charged in 2015 and Rs. 870,000 in 2016.
Required
Show how the change in accounting policy must be reflected in the statement of changes
in equity for the year ended 31 December 2016. Work to the nearest Rs.000.

18. Change in Estimates, Policies and Error (Concept) - Financial Reporting Standards

a. X Ltd is considering acquiring a machine. It has two options; cash purchase at a cost of Rs.
11,420,000 or a lease. The terms of the lease are as follows:
I. The lease period is for four years from 1 January 2016 with an annual rental of Rs.
4,000,000 payable on 31 December each year.
Page 198 of 328
Paper1: Advanced Financial Reporting [RTP 2019 December]
II. The lessee is required to pay all repairs, maintenance and other incidental costs. (iii)
The interest rate implicit in the lease is 15% p.a.

Note: Estimated useful economic life span of the machine is four years.

Required
(a) Prepare a schedule of the allocation of the finance charges in the books of X Limited for
the entire lease period.
(b) Prepare an extract of the Statement of Financial Position of X Limited for the year ended
31 December 2016.

19. Financial Instruments (Concept and Numerical) - Financial Reporting Standards

Waters Ltd acquired the following financial assets and liabilities in 2016.
1. On 1 September, Waters acquired 2,000 Rs. 100 nominal units of 7% treasury stock 2022 for Rs.
104.10 per unit. The gross redemption yield at the date of purchase was 6.30%. Waters does not
intend to hold the treasury stock until maturity, as the cash may be required in the meantime.
Interest is paid annually in arrears.
2. Waters buys and sells goods in Constantia, a country whose currency is the Prif (PR). On 3
December Waters enters into a futures contract to sell PR500,000 on 30 April 2017 at an agreed
price of PR1.98/Rs. 1. This contract is not part of a designated hedge. The cost of entering into the
contract was Rs. 750.
3. On 5 February Waters acquired 250,000 ordinary shares in Gilmour Ltd at Rs. 4.85 per share
incurring Rs. 35,000 attributable transaction costs.
4. On 1 July Waters sells goods to Mason for Rs. 500,000 on interest free credit payable 30 June
2017. The imputed rate of interest is 11%.
5. On 30 April Waters acquired 1,000 Rs. 100 nominal units of 8.5% treasury stock 2018 at Rs.
107.10 per unit. The gross redemption yield is 5.9%. Waters intends to hold the investment to
maturity. Interest is paid annually in arrears.
6. On 26 December Waters purchased Rs. 25,000 of quoted company loan notes. This asset has been
designated as being held for short-term trading purposes.
7. On 24 December Waters sold 10,000 shares 'short' in Wright Ltd for Rs. 3.60 each, hoping that
the share price would fall so that it could clear its position by buying the shares in January 2017 at
a lower price.

On 31 December 2016, the values are as follows:


1. Rs. 100 nominal units of 7% treasury stock 2022 are trading at Rs. 98.07 per unit at 31
December 2016. The gross redemption yield at that date is 7.3%.
2. The futures rate for a Prif contract with a delivery date of 30 April 2017 is PR1.99/Rs. 1.
3. The shares in Gilmour are now trading at Rs. 5.20 – Rs. 5.25 per share, having an average of
Rs. 5.05 during the year. Disposal costs would be 2% of the sale proceeds.
4. Amounts receivable from Mason remain outstanding at the reporting date. The imputed
interest rate for current sales is 12%.
5. The 8.5% treasury stock 2018 is now trading at Rs. 101.50 per unit and the gross redemption
yield is currently quoted at 7.48%.
6. The loan notes are now worth Rs. 25,500 due to the market being more confident that the
interest will be paid in full and on time.
7. Shares in Wright Ltd are now trading at Rs. 3.30 each.

Required Explain and calculate the impact of the above transactions on the financial statements of
Waters Ltd for the year ended 31 December 2016.
Page 199 of 328
Paper1: Advanced Financial Reporting [RTP 2019 December]
20. General Purpose Financial (Framework) - Financial Reporting Standards

The IFRS are generally accepted as accounting standards in the preparation of general purpose
financial statements in many countries of the world.

Required
a. Briefly explain the meaning of general purpose financial statements in accordance with IAS 1
(Presentation of Financial Statements)
b. Explain briefly any FOUR possible reasons for the prevalence of IFRS in many countries of the
world.
c. Explain the arguments in support and against financial reporting standards.
Page 200 of 328
Paper1: Advanced Financial Reporting [RTP 2019 December]
Suggested Answers/Hints:

1. Investment Property - Financial Reporting Standards


From 1 October 2000, the property would be regarded as an investment property since it is being
held for its investment potential rather than being owner occupied or developed for sale.

The property would be measured under the fair value model. This means it will be measured at its
fair value each year end, with any gains or losses on remeasurement recognised in profit or loss.

On 31 March 2015, the property ceases to be an investment property because Delta begins to
develop it for sale as flats.

The increase in the fair value of the property from 30 September 2014 to 31 March 2015 of $3
million ($29 million – $26 million) would be recognised in P/L for the year ended 30 September
2015. Since the lease of the property is an operating lease, rental income of $1 million (($2
million x 6/12) would be recognised in P/L for the year ended 30 September 2015.

When the property ceases to be an investment property, it is transferred into inventory at its then
fair value of $29 million. This becomes the initial ‘cost’ of the inventory.

The additional costs of $6 million for developing the flats which were incurred up to and
including 30 September 2015 would be added to the ‘cost’ of inventory to give a closing cost of
$35 million.

The total selling price of the flats is expected to be $50 million (10 x $5 million). Since the further
costs to develop the flats total $4 million, their net realisable value is $46 million ($50 million –
$4 million), so the flats will be measured at a cost of $35 million. The flats will be shown in
inventory as a current asset.

2. Earnings Per Share (Simple)- Financial Reporting Standards

Working notes for identifying Dilutive or Antidilutive


Earnings
Impact on per
Impact on Earning Increase in No. incremental Dilutive
Particulars Earning (after Tax) of Shares shares /Antidilutive
Ordinary Number 125,380,000 125,380,000 85,220,000 1.47 Basic EPS
Convertible Debentures 7,500,000 5,250,000 3,000,000 1.75 Antidilutive
Convertible Preference Dilutive
Shares 2,450,000 2,450,000 4,000,000 0.6125
150,000 Dilutive
Options - - 1.5m (11-9.9)/11 -

Notes to the financial statements for the year ended December 31, 2016

EARNINGS PER SHARE 2016

Basic attributable to ordinary shareholders Profit (Rupees) 125,383,000


Rs. 127,830,000 – Rs. 2,450,000 = Rs. 125,380,000

Weighted average number of ordinary shares outstanding during the year 85,220,000

Earnings per share - basic (Rupees) 1.47


Page 201 of 328
Paper1: Advanced Financial Reporting [RTP 2019 December]
Diluted profit after taxation (Rupees) 127,833,000

Weighted average number of ordinary shares, options and convertible


preference shares outstanding during the year 89,370,000
(85,220,000+4,000,000+150,000)

Earnings per share - diluted (Rupees) 1.430

3. Earnings Per Share (Complex)- Financial Reporting Standards

Extract from notes of ABC Limited


Notes to consolidated financial statements for the year ended March 31, 2016
Rs. in '000
Earnings per share basic
Profit after tax and non-controlling interest (15,000-2,000) 13,000
Dividend paid during the year to ordinary shareholders (Rs. 4,000) -
10% Cumulative preference dividend for 2015 (Rs. 2,000) -
10% Cumulative preference dividend for 2016 (2,000)
Dividend declared on 12% non-cumulative preference shares for 2016 (2,400)
Profit available for distribution to ordinary share holders 8,600

Diluted earnings per share


Profit available for distribution to ordinary share holders 8,600
Effect of dividend declared on 12% non-cumulative preference shares
Convertible into ordinary shares on or before December 31, 2017 2,400
11,000

April July Oct Jan Mar

3m 3m 3m 3m

No of shares
10,000,000 416,667 1,200,000 2,323,333 13,940,000

Right factor
Outstanding shares before the exercise of
rights at fair value 10,416,667 12.50 130,208,337.50
Rights issued at a premium of Rs. 1.5 1,200,000 11.50 13,800,000.00
11,616,667 144,008,337.50

Theoretical ex-right value per share 1,44,008,337.5 / 11,616,667 12.40


Right Factor 12.50/12.40 1.00833
Page 202 of 328
Paper1: Advanced Financial Reporting [RTP 2019 December]
Bonus Factor
Bonus issued on January 01, 2016 (20%) Adjusting factor (6 shares for 5 shares) = 1.2
Time Right
Month No of Shares Factor Factor Bonus Factor Total
Apr - June 10,000,000 3/12 1.00833 6/5 3,024,998.55
Jul - Sep 10,416,667 3/12 1.00833 6/5 3,151,040.26
Oct - Dec 11,616,667 3/12 - 6/5 3,485,000.10
Jan - Mar 13,940,000 3/12 - - 3,485,000.00
13,146,038.91
Total 13,146 ('000)

Basic Earnings per Shares = 8600 / 13146 = Rs. 0.65 per shares

Diluted EPS

April July Oct Jan Mar

3 months 9 months

No of Non-cumulative Pref. shares


2,500,000 500,000 (given) 2,000,000
(b/f) Total

Weighted no of convertible Pref Shares


= 2,500,000 * 3/12 * 10/ 12 + 2,000,000 * 9/12 * 10/ 12
= 1,770,833.33
= 1,770 ('000)

Diluted Earnings per Shares = (8600 +2400) / (13146 +1,770) = Rs. 0.74 per shares
The non-cumulative preference shares are anti-dilutive

4. Share Based Payment (Logical)- Financial Reporting Standards


i. The need for accounting standard regulation Share options are often granted to employees at an
exercise price that is higher than the market price of the shares.
Therefore, the options have no intrinsic value to the company and, prior to the issue of IFRS 2,
these transactions were not generally recognised until such time as the shares were issued. This
approach could be seen as resulting in a distortion of reported results between accounting
periods and leaving liabilities unrecorded.
In addition, the subject of accounting for share-based payments contains a number of other
contentious issues, notably relating to the measurement principles to be applied in recognising
the transactions. If employees agree to stay until their options vest, the organisation must
recognise the service they will provide in return, but how should this be valued? IFRS 2 was
therefore issued in February 2004 to provide comprehensive guidance on these matters.

ii.The three types of share based payments


These can be summarised as follows:
Equity-settled share-based payment transactions
The entity pays for goods or services by issuing equity instruments in the form of shares or
share options.
Page 203 of 328
Paper1: Advanced Financial Reporting [RTP 2019 December]
Cash-settled share-based payment transactions
The entity incurs a liability for goods or services and the settlement amount is based on the
price (or value) of the entity’s shares or other equity instruments.

Share based payments with cash alternatives


Transactions where an entity acquires goods or receives services and either the entity or the
supplier can choose payment to be a cash amount based on the price (or value) of the entity’s
shares or other equity instruments, or equity instruments of the entity.

5. Intangible Assets (Comprehensive) - Financial Reporting Standards

i. Development expenditure
NAS 38 on intangibles requires that research and development be considered separately:
Research – which must be expensed as incurred
Development – which must be capitalised where certain criteria are met.
It must first be clarified how much of the Rs. 3 million incurred to date (10 months at Rs.
300,000) is simply research and how much is development. The development element will only
be capitalised where the NAS 38 criteria are met. The criteria are listed below together with the
extent to which they appear to be met.

 The project must be believed to be technically feasible. This appears to be so as the feasibility
has been acknowledged.
 There must be an intention to complete and use/sell the intangible. Completion is scheduled
for June 2017
 The entity must be able to use or sell the intangible. Interest has been expressed in purchasing
the knowhow on completion
 It must be considered that the asset will generate probable future benefits. Confirmation is
required from Brooklyn as to the extent of interest shown by the pharmaceutical companies
and whether this is of a sufficient level to generate orders and to cover the deferred costs.
 Availability of adequate financial and technical resources must exist to complete the project.
The financial position of Brooklyn must be investigated. A grant is being obtained to fund
further work and the terms of the grant, together with any conditions, must be discussed
further.
 Able to identify and measure the expenditure incurred. A separate nominal ledger account has
been set up to track the expenditure.

If all of the above criteria are met, then the development element of the Rs. 3m incurred to date
must be capitalised as an intangible asset. Amortisation will not begin until commercial
production commences.

ii. Provision
Although the claim was made after the reporting period, NAS 10 considers this to be an adjusting
event after the reporting period. The employment of the individual dates back to 20X2 and so the
lawsuit constitutes a current obligation for the payment of damages as a result of this past event
(the employment).
The amount and the timing are not precisely known but the likelihood of payment of damages by
Brooklyn is probable and so a provision should be made for the estimated amount of the liability,
as advised by the lawyer. Disclosure, rather than provision, would only be appropriate if the
expected settlement was possible or remote, and the lawyer’s view is that a payment is more
likely than not.
Page 204 of 328
Paper1: Advanced Financial Reporting [RTP 2019 December]
It is not appropriate to calculate an expected value where there is only one event, instead a
provision should be made for the most likely outcome. The lawyer has various views on the
possible, but the most likely payout is Rs. 500,000 as this has a 50% probability. As settlement of
the provision is not anticipated until 2019, the provision should be discounted back at 8% to give
a liability of Rs. 476,280.
Provided that the payment from the insurance company is virtually certain, this should be shown
as an asset, also at its discounted value of Rs. 47,628, being 10% of the provision.

In both cases the discounting should be unwound over the coming three years through profit or
loss.

iii. Revaluation
NAS 16 on Property, Plant and Equipment does not impose a frequency for updating revaluations.
It simply requires a revaluation where it is believed that the fair value of the asset has materially
changed. Hence, if in the past there have been material differences between the carrying amount
and fair value at the 5 yearly review then Brooklyn should consider having more frequent
valuations following on from this year’s valuation.

Revaluations should be regular and not timed simply when property prices are at a peak. It is not
acceptable for Brooklyn to defer its next revaluation while values are low. If property prices do
fall in 2017, then it may be necessary to perform an impairment test in accordance with NAS 36
Impairment of assets.
If it is believed that an asset value has moved materially, then all assets in that class must be
revalued. Hence it is not sufficient for Brooklyn to just revalue the London property. NAS 16
does not require the valuation to be performed by an external party, and so the use of the property
manager to conduct the valuations is acceptable.

Notes to the financial statements will disclose that he is not independent of the company.

6. Consolidation, First Time Adoption and share based payment (Comprehensive Logic) -
Financial Reporting Standards

Dear Dipesh,

I am pleased to write you back addressing your queries.

Query One
When an entity adopts International Financial Reporting Standards (IFRSs) for the first time, the
entity needs to prepare an opening IFRS statement of financial position at the date of transition to
IFRS. This is a requirement of IFRS 1 First Time Adoption of International Financial Reporting
Standards. The date of transition to IFRS is the beginning of the earliest period for which the
entity provides comparative information. In our case, this date is 1 April 2015.

The opening IFRS statement of financial position should be prepared in accordance with IFRSs
which are in force for the current reporting period – in this case, the year ended 31 March 2017.

The statement of profit or loss and other comprehensive income, and the statement of changes in
equity, which are presented as comparative figures in the financial statements for the year ended
31 March 2017, shall also be prepared in accordance with IFRSs which are in force for the year
ended 31 March 2017.
Page 205 of 328
Paper1: Advanced Financial Reporting [RTP 2019 December]
In the first set of financial statements we will need a reconciliation of those amounts which were
previously reported under local standards in the previous year’s financial statements.

The reconciliation will be between the amounts reported in previous periods under local standards
and the equivalent amounts reported as comparatives in the current period under IFRSs. For us,
this will mean reconciling equity at 1 April 2015 and 31 March 2016, plus total comprehensive
income for the year ended 31 March 2016

Query Two
The measurement basis for financial assets is set out in IFRS 9 Financial Instruments. The
measurement basis depends on the business model for managing the financial asset and the
contractual cash flow characteristics of the financial asset.

In order for the financial asset to be measured at amortised cost, the contractual terms should give
rise to cash flows on specified dates which are solely payments of principal and interest on the
amounts outstanding. This condition is satisfied in the case of the loan you are querying.

There is, however, another condition to be satisfied. The asset should be held under a business
model whose objective is to hold the financial asset in order to collect the contractual cash flows.
This condition is not satisfied, given the possibility of assigning the loan should interest rates rise.

A financial asset is measured at fair value through other comprehensive income where the
‘contractual cash flow test’ is passed and the asset is held under a business model whose objective
is achieved both by collecting the contractual cash flows and by selling the financial asset. This
appears to be the case here, so classification as fair value through other comprehensive income
seems appropriate.
Where a financial asset is measured at fair value through other comprehensive income, the
interest income which is included in profit or loss is the same amount as would be recorded were
the asset to be measured at amortised cost. Therefore interest income of $2·5 million will be
recorded in profit or loss.
The increase in fair value of $2 million ($52 million – $50 million) will be recorded in other
comprehensive income.

Query Three
Under the provisions of IFRS 2 Share-based Payment, this arrangement is an equity settled share-
based payment.

IFRS 2 regulates the treatment of vesting conditions based on whether they are market based or
non-market based. A market based vesting condition is taken into account by reflecting it in the
measurement of the fair value of the option. It does not need to be considered subsequently as to
do so would result in double-counting. Therefore the condition relating to the share price can be
ignored after the fair value of $1 is determined. A non-market condition is taken into account by
reflecting it in the calculation of the number of options ultimately expected to vest. In this case,
that number would be 90,000 (900 x 100). The cost of the arrangement is recognised over the
vesting period, based on the fair value of the option at the grant date.

The amount recognised for the year ended 31 March 2017 would be $18,000 (90,000 x $1 x 1/5).
This amount is recognised as an employment cost (probably in profit or loss) and a corresponding
credit to equity.
Page 206 of 328
Paper1: Advanced Financial Reporting [RTP 2019 December]
I hope I have cleared out your queries. Please feel to reach for further information.

Yours,
Bigreko Senior

7. Lease, Going Concern, Joint Arrangement (Comprehensive Logic with Numerical) -


Financial Reporting Standards

Event a
It would appear that the lease of the asset to entity X is a finance lease. This is because entity X is
responsible for repairs, maintenance and insurance of the asset and because the present value of
the minimum lease payments by entity X is $760,000 (200,000 x $3·80). This is 98·6% of the fair
value of the asset at the inception of the lease ($771,000). Because the lease is a finance lease,
Delta will show a lease receivable – net investment in finance leases under non-current assets.

The carrying amount of the lease receivable on 1 April 2016 will be $791,000 ($771,000 +
$20,000). During the year ended 31 March 2017, Delta will recognise income from finance leases
in the statement of profit or loss. The amount recognised will be $79,100 ($791,000 x 10%).

Following recognition of the lease income and the rental payment from Delta on 31 March 2017,
the net investment in finance leases in the statement of financial position of Delta at 31 March
2017 will be $670,100 ($791,000 + $79,100 – $200,000).

Event b
The joint arrangement with entity Y is a joint operation because Delta and entity Y have equal
rights to the assets and joint obligations for the liabilities relating to the arrangement. In a joint
operation, the operators include their share of any jointly held assets.
Therefore the property, plant and equipment of Delta at 31 March 2017 will include:
– Leasehold property of $25m x 24/25 = $24m
– Plant and equipment of $7·5m x 4/5 = $6m

In a joint operation, the operators include their share of jointly incurred costs. Therefore the
statement of profit or loss of Delta for the year ended 31 March 2017 will include the following
costs:
– Amortisation of lease premium $1m.
– Depreciation of plant and equipment $1·5m
–Cash cost of operating the depot $4m.

Delta will also include its own discretionary delivery charges of $2m as a reduction in its
operating costs.

Event c
Doubts regarding the going concern status of a customer would normally be regarded as prima
facie evidence that any trade receivable had suffered impairment. In such circumstances an
impairment allowance equal to the expected losses would normally be appropriate. However,
IFRS 9 Financial Instruments requires the impairment assessment to be made at the reporting
date.
At the reporting date, the going concern status of Z was not in doubt, so in this case no allowance
is necessary. However, the information about the decline in the going concern status of Z after the
reporting date is a non-adjusting event after the reporting date.
Page 207 of 328
Paper1: Advanced Financial Reporting [RTP 2019 December]
Therefore whilst no impairment allowance is necessary, it will be necessary to disclose details of
the 20 April event at Z’s business premises and its impact on the collectability of Delta’s trade
receivable.

8. Income Taxes (Comprehensive Numerical) - Financial Reporting Standards

COMPUTATION OF TAX EXPENSE FOR THE YEAR ENDED DECEMBER 31, 2016

Rs.in million
Profit before tax 15.80
Add: Inadmissible expenses
Accounting depreciation (Rs. 1.1 million + Rs.0.7 million) 1.80
Financial charges on finance lease (Allowable in Nepal 0.15
Penalty paid 0.70
Provision for gratuity 2.40
20.85

Less: Admissible expenses


Tax depreciation 1.65
Lease payments 0.65
Payment of gratuity 1.60
Borrowing cost capitalised 2.30

Taxable profit for the year 14.50


Current tax expense @ 35% 5.075

Items Carrying amount Tax base Temp difference


Fixed Assets 16.70 13.85 2.85
CWIP 2.30 - 2.30
Gratuity Provision 1.50 - (1.50)
Total 3.65
Deferred tax expense @ 35% 1.28

Deferred tax liability (Opening) 0.55


Deferred tax expenses for the year 0.73
Deferred tax liability (Closing) 1.28

9. Income Taxes (Tax expenses and Deferred Tax – Simple Numerical) - Financial Reporting
Standards

Assets: Carrying amount Tax base Temp difference


Property, plant, and
10,000
equipment 2,400 (7,600)
Goodwill 6,000 6,000 -
Other intangible assets 5,000 - (5,000)
Financial assets (cost) 10,500 9,000 (1,500)
Trade receivables 7,000 7,500 500
Other receivables 4,600 5,000 400
Cash and cash equivalents 6,700 -
Page 208 of 328
Paper1: Advanced Financial Reporting [RTP 2019 December]
6,700
Sub Total (13,200)
Liabilities
Long term borrowings 9,600 10,000 (400)
Employee benefit liability 4,000 5,000 (1,000)
Current tax liability 3,070 3,070 -
Trade and other payables 5,000 4,000 1,000
Sub Total (400)
Total (13,600)
Deferred tax expense @ 30% (4,080.00)
Less existing liability (3600)
Charged to P & L (480.00)

10. Difference Between Principles Based System and a Rules Based System of developing
accounting standards.

A rules-based system requires preparers to understand and apply detailed rules to report specific
transactions.
A principles-based system requires preparers to use judgment in order to develop accounting
policies to report specific types of transactions and events.

Consider accounting for tangible non-current assets. An extreme rules-based approach would set
out precise requirements for each type of asset, for example:
'Plant and equipment should be depreciated on the straight line basis over a period not
exceeding four years.'

A principles-based approach would contain more general requirements, for example:


'The depreciable amount of an asset shall be allocated on a systematic basis over its useful
life…The depreciation method used shall reflect the pattern in which the asset's future economic
benefits are expected to be consumed by the entity.

Rules-based accounting standards often need to contain complex definitions and scope
exceptions. For example, if plant and equipment must be depreciated over four years, while other
classes of asset are depreciable over a longer period, the standard must define what is meant by
plant and equipment. Standards often contain further material that explains and interprets the rules
and this in turn may be supplemented by guidance and regulations relating to particular industries
or types of transaction. As a result, accounting standards and guidance can be voluminous.

A purely rules-based system has the following advantages:


 in theory, it results in financial statements being comparable between entities (or between
entities in the same industry);
 it may reduce the volume of explanation necessary in financial statements (as in theory there
is only one allowed accounting treatment for each type of transaction or event);
 it is suitable for large, complex economies; and
 it provides the 'answers' in almost all situations; preparers do not have to make judgments and
risk the consequences (e.g. litigation or reputational damage if a user makes a wrong decision
based on information in the financial statements).

A purely or mainly principles-based system has the following advantages:


 in theory, it is more likely than a rigid-rules based system to result in financial statements that
show a true and fair view/give a fair presentation;
 it encourages the use of professional judgment;
 it is less open to 'creative accounting' abuses as principles are harder to evade than rules; and
Page 209 of 328
Paper1: Advanced Financial Reporting [RTP 2019 December]
 arguably it is more flexible than a system of rules and can therefore cope better with a rapidly
changing business and economic environment.

11. Share Based payments (Numerical)- Financial Reporting Standards

Solution to Part a
The expected total cost of the arrangement at 31 March 2014 is

400 X $1·50 X (500 – 10 – 20) = $282,000.

Therefore $70,500 ($282,000 X ¼) would be credited to equity and debited to profit or loss for
the year ended 31 March 2014.

For the year ended 31 March 2015, the expected total cost of the originally granted options would
be
400 X $1·50 X (500 – 10 – 5 – 10) = $285,000.

The cumulative amount taken to profit or loss and recognised in equity at 31 March 2015 is
$142,500.

The additional cost of the repriced options must also be recognised over the three-year period to
31 March 2017. The total additional cost is

400 X ($1·45 – $0·25) X 475 = $228,000.

Therefore the amount recognised in the year ended 31 March 2015 is $76,000 ($228,000 X 1/3).

Therefore the total recognised in equity at 31 March 2015 is $218,500 ($142,500 + $76,000).

The amount recognised in equity would be shown as ‘other components of equity’. And the
charge to profit or loss for the year ended 31 March 2015 is $148,000 ($142,500 + $76,000 –
$70,500).

The amount recognised in profit or loss would be shown as an employment expense.

Solution to Part b

For the year ended 31 March 2014, the expected total cost will be

50 X 1,000 X $0·90 = $45,000.

The amount taken to profit or loss in the prior period, and recognised as a liability, will be

$15,000 ($45,000 X 1/3).

At 31 March 2015, the liability will be 50 X 2,000 X $1·20 X 2/3 = $80,000

Since the rights are exercisable on 30 June 2016, the liability will be non-current. The charge to
profit or loss for the year ended 31 March 2015 will be $65,000 ($80,000 – $15,000). This will be
included in employment expenses.
Page 210 of 328
Paper1: Advanced Financial Reporting [RTP 2019 December]
12. Calculation of Distributable Profit – Banking Industry
Amount in Million
Statement of Distributable Profit
Particulars Current Year
Net Profit/(loss) as per Statement of Profit or Loss 150.00
Appropriation
a. General Reserve (30.00)
b. Foreign Exchange Fluctuation fund (10*25%) (2.50)
c. Capital redemption reserve (50 / 5 yrs) (10.00)
d. Corporate social responsibility fund (1.50)
e. Employees training fund -
f. Other -
Profit/(loss) before regulatory adjustment 106.00
Regulatory Adjustments:
a. Interest Receivable (-)/previous accrued interest received(+) (316-300) (16.00)
b. Short loan loss provision in accounts(-)/reversal(+) -
c. Short provision for possible losses on investment(-)/reversal(+) -
d. Short provision for possible losses on Non-Banking Assets (-)/reversal(+) (25.00)
e. Deferred Tax Assets recognized(-)/reveral(+) (15.00)
f. Goodwill recognized (-)/Impairment of Goodwill(+) -
g. Bargain purchase gain recognized (-)/reversal(+) -
h. Actuarial Loss recognized (-)/reversal(+) (5.00)
i. Other (+/-) -
Profit or (Loss) after Regulatory Adjustments 45.00

13. Foreign Exchange (Numerical) - Financial Reporting Standards


Date Particulars Dr Cr
1-Jul-16 Advance to suppliers 1,210,000
Cash 1,210,000
(Amount paid on signing the contract. Exchange rate was Rs. 60.5/US$)

30-Sep-16 Advance to suppliers 3,050,000


Cash 3,050,000
(Amount paid on delivery. Exchange rate was Rs. 61/US$)

30-Sep-16 PPE in transit/ CWIP 6,090,000


Advance to suppliers 4,260,000
Payable to suppliers 1,830,000
(Recording of asset on the delivery date as risk and rewards are transferred to the company)

31-Dec-16 Exchange loss 6,000


Payable to suppliers 6,000
(Adjustment of exchange rate as of reporting date. Exchange rate was Rs. 60.5/US$)

31-Jan-17 Property, plant and Equipment 6,090,000


PPE (In transit/ in progress) 6,090,000
(Transfer the new plants and machineries to Property, Plant and Equipment)

31-Jan-17 Payable to suppliers 1,836,000


Exchange loss (Bal.) 9,000
Cash 1,845,000
(Final payment to supplier. Exchange rate was Rs. 61.5/US$1)
Page 211 of 328
Paper1: Advanced Financial Reporting [RTP 2019 December]
14. Concept of Various entities for applying NFRS

Any entity that publishes general purpose financial statements for external users and does not
have public accountability can use the NFRS for SMEs. An entity has ‘public accountability’ if it
files or is in the process of filing its financial statements with a securities commission or other
regulatory organization for the purpose of issuing any class of instrument in a public market or if,
as a main part of its business, it holds assets in a fiduciary capacity for a broad group of third
parties. Banks, insurance entities, securities brokers, and dealers and pension funds are examples
of entities that hold assets in a fiduciary capacity for a broad group of third parties. Note that size
is not the determining factor as to which entitles can use the NFRS for SMEs – the applicability is
based entirely on whether the entity has public accountability or not.

NFRS for SMEs is formulated and issued for application on the basis of IFRS for SMEs 2015
issued by IASB. The term small and medium-sized entities around the world have developed
their own definitions of SMEs for a broad range of purposes including prescribing financial
reporting obligations. Definitions include quantified criteria based on revenue, assets, employees
or other factors. Frequently, the term SMEs is used to mean or to include very small entities
without regard to whether they publish general purpose financial statements for external users.
SMEs often produce financial statements only for the use of owner-managers or only for the use
of tax authorities or other governmental authorities.

The NFRS for SMEs is intended for use by small and medium-sized entities (SMEs). Small and
medium-sized entities are entities that:
a) Do not have public accountability; and
b) Publish general purpose financial statements for external users.

For financial reporting purpose the entities can be classified as:


- Those having public accountability: NFRS shall be applicable.
- Those not having public accountability: NFRS for SME shall be applicable.
- Those not having public accountability other than SMEs: Micro Entities are required to
prepare financial statements in accordance with the Accounting Guidelines for Micro
Entities on consistent basis.

Definition of Public Accountability


Entities having economic significance are:
- Borrowings from banks or financial institutions or public funds or from entities holding
assets in fiduciary capacity of NRs 500 million or more;
- Balance Sheet total (without off-setting current liabilities with current assets) of NRs
1,000 million or more;
- Employing more than 300 employees including workers in an average;
- Annual Turnover of NRs 1,000 million or more;
- Holding assets in fiduciary capacity in excess of NRs 500 million (includes security
brokers handling demat account, micro finance and cooperatives)

What are the Micro Entities?


Micro entities are those entities with the following thresholds (all):
- Annual Turnover of NRs 100 million or less
- Borrowings from banks or financial institutions or public funds or from entities holding
assets in fiduciary capacity of NRs 50 million or less
- Balance Sheet Total of NRs 100 million (without off-setting current liabilities in current
assets) or less
- Holding assets in fiduciary capacity of NRs 50 million or less (includes security brokers
handling demat account, micro finance and cooperatives.
Page 212 of 328
Paper1: Advanced Financial Reporting [RTP 2019 December]
- An entity must meet all of the above thresholds in 2 consecutive years to qualify as a
micro-entity and once qualified, must exceed at least 1 of the above thresholds for 2
consecutive years to cease to qualify.

15. Financial Instruments (Concept) - Financial Reporting Standards


a) The face value of the debentures
Rs. 100 X 20,000 = Rs. 2,000,000
The amount accrued to the company as proceeds = Rs. 97.5 X 20,000 = Rs. 1,950,000

b) The difference between the face value and the market value of the debentures is Rs. 50,000.
This is as a result of discount allowed on the issue on the debentures. Discount on debentures
attracts investors.
c) Nominal interest rate is the rate based specifically on the face value of the loan capital. In
case of Passila Ltd., the nominal interest rate on the debentures is 8% per annum on Rs.
2,000,000. The effective interest is the rate based on the market value.
This is the actual value collected on issue which can be at par, discount or premium. For
Passila Ltd., the effective interest rate will be 8% of Rs. 1,950,000

d) The nominal interest payable Rs. 2,000,000 X 8% X 6 months ÷ 12 months = Rs. 80,000

e) (i) The face value of Rs. 2,000,000 will be the most appropriate valuation to be disclosed in
the Statement of financial position. The management may be interested in the quoted market
value or the proceeds, but for the sake of outside investors who would only be interested in
the company having good reputations devoid of trading losses, it is advisable that the face
value be adopted.

(ii) Disclosing the debentures’ liability at face value plus interest payment for five years may
seem proper in the eyes of external investors and credit institutions, but principally, it would
be wrong to credit debentures’ account with both the face value and the interest payments.
An interest payment on debentures is a revenue item which is debited to the Profit and Loss
Account.

(iii)Disclosing debentures’ liability at market value on the Statement of financial position


will amount to disclosure at replacement value. The market value should be disclosed.

16. Related Party (Concept) - Financial Reporting Standards

a. NAS 24 does not require disclosure of transactions between companies and providers of
finance in the ordinary course of business. As RP is a merchant bank, no disclosure is needed
between RP and AB. However, RP owns 25% of the equity of AB and it would seem
significant influence exists (NAS 28, greater than 20% existing holding means significant
influence is presumed) and therefore AB could be an associate of RP.NAS 24 regards
associates as related parties. The decision as to associate status depends upon the ability of RP
to exercise significant influence especially as the other 75% of votes are owned by the
management of AB. Merchant banks tend to regard companies which would qualify for
associate status as trade investments since the relationship is designed to provide finance.
NAS 28 presumes that a party owning or able to exercise control over 20% of voting rights is
a related party. So an investor with a 25% holding and a director on the board would be
expected to have significant influence over operating and financial policies in such a way as
to inhibit the pursuit of separate interests. If it can be shown that this is not the case, there
is no related party relationship. If it is decided that there is a related party situation then
all
Page 213 of 328
Paper1: Advanced Financial Reporting [RTP 2019 December]
material transactions should be disclosed including management fees, interest, dividends and
the terms of the loan.

b. NAS 24 does not require intragroup transactions and balances eliminated on consolidation to
be disclosed. NAS 24 does not deal with the situation where an undertaking becomes, or
ceases to be, a subsidiary during the year. Best practice indicates that related party
transactions should be disclosed for the period when X was not part of the group.

Transactions between RP and X should be disclosed between 1 July 20X9 an 31 October


20X9 but transactions prior to 1 July will have been eliminated on consolidation. There is no
related party relationship between RP and Z since it is a normal business transaction unless
either parties interests have been influenced or controlled in some way by the other party.

c. Employee retirement benefit schemes of the reporting entity are included in the NAS 24
definition of related parties. The contributions paid, the non current asset transfer ($10m) and
the charge of administrative costs ($3m) must be disclosed.

The pension investment manager would not normally be considered a related party. However,
the manager is key management personnel by virtue of his non-executive directorship.
Directors are deemed to be related parties by NAS 24, and the manager receives a $25,000
fee.

NAS 24 requires the disclosure of compensation paid to key management personnel and the
fee falls within the definition of compensation. Therefore, it must be disclosed.

17. Change in Estimates, Policies and Error (Concept) - Financial Reporting Standards

Statement of changes in equity (extract) for the year ended December 31, 2016
Retained earnings Retained earnings
2016 2015
Rs.000 Rs.000
Opening balance as reported 23,950 22,500
Change in accounting policy (W2) 450 400
––––––– –––––––
Re-stated balance 24,400 22,900
Profit after tax for the period (W1) 4,442 3,250
Dividends paid (2,500) (1,750)
––––––– –––––––
Closing balance 26,342 24–,400

Workings
(1) Revised profit
2016 2015
Rs.000 Rs.000
Per question 4,712 3,200
Add back: Expenditure for the year 600 500
Minus: Depreciation (870) (450)
–––––– ––––––
Revised profit 4,442 3,250
–––––– ––––––
Page 214 of 328
Paper1: Advanced Financial Reporting [RTP 2019 December]
(2) Prior period adjustment

The prior period adjustment is the reinstatement of the Rs. 400,000 asset on 1 January 2015
and the Rs. 450,000 asset at 1 January 2016. On 31 December 2016 the closing balance above
of Rs. 26,342,000 can be reconciled as the original Rs. 26,162,000 plus the reinstatement of
the remaining asset of Rs. 180,000.

18. Change in Estimates, Policies and Error (Concept) - Financial Reporting Standards

Year Opening Balance Finance Charge at 15% Rentals Closing Balance


2016 11,420 1,713 4,000 9,133
2017 9,133 1,370 4,000 6,503
2018 6,503 975 4,000 3,478
2019 3,478 522 4,000 -
Total 4,580 16,000

Statement of Financial Position (Extract) as at 31 December 2016


Rs.’000

Non-Current assets (Rs.11,420,000 – Rs.2,855,00) 8,565

Non-Current Liabilities (Obligation under lease) 6,503

Current Liabilities Obligation under lease


(Rs.9,133,000 – Rs.6,503,000) 2,630

Note: Annual Depreciation = 11,420,000 / 4 = 2,855,000

19. Financial Instruments (Concept and Numerical) - Financial Reporting Standards

Investment in 7% treasury stock 2022


As there is not an intention to hold the investment to maturity, the investment should be classified
as at fair value (with gains and losses recognised in profit or loss).

At initial recognition it will be measured at fair value which is the consideration given of Rs.
208,200. There is no interest received up to year end (first payment will be received on 31
October 2017)

The market value of the stocks at the reporting date is Rs. 196,140 and the revaluation loss of Rs.
12,060 will be recognised in profit or loss.

Futures Prif contract


The derivative will be classified as at fair value through profit or loss. Initial transaction costs
cannot be included as part of the carrying amount and therefore the fee of Rs. 750 will be
immediately charged to profit or loss. At the reporting date the contract is valued at the fair value
of PR1.99/Rs. 1 so the loss is Rs. 1,269 to be included in profit or loss and as a liability on the
statement of financial position

Investment in Gilmour Ltd


This would normally be classified as at fair value (with gains and losses recognised in profit or
loss).
Page 215 of 328
Paper1: Advanced Financial Reporting [RTP 2019 December]
On initial recognition it would be valued at fair value which would be the cost of Rs. 1,212,500.
The directly attributable transaction costs (Rs. 35,000) would be expensed to profit or loss.

At the reporting date the shares will be valued at fair value (Rs. 5.20 per share) ignoring selling
costs = Rs. 1,300,000. The revaluation gain of Rs. 87,500 will be recognised in profit or loss.

Amount receivable from Mason


On recording the sale, the revenue needs to be discounted at the imputed rate of interest of 11%.
Revenue recognised on 1 July is therefore Rs. 450,450 (500,000 * 1.11).
The receivable on the statement of financial position will include the accrued interest element of
Rs. 24,775 (Rs. 450,450 x 0.11 x 6/12) and so will be Rs. 475,225 (Rs. 450,450 + Rs. 24,775) in
total.
The accrued interest of Rs. 24,775 will be recognised as finance income. The receivable would
not be adjusted for any change in interest rates.

Investment in 8.5% treasury stock 2018


This would be classified as to be subsequently measured at amortised cost. On initial recognition,
it will be recorded at fair value, the cost of Rs. 107,100. Finance income will be credited to profit
or loss using the gross redemption yield of 5.9%.

Interest recognised in profit or loss will be Rs. 4,213 (Rs. 107,100 * 5.9% * 8/12).

The investment in the statement of financial position at 31 December 2016 will be at Rs. 107,100
plus Rs. 4,213 = Rs. 111,313. (No interest will have been received to date as it is paid annually in
arrears).

The market value is not reflected in the statement of financial position at 31 December 2016 but it
would be disclosed in accordance with IFRS 7.

Investment in loan notes


The investment has been classified as held for trading so it is accounted for as a financial asset at
fair value through profit or loss. On acquisition it will be recorded at its cost of Rs. 25,000. At the
reporting date the notes will be revalued to their fair value of Rs. 25,500 with the Rs. 500 uplift
being recognised in profit or loss

Selling shares short


On initial recognition, the journal would be:
Rs. Rs.
Dr Cash (10,000 *Rs. 3.60) 36,000
Cr Financial liability 36,000

At the reporting date the financial liability must be revalued to its fair value of Rs. 33,000:
Rs. Rs.
Dr Financial liability 3,000
Cr Statement of profit or loss 3,000

20. General Purpose Financial (Framework) - Financial Reporting Standards

a. A general purpose financial statement is a statement that is intended to meet the needs of users
who are not in a position to demand information that are tailored to their needs. Such information
is useful to existing and potential investors.
Page 216 of 328
Paper1: Advanced Financial Reporting [RTP 2019 December]
b. Reasons for the prevalence of IFRS in many countries

(i) Production of high quality financial statements It involves the preparation of financial
statements that have qualitative features, that is, faithful representation, error free,
neutrality, understandability etc.
(ii) Preparation of user-friendly financial statements Production of financial statements that
contains necessary information that will assist users of financial statements to make
crucial economic decisions.
(iii) Uniformity of financial statements prepared Financial accounting is a language of
business. This language must not be different across countries of the world for it to assist
international investors.
(iv) Access to international finances With a General Purpose Financial Statements, it would
be easier for multi-national entities to have more access internationally.
(v) Enhancement of major economic decisions High quality financial statements will assist
users of the statements to make informed and important financial decisions.
(vi) Comparability of financial statements General purpose financial statements enhance
comparability of financial information among similar industries.
(vii) Globalisation and integration With IFRS, Chartered Accountants become more mobile,
since the standards are the same across the countries.
(viii) Job Creation Jobs are created across countries and a Chartered Accountant in Nepal can
practice in other countries with convenience.

c. Arguments for and against financial reporting standards


Arguments for:
(i) They guide preparers and users of financial statements
(ii) Their use enhances objectivity and comparability of financial statement which would in
turn engender reliability
(iii) Their use helps to curtail or significantly narrow the divergence in the principles adopted
by preparers of financial statements.
(iv) Standards improve the uniformity of treatment of transactions in the financial statements
among companies thereby increasing the credibility and comparability of financial
statement.
(v) Standards compel organisations to disclose information which they may not want to
disclose had the standards not been in existence.
(vi) Standards reduce the number of choices in the methods used to prepare financial
statement thereby reducing the risk of creative accounting.
(vii) Foreign companies which are targets for takeovers or mergers can be more easily
evaluated.

Arguments against:
(i) The cost of setting up and maintaining a standard-setting apparatus is quite significant
and not all countries can afford it.
(ii) The standards cannot address all issues or transactions. There are some which are unique
and so rare/unusual that global standards are not and cannot be available for them.
(iii) Low level of details or explanations.
Page 217 of 328
Paper1:AdvancedFinancial Reporting [RTP 2020 June]
Revision Questions:
1. Accounting Standards Mix (Numerical and Logical)
Delta is an entity which prepares financial statements to 31 March each year. Each year, the
financial statements are authorized for issue on 25 May. The following events have occurred which
are relevant to the year ended 31 March 2017:

Event (a)
On 1 April 2016, Delta purchased an asset for $771,000 and immediately leased this asset to entity
X. The lease term was for five years and the lease rental, receivable annually in arrears on 31 March,
was $200,000. Delta incurred direct costs of $20,000 in arranging this lease. The annual rate of
interest implicit in this lease was 10%. Under the terms of the lease, entity X is responsible for
ensuring the asset and for carrying out any necessary repairs and maintenance of the asset. At a
discount rate of 10% per annum the present value of $1 receivable annually in arrears for five years
is $3·80.

Event (b)
On 1 April 2016, Delta entered into a joint arrangement with entity Y to jointly operate a delivery
depot. Entity Y is located, and has major customers in, the same geographical region as Delta. Delta
and entity Y each made the following payments in respect of the arrangement on 1 April 2016:

- $25 million each to purchase a joint 25-year leasehold interest in a depot which was close to
both Delta and entity Y’s business premises. This depot was to act as headquarters for the
delivery vehicles (see below).

- $7·5 million each to purchase a fleet of delivery vehicles. The vehicles have an expected useful
life of five years, with no expected residual value.

Delta and entity Y agreed to jointly use the delivery vehicles to deliver products to their customers,
and to share the operating costs of the depot equally. Any delivery charges to customers were levied
by Delta and entity Y directly at the discretion of the individual entities. During the year ended 31
March 2017, the total cash cost of operating the depot was $8 million. This was paid equally by
Delta and entity Y. In the year ended 31 March 2017, Delta charged its customers a total of $2
million in delivery charges.

Event (c)
On 31 March 2017, Delta was owed $10m by entity Z. The amount was due for payment by 30
April 2017. Entity Z has been a customer for many years and has an excellent payment record. At
31 March 2017, there was no reason to suppose that entity Z would fail to pay the $10m owed to
Delta by 30 April 2017. By 20 April 2017, entity Z’s going concern status was in considerable
doubt.

Required: Explain and state (where possible by quantifying amounts) how the events would
be reported in the financial statements of Delta for the year ended 31 March 2017.

2. Earnings Per Share – (Simple) Financial Reporting Standards


Following information pertains to Sajjad Limited (SL) for the year ended 31 December 2016:

i. The share capital of SL comprises of:


Particulars Rs. in million
Ordinary share capital (Rs. 100 each) 1,000
9% Class A preference shares (Rs. 100 each) 200
6% Class B preference shares (Rs. 100 each) 300
Page 218 of 328
Paper1:AdvancedFinancial Reporting [RTP 2020 June]
ii. Class A preference shares which were issued on 1 January 2014 are cumulative, nonconvertible
convertible and non-redeemable. These shares were issued at Rs. 77.22 per share i.e. at a
discount of Rs. 22.78 per share. These shareholders are entitled to annual dividend of 9% with
effect from 1 January 2017. At the time of issue, the market dividend yield on such type of
preference shares was 9% per annum.

iii. Class B preference shares which were issued on 1 January 2016 are noncumulative, non-
convertible and non-redeemable. The payment of dividend of these shares was made on 29
December 2016. These shareholders are also entitled to participate in any remaining profits
after adjusting dividend to ordinary and preference shareholders. Such remaining profits are
allocated between the Class B shareholders and the ordinary shareholders in such a manner that
the profit per share of ordinary shareholders is twice the profit per share of Class B shareholders.

iv. SL earned profit after tax of Rs. 150 million during the year ended 31 December 2016 and paid
interim dividend of Rs. 2.50 per share to ordinary shareholders.

Required:
Compute basic earnings per share for the ordinary shareholders for the year ended 31 December
2016.

3. Earnings Per Share (Complex)- Financial Reporting Standards


Following information pertains to Tiger Limited (TL):
Particulars Quarter ended 31-Dec-17 Half year ended 31-Dec-17
Profit after tax (Rs. in million) 140 239
Average market price per share (Rs.) 330 360
Ordinary shares
20 million shares of Rs. 100 each were outstanding as at 1 July 2017.
4 million shares were issued on 1 August 2017 at market price of Rs. 355 per share.
Convertible bonds
On 1 November 2016 TL issued 0.8 million 7% convertible bonds at par value ofRs. 1,000 each.
Each bond is convertible into 3 ordinary shares at any time prior tomaturity date of 31 October
2019. On inception the liability component was calculatedas Rs. 760 million. On the date of issue,
the prevailing interest rate for similar debt without conversion option was 9% per annum. 50% of
these bonds were converted into ordinary shares on 1 November 2017.
Warrants
On 1 January 2016, TL issued share warrants giving the holders right to buy 6 million ordinary
shares at Rs. 340 per share. The warrants are exercisable within a period of 2 years.
Applicable tax rate is 30%.
Required:
Compute basic and diluted earnings per share to be disclosed in statement of profit or loss for the
following periods: (Show all relevant workings)
(a) Quarter ended 31 December 2017 (06)
(b) Half year ended 31 December 2017 (07)

4. Consolidated Financial Statement (Complex)- Financial Reporting Standards


The draft statements of financial position of Ant Limited (AL), Bee Limited (BL) and Fly Limited
(FL) as at 31 December 2017 are as follows:
Rs. in million
Particulars AL BL FL
Assets
Property, plant and equipment 3,510 2,835 2,200
Page 219 of 328
Paper1:AdvancedFinancial Reporting [RTP 2020 June]
Investment property 130 45 -
Investment in BL at cost 3,540 - -
Investment in FL at cost - 2400 -
Current assets 2,120 1,420 2,800
TOTAL ASSETS 9,300 6,700 5,000
Equity and liabilities
Share capital (Rs. 10 each) 5,500 4,000 2,500
Retained earnings 2,000 1,314 1,000
Gratuity 25 - -
Current liabilities 1,775 1,386 1,500
TOTAL EQUITY AND LIABILITIES 9,300 6,700 5,000

Other information:
(i) Details of investments are as follows:
Rs. in million
Cost of Retained earnings
Date of investment Investor % holding Investee
investment of investee
1-Jan-2015 AL 65% BL 3100 520
1-Apr-2015 AL 10% BL 440 815
20-Jun-2017 BL 60% FL 2400 1150

(ii) On acquisition date of BL, fair value of its net assets was equal to their carrying value except a
plant whose fair value was Rs. 120 million whereas its carrying amount was Rs. 140 million.
Value in use and remaining useful life of the plant were Rs. 150 million and 10 years respectively
at that date.

(iii) At the date of acquisition of FL, fair value of its net assets recorded in the books was equal to
their carrying value. Further, a contingent liability of Rs. 70 million was disclosed in the financial
statements of FL. AL's legal adviser had at that time estimated that this claim would be settled
at Rs. 50 million. However, it was actually settled on 15 February 2018 at Rs. 40 million. Date
of authorization of FL's financial statements was 10 February 2018 and the claim were disclosed
as contingent liability in FL's financial statements.

(iv) On 1 July 2017 AL sold its office building having carrying value of Rs. 43 million to BL at its
fair value of Rs. 50 million. The building had a remaining useful life of 5 years on the date of
disposal. On the same date, BL rented out the building to Monkey Limited for one year.

AL group follows fair value model for investment property whereas BL uses cost model for
investment property. Fair value of the building on 31 December 2017 was Rs. 58 million.
(v) (v) On 31 December 2017 FL’s recoverable amount was estimated at Rs. 3,700 million.

(vi) AL group follows a policy of valuing the non-controlling interest at its proportionate share of
the fair value of the subsidiary's identifiable net assets.

(vii) The following information relates to AL's gratuity scheme for the year ended 31 December 2017:

Rs. in million
Contribution paid 70
Benefits paid 55
Current service cost 85
Re-measurement gain 10
Page 220 of 328
Paper1:AdvancedFinancial Reporting [RTP 2020 June]
During the year, payments made by AL were charged to profit or loss account. No further
adjustments have been made.

Discount rate and fair value of plan assets at 1 January 2017 were 12% per annum and Rs. 320
million respectively.

Required:
Prepare AL's consolidated statement of financial position as on 31 December 2017 in accordance
with the requirements of NFRSs.

5. Consolidated Statement of Cash Flows (Complex)- Financial Reporting Standards

Summarized consolidated statement of financial position of Vitz Limited (VL) as at 30 June 2018 is
presented below:
Assets 2018 2017 Equity & Liabilities 2018 2017
Rs in million Rs. in million
Property, plant & equipment 3,678 4,173 Share capital (Rs. 10 each) 2,800 2,500
Goodwill 569 639 Share Premium 300 -
Investment in associate 670 - Other group Reserves 3,519 2,451
Inventories 1,850 1,050 Non-Controlling Interest 1,638 874
Trade & Other receivables 975 823 Trade & Other Payables 912 1630
Cash and Bank 1,568 770 Deferred Consideration 223 -
9,392 7,455 9,392 7,455

Following information is available for preparation of consolidated statement of cash flows:

i. On 1 January 2018, VL acquired 40% shares in Audi Limited (AL) by paying Rs. 600 million. On
that date, cash balance of AL was Rs. 100 million. AL earned profit of Rs. 800 million (accrued
evenly) during the year ended 30 June 2018. Further, VL sold goods for Rs. 400 million to AL in
2018 at 30% profit margin. 25% of these goods remained unsold on 30 June 2018.

ii. On 1 April 2018, VL disposed of its 100% shareholdings in Subaro Limited (SL) for Rs. 1,600
million. On that date, carrying value of SL’s net assets was as follows:
Rs. in million
Property, plant and equipment 1,300
Working capital (other than bank balances) (150)
Bank balances 100
1,250
On the date of disposal, carrying value of SL's goodwill was Rs. 200 million. SL earned profit of Rs.
185 million (accrued evenly) during the year ended 30 June 2018.

iii. A building having carrying value of Rs. 170 million was disposed of during the year for Rs. 350
million in cash. Another machine having carrying value of Rs. 250 million was disposed of during
the year for Rs. 230 million which will be received in August 2018.

iv. During the year, VL disposed of 30% shareholdings (leaving 60% with VL) in Wing Limited (WL)
for Rs. 450 million when WL’s net assets and goodwill were Rs. 1,000 million and Rs. 150 million
respectively.

v. On 1 July 2017, VL acquired its first foreign subsidiary, Ford Limited (FL) by purchasing 80%
shareholdings against:
• immediate cash payment of Rs. 495 million (USD 4.5 million)
• issuance of 15 million shares of VL at market value of Rs. 25 each.
Page 221 of 328
Paper1:AdvancedFinancial Reporting [RTP 2020 June]
• deferred payment of USD 2 million payable after two years. Applicable discount rate is 8%.

The fair value of net assets of FL at the date of acquisition was as follows:
USD in million
Property, plant and equipment 5.5
Working capital (other than bank balances) 3.5
Bank balances 1.0
10.0
FL earned profit of USD 1.5 million (accrued evenly) during the year ended 30 June 2018. FL’s
goodwill was not impaired at year-end.

Exchange reserve on translation of FL comprises of Rs. 13 million for bank balances, Rs. 36 million
for working capital (other than bank balances) and the remaining relates to goodwill and property,
plant and equipment.

vi. Following exchange rates are available:


1-Jul-17 31-Dec-17 30-Jun-18 Average
1 USD to Rs. 110 115 120 116

vii. No dividend was paid during the year by the group.

viii. Depreciation for the year was Rs. 480 million.

ix. VL values non-controlling interest on the date of acquisition at its proportionate share of the fair
value of the subsidiaries' identifiable net assets.

Required
Prepare VL’s consolidated statement of cash flows for the year ended 30 June 2018 using 'indirect
method' in accordance with NFRS. (Ignore corresponding figures)

6. Share based payment (Comprehensive logic with numerical) - Financial Reporting Standards

Query One
On 1 January 2014, Corolla Limited (CL) granted share options to each of its 50 executives to
purchase CL’s shares at Rs. 1,000 per share. In this respect following information is available:

(i) The share options will vest and become exercisable upon completion of 3 years provided that:
• The executives remain in service till the vesting date.
• CL’s share price increases to Rs. 1,500 per share.

(ii) Each executive will receive 4,000 share options if average annual gross profit during the vesting
period is atleast Rs. 900 million. However, if the average gross profit exceeds Rs. 1,000 million
each executive would be entitled to 6,000 share options.

(iii) On 1 January 2016, CL extended the vesting period to 31 December 2017 and reduced the
exercise price to Rs. 900 per share. On 1 January 2016, fair value of each share option was Rs. 580
for the original share option granted (i.e. before taking into account the re-pricing) and Rs. 710 for
re-priced share option.

Following further information is also available:


1st Jan 31 December
2014 2014 2015 2016 2017
Executives in employment 50 47 44 43 42
Executives expected to leave during 12 8 4 2 -
Page 222 of 328
Paper1:AdvancedFinancial Reporting [RTP 2020 June]
remaining vesting period
Gross profit for the year (Rs. in million) - 940 820 1,270 1,200
Fair value of each share (Rs.) 1,400 1,450 1,550 1,480 1,650
Fair value of each option (Rs.) 600 650 580 650 750

At each year-end, CL estimated that gross profit for the future years would approximately be the
same as of current year.
Required:
Calculate the amounts recorded in respect of share options in CL’s financial statements for the years
ended 31 December 2014, 2015, 2016 and 2017 and explain the basis of your calculations.

Query Two
Newtown Ltd granted 1,000 share appreciation rights (SARs) to each of its 500 employees on 1
July 2014. To be eligible for the rights, employees must remain employed by Newtown Ltd for 3
years from the date of grant. The rights must be exercised in July 2017, with settlement due in cash.

In the year to 30 June 2015, 42 employees left and a further 75 were expected to leave over the
following two years.

In the year to 30 June 2016, 28 employees left and a further 25 were expected to leave in the
following year.

The fair value of each SAR was Rs. 90 at 30 June 2015 and Rs. 110 at 30 June 2016.

Required:
Prepare the journal entry to record the expense associated with the SARs for the year ended 30 June
2016, in accordance with NFRS 2 Share-based payment.

7. Employee Benefits (Comprehensive Logic with Numerical) - Financial Reporting Standards

(a) Explain the following as used in NAS 19 Employee Benefits:


(i) The term ‘defined benefit pension plan’
(ii) The basis to be adopted in measuring scheme assets
(iii) The basis to be adopted in measuring scheme liabilities
(iv) Actuarial gains and losses.

(b) Universal Solutions operates a defined benefit pension scheme on behalf of its employees.
The company conducts an annual review of funding in conjunction with their actuaries who
have supplied the following information:
At Dec 31 20X3 At Dec 31 20X4
Rs. Rs.
Present value of pension fund obligations 1,200 1,300
Market value of pension fund assets 1,000 1,100

Information relevant to the actuarial valuation:


Discount rate used to determine pension fund liabilities 5%
Current service cost Rs. 100
Contributions to the pension fund Rs. 140
Benefits paid out amounted to Rs. 95

Required:
Explain and state (where possible by quantifying amounts) how the three events would be reported
in the financial statements of Delta for the year ended 31 March 2017.
Page 223 of 328
Paper1:AdvancedFinancial Reporting [RTP 2020 June]
8. (Simple Logical) - Financial Reporting Standards

Epsilon is an entity which prepares financial statements to 30 September each year.

(a) Purchase of machine

On 1 April 2018, Epsilon accepted delivery of a large and complex machine from an overseas
supplier. The agreed purchase price for the machine was 20 million francs – the functional currency
of the supplier. Under the terms of the agreement with the supplier 12·6 million francs was payable
on 31 July 2018, with the balance of 7·4 million francs being payable on 30 November 2018. The
payment due on 31 July 2018 was made in accordance with the terms of the agreement. Epsilon
does not use hedge accounting.

On 1 April 2018, Epsilon incurred direct costs of $250,000 in installing the machine at its premises.
Although the machine was ready for use from 1 April 2018, Epsilon did not bring the machine into
use until 30 April 2018.
During April 2018 Epsilon incurred costs of $200,000 in training relevant staff to use the machine.
The directors of Epsilon estimate that the machine is capable of being usefully employed in the
business until 31 March 2023, and that it will have no residual value at that date.

(b) Decommissioning

On 31 March 2023, Epsilon will be legally required to decommission the machine using the original
supplier.
The directors of Epsilon estimate that the cost of safely decommissioning the machine on 31 March
2023 will be 3 million francs.
Note: A relevant annual rate to be used in any discounting calculations is 8% and the appropriate
discount factor is 0·681.

(c) Impairment review

During the final few months of the accounting period ending on 30 September 2018, Epsilon
experienced difficult trading conditions. These difficulties did not affect the ability of Epsilon to
operate as a going concern. In an impairment review of the machine at 30 September 2018, the
directors of Epsilon estimated that the machine’s recoverable amount was $2·5 million.

Relevant exchange rates (francs to $1) are as follows:


– 1 April 2018 – 10 francs to $1.
– 30 April 2018 – 9·5 francs to $1.
– 31 July 2018 – 9 francs to $1.
– 30 September 2018 – 8 francs to $1.
– Average rate for the period from 1 April 2018 to 30 September 2018 – 9·2 francs to $1.

Required:
Explain and show with appropriate calculations how the above events would be reported in the
financial statements of Epsilon for the year ended 30 September 2018.

9. Income Taxes (Tax expenses and Deferred Tax – Simple Numerical) - Financial Reporting
Standards

The following statement of financial position relates to Model Town Group, a public limited
company at 30 June 2016:
Rs.000
Assets:
Non-current assets:
Page 224 of 328
Paper1:AdvancedFinancial Reporting [RTP 2020 June]
Property, plant, and equipment 10,000
Goodwill 6,000
Other intangible assets 5,000
Financial assets (cost) 9,000
a. Total 30,000
Trade receivables 7,000
Other receivables 4,600
Cash and cash equivalents 6,700
b. Total 18,300
c. Total (a+b) 48,300

Equity and liabilities


Share capital 9,000
Other reserves 4,500
Retained earnings 9,130
a. Total equity 22,630

Non-current liabilities
Long term borrowings 10,000
Deferred tax liability 3,600
Employee benefit liability 4,000
b. Total non-current liabilities 17,600
Current tax liability 3,070
Trade and other payables 5,000
c. Total current liabilities 8,070
Total liabilities (b+c) 25,670
Total equity and liabilities (a+b+c) 48,300

The following information is relevant to the above statement of financial position:

(i) The financial assets are investments in equity. Model Town has made an irrevocable
election to recognise gains and losses on these assets in other comprehensive income.
However, they are shown in the above statement of financial position at their cost on 1
July 2015. The market value of the assets is Rs. 10.5 million on 30 June 2016. Taxation
is payable on the sale of the assets.

(ii) The stated interest rate for the long term borrowing is 8 per cent. The loan of Rs. 10
million represents a convertible bond which has a liability component of Rs. 9.6 million
and an equity component of Rs.0.4 million. The bond was issued on 30 June 2016.

(iii) The tax bases of the assets and liabilities are the same as their carrying amounts in the
statement of financial position at 30 June 2016 except for the following:
(a)
Rs.000
Property, plant, and equipment 2,400
Trade receivables 7,500
Other receivables 5,000
Employee benefits 5,000
(b) Other intangible assets were development costs which were all allowed for tax
purposes when the cost was incurred in 2015.

(c) Trade and other payables include an accrual for compensation to be paid to
employees. This amounts to Rs. 1 million and is allowed for taxation when paid.
Page 225 of 328
Paper1:AdvancedFinancial Reporting [RTP 2020 June]
(iv) Goodwill is not allowable for tax purposes in this jurisdiction.

(v) Assume taxation is payable at 30%.

Required
Calculate the provision for deferred tax at 30 June 2016 after any necessary adjustments to the
financial statements showing how the provision for deferred taxation would be dealt with in the
financial statements. (Assume that any adjustments do not affect current tax. You should briefly
discuss the adjustments required to calculate the provision for deferred tax).

10. What are the agreed Actions under the Nepal Portfolio Performance Review (NPPR )14
Action Plan 2010? (theory)

11. Define following terms with respect to Public Financial Management: (Theory)
a. Treasury Single Account (TSA)
b. PEFA Assessment
c. NPSAS

12. Related Parties (Simple Logical) - - Financial Reporting Standards


a. Define the following:
• Related Party
• Key Management Personnel

b. What are the disclosure requirements as per NAS 24?

13. Foreign Exchange (Numerical) - Financial Reporting Standards


KL purchased an investment property in United States for USD 2.6 million. 10% advance payment
was made on 1 May 2017 and 70% payment was made on 1 July 2017 on transfer of title and
possession of the property. The remaining amount was paid on 1 August 2017.

On 1 September 2017, KL rented out this property at annual rent of USD 0.24 million for one year
and received full amount in advance on the same date.

KL uses fair value model for its investment property. On 31 December 2017, an independent valuer
determined that fair value of the property was USD 2.5 million.

Following spot exchange rates are available:


1-May-17 1-Jul-17 1-Aug-17 1 Sept-17 31-Dec-17
1 USD to Rs. 110 105 108 110 116

Following average exchange rates are also available:


2017 Jul to Dec 2017 Sep to Dec 2017
1 USD to Rs. 105 111 113

Required
Prepare the extracts relevant to the above transactions from KL’s statements of financial position
and comprehensive income for the year ended 31 December 2017, in accordance with the NFRSs.

14. Inventories (Simple Numerical) – Financial Reporting Standards


On 31 March 20X1, the inventory of ABC includes spare parts which it had been supplying to a
number of different customers for some years. The cost of the spare parts was ` 10 million and
based on retail prices at 31 March 20X1, the expected selling price of the spare parts is ` 12 million.
On 15 April 20X1, due to market fluctuations, expected selling price of the spare parts in stock
Page 226 of 328
Paper1:AdvancedFinancial Reporting [RTP 2020 June]
reduced to ` 8 million. The estimated selling expense required to make the sales would ` 0.5 million.
Financial statements were authorised by Board of Directors on 20th April 20X1.

As at 31st March 20X2, Directors noted that such inventory is still unsold and lying in the
warehouse of the company. Directors believe that inventory is in a saleable condition and active
marketing would result in an immediate sale. Since the market conditions have improved, estimated
selling price of inventory is 11 million and estimated selling expenses are same 0.5 million.

What will be the value inventory at the following dates?

(a) 31st March 20X1


(b) 31st March 20X2

15. Conceptual - Financial Reporting Standards


Company A has taken a long term loan arrangement from Company B. In the month of December
20X1, there has been a breach of material provision of the arrangement. As a consequence of which
the loan becomes payable on demand on March 31, 20X2. In the month of May 20X2, the Company
started negotiation with the Company B for not to demand payment as a consequence of the breach.
The financial statements were approved for the issue in the month of June 20X2. In the month of
July 20X2, both company agreed that the payment will not be demanded immediately as a
consequence of breach of material provision.

Advise on the classification of the liability as current / non –current.

16. Value Added Statement (Simple Numerical)

The following data is given in respect of Samriddhi Ltd. for the year ended 31 3-20X1:
Abstract of Statement of Profit & Loss for the year ended 31 3-20X1
` in ‘000 ` in ‘000
Income
Sale 2,380
Other Income 370 2,750
Expenditure
Operating Cost 1,855
Administrative Expenses 150
Interest Cost 215
Depreciation 240 2,460
Profit before tax 290
Provision for tax 87
Profit after tax 203
Credit balance as per last balance sheet 60
263
Other Information:
` in ‘000
1. Operating cost consists of:
Material cost 1,220
Wages, salaries & other benefits to employees 330
Local taxes including cess 70
Other manufacturing expenses 235
Page 227 of 328
Paper1:AdvancedFinancial Reporting [RTP 2020 June]
2. Administrative expenses consist of:
Directors’ Remuneration 55
Audit Fee 25
Provision for doubtful debts 8
Others 62
3. Interest cost consists of:
Interest on 10% debentures 180
Interest on temporary bank overdraft 35
4. The capital structure of the company consists of:
Equity share capital 1,500
9% Preference share capital 600

You are required to prepare a Gross Value Added (GVA) statement and calculate the
following ratios:
(a) GVA to Material Cost Ratio (Industry average 0.80)
(b) GVA to Employee Cost Ratio (Industry average 3.82)
(c) GVA to Sales Ratio (Industry average 0.70)
(d) GVA to Capital Employed Ratio (Industry average 0.30)
Also advise on the utility of the above ratios in comparison to the Industry average.

17. Financial Instruments (Conceptual Numerical) - Financial Reporting Standards


On 15 October 2016, Syangjali Industries Limited (SIL) made the following investments:
Name of Investee No. of shares Percentage of Cost of Investment
shareholding acquired (Rs in million)
Laure Ltd. (LL) 155,000 4% 20
Shaili Ltd (SL) 135,000 2% 65
*including transaction costs

a. Investment in LL was made with no intention to sell the shares while investment in BL was
made with the intention to sell the shares before 31 December 2016.
b. The board of directors in its meeting held on 30 November 2016 decided that since the future
prospects of SL are quite attractive, its shares should be held till 30 June 2018. The market rate
on 30 November 2016 was Rs. 621.
c. On 31 December 2016, RIL decided to record an impairment loss of Rs. 5 million against
investment in KL. The market price of shares of KL and BL as on 31 December 2016 was Rs.
80 and Rs. 600 respectively.
d. SIL’s broker normally charges transaction costs of 0.2%.

Required:
Explain the accounting treatment of above transactions in accordance with Nepal Financial
Reporting Standards.

18. Leases (Simple Numerical) - Financial Reporting Standards

On 1 April 2015 Acacia Ltd entered into the following lease agreements. The terms of each lease
are as follows:

1. Plant with a fair value of Rs. 275,000 was leased under an agreement which requires Acacia
Ltd to make annual payments of Rs. 78,250 on 1 April each year, commencing on 1 April 2015,
for four years. After the four years Acacia Ltd has the option to continue to lease the plant at a
nominal rent for a further three years and is likely to do so as the asset has an estimated useful
Page 228 of 328
Paper1:AdvancedFinancial Reporting [RTP 2020 June]
life of six years. The present value of the lease payments is Rs. 272,850. Acacia Ltd is
responsible for insuring and maintaining the plant during the period of the lease.

2. Office equipment with a fair value of Rs. 24,000 was leased under a non-cancellable agreement
which requires Acacia Ltd to make annual payments of Rs. 6,000 on 1 April each year,
commencing on 1 April 2015, for three years. The lessor remains responsible for insuring and
maintaining the equipment during the period of the lease. The equipment has an estimated
useful life of ten years. The present value of the lease payments is Rs. 16,415.Acacia Ltd
allocates finance charges on an actuarial basis. The interest rate implicit in both of the leases is
10%.
Required:
Prepare all relevant extracts from Acacia Ltd's financial statements for the year ended 31 March
2016 in respect of the above leases. The only notes to the financial statements required are those in
respect of lease liabilities or commitments.

19. Intangible Assets (Concept and Numerical) - Financial Reporting Standards

On 1 April 2017 ZL acquired a licence for operating a TV channel for Rs. 86.3 million out of which
Rs. 50 million was paid immediately. The balance amount is payable on 1 April 2019. A mega
social media and print media campaign was launched to promote the channel at a cost of Rs. 10
million. The transmission of the channel started on 1 August 2017.

The license is valid for 5 years but is renewable every five years at a cost of Rs. 35 million. Since
the renewal cost is significant, the management intends to renew the license only once and sell it at
the end of 8 years.

In the absence of any active market, the management has estimated that residual value of the license
would be Rs. 15 million and Rs. 20 million at the end of 5 years and 8 years respectively.
Applicable discount rate is 10% p.a.

Required:
Discuss how these transactions should be recorded in ZL’s books of accounts for the year ended 31
December 2017.

20. Provisions, Contingent Assets and Contingent Liabilities (Conceptual) - Financial Reporting
Standards

You will be aware that the board of directors met on 10 March 20X7 to discuss over-capacity in
parts of the group. The decision was reluctantly taken to implement a programme of redundancies.
The programme was to be implemented in two phases:

- Phase 1 involves 300 redundancies on 30 June 20X7. This phase of the programme was planned
out in detail at the meeting on 10 March 20X7. The redundancy costs were calculated in some
detail at the meeting and this first phase was made public to all affected parties on 25 March
20X7.

- Phase 2 involves 200 redundancies on 30 September 20X7. This phase of the programme was
also planned out in detail at the meeting on 10 March. The redundancy costs were estimated at
the meeting and this second phase was announced on 25 April 20X7.

The financial statements for the year ended 31 March 20X7 include a provision for the first phase
of the redundancies but not the second phase. Both phases were agreed and the costs calculated at
the same meeting. Surely both costs should be accounted for consistently?
Page 229 of 328
Paper1:AdvancedFinancial Reporting [RTP 2020 June]
Answers/Hints:
1. Accounting Standards Mix (Numerical and Logical)

Event (a)
It would appear that the lease of the asset to entity X is a finance lease. This is because entity X is
responsible for repairs, maintenance and insurance of the asset and because the present value of the
minimum lease payments by entity X is $760,000 (200,000 x $3·80). This is 98·6% of the fair value
of the asset at the inception of the lease ($771,000).

Because the lease is a finance lease, Delta will show a lease receivable – net investment in finance
leases under non-current assets. The carrying amount of the lease receivable on 1 April 2016 will
be $791,000 ($771,000 + $20,000).

During the year ended 31 March 2017, Delta will recognize income from finance leases in the
statement of profit or loss. The amount recognized will be $79,100 ($791,000 x 10%).

Following recognition of the lease income and the rental payment from Delta on 31 March 2017,
the net investment in finance leases in the statement of financial position of Delta at 31 March 2017
will be $670,100 ($791,000 + $79,100 – $200,000).

Event (b)
The joint arrangement with entity Y is a joint operation because Delta and entity Y have equal rights
to the assets and joint obligations for the liabilities relating to the arrangement. In a joint operation,
the operators include their share of any jointly held assets. Therefore the property, plant and
equipment of Delta at 31 March 2017 will include:

– Leasehold property of $25m x 24/25 = $24m

– Plant and equipment of $7·5m x 4/5 = $6m

In a joint operation, the operators include their share of jointly incurred costs. Therefore the
statement of profit or loss of Delta for the year ended 31 March 2017 will include the following
costs:
– Amortisation of lease premium $1m.
– Depreciation of plant and equipment $1·5m.
– Cash cost of operating the depot $4m.

Delta will also include its own discretionary delivery charges of $2m as a reduction in its operating
costs.

Event (c)
Doubts regarding the going concern status of a customer would normally be regarded as prima facie
evidence that any trade receivable had suffered impairment. In such circumstances an impairment
allowance equal to the expected losses would normally be appropriate.

However, IFRS 9 Financial Instruments requires the impairment assessment to be made at the
reporting date. At the reporting date, the going concern status of Z was not in doubt, so in this case
no allowance is necessary. However, the information about the decline in the going concern status
of Z after the reporting date is a non-adjusting event after the reporting date. Therefore whilst no
impairment allowance is necessary, it will be necessary to disclose details of the 20 April event at
Z’s business premises and its impact on the collectability of Delta’s trade receivable
Page 230 of 328
Paper1:AdvancedFinancial Reporting [RTP 2020 June]
2. Earnings Per Share – (Simple) Financial Reporting Standards

Rs. in million
Profit for the year 150.00
Less: Dividend
Class A Preference shareholders (9÷1.09×2) 16.51
Class B Preference shareholders (300×6%) 18.00
Profit attributable to class B preference shareholders 11.80
[90.49(W-1)×3÷(20+3)(W-2)]
46.31
Profit available for ordinary shareholders 103.69

Earnings per share (103.16÷10) 10.37

W-1 Undistributed earnings

Profit after tax 150.00


Less: Imputed dividend (16.51)
Dividend to class B preference shares (18.00)
Dividend to ordinary shareholders (25.00)
Undistributed earnings 90.49

W-2: Determination of ratio for distribution of undistributed earnings between ordinary and
class B preference shareholders
No. of outstanding Weight Product
shares (in million)
Ordinary shareholder 10 2 20
Class B preference shareholder 3 1 3
23

3. Earnings Per Share (Complex)- Financial Reporting Standards

Tiger Limited
a. EPS for quarter ended 31 December 2017
Numerator Denominator EPS Effect
Rs. in million Shares in million Rs. / share
Basic EPS 140 24.80 (W-1) 5.65
Warrant 0 0 No effect
140 24.8 5.65
1.6
8.05 0.8(2.4×1/3)+0.8(1.2×2/3)
Bonds (W-3) OR 1.2+0.4(1.2÷3) 5.03
148.05 26.40 5.61 Dilutive

Rs. per share


Basic EPS 5.65
Diluted EPS 5.61

b. EPS for half year ended 31 December 2017


Numerator Denominator EPS Effect
Rs. in million Shares in million Rs. / share
Basic EPS 239 23.73 (W-2) 10.07
Warrant 0 0.333
Page 231 of 328
Paper1:AdvancedFinancial Reporting [RTP 2020 June]
[6- (340÷360×6)]
239 24.06 9.93 Dilutive
2
20.02 1.6(2.4 ×4/6)+0.4(1.2×2/6)
Bonds (W-3) OR 1.2+0.8(1.2×4÷6) 5.03
Anti-
259.02 26.067 9.94 Dilutive

Rs. per share


Basic EPS 10.07
Diluted EPS 9.93

W-1: Weighted average shares for quarter ended 31 December 2017


Date Shares Period Total
1 Oct (20+4) 24 1/3 8
0.333
1 Nov (0.8*3*50%) 1.2 [6- (340÷360×6)]
25.20 2/3 16.80
24.80

W-2: Weighted average shares for half year ended 31 December 2017
Date Shares Period Total
1 Jul 20 1/6 3.33
1 Aug 4
24 3/6 12
1 Nov 1.20
25.20 2/6 8.40
23.73

W-3: Interest on Bonds for half year (net of tax):


Rs. in million
First quarter
July to Sep [(760×(9%×3/12×70%) ] 11.97
Second quarter
Oct [(760×(9%×1/12×70%)] 3.99
Nov to Dec [386.20(W-4)×(9%×2/12×70%) 4.06
8.05
20.02

Rs. in million
First quarter
July to Sep [(760×(9%×3/12×70%) ] 11.97
Second quarter
Oct [(760×(9%×1/12×70%)] 3.99
Nov to Dec [386.20(W-4)×(9%×2/12×70%) 4.06
8.05
20.02

W-4: Carrying value of bonds after conversion:


Rs. in million
Initial recognition 760.00
Interest for the year (760×9%) 68.40
Page 232 of 328
Paper1:AdvancedFinancial Reporting [RTP 2020 June]
Interest paid (800×7%) (56.00)
772.40
Conversion (772.40×50%) (386.20)
386.20

4. Consolidated Financial Statement (Complex)- Financial Reporting Standards

Ant Limited
Consolidated Statement of Financial Position
As on 31 December 2017
Particulars Rs. in million
Assets
Property, plant and equipment [3,510+2,835+ 2,200– (20 – 6(W-1))] 2,200.00
Goodwill [175 (W-2) + 108 (W-4)] 283.00
Investment property (130 + 45 + 5(W-1)+ 8(W-1)) 188.00
Current assets (2,120 + 1,420 + 2,800) 6340.00
TOTAL ASSETS 15,342
Equity and liabilities
Share capital (Rs. 10 each) 5,500.00
Group Reserves (W-5) 2476.75
NCI (W-7) 2631.25
Gratuity [25 + 8 (W-9)] 33.00
Current liabilities (1,775 + 1,386+ 1,500+ 40(W-3)) 4701.00
TOTAL EQUITY AND LIABILITIES 15,342

W-1: Net Assets- BL


Rs. in million
Acquisition
Date 1 Apr-17 Reporting Date
Share Capital 4,000 4,000 4,000
Retained Earning 520 815 1,314
Decrease in FV of machine (20) (20) (20)
Depreciation expense (20×10%×2.25),
(20×10%×3) - 4.50 6
Adjustment for uniform accounting
policy [58-45] - - 13
4500 4799.50 5313

299.50 513.50
Post-Acquisition Profit

W-2: Goodwill- BL
Rs in million
Cost 3,100
Decrease in FV of machine (2,925)
175

W-3: Net Assets- FL


Acquisition
Date Reporting Date
Share Capital 2,500 2,500
Retained Earning 1,150 1,000
Page 233 of 328
Paper1:AdvancedFinancial Reporting [RTP 2020 June]
Decrease in FV of machine (50) (40)
3,600 3,460

(140)
Post-Acquisition Profit

W-4: Goodwill- FL
Rs in million
Cost (2400*75%) 1,800
Net Assets [3,600 × 45%(60%×75%)] (1,620)
On acquisition 180
Impairment (W-8) (72)
On reporting date 108

W-5: Group Reserves


Rs in million
AL 2,000
Post acquisition - BL (Up to Mar 2017) - [(299.5 (W-1) × 65%) 194.68
(Apr to Dec 2017) (513.5 (W-1) × 75%)] 385.12
Post acquisition - FL (140 (W-3) × 45%) (63.00)
Equity adjustment on further holding of 10% (W-6) 39.95
Gratuity expense (W-9) (8)
Impairment of goodwill of FL (W-8) (72)
2,476.75

W-6: Equity adjustment on further holding of 10%


Rs in million
Net Assets acquired (4,799.5 (W-1)*10%) 479.95
Cost (440.00)
Increase in equity 39.95

W-7: NCI
Rs in million
Acquisition – BL(4,500*35%) 1,575
Post acquisition (Up to Mar 2017) - [(299.5 (W-1) × 35%) 104.82
(Apr to Dec 2017) (513.5 (W-1) × 2%)] 128.38
10% further acquisition (4,799.50 (W-1) × 10%) (479.95)
Acquisition - FL (3,600 × 55%) 1,980
Post Acquisition – FL (140 (W-3) × 55%) (77)
Indirect Holding (2400 *25%) (600)
2,631.25
W-8: Impairment of Goodwill – FL
Rs in million
Grossing up of goodwill (180/0.45) 400
Net assets on 31 December 2017 (W-3) 3,460
3860
Recoverable amount 3,700
Notional write off 160
Impairment to be recorded (160 × 45%) 72
Page 234 of 328
Paper1:AdvancedFinancial Reporting [RTP 2020 June]
W-9: Gratuity scheme
Rs in million
Charge for the year (P&L and OCI)
Current service cost 85
Interest cost (25×12%) 3
Remeasurment gain (10)
78
Contribution paid (70)
Net Increase 8

5. Consolidated Statement of Cash Flows (Complex)- Financial Reporting Standards

Vitz Limited Rs. in million


Cash Flow from Operating activities
Profit (W-5)817.2+(W-6)222.8 1,040
Adjustments for:
Share of associate profit (W-3) (160–12) (148)
Gain on disposal of subsidiary SL (1,600–1,250–200) (150)
Gain on disposal of property, plant & equipment (350+230)–(170+250) (160)
Unwinding of interest on deferred consideration [189(W-7)×8%] 15
Exchange loss on deferred consideration [223–(189+15)] 19
Depreciation 480
Impairment of Goodwill (W-2) 65
1,161
Increase in working capital (W-4) (951)
210
Cash Flow from Investing activities
Acquisition of shares in associate – AL (600)
Proceeds from disposal of subsidiary - SL (1,600–100) 1,500
Proceeds from disposal of property, plant & 350
equipment
Acquisition of foreign subsidiary - FL (W-7) (495–110) (385)
Purchase of property, plant and equipment (W-1) (1,043)
Dividend received from associate (W-3) 78
(100)
Cash Flow from Financing activities
Proceeds from sale of shares of subsidiary – WL 450
Proceeds from issue of shares at premium (2,800+300)–(2,500+375) 225
675
Net increase in cash and cash equivalents 785
Effect of exchange rate movement 13
798
Cash and cash equivalents – beginning 770
Cash and cash equivalents – ending 1,568

W-1: Additions to property, plant and equipment


Rs in million
Closing balance 3,678
Opening balance 4,173
Transfer-in on acquisition of FL (W-7) 605
Exchange gain relating to FL’s PPE (122–13–36–16) 57
Transfer-out on disposal of SL (1,300)
Carrying value of PPE disposed of (170+250) (420)
Page 235 of 328
Paper1:AdvancedFinancial Reporting [RTP 2020 June]
Depreciation (480)
(2,635)
Additions 1,043

W-2: Impairment of Goodwill


Rs in million
Opening balance 639
Goodwill on acquisition of FL (W-7) 179
Exchange gain on FL’s goodwill (W-8) 16
Goodwill de-recognised on disposal of SL (200)
634
Closing Balance (569)
Impairment of Goodwill 65

W-3: Dividend from associate AL


Rs in million
Cost of acquisition of associate 600
Share of profit (800×6/12×40%) 160
Unrealised profit on inter-co inventory (400×30%×25%×40%) (12)
748
Closing Balance (670)
Dividend from associate 78

W-4: (Increase)/Decrease in working capital


Rs in million
Opening balance (1050+823-1630) 243
Working capital of subsidiary FL (W-7) 385
Exchange gain on working capital of FL 36
Working capital pertaining to SL disposed of during the year 150
814
Closing Balance (1950+957-912) (1,995)
Receivable for property, plant & equipment disposed off 230
(1,765)
951

W-5: Other group reserves/Profit attributable to parent


Rs in million
Closing Balance 3,519
Opening balance 2,451
Equity adjustment on sale of 30% shareholdings in WL (450–300) 150
Exchange gain - attributable to parent [122–21.2(W-6)] 100.8
2,701.8
Profit attributable to parent 817.2

W-6: Non-controlling interest/Profit attributable to NCI


Rs in million
Closing Balance 1,638
Opening balance 874
NCI share of 30% in subsidiary WL (1000*30%) 300
NCI share of 20% in subsidiary FL (W-7) 220
Share of exchange gain on translation of operation – FL (106×20%) 21.2
Page 236 of 328
Paper1:AdvancedFinancial Reporting [RTP 2020 June]
(1,415.2)
Profit attributable to NCI 222.8

W-7: Goodwill - FL
USD in Rate Rs in
million million
Purchase Consideration:
- Cash 4.5 110 495
- Shares at market value (15×25/110) 3.41 110 375
- Deferred consideration payable after 1.714 110 189
two year (2/(1.08)2
9.624 1,059
NCI (10×20%) 2 110 220
Fair value of net assets:
- Property, plant & equipment 5.5 110 605
- Working Capital 3.5 110 385
- Cash 1 110 110
(10) (1,100)
FL - Goodwill at the date of acquisition 1.624 179

W-8: Exchange gain reserve – FL


Rs in million
Exchange gain on FL goodwill (W-7) 179/110×(120–110) 16
Exchange gain on translation of FL operations:
- Net assets at year-end date rate (10+1.5)×120 1,380
- Net assets on acquisition date rate 10×110 (1,100)
- Profit for the year at average rate 1.5×116 (174)
106
122

6. Share based payment (Comprehensive Logic) - Financial Reporting Standards

Query One
Amounts recorded in respect of share options in CL’s financial statements:
Year No. of No. of Fair value per Period Equity Expense
executives share option balance for the
options (Rs.) at year- year
end
2014 39 (47-8) × 4,000 × 600 × 1/3 = 31.20 31.20
2015 40 (44-4) × - × 600 × 2/3 = - (31.20)
2016 43 × 6,000 × 600 × 3/3 = 154.80 154.80
41 (43-2) × 6,000 × 130 (710-580) × 1/2 = 15.99 15.99
170.79 170.79
43 × 6,000 × 600 × 3/3 = 154.80 -
2017 42 × 6,000 × 130 (710-580) × 2/2 = 31.76 16.77
187.56 16.77

Explanation of basis of calculation:


Service condition/No. of executives:
Service condition shall be taken into account by adjusting the number of share options based on
expected number of executives that would remain in service till the vesting date at each year end.
Page 237 of 328
Paper1:AdvancedFinancial Reporting [RTP 2020 June]

Performance condition/No. of share options:


Performance condition other than market condition shall be taken into account by adjusting the
number of equity instruments included in the measurement of the transaction amount at each year
end. In this respect, number of options are based on expected average annual gross profit during the
vesting period as worked-out below:
Year Average gross profit for vesting period No. of Options
2014 (940×3÷3) = 940 4,000
2015 (940+820×2) ÷ 3 = 860 Nil
2016 (940+820+1,270) ÷3= 1,010 6,000
2017 (940+820+1,270+1,200) ÷4=1,058 6,000

Market condition/Fair value per option:


Market conditions are only taken into account when estimating the fair value of the share options
at the measurement date.
CL should recognize an expense irrespective of whether market conditions are satisfied at year end
provided all other vesting conditions are satisfied.

Vesting period:
The expense is spread over the vesting period. At the grant date the vesting period was three years
which was subsequently revised to four years on 1 January 2016.

Query Two
2015
Total expected expense (at end of 2016)
1,000 SARs x Rs. 110 x 405 (500 – 42 – 28 – 25) Rs. 44,550,000
Fraction of vesting period by the year end 2/3
Liability to be recognised by the year end Rs. 29,700,000
Less opening liability:
Total expected expense (at end of 2015)
1,000 SARs x Rs. 90 x 383 (500 – 42 – 75) Rs. 34,470,000
Fraction of vesting period by the year end 1/3
Liability recognised by the end of 2015 Rs. 11,490,000
To be recognised in 2016 Rs. 18,210,000

Entry in 2016:
Dr Statement of profit or loss – staff costs Rs. 18,210,000
Cr Liability Rs. 18,210,000

7. Employee Benefits (Comprehensive Logic with Numerical) - Financial Reporting Standards

a. As per NAS 19 Employee Benefits:

(i) Defined benefit pension scheme:


The employees of a defined benefit scheme will be guaranteed a pension based on their final salary
and their number of years of service. Accordingly, the higher paid the employee is on retirement
and the longer the length of service:
- the greater the employee’s pension entitlement and
- the greater the liability of the pension fund.
Page 238 of 328
Paper1:AdvancedFinancial Reporting [RTP 2020 June]
An actuary will advise the company of the cash contributions to be paid into the plan each year in
order to provide the promised pensions. This is a complicated calculation involving many estimates
such as employee mortality, future increases in salary and expected future investment returns.
The employer has an open-ended liability to make additional contributions should there be a deficit
in the defined benefit pension fund. A deficit may arise, for example, if salary levels rise more than
expected or staff turnover reduces, increasing service years.
It will be necessary for the actuary to regularly re-value the pension fund’s assets and liabilities to
assess the surplus or deficit position and revise the company’s contributions.

(ii) The basis to be adopted in measuring scheme assets:


Assets should be measured at their fair value. For quoted securities, for example, this means their
market price.

(iii) The basis to be adopted in measuring scheme liabilities:


Liabilities should be measured on an actuarial basis (i.e. discounted cash flow), using the projected
unit method.
The projected unit method is an accrued benefits valuation method in which the scheme’s liabilities
reflect projected future earnings. To derive the scheme liabilities, the expected future pension
payments should be discounted at a rate that reflects the time value of money, for example, using
an AA (high quality) corporate bond rate.

(iv) Actuarial gains and losses:


Actuarial gains and losses are deficits or surpluses that arise because:
• events have not coincided with the actuarial assumptions made at the last valuation (experience
gains and losses) or
• the actuarial assumptions have changed.

For example, if the actuary forecast that investment returns were going to be 7% in a year, but in
fact the return actually achieved was only 5%, this would give rise to an actuarial deficit.

b. In the books of Universal Solutions


Statement of financial position – extract
Year 3 Year 4
Defined benefit net liability 200 300
Journal Entries:
Dr. Cr.
P&L:
Interest Cost 10
Current Service Cost 100
110
OCI 130
Cash 140
Defined benefit met liability 100

Workings:
Pension Fund Company Position
Liabilities Assets SFP
Rs. Rs. Rs.
Opening balance 1 Jan 20X4 (1,200) 1,000 (200)
Interest cost (5%) (60) 50 (10)
Current service cost (100) - (100)
Page 239 of 328
Paper1:AdvancedFinancial Reporting [RTP 2020 June]
Contributions to the pension fund - 140 140
Benefits paid out 95 (95) -
Amounts recorded by company (1,265) 1,095 (170)
Actuarial difference (balance) (135) 5 (130)
Closing balance 31 Dec Year 4 (1,400) 1,100 (300)

8. (Simple Logical) - Financial Reporting Standards

(a) Purchase of machine


The cost of purchasing the machine from the foreign supplier (20 million francs) will initially be
recognized in the financial statements using the rate of exchange at the date of delivery (10 francs
to $1). Therefore $2 million (20 million/10) will be included in Epsilon’s property, plant and
equipment (PPE).

PPE is a non-monetary item, so even though the exchange rate (francs to the $) fluctuates during
the accounting period, this will cause no change to the $2 million carrying amount.
The liability to pay the supplier will initially be recognized at $2 million (the $ cost of the machine).
The part payment of the liability on 31 July 2018 will be recorded using the rate of exchange on
that date. Therefore $1,400,000 (12,600,000/9) will be credited to cash and debited to the liability.
The closing liability is a monetary item, so on 30 September 2018 it needs to be re-measured using
the rate of exchange in force at that date.
The amount of the closing liability in $ is $925,000 (7·4 million /8). This will be shown as a current
liability.
Due to the strengthening of the franc against the $, there will be an exchange loss on the
re‑measurement of the liability which must be recognized in the statement of profit or loss. The
amount of the exchange loss is $325,000 ($925,000 – ($2,000,000 – $1,400,000)).
The $250,000 cost of installing the machine is a directly attributable cost of getting the machine
ready for use and this amount will be added to the cost of PPE.
The costs of $200,000 incurred in training staff to use the machine are revenue items and cannot be
included in the cost of PPE. These must be charged in the statement of profit or loss as an expense.

(b) Decommissioning
Epsilon has a legal obligation to dispose of the machine safely at the end of its useful life. This
obligation is reliably measurable and so it must be recognized as a provision on 1 April 2018. The
provision is recognized at the present value of the estimated future expenditure of 3 million francs
(3 million x 0·681 = 2,043,000 francs).

The provision is added to the cost of the asset using the rate of exchange on 1 April 2018 (10 francs
to $1). Therefore $204,300 (2,043,000/10) is added to the cost of PPE.
As the date for payment of the disposal costs draws closer the provision increases. This ‘unwinding
of the discount’ is shown as a finance cost in the statement of profit or loss.

The finance cost in francs is 81,720 (2,043,000 x 8% x 6/12). This will be translated into $ using
the average rate for the period from 1 April 2018 to 30 September 2018 (9·2 francs to $1). Therefore
the charge to the statement of profit or loss for the finance cost will be $8,883 (81,720/9·2). The
closing provision for decommissioning is a monetary item, so on 30 September 2018 it needs to be
re-measured using the rate of exchange in force at that date.

The provision in francs is 2,124,720 (2,043,000 + 81,720). The $ equivalent of this is $265,590
(2,124,720/8).
Page 240 of 328
Paper1:AdvancedFinancial Reporting [RTP 2020 June]
The provision will be shown as a non-current liability in the statement of financial position at 30
September 2018. Due to the strengthening of the franc against the $, there will be an exchange loss
on the remeasurement of the provision which must be recognized in the statement of profit or loss.
The amount of the exchange loss is $52,407 ($265,590 – ($204,300 + $8,883)).

(c) Impairment review


The total initial cost of the machine will be $2,454,300 ($2 million + $250,000 + $204,300).

The machine will be depreciated from 1 April 2018 over its five-year useful life, so the depreciation
charge for the year ended 30 September 2018 will be $245,430 ($2,454,300 x 1/5 x 6/12). The
closing carrying amount of the machine in PPE will be $2,208,870 ($2,454,300 – $245,430).

This will be shown as a non-current asset in the statement of financial position at 30 September
2018.

The difficult trading conditions experienced by Epsilon in the final few months of the financial year
is an indicator that the machine could have suffered impairment. Therefore, a review is required.
However, since the recoverable amount ($2·5 million) of the machine is higher than its carrying
amount, no impairment loss needs to be recorded.

9. Income Taxes (Tax expenses and Deferred Tax – Simple Numerical) - Financial Reporting
Standards

Assets: Carrying amount Tax base Temp difference


Property, plant, and equipment 10,000 2,400 (7,600)
Goodwill 6,000 6,000 -
Other intangible assets 5,000 - (5,000)
Financial assets (cost) 10,500 9,000 (1,500)
Trade receivables 7,000 7,500 500
Other receivables 4,600 5,000 400
Cash and cash equivalents
6,700 6,700 -
Sub Total (13,200)
Liabilities
Long term borrowings 9,600 10,000 (400)
Employee benefit liability 4,000 5,000 (1,000)
Current tax liability 3,070 3,070 -
Trade and other payables 5,000 4,000 1,000
Sub Total (400)
Total (13,600)
Deferred tax expense @ 30% (4,080.00)
Less existing liability (3600)
Charged to P & L (480.00)

10. The agreed Actions under the Nepal Portfolio Performance Review (NPPR )14 Action Plan
2010 are mentioned as under: (Theory)

1. Develop and adopt effective selection criteria for deploying accounting staff in development
projects.
2. Develop transparent placement and transfer criteria of accounts staff in development projects.
Page 241 of 328
Paper1:AdvancedFinancial Reporting [RTP 2020 June]
3. Provide connectivity to FCGO’s FMIS system for selected development projects (selected
programs/projects Education SWAp, Health SWAp, Road Sector Development Project,
LGCDP, Nepal Peace Trust, RRRSDP, Melamchi, PAF, Rural Water Supply, Irrigation Water
Resources Management Project) to implement computerized government accounting system
(CGAS) which facilitates effective monitoring and reporting of financial information.
4. Provide basic financial management training (mainly dealing with foreign assisted development
projects) to all accounts staff, and then a refresher training program in an interval of six months.
5. Monitor the accounts staff position in all foreign assisted development projects and ensure that
there are no vacant positions.
6. Develop Performance Based Reward System for Finance Staff working in foreign aided
development projects based on certain result indicators to recognize the contribution of Finance
Staff.
7. Discontinue the system of allowing four months grace period to fiscal year for 12 remote
districts which is having serious impacts in the financial management system.
8. Make an arrangement to involve FCGO in making decisions on accounts staff related to
creating new positions, upgrading or canceling the positions.
9. FCGO representative to be mandatorily included in discussions during project preparation,
appraisal and negotiations.
10. Issue the revised guidelines regarding Conditional Grants in line with the budget spirit.
11. Form a working group in each line ministry headed by the Planning Chief to internalize the
MTEF into the line ministry.
12. All line ministries to be mandated to prepare sectoral MTEF prior to preparation of annual
budget.
13. Develop MTEF software and Manual.
14. Provide basic financial management and auditing training (mainly dealing with foreign assisted
development projects) to all audit staff, and then a refresher training programs.
15. Develop Risk-based Financial Audit Manual, and pilot in selected foreign aided development
projects, and then finalize the Manual.
16. Review and update the existing Performance Audit Guidelines.

11. Define following terms with respect to Public Financial Management: (Theory)
a. Treasury Single Account (TSA)
The Treasury Single Account (TSA) system is a unified structure of government bank accounts
that gives a consolidated view of government cash resources. In Nepal, the TSA has been rolled
out in all 75 districts a year ahead of the planned schedule, a process which takes several years
in most countries. This means processing budget expenditures, which previously would take
hours, even days, has now shortened to simply a few minutes.

Alongside improved management of idle cash balances, this complete rollout has helped
centralize payments at DTCOs. Once the TSA regime is fully operationalized, most of the
responsibilities related to payment services,management of bank accounts and government
accounting and reporting will be shifted from the Nepal Rastra Bank (NRB), banks and the
spending units to the DTCOs. The available accurate real-time or prompt information will be
useful for cash forecasting, cash management, working on the better debt management; and just
in time release of funds for the budget execution. Besides, the DTCO’s will have the accurate
and authentic information on the expenditure, revenue and deposit accounts of the spending
units on a day to day basis.

b. PEFA Assessment
The Public expenditure and Financial Accountability (PEFA) framework is an internationally
approved tool to measure performance in PFM using standard indicators. It provides the
foundation for evidence-based measurement of countries’ PFM systems. In Nepal, the PEFA
assessment has been carried out twice by the government with support from the World Bank.
Page 242 of 328
Paper1:AdvancedFinancial Reporting [RTP 2020 June]
With the completion of the second PEFA assessment, which has shown progress in 61% of
per4formance indicators from 2010 to 2015, the government has developed a new PFM reform
action plan that adopts a holistic approach to PFM reforms encompassing both institutional and
technical aspects. It has been approved by the National PEFA Steering Committee in March
2016.

c. NPSAS
GON is committed to implement Nepal Public Sector Accounting Standard (NPSAS), in line
with cash based International Public Sector Accounting Standard (IPSAS). On 5th September
2009, the government also approved NPSAS to be used for use in public entities; as
recommended and pronounced by the Accounting Standards Board of Nepal. It is expected that
the Financial Comptroller General Office is gearing to start its financial statements in NPSAS
format from financial year 2012-2013. The actual implementation will have to be seen on
ground.

12. Related Parties (Simple Logical) - - Financial Reporting Standards

a. Define:
• Related Party:
A party is related to an entity if it either:
 controls, is controlled by, or is under common control with, the entity
 has an interest in the entity that gives a significant influence over the entity
 has joint control over the entity
 is an associate
 is a joint venture in which the entity is a venturer
 is a member of the key management personnel of the entity or its parent
 is a close family member of any of the above
 is a post-employment benefit plan for the employees of the entity or of any entity that
is a related party of the entity

• Key Management Personnel:


Those persons having authority and responsibility for planning, directing and controlling the
activities of the entity, directly or indirectly, including any director of that entity.

b. Disclosure of transactions and balances generally:


Relationships between parents and subsidiaries shall be disclosed irrespective of whether there
have been transactions between those related parties.
 Name of entity’s parent and;
 If different the ultimate controlling party

If there have been transactions between related parties, an entity should disclose the nature of
the related party relationships as well as information about the types of transactions and the
outstanding balances necessary for an understanding of the financial statements. Disclosure
should be made irrespective of whether a price is charged.

At a minimum the disclosure should include:


 The amount of the transactions
 The amount of outstanding balances, including terms and conditions, whether they are
secured and the nature of the consideration to be provided
 Provisions for doubtful debts based on the amount outstanding
 The expense recognised during the period in relation to bad and doubtful debts
Page 243 of 328
Paper1:AdvancedFinancial Reporting [RTP 2020 June]
13. Foreign Exchange (Numerical) - Financial Reporting Standards

Kangaroo Ltd.
Statement of Financial Position
As on 31 December 2017
Assets Rs in million
Investment property (W-1) 290
Investments (105 + 130) (W-2) 235

Liabilities
Unearned rent (0.24 × 8 ÷12 × 110) 17.60

Statement of comprehensive income


For the year ended 31 December 2017
Profit & Loss Account Rs in million
Exchange loss on 20% payment (2.6 × 20% × (105 – 108) (1.56)
Increase in fair value of investment property (W-1) 18.30
Rent income (0.24 × 4÷12 × 110) 8.80
Transaction cost – Investment-A (2)
Dividend income (12 + 9) 21
Realised gain on investment-A [(23 × 0.98 – (100 × 20%)] 2.54
Unrealised Gain – Investment-A (W-2) 25

Other Comprehensive Income


Unrealized gain- Investment-B (W-2) 22.90
Realised gain on investment-B [(50 × 0.98 – (153 × 0.3)] 3.10

W-1: Investment property


Rs. in million
Advance payment (2.6 × 10% × 100) 26.00
Initial recognition (2.6 × 70% × 105) 191.10
(2.6 × 20% × 105) 54.60
Total cost 271.70
Fair value (2.5 × 116) 290.00
Gain (P & L) 18.30

W-2: Investments
Assets Investment A Investment B
Rs in million
Purchase price 100 150
Transaction cost - 3
Total cost 100 153
Cost of shares held at 31 Dec 2017 (100×80%) 80.00 (153×70%) 107.10
Fair value - 31 Dec 2017 105 130
Gain 25 22.90

14. Inventories (Simple Numerical) – Financial Reporting Standards

As per NAS 2 ‘Inventories’, inventory is measured at lower of ‘cost’ or ‘net realisable value’.
Further, as per NAS 10: ‘Events after Balance Sheet Date’, decline in net realisable value below
cost provides additional evidence of events occurring at the balance sheet date and hence shall be
considered as ‘adjusting events’.
Page 244 of 328
Paper1:AdvancedFinancial Reporting [RTP 2020 June]
• In the given case, for valuation of inventory as on 31 March 20X1, cost of inventory would be
10 million and net realisable value would be 7.5 million (i.e. Expected selling price 8 million-
estimated selling expenses ` 0.5 million). Accordingly, inventory shall be measured at 7.5
million i.e. lower of cost and net realisable value. Therefore, inventory write down of 2.5
million would be recorded in income statement of that year.

• As per para 33 of NAS 2, a new assessment is made of net realizable value in each subsequent
period. It Inter alia states that if there is increase in net realizable value because of changed
economic circumstances, the amount of write down is reversed so that new carrying amount is
the lower of the cost and the revised net realizable value. Accordingly, as at 31 March 20X2,
again inventory would be valued at cost or net realisable value whichever is lower. In the
present case, cost is 1 million and net realisable value would be 10. 5 million (i.e. expected
selling price 11 million – estimated selling expense 0.5 million). Accordingly, inventory would
be recorded at 10 million and inventory write down carried out in previous year for 2.5 million
shall be reversed.

15. Conceptual - Financial Reporting Standards

As per para 67 of NAS 1 “Presentation of Financial Statements”, Some borrowing agreements


incorporate undertakings by the borrower (covenants) which have the effect that the liability
becomes payable on demand if certain conditions related to the borrower’s financial position are
breached. In these circumstances, the liability is classified as non – current only when:
a. The lender has agreed, prior to the approval of the financial statements, not to demand payment
as a consequence of the breach; and
b. It is not probable that further breaches will occur within twelve months of the balance sheet
date.

In the given case, Company B (the lender) agreed for not to demand payment but only after the
financial statements were approved for issuance. The financial statements were approved for
issuance in the month of June 20X2 and both companies agreed for not to demand payment in the
month of July 20X2 although negotiation started in the month of May 20x2 but could not agree
before June 20X2 when financial statements were approved for issuance.

Hence, the liability should be classified as current in the financial statement for the year ended
March 31, 20X2.

16. Value Added Statement (Simple Numerical)

Samriddhi Ltd.
Value Added Statement for the year ended 31st March, 20XI
(in Rs.‘000)
Sales 2,380
Less: Cost of Bought in Materials and Services:
Operational Cost ` (1,220 + 235) 1,455
Administrative Expenses ` (25+8+62) 95
Interest cost 35 (1,585)
Value addition by manufacturing and trading activities 795
Add: Other Income 370
Gross Value Added 1,165
Page 245 of 328
Paper1:AdvancedFinancial Reporting [RTP 2020 June]
Application of Gross Value Added
(Rs in ‘000) (Rs in ‘000) %
To Pay Employees:
Wages/ Salaries to Administrative Staff 330 0.28
To Pay Directors:
Directors’ Remuneration 55 0.05
To Pay Government:
Local taxes including cess 70
Provision for tax 87 157 0.14
To Pay Providers of Capital
Interest on Debentures 180 0.15
To Provide for Maintenance:
Depreciation 240
Retained Profit 203 443 0.38
1,165 1.00

Ratios:
(a) Gross Value Added to Material Cost Ratio =Gross Value Added ÷Material cost
= 1165 ÷ 1220
= 0.95
Higher GVA ratio of Samriddhi Ltd. in comparison to Industry shows that it has better material
utilisation policy than industry’s material utilisation policy.

(b) Gross Value Added to Employee Cost Ratio =Gross Value Added ÷ Employee Cost
= 1165÷ 330
= 3.53
Higher GVA ratio of Industry in comparison to Samriddhi Ltd. shows that Industry’s labour
productivity or policy is better than Samriddhi Ltd.’s labour productivity or policy.

(c) Gross Value Added to Sales Ratio =Gross Value Added÷ Sales
= 1165÷ 2380
= 0.49
Higher GVA ratio of Industry in comparison to Samriddhi Ltd. shows that Industry’s sales policy
is better than Samriddhi Ltd.’s sales policy.

(d) Gross Value Added to Capital Employed Ratio = Gross Value Added÷ (Equity share capital
+ Preference share capital + Retained Earnings)
=1165÷ (1500+600+263)
= 0.49
Higher GVA ratio of Samriddhi Ltd. in comparison to Industry shows that managerial efficiency
of Samriddhi Ltd. is better than Industry. Samriddhi Ltd. is able to efficiently utilise its capital in
the generation of profit and in addition of value to its organisation.

17. Financial Instruments (Concept) - Financial Reporting Standards


Investment in LL:
Initial measurement
According to NFRS 9, at initial recognition, RIL may make irrevocable election to present
subsequent changes in fair value in equity investment in other comprehensive income instead
of profit or loss account.
If SIL opted as above, investment in LL would initially be recognized at fair value plus
transaction costs i.e. Rs. 20 million. However, if SIL opted to measure the investment at fair
value through profit and loss (FVTPL), investment should initially be measured at Rs. 19.96
Page 246 of 328
Paper1:AdvancedFinancial Reporting [RTP 2020 June]
million (20/1.002) and transaction costs of Rs. 0.04 million (20–19.96) should be charged to
profit and loss account.

Subsequent measurement
On 31 December 2016, if fair value through other comprehensive income has been opted,
investment in LL should be measured at fair value of Rs. 12.4 million and a loss of Rs. 7.6
million [20–12.4(155,000×80)] (instead of Rs. 5 million) should be booked through other
comprehensive income.
According to NFRS 9, amount presented in other comprehensive income shall not be
subsequently transferred to profit or loss. However, the entity may transfer the cumulative gain
/(loss) within equity.
If fair value through profit or loss has been opted, then SIL should account for the loss of Rs.
7.56 million (20–0.04(transaction cost)–12.4) through profit and loss account.

Investment in SL:
Initial measurement
The investment in SL should be recognized as held for trading at fair value of Rs. 64.87 million
(65÷1.002) and transaction cost of Rs. 0.13 million should be charged to profit and loss account.

Subsequent measurement
As at 30 November 2016, the investment should be re-measured to fair value at the market price
of Rs. 83.835 million (135,000×621) and a gain of Rs. 18.965 million (83.835–64.87) shall be
booked in the profit and loss account.

Reclassification of asset
On 30 November 2016 when SIL decided to hold the shares for a longer period, investment in
BL should be reclassified from held for trading to non-trading investment. Further, SIL may
make irrevocable election that investment in SL would be re-measured at fair value through
other comprehensive income, as discussed in the case of LL above. Similarly, treatment on 31
December 2016 would depend on whether SIL opted to re-measure at fair value through OCI
or not.

18. Leases (Simple Numerical) - Financial Reporting Standards


Relevant Extracts:

Statements of profit or loss for the year ended 31 March 2016 (extracts)
Rs.
Depreciation (272,850 ÷ 6) 45,475
Lease payments 6,000*
Finance costs (W) 19,460
* Considering low value item as described in NFRS16

Statement of financial position as at 31 March 2016 (extracts)


Rs.
Non-current assets:
Right of use (272,850 – 45,475) 227,375
Non-current liabilities
Lease liabilities (Note 1) 135,810
Current liabilities
Lease liabilities (Note 1) 78,250

Statement of cash flows for the year ended 31 March 2016 (extracts)
Cash flows from financing activities
Payment of lease liabilities (78,250)
Page 247 of 328
Paper1:AdvancedFinancial Reporting [RTP 2020 June]
Notes to the financial statements (extracts)
Rs.
Lease liabilities include the following:
Amounts due within
One year 78,250
Two to five years 135,810
214,060

WORKING:
Lease of plant (Amount in Rs.)
B/f Payment Capital Interest @ 10% C/f
2016 272,850 (78250) 194,600 19,460 214,060
2017 214,060 (78250) 135,810

19. Intangible Assets (Conceptual) - Financial Reporting Standards

Since a part of the payment for the license has been deferred beyond normal credit terms so the
license will be initially recognised at cash price equivalent of Rs. 80 million i.e. Rs. 50 million plus
Rs. 30 million (i.e. present value of Rs. 36.3 million discounted at 10% for 2 years.)

The advertisement cost of Rs. 10 million incurred on launching of the channel cannot be included
in the cost of the license and will be charged to Profit and loss account.
Since the renewal cost is significant so the useful life of the license will be restricted to the original
5 years only.

The residual value of the license will be assumed to be zero since there is no active market for the
license and there is no commitment by 3rd party to purchase the license at the end of useful life.
The amortization for the year will be Rs. 12 million [(80 – 0) × 1/5 ×9/12] calculated from 1 April
2017 when the license was available for use:

Unwinding of interest expense of Rs. 2.25 million (30 × 10% × 9/12) shall be recorded with
increasing the liability of payable for license with same amount.

20. Provisions, Contingent Assets and Contingent Liabilities (Conceptual) - Financial Reporting
Standards

As per provisions stated in NAS 37:


NAS 37 states that in order for a provision to be recognised, an obligation needs to exist at the
reporting date which can be measured reliably.

The costs of both phases of the redundancy programme have been either estimated or calculated,
so for both phases the potential obligation can be measured reliably.
The reason for the different treatments of the two phases is due to whether or not an obligation
exists at the reporting date.

An obligation can be legal or constructive; in this case the redundancy programme was determined
internally by the company so the obligation is not a legal one.
In the case of phase 1 of the programme, a constructive obligation does exist at the reporting date
because the details have been announced to those affected by it, giving them a valid expectation
that it will be carried through. Therefore NAS 37 requires a provision for the costs to be included
in the financial statements. As no such obligation exists for phase 2 at the reporting date, since the
announcement had not been made at that time, neither a provision nor disclosure of a contingent
liability is required.
Page 248 of 328
Paper1: Advanced Financial Reporting [RTP 2020 December]
Revision Questions:
1. Borrowing Costs(Conceptual) – Financial Reporting Standards

Part i
Identify whether or not the following are qualifying assets.
a. A construction company constructing a bridge for government which will take 6 years to
complete.
b. A very sophisticated integrated circuits being made by an entity who manufactures and sales
10,000 to 12,000 units every month.
c. A power plant under construction, it may take 10 months to complete this.
d. An equipment purchased by X Limited, the equipment may be used immediately after it is
delivered.
e. Special order from a customer to manufacture a machine for him which will take 11 months
at the least.
f. An entity is constructing office building which will take 8 months to complete.

Part ii
Cord Limited is engaged in the manufacturing of automobiles. Currently the company is
manufacturing its power generation plant. The project was started on January 15, 2011 with
company’s own funds. Subsequently, Cord Limited borrowed a loan from ZBL Bank to finance
the project on February 22, 2011. The first payment out of the loan was made on March 04, 2011.
Due to some law and order situation, the project remained closed from April 25, 2011 to May 9,
2011. The work was also stopped for a week from May 23, 2011 to May 30, 2011 so that
necessary plan and layout can be finalized after testing of project completed so far. The plant was
completed on July 31, 2011 except that some sign board could not be installed until August 10,
2011. Loan was repaid on August 31, 2011. Cord Limited started using the plant on September 1,
2011.

Required:
(a) From when Cord Limited should start capitalising borrowing costs?
(b) Should Cord Limited suspend capitalisation from April 25, 2011 to May 9, 2011?
(c) Should Cord Limited suspend capitalisation from May 23, 2011 to May 30, 2011?
(d) When Cord Limited should cease to capitalise borrowing costs?

2. Borrowing Costs(Numerical) – Financial Reporting Standards


Company A borrowed Rs. 9,000 @ 15% per annum to fund a project on 1st Jan 2016. The
following expenditures were made on the project during the year ending 31 December 2016
Date: 1st March 2016: Rs. 2,500
Date: 1st October 2016: Rs. 4,200
Date: 1st December 2016: Rs. 2,300

Surplus funds were invested @10% whenever available.

The project activities started on 1 March 2016. Work on the project was suspended during the
month of August and resumed in early September. Construction was completed on 31 December
2016.

Required: Calculate the borrowing costs to be capitalised and to be charged to Profit or Loss.
Page 249 of 328
Paper1: Advanced Financial Reporting [RTP 2020 December]
3. Employee Benefits (Numerical) – Financial Reporting Standards
Honey Company operates a defined benefit plan and following information is related to its plan as
on June 30, 2012:
Amount in Rs.
Fair value of plan assets as on June 30, 2011 6,800,000
Present value of pension liability as on June 30, 2011 6,500,000
Current service cost 750,000
Benefit paid 500,000
Contribution to plan assets 800,000
Re-measurement component in obligation (loss) 100,000
Re-measurement component in assets (gain) 420,000
Present value of pension liability as on June 30,2012 ??????
Fair value of plan assets as on June 30, 2012 ??????

The total present value of economic benefits available in the form of refunds from the plan or
reduction in future contributions to the plan is Rs. 300,000 as on June 30, 2011 and 2012. The
company uses the discount rate of 10%.

Required:
Calculate the amount of asset / liability to be presented in Statement of Financial Positions on
June 30, 2012.

4. Property Plant and Equipment (Numerical) – Financial Reporting Standards


Following information pertains to Rose Enterprises for the year ended 31 December 2017:
i. Acquisition of land and construction of a factory building:
Rs. in '000
Cost of freehold land purchased with old building structure 25,000
Cost of demolition of the old building structure 1,500
Proceeds from sale of scrap of the old building 250
Fee paid to ABC Architects for site plan and drawings 800
Advance paid to Quality Construction (QC) for construction of the building 6,000
Further payment to QC 35,000

ii. Acquisition and installation of new plant:


Rs. in '000
25% cost of the plant paid in advance 4,000
Transportation and import charges 1,250
Cost of installation 400

iii. Other information:


- Cost of freehold land includes property tax for 2017-18 and transfer fee of Rs. 120,000 and
Rs. 850,000 respectively.
- Factory building was available for use from 1 July 2017. The final invoice of Rs. 19,000,000
is still unpaid.
- Transportation and import charges of the plant include annual fire insurance premium and
insurance in-transit of Rs. 350,000 and Rs. 60,000 respectively.
- The plant started operations on 1 August 2017. Remaining amount was paid on 31 August
2017.
- Old plant was sold on 1 September 2017 at its written down value plus 20%. The plant was
purchased on 1 April 2015 at a cost of Rs. 8,500,000.
- Building and plant are depreciated at the rate of 5% and 10% respectively on reducing
balance method.
Page 250 of 328
Paper1: Advanced Financial Reporting [RTP 2020 December]
Required:
- Pass journal entry to record disposal of the old plant.
- Determine written down value of the fixed assets as at 31 December 2017.

5. Property Plant and Equipment (Numerical) – Financial Reporting Standards

Adjustments Limited has carried out a review of its non-current assets.


a. A lathe was purchased on 1 January 2009 for Rs. 150,000. The plant had an estimated useful
life of twelve years, residual value of nil. Depreciation is charged on the straight line basis.
On 1 January 2015, when the asset’s net book value is Rs. 75,000, the directors decide that
the asset’s total useful life is only ten years.

b. A grinder was purchased on 1 January 2012 for Rs. 100,000. The plant had an estimated
useful life of ten years and a residual value of nil. Depreciation is charged on the straight line
basis. On 1 January 2015, when the asset’s net book value is Rs. 70,000, the directors decide
that it would be more appropriate to depreciate this asset using the sum of digits approach.
The remaining useful life is unchanged.
c. The company purchased a fifty year lease some years ago for Rs. 1,000,000. This was being
depreciated over its life on a straight line basis. On 1 January 2015, when the net book value
is Rs. 480,000 and twenty-four years of the lease are remaining, the asset is revalued to Rs.
1,500,000. This revised value is being incorporated into the accounts.

Required
Explain the effects of these changes on the depreciation for the year to 31 December 2015.

6. Intangible Assets (Numerical) – Financial Reporting Standards


Raisin International (RI) is planning to expand its line of products. The related information for the
year ended 31 December 2015 is as follows:
i. Research and development of a new product commenced on 1 January 2015. On 1
October 2015, the recognition criteria for capitalization of an internally generated
intangible asset were met. It is estimated that the product would have a useful life of 7
years. Details of expenditures incurred are as follows:
Rs. million
Research work 4.50
Development work 9.00
Training of production staff 0.50
Cost of trial run 0.80
Total costs 14.80

ii. The right to manufacture a well-established product under a patent for a period of five
years was purchased on 1 March 2015 for Rs.17 million. The patent has an expected
remaining useful life of 10 years. RI has the option to renew the patent for a further period
of five years for a sum of Rs.12 million.

iii. RI has acquired a brand at a cost of Rs. 2 million. The cost was incurred in the month of
June 2015. The life of the brand is expected to be 10 years. Currently, there is no active
market for this brand. However, RI is planning to launch an aggressive marketing
campaign in February 2016.

iv. In September 2014, RI developed a new production process and capitalized it as an


intangible asset at Rs.7 million. The new process is expected to have an indefinite useful
life. During 2015, RI incurred further development expenditure of Rs. 3 million on the
new process which meets the recognition criteria for capitalization of an intangible asset.
Page 251 of 328
Paper1: Advanced Financial Reporting [RTP 2020 December]

Required:
In the light of International Financial Reporting Standards, explain how each of the above transaction
should be accounted for in the financial statements of Raisin International for the year ended 31
December 2015.

7. Agriculture (Numerical) – Financial Reporting Standards


Mishall Limited a public limited company, Dairy, produces milk on its farms. It produces 30% of
the country’s milk that is consumed. Dairy owns 450 farms and has a stock of 210,000 cows and
105,000 heifers. The farms produce 8 million kilograms of milk a year, and the average inventory
held is 150,000 kilograms of milk. However, the company is currently holding stocks of 500,000
kilograms of milk in powder form.

At December 31, 2018, the herds are:


▪ 210,000 cows (3 years old), all purchased on or before January 1, 2018
▪ 75,000 heifers, average age 1.5 years, purchased on June 1, 2018
▪ 30,000 heifers, average age 2 years, purchased on January 1, 2018

No animals were born or sold in the year. The unit values less estimated selling costs were:
Rs.
1-year-old animal at December 31, 2018 32
2-year-old animal at December 31, 2018 45
1.5-year-old animal at December 31, 2018 36
3-year-old animal at December 31, 2018 50
1-year-old animal at January 1, 2018 and June 1, 2018 30
2-year-old animal at January 1, 2018 40

The company has had problems during the year: Contaminated milk was sold to customers. As a
result, milk consumption has gone down. The government has decided to compensate farmers for
potential loss in revenue from the sale of milk. This fact was published in the national press on
November 1, 2018. Dairy received an official letter on December 10, 2018, stating that Rs.5
million would be paid to it on March 2, 2019.

The company’s business is spread over different parts of the country. The only region affected by
the contamination was Lahore, where the government curtailed milk production in the region.
The cattle were unaffected by the contamination and were healthy.

The company estimates that the future discounted cash flow income from the cattle in the Lahore
region amounted to Rs.4 million, after taking into account the government restriction order. The
company feels that it cannot measure the fair value of the cows in the region because of the
problems created by the contamination. There are 60,000 cows and 20,000 heifers in the region.
All these animals had been purchased on January 1, 2018.

A rival company had offered Dairy Rs.3 million for these animals after selling costs and further
offered Rs.6 million for the farms themselves in that region. Dairy has no intention of selling the
farms at present. The company has been applying IAS 41 since January 1, 2018.

Required:
Advise the directors on how the biological assets and produce of Dairy should be accounted for
under IAS 41, discussing the implications for the financial statements.

8. Non-Current Assets Held For Sale And Discontinued Operations (Logical) – Financial
Reporting Standards
Page 252 of 328
Paper1: Advanced Financial Reporting [RTP 2020 December]

Victoria owns several properties and has a year end of 31 December. Wherever possible, Victoria
carries investment properties under the fair value model

Property 1 was acquired on 1 January Year 1. It had a cost of Rs. 1 million, comprising Rs.
500,000 for land and Rs. 500,000 for buildings. The buildings have a useful life of 40 years.
Victoria uses this property as its head office.
Property 2 was acquired many years ago for Rs. 1.5 million for its investment potential. On 31
December Year 7 it had a fair value of Rs. 2.3 million. By 31 December Year 8 its fair value had
risen to Rs. 2.7 million. This property has a useful life of 40 years.

Property 3 was acquired on 30 June Year 2 for Rs. 2 million for its investment potential. The
directors believe that the fair value of this property was Rs. 3 million on 31 December Year 7
and Rs. 3.5 million on 31 December Year 8. However, due to the specialised nature of this
property, these figures cannot be corroborated. This property has a useful life of 50 years.

Required
For each of the above properties briefly state how it would be treated in the financial statements
of Victoria for the year ended 31 December Year 8, identifying any impact on profit or loss.

9. Exploration for and Evaluation of Mineral Resources (Logical) – Financial Reporting


Standards
If an entity chooses an accounting policy to capitalise E&E expenditure, what approaches are
acceptable?

10. Impairment of Assets (Numerical) – Financial Reporting Standards

A company runs a unit that suffers a massive drop in income due to the failure of its technology
on 1 January 2008. The following carrying values were in the books immediately prior to the
impairment:

Rs. m
Goodwill 20
Technology 5
Brands 10
Land 50
Buildings 30
Other net assets 40
155

The recoverable value of the unit is estimated at Rs. 85 million. The technology is worthless,
following its complete failure. The other net assets include inventory, receivables and payables. It
is considered that the book value of other net assets is a reasonable representation of its net
realizable value.

Required:
(a) Show the impact of the impairment on 1 January 2008.
(b) Show the impact of the impairment on 1 January 2008 assuming that net selling price of land
is Rs. 29 million

11. Impairment of Assets (Numerical) – Financial Reporting Standards


Sky-Line Limited (SL) operates a 4 Star Hotel facility in Muree. The hotel was constructed at a
cost of Rs.300 million, 5 years back and it is depreciated on a straight-line basis (total useful life
Page 253 of 328
Paper1: Advanced Financial Reporting [RTP 2020 December]
of 15 years and residual value of 20%). There are indications that the property is not performing
as expected due to:
(a) opening of a competing hotel nearby,
(b) a significant drop in number of tourists to the area because of terrorism.

There is a 40% probability that the hotel will generate net cash flows of Rs.40 million per annum
and 60% probability that the cash flows would only be Rs.20 million per annum.

The property’s net operating income is Rs.30 million which is at the rate of 15%. 5% of the
proceeds from sale would be expended in closing the deal.

Required:
If the appropriate discount rate is 10% then compare the carrying value with the recoverable
amount to arrive at the impairment loss.

12. Presentation of Financial Statements (Numerical) – Financial Reporting Standards

Following is the trial balance of Shaheen Limited (SL) as at June 30, 2015:
Rs. in ‘000
Dr
Sales revenue 200,000
Manufacturing costs 100,000
Selling and distribution costs 35,000
Administrative costs 30,000
Opening inventories 23,000
Interest on borrowings 5,000
Provision for income tax 2,000
Advance income tax paid 6,000
Property, plant and equipment 86,000
Accumulated depreciation on property, plant and equipment 12,000
Export licence 6,000
Trade receivables 37,800
Cash and bank balances 4,725
Other receivable and prepayments 14,000
Trade payables 12,000
Provisions for litigation 5,000
Long term borrowings 31,525
Deferred tax 5,000
Share capital (Rs. 10 each and fully paid) 60,000
Retained earnings 20,000
347,525 347,525

Additional information
(i) Sales last year (year ended 30 June 2014) included goods invoiced at Rs 10 million which
were sent to a customer on June 25, 2014 under a sale or return agreement, at cost plus 20%.
The goods were returned on August 25, 2014. No correction has been made for the return.
(ii) The export licence has been obtained for exporting a new product and is effective for five
years up to December 31, 2019. However, the exports commenced from July 1, 2015.
(iii) Closing inventories are valued at Rs. 30 million.
(iv) Details of property, plant and equipment are as follows:
Page 254 of 328
Paper1: Advanced Financial Reporting [RTP 2020 December]
Plant and
Land Buildings equipment
Rs in ‘000
Cost as at June 30, 2014 20,000 36,000 30,000
Fully depreciated amounts included in cost 3,000
Estimated useful life at the date of purchase 20 years 10 ears

The company uses straight line method for charging depreciation. Depreciation is allocated
to manufacturing, distribution and administrative costs at 75%, 15% and 10% respectively.
(v) Rs. 6 million of the long term borrowings is of current maturity (i.e. will be repaid within 12
months).
(vi) During the year Rs. 5 million was paid in full and final settlement of income tax liability
against which a provision of Rs. 7.0 million had been made in the previous year. Current
year’s taxable income exceeds accounting income by Rs. 5 million of which 0.8 million are
permanent differences. Applicable tax rate for the company is 35%.
(vii) On July 30, 2015 the board of directors proposed a final dividend at 15% for the year ended
June 30, 2015 (2014: at 20%)
Required
In accordance with the requirements of the International Financial Reporting Standards, prepare:
(a) The statement of financial position as of June 30, 2015
(b) The statement of profit or loss for the year ended June 30, 2015
(c) The statement of changes in equity for the year ended June 30, 2015.
(Comparative figures and notes to the financial statements are not required)

13. Provisions, Contingent Liabilities, and Contingent Assets (Numerical) – Financial Reporting
Standards

The financial statements of Bravo Limited (BL) for the year ended 30 September 2013 are under
finalisation and the following matters are under consideration:

BL’s plant was commissioned and became operational on 1 April 2008 at a cost of Rs. 130
million.

At the time of commissioning its useful life and present value of decommissioning liability was
estimated at 20 years and Rs. 19 million respectively.

BL’s discount rate is 10%.

There has been no change in the above estimates till 30 September 2013 except for the
decommissioning liability whose present value as at 1 April 2013 was estimated at Rs. 25 million.

Required:
Compute the related amounts as they would appear in the statements of financial position and
comprehensive income of Bravo Limited for the year ended 30 September 2013 in accordance
with IFRS. (Ignore corresponding figures)

14. Provisions, Contingent Liabilities, and Contingent Assets (Logical) – Financial Reporting
Standards
In which of the following circumstances might a provision be recognised?
(a) On 13 December 20X9 the board of an entity decided to close down a division. The
accounting date of the company is 31 December. Before 31 December 20X9 the decision was
not communicated to any of those affected and no other steps were taken to implement the
decision.
Page 255 of 328
Paper1: Advanced Financial Reporting [RTP 2020 December]
(b) The board agreed a detailed closure plan on 20 December 20X9 and details were given to
customers and employees.
(c) A company is obliged to incur clean up costs for environmental damage (that has already
been caused).
(d) A company intends to carry out future expenditure to operate in a particular way in the future.

Answer
(a) No provision would be recognised as the decision has not been communicated.
(b) A provision would be made in the 20X9 financial statements.
(c) A provision for such costs is appropriate.
(d) No present obligation exists and under IAS 37 no provision would be appropriate. This is
because the entity could avoid the future expenditure by its future actions, maybe by changing
its method of operation.

15. Accounting Policies, Changes in Accounting Estimates and Errors (Numerical) – Financial
Reporting Standards

G Ltd adopted IFRSs from the beginning of year 2012. As a consequence, G Ltd changed its
accounting policy for the treatment of borrowing costs that are directly attributable to the
acquisition of a hydroelectric power station under construction for use by G Ltd. In previous
periods, G Ltd had charged such costs as an expense. G Ltd has now decided to capitalise these
costs, rather than treating them as an expense as a result of adopting IAS 23 and its transitional
provisions.

G Ltd expensed borrowing costs directly related to construction of qualifying asset incurred of
$2,600 during 2011 and $5,000 in 2010 and $4,000 in 2009. G Limited accounting records for
2012 show profit before tax of $27,000 (after deducting $3,000 borrowing costs relating to
qualifying assets). The income tax is $8,100. G Ltd has not yet recognised any depreciation on the
power station because it is not yet in use.

In 2011, G Limited reported: $


Profit before interest and tax 20,600
Interest expense (all on qualifying assets) (2,600)
Profit before tax 18,000
Tax (5,400)
Profit 12,600

Year 2011 reported retained earnings was $20,000 and closing retained earnings was $32,600. G
Ltd.’s tax rate was 30% for 2012, 2011 and prior periods. G Ltd had $10,000 of share capital
throughout, and no other components of equity except for retained earnings.
Required: Relevant extracts of financial statements.

G Ltd – Statement of profit or loss and other comprehensive income (extracts)


2012 2011
Profit before interest and tax 27,000+3,000 30,000 20,600
Interest 0 0
Profit before tax 30,000 20,600
Tax @ 30% (9,000) (6,180)
Profit after tax 21,000 14,420
Page 256 of 328
Paper1: Advanced Financial Reporting [RTP 2020 December]
G Ltd – Statement of changes in equity
Share Restated Total
capital Retained
earnings
Balance as at January 1, 2011 (previously reported) 10,000 20,000 30,000
Change in accounting policy (net of tax @ 30%) 6,300 6,300
Balance as at January 1, 2011 (Restated) 10,000 26,300 36,300
Profit for the year ended 2011 14,420 14,420
Balance as at December 31, 2011 10,000 40,720 50,720
Profit for the year ended 2012 21,000 21,000
Balance as at December 31, 2012 10,000 61,720 71,720

16. Related Parties (Logical) – Financial Reporting Standards


Following transactions were carried out by Yellow Limited during the year ended June 30, 2006.
(i) Mr. Sharp, a well-known management consultant was hired, to conduct a three weeks
workshop on time management for the staff of the company at a fee of Rs. 0.5 million.
Mr. Sharp is the son of the Chief Executive Officer.
(ii) A loan of Rs. 30 million was obtained from Blue Bank Limited. The loan was negotiated
by Mr. Slim, General Manager Finance of Yellow Limited, who was formerly a senior
executive of the Bank.
(iii) Three used delivery trucks of the company were sold to Red Supplies Limited, which
supplies approximately 60% of the total raw material used by the company.
(iv) Granted interest bearing loan to its Chief Executive Officer for construction of house in
accordance with the company’s policy relating to employees’ benefit.
(v) Paid mobilization advance of Rs. 9 million against a construction contract to Orange
Limited which is owned by Mr. Clear, a member of a reputed business family. Two
influential directors of the company are nephews of Mr. Clear.
(vi) The company awarded a contract for plant maintenance services to its subsidiary Brown
(Pvt.) Limited effective August 01, 2007.
(vii) The company has nominated a director in Purple Limited who participates actively in its
operational decisions. However, no business relations exist between the two companies.

Required:
For each case, discuss the requirement of IAS 24 (Related Party Disclosures) as regards the
following disclosures in the financial statements for the year ended June 30, 2006:
(a) Related party relationship; and
(b) Related party transactions.

17. Share Based Payment (Numerical) – Financial Reporting Standards

An entity granted 2,000 share options at an exercise price of $18 to each of its 25 key
management personnel on 1 January 2004. The options only vest if the managers are still
employed by the entity on 31 December 2006. The fair value of the options was estimated at $33
and the entity estimated that the options would vest with 23 managers. This estimate was
confirmed on 31 December 2004.

In 2005 the entity decided to base all incentive schemes around the achievement of performance
targets and to abolish the existing scheme for which the only vesting condition was being
employed over a particular period. The scheme was cancelled on 30 June 2005 when the fair
value of the options was $60 and the market price of the entity's shares was $70. Compensation
was paid to the 24 managers in employment at that date, at the rate of $63 per option.
Page 257 of 328
Paper1: Advanced Financial Reporting [RTP 2020 December]
Required:
How should the entity recognise the cancellation, pass journal entries?

18. Earning Per Shares (Numerical) – Financial Reporting Standards


On 1 April 2011, a company issued $1,250,000 8% convertible unsecured bonds for cash at par.
Each $100 nominal of loan stock will be convertible in 2016 or 2017 into the number of ordinary
shares set out below:
On 31 December 2016 124 shares
On 31 December 2017 120 shares

Issued share capital:


$500,000 in 10% cumulative irredeemable preference shares of $1
$1,000,000 in ordinary share of 25 cents each = 4,000,000 shares

Income tax rate are 30%

Trading results for the years ended 31 December were as follows:


2012 $ 2011 $
Operating profit 1,100,000 991,818
Interest on 8% bonds (100,000) (75,000)
Profit before tax 1,000,000 916,818
Income tax (300,000) (275,045)
Profit after tax 700,000 641,773

Calculate the basic and diluted EPS.

19. Financial Instruments (Numerical) – Financial Reporting Standards

Part a
On July 1, 2010, Star Company made a loan to Moon Company of Rs. 10 million for 5 years. The
coupon rate is same as the effective rate of interest i.e. 9%.

On June 30, 2012, it becomes clear that Moon Company is financial crisis and it has become
evident of impairment. It is estimated that the remaining future cash flows from the loan will only
be Rs. 7.50 million at the end of the fifth year (inclusive of interest of remaining years).

Interest is received at the end of each year and current market interest rate is 10.50%.

Required:
Show the effect of the above transactions on statement of financial position and Statement of
Comprehensive Income for the year ended June 30, 2012.

Part b
Rich Limited made a 12% loan of Rs.500,000 to Poor Limited on January 1, 2005. Loan is
payable after six years. The coupon and effective rate of interest are the same and interest is
received at the end of each year.
On January 1, 2010, Rich Limited discovered that Poor Limited was facing financial problems. It
was estimated that Poor Limited would be able to pay only Rs.336,000 instead of Rs.560,000 at
the end of 2010.
Required:
(i) How above situation will be accounted for in the financial statements of Rich Limited on
January 1, 2010.
Page 258 of 328
Paper1: Advanced Financial Reporting [RTP 2020 December]
(ii) What amount will be recognised as interest income for the year ended December 31,
2010.

20. Business Combination and Consolidations (Numerical) – Financial Reporting Standards

On 1 July 2014, Galaxy Limited (GL) acquired controlling interest in Beta Limited (BL). The
following information has been extracted from the financial statements of GL and BL for the year
ended 30 June 2015.
Rs. in million
GL BL
Share capital (Rs. 100 each) 100 50
Retained earnings – 1 July 2014 40 18
Profit for the year ended 30 June 2015 20 6
Shareholders’ equity/Net assets 160 74

Investment in BL (300,000 shares) 50 –


Inter-company sales (at invoice value) 25 30
Inter-company purchases remained unsold at year-end 9 5
Inter-company current account balances 7 (4)

Other relevant information is as under:


(i) On the date of acquisition, fair value of BL's net assets was equal to their book value
except for the following:
a. Fair value of a land exceeded its carrying value by Rs. 20 million.
b. The value of a plant was impaired by Rs. 10 million. The impairment was also
recorded by BL on 2 July 2014
c. BL measures its property, plant and equipment using cost model.
(ii) There is no change in share capital since 1 July 2014.
(iii) Inter-company sales are invoiced at cost plus 20%. The difference between the current
account balances is due to goods dispatched by GL on 30 June 2015 which were received
by BL on 5 July 2015.
(iv) GL values non-controlling interest at the acquisition date at its fair value which was Rs.
35 million.
(v) As at 30 June 2015, goodwill of BL was impaired by 10%.

Required: Compute the amounts of goodwill, consolidated retained earnings and non-controlling
interest as they would appear in GL's consolidated statement of financial position as at 30 June
2015.
Page 259 of 328
Paper1: Advanced Financial Reporting [RTP 2020 December]
Answers/Hints:
1. Borrowing Costs(Conceptual) – Financial Reporting Standards
Part i
a. Qualifying asset
b. Not a qualifying asset
c. Qualifying asset
d. Not a qualifying asset
e. Qualifying asset
f. Qualifying asset

Part ii
From when Cord Limited should start capitalising borrowing costs? 22 Feb 2011
Should Cord Limited suspend capitalisation from April 25, 2011 to May 9, 2011? Suspended
Should Cord Limited suspend capitalisation from May 23, 2011 to May 30, 2011? Not suspended
When Cord Limited should cease to capitalise borrowing costs? 31 Jul 2011

2. Borrowing Costs(Numerical) – Financial Reporting Standards


Borrowing costs to be capitalised Rs.
Borrowing costs incurred 9,000 x 15% x 9/12 1,012.5
Less: Temporary investment income 6,500 x 10% x 6/12 (325)
2,300 x 10% x 2/12 (38.3)
649.2

The project commenced on 1st March resulting in a period of 10 months up to the year end.
However, interest cannot be capitalised during the period of suspension. Therefore, interest is
capitalised only for 9 months.

Borrowing costs to be expenses Rs.


Borrowing costs incurred 9,000 x 15% x 3/12 337.5
Less: Temp. Inv. income 9,000 x 10% x 2/12 (before commencement) (150)
6,500 x 10% x 1/12 (suspension) (54.17)
133.33
3. Employee Benefits (Numerical) – Financial Reporting Standards
2012
Liability in SFP Rs
Present value of the obligation at year end 7,500,000
Less: Fair value of the plan assets at year end (8,200,000)
(700,000)
Asset ceiling loss 400,000
Net Surplus to be shown in SFP (PV of future refund etc) (300,000)

Workings 2012
Actuarial gains or losses on Liability Rs
Present value of obligation – 1 January 6,500,000
Interest cost (10%) 650,000
Current service costs 750,000
Benefits paid (500,000)
Expected value 7,400,000
Actuarial loss (gain) β 100,000
Present value of obligation – 31 December 7,500,000
Page 260 of 328
Paper1: Advanced Financial Reporting [RTP 2020 December]
Actuarial gains or losses on Assets Rs
Fair value of plan assets – 1 January 6,800,000
Return on plan assets (10%) 680,000
Contribution received by fund 800,000
Benefits paid (500,000)
Expected value 7,780,000
Actuarial (loss) gain 420,000
Fair value of plan assets – 31 December 8,200,000

4. Property Plant and Equipment (Numerical) – Financial Reporting Standards


Debit Credit
--------- Rs. in '000 ---------
Bank/Cash/Receivable (8,500–1,896) × 120% 7,925
Accumulated depreciation (W1) 1,896
Fixed assets (Plant) 8,500
Gain on disposal (Balancing figure) 1,321

W-1: Accumulated depreciation: Rs. in ‘000


From 1 April to 31 December 2015 (8,500 x 10% x 9 / 12) 638
For the year ended 31 December 2016 (8,500 – 638) × 10% x 12 / 12 786
From 1 January to 31 August 2017 (8,500 – 638 – 786) × 10% × 8 / 12 472
1,896
Part (b) Ref Freehold Building Plant
Land
WDV as at 31 December 2017 -------------- Rs. in '000 --------------
Purchase price (i) 25,000
Demolition of old building Rs. 1,500 – 250 (i) 1,250
Architect fee paid to ABC consultant (i) 800
Construction cost – Advance (i) 6,000
Construction cost – Further payment (i) 35,000
Plant cost – advance 25% (ii) 4,000
Plant cost – remaining 75% (ii) 12,000
Transportation and import charges (ii) 1,250
Installation charges (ii) 400
Property tax year 2017-18 (should be PL) (iii) (120)
Transfer fee (correctly included already) (iii) -
Construction costs – unpaid amount (iii) 19,000
Annual fire insurance (should be expense) (iii) (350)
Insurance in transit (correct already) (iii) -
Cost to be capitalised 26,130 60,800 17,300
Depreciation for 2017 -
- Land (1,520)
- Building (60,800 × 5% × 6 / 12)
- Plant (17,300 × 10% × 5 / 12) (721)
WDV of PPE as at 31 December 2017 26,130 59,280 16,579
Page 261 of 328
Paper1: Advanced Financial Reporting [RTP 2020 December]
5. Property Plant and Equipment (Logical) – Financial Reporting Standards
Part (a)
The lathe was purchased in 2009 and was originally being written off over an estimated useful life
of twelve years. As at 1 January 2015 six of the years have elapsed with a further six years
remaining. It was decided that the machine will now only be usable for a further four years.

IAS 16 requires that where the original estimate of useful life is revised, adjustments should be
made in current and future periods (not in prior periods). The unamortized cost of the asset should
be charged to revenue over the remaining useful life of the asset. The net book value of Rs.
75,000 should therefore be charged over the remaining four years of useful life, giving an annual
depreciation charge of Rs. 18,750. The revision is not a change in accounting policy, or a
fundamental error but a change in accounting estimate. It is therefore not appropriate to deal with
any excess depreciation by adjusting opening retained earnings.

Part (b)
The grinder was purchased in 2012 and was originally being depreciated on a straight line basis. It
has now been decided to depreciate this on the sum of digits basis.

IAS 16 requires that depreciation methods be reviewed periodically and if there is a significant
change in the expected pattern of economic benefits, the method should be changed. Depreciation
adjustments should be made in current and future periods. This change might be appropriate if,
for instance, usage of the machine is greater in the early years of an asset’s life when it is still new
and consequently it is appropriate to have a higher depreciation charge. If the change is
implemented, the unamortized cost (the net book value) of the asset should be written off over the
remaining useful life commencing with the period in which the change is made.
The depreciation charge for the remaining life of the asset will therefore be as follows.
Year No of digits Depreciation
2015 7 (7/28 x 70,000) 17,500
2016 6 (6/28 x 70,000) 15,000
2017 5 12,500
2018 4 10,000
2019 3 7,500
2020 2 5,000
2021 1 2,500
½ x 7 (7 + 1) 28 70,000
Disclosure will need to be made in the accounts of the details of the change, including the effect
on the charge in the year.

Part (c)
IAS 16’s allowed alternative treatment in respect of measurement of property plant and
equipment (subsequent to initial recognition), is that of revaluation. Revaluation is made at fair
value. Where any item of property plant or equipment is revalued, the entire class to which the
asset belongs should be revalued. Revaluations must be kept up to date. Where there are volatile
movements in fair value, the revaluation should be performed annually. Where there are no such
movements, revaluations every three to five years may be appropriate.

Accumulated depreciation at the date of revaluation is either:


- Restated proportionately with the change in the gross carrying amount so that the carrying
amount after the revaluation equals the revalued amount (e.g. where revaluations are made to
depreciated replacement cost using indices)
- Eliminated against the gross carrying amount of the assets and the net amount restated to the
revalued amount of the asset (e.g. where buildings are revalued to their market value).
Page 262 of 328
Paper1: Advanced Financial Reporting [RTP 2020 December]
IAS 16 requires that the subsequent charge for depreciation should be based on the revalued
amount. The annual depreciation will therefore be Rs. 62,500, i.e. Rs. 1,500,000 divided by the 24
years of remaining life. There will then be a difference between the revalued depreciation charge
and the historical depreciation charge. The resulting excess depreciation may be dealt with by a
movement in reserves, i.e. by transferring from the revaluation reserve to retained earnings a
figure equal to the depreciation charged on the revaluation surplus each year.

6. Intangible Assets (Numerical) – Financial Reporting Standards

(i) Since the product met all the criteria for the development of the product, it should be
recognized as an intangible in the statement of financial position (SOFP) of the company.
However, RI should capitalize only the development work (i.e. Rs.9.80 million) as intangible
asset. IAS-38 does not allow capitalization of cost relating to the research work, training of
staff. Since the product has a useful life of 7 years, the amortization expense amounting to
Rs.0.35 million (Rs.9.8 million × 3/12 ÷ 7 years) should be recorded in the statement of profit
or loss.

(ii) This purchasing of right to manufacture should be recognized as an intangible in the SOFP
because:
▪ It is for an established product which would generate future economic benefits.
▪ Cost of the patent can be measured reliably. Since there is a finite life, the patent
must be amortized over its useful life. The useful life will be shorter of its actual life
(i.e. 10 years) and its legal life (i.e. 5 years. The amortization to be recorded in SOCI
is Rs.2.83 million (Rs.17 million × 10/12 ÷ 5).

(iii) The acquired brand should be recognized as an intangible in the SOFP because acquisition
price is a reliable measure of its value. The amortization to be recorded in SOCI is Rs.0.12
million (Rs.2 million ÷ 10 years x 7/12).

(iv) The carrying value of the intangible asset should be increased to Rs.10 million in the SOFP.
Since there is an indefinite useful life of the intangible assets, it should not be amortized.
Instead, RI should test the intangible asset for impairment by comparing its recoverable amount
with its carrying amount.

7. Agriculture (Numerical) – Financial Reporting Standards

Biological assets should be measured at each reporting date at fair value less estimated selling
costs unless fair value cannot be measured reliably. The Standard encourages companies to
separate the change in fair value less estimated selling costs between those changes due to
physical reasons and those due to price.

Fair value of cattle excluding Lahore region: Rs. 000 Rs. 000
Fair value at January 1, 2018
Cows (210,000 – 60,000) × 40 6,000
Heifers (30,000 – 20,000) × 30 300
Purchase 75,000 heifers × 30 2,250
8,550
Increase due to price change 150,000 × (45 – 40) 750
10,000 × (32 – 30) 20
75,000 × (32 – 30) 150
Increase due to physical change 920
150,000 × (50 – 45) 750
10,000 × (45 – 32) 130
Page 263 of 328
Paper1: Advanced Financial Reporting [RTP 2020 December]
75,000 × (36 – 32) 300
1,180 10,650
Fair value less estimated POS costs at October 31, 20X4
150,000 × 50 7,500
10,000 × 45 450
75,000 × 36 2,700 10,650

Lahore region—fair value of cattle:


This region has an inventory of cattle of 60,000 cows and 20,000 heifers. Fair value is difficult to
ascertain because of the region’s problems. However, according to IAS 41, if fair value was used
on initial recognition, then it should be continued to be used. The cattle in this region would have
been fair valued at January 1, 2018, under the Standard. Therefore, the cattle must be valued at
fair value less estimated selling costs as at December 31, 2018.

Although Rs.3 million has been offered for these animals, this may be an onerous contract as rival
companies are likely to wish to take advantage of the problems in this region. The future
discounted income is again an inappropriate value as the cattle are healthy and could be moved to
another region and sold.
The cattle in this region would therefore be valued at Rs.000
60,000 cows × 50 3,000
20,000 heifers × 45 900
3,900

8. Non-Current Assets Held For Sale And Discontinued Operations (Logical) – Financial
Reporting Standards

Property 1
Treatment in the financial statements for the year ended 31 December Year 8 (IAS16). This is
used by Victoria as its head office and therefore cannot be treated as an investment property. It
will be stated at cost minus accumulated depreciation in the statement of financial position. The
depreciation for the year will be charged in the statement of profit or loss.

Property 2
This is held for its investment potential and should be treated as an investment property. It will be
carried at fair value, Victoria’s policy of choice for investment properties. It will be revalued to
fair value at each year end and any resultant gain or loss taken to the statement of profit or loss
(Rs. 400,000 gain in Year 8).

Property 3
This is held for its investment potential and should be treated as an investment property.
However, since its fair value cannot be arrived at reliably it will be held at cost minus
accumulated depreciation in the statement of financial position. The depreciation for the year will
be an expense in the statement of profit or loss.

This situation provides the exception to the rule whereby all investment properties must be held
under either the fair value model, or the cost model.

9. Exploration for and Evaluation of Mineral Resources (Theoretical) – Financial Reporting


Standards
A number of different methods of capitalising E&E expenditure have historically been used by
entities in the extractives sector, in particular ‘successful efforts’, ‘area-of-interest’ and ‘full cost
accounting’. A partial capitalisation approach is also commonly followed in practice.
Page 264 of 328
Paper1: Advanced Financial Reporting [RTP 2020 December]

a) Successful efforts
Under this approach, in general only those costs that lead directly to the discovery, acquisition, or
development of specific, discrete mineral reserves are capitalised. Costs that are known, when
they are incurred, to fail to meet this criterion are generally charged to profit or loss as incurred.
However, some interpretations of the successful efforts method would result in capitalising the
cost of unsuccessful development wells. This might be where an entity views a specified area as
being a single cost centre (this may be smaller than an area-of-Interest – see below).

b) Area-of-Interest
An area of interest is an individual geological area which is considered to constitute a favourable
environment for the presence of a mineral deposit or an oil or natural gas field. Under this
approach, all E&E expenditure relating to an area of interest are grouped and capitalised, to the
extent that the costs are expected to be recouped either through the successful development and
exploitation of the area, or by its sale.

c) Full cost
The full cost method generally results in capitalising all costs incurred in prospecting, acquiring
mineral interests, exploration, appraisal, development and construction which are then
accumulated in large cost centres. There are certain aspects of full cost accounting that are not
consistent with the requirements of IFRS. This includes capitalizing all pre licence acquisition
costs (which are not within the scope of IFRS 6, as these do not form E&E activities), the need to
classify E&E assets as tangible or intangible which is often not carried out for large asset pools,
and the need to test E&E assets for impairment at the point at which E&E activity ceases and the
assets are reclassified under IAS 16 or IAS 38(which is typically not possible, as full cost
accounting does not disaggregate cost pools in the required level of detail).

d) Partial capitalisation
Under this approach, only some costs that are eligible for capitalisation are included in an entity’s
E&E asset. In some jurisdictions, a common approach is to capitalise initial acquisition costs for a
particular mining asset and to expense all subsequent costs.

10. Impairment of Assets (Numerical) – Financial Reporting Standards


Part a
An impairment of $70 million is required ($155m carrying amount - $85m recoverable amount)

Items Carrying value Impairment Carrying value


before impairment allocated after impairment
$m $m $m
Goodwill 20 (20) 0
Technology 5 (5) 0
Brands 10 (5) 5
Land 50 (25) 25
Buildings 30 (15) 15
Other 40 (0) 40
CGU 155 (70) 85

Total impairment: $70m


Allocated towards
Technology ($ 5m)
Goodwill ($20m)
Remaining ($45m)
Page 265 of 328
Paper1: Advanced Financial Reporting [RTP 2020 December]

Prorate based on carrying value of Remaining Impairment:


Brands $45m × 10/(10 + 50 + 30) = $5m
Land $45m × 50/(10 + 50 + 30) = $25m
Buildings $45m × 30/(10 + 50 + 30) = $15m

Part b

Items Carrying value Impairment Carrying value


before impairment allocated after impairment
$m $m $m
Goodwill 20 (20) 0
Technology 5 (5) 0
Brands 10 (5)+(1)=(6) 54
Land 50 (25) (21) 25 29
Buildings 30 (15)+(3)=(18) 15 12
Other 40 (0) 40
CGU 155 (70) 85
Total impairment: $70m
Allocated towards
Technology ($ 5m)
Goodwill ($20m)
Remaining ($45m)
Prorate based on carrying value of Remaining Impairment:
Brands $45m × 10/(10 + 50 + 30) = $5m
Land $45m × 50/(10 + 50 + 30) = $25m
Buildings $45m × 30/(10 + 50 + 30) = $15m

The maximum loss that can be charged to land is $21m as carrying amount of land cannot be
taken below $29m. The pro rata loss is $25m for land. The remaining loss of $4m shall be again
pro-rated in other assets.

Prorate based on carrying value:


Brands $4m × 10/(10 + 30) = $1m
Buildings $4m × 30/(10 + 30) = $3m

11. Impairment of Assets (Numerical) – Financial Reporting Standards


The detailed calculation is as under:
Carrying value of asset = Rs.300 million – [5 x (300 – 60) / 15 years]
= Rs.220 million

Fair value = Net operating income / return rate (since no active market)
= Rs.30 m / 15% = 200 million
Fair value less costs to sell = Rs.200 million – 10 million (5%) = 190 million

Value in use = PV of net cash flows discounted at 10% for 10 years


= (Rs.40 m x 0.4 + Rs.20 m x 0.6) x [1- (1.10)-10]/ 10%]
= Rs.28 m x 6.145
= Rs.172 m

Recoverable amount = Rs.190 million (Higher)

Impairment loss = Rs.30 million (i.e. Rs. 220 million – Rs. 190 million)
Page 266 of 328
Paper1: Advanced Financial Reporting [RTP 2020 December]
12. Presentation of Financial Statements (Numerical) – Financial Reporting Standards

SHAHEEN LIMITED
Statement of Financial Position as at 30 June 2015
2015 Rs. 000
EQUITY AND LIABILITIES
Equity
Issued, subscribed and paid up capital 60,000
Un-appropriated profits 35,956
95,956
Non-current liabilities
Long term borrowings 31,525 – 6,000 J5 25,525
Deferred tax liability 5,000 – 1,470 J7 3,530
29,055
Current liabilities
Trade and other payables 12,000
Provision for litigation 5,000
Current portion of long-term borrowings J5 6,000
Provision for taxation J6 9,414
32,414
Total equity and liabilities 157,425

ASSETS
Fixed assets
Property, plant and equipment 86,000 – 12,000 – 4,500 J4 69,500
Intangible assets (Export licence) 6,000 – 600 J2 5,400
74,900
Current assets
Stock-in-trade J3 30,000
Trade debts 37.800 - 10,000 J1 27,800
Other receivables and prepayments 6,000 + 14,000 20,000
Cash and bank balances 4,725
82,525
Total assets 157,425

SHAHEEN LIMITED
Statement of Comprehensive Income for the year ended 30 June 2015
2015 Rs000
Revenue 200,000
Cost of sales (100,000+23,000 +8,333 J1– 30,000 J3+ 3,375 J4) (104,708)
Gross profit 95,292
Selling and distribution costs35,000 + 600 J2 + 675 J4 (36,275)
Administrative costs 30,000+ 450 J4 (30,450)
Operating profit 28,567
Finance costs (5,000)
Profit before tax 23,567
Income tax expense (9,998 J6 – 1,470 J7- 2,000) (6,528)
PROFIT FOR THE YEAR / TOTAL COMPREHENSIVE INCOME 17,039
Page 267 of 328
Paper1: Advanced Financial Reporting [RTP 2020 December]
Shaheen Limited
Statement of Changes in Equity
For the year ended 30 June 2015
Share Capital R/E Total
Balance as at July 01, 2014 60,000 32,000* 92,000
Correction of error 1,667 J1 – 584 J6 - (1,083) (1,083)
Balance as at July 01, 2014 – restated 60,000 30,917 90,917
Profit for the year - 17,039 17,039
Dividend paid 60,000 x 20% - (12,000) (12,000)
Balance as at June 30, 2015 60,000 35,956 95,956

*Rs. 20,000 + 12,000 dividend = Rs. 32,000

Journal Entries
1. RE (Sales last year) 10,000
RA 10,000

Inventory 8,333
RE (COS last year) 8,333

COS (This year) 8,333


Inventory 8,333
Correction of error for last year and current year for goods sold on return basis during last
year but returned this year. Cost of inventory Rs. 10,000 x 100/120 = Rs. 8,333
Net effect on last year = Rs. 10,000 – 8,333 = Rs. 1,667
2. Selling & distribution costs 600
Intangible assets (licence) 600
Rs. 6,000 / 5 years x 6/12 = Rs. 600
3. Stock in trade 30,000
COS 30,000
Closing inventory recorded
4. COS 75% 3,375
SE 15% 675
AE 10% 450
PPE 4,500
Depreciation on building Rs. 36,000 / 20 years = Rs. 1,800
Depreciation on plant and equipment(Rs. 30,000 – 3,000) / 10 years = Rs. 2,700
Total = Rs. 4,500
5. Long term borrowings 6,000
Current maturity of long term loan 6,000
Current portion separated
6. Tax expense 9,988
RE (correction of error) 584
Provision for income tax 9,414
Current tax payable = Rs. 9,998 – 584 = Rs. 9,414
Current year Rs. 23,567 + 5,000 inadmissible = Rs. 28,567 x 35% = Rs. 9,998
Prior year J1A = Rs. 1,667 x 35% = Rs. 584
7. Deferred tax liability 1,470
Tax expense 1,470
Decrease in deferred tax liability
Rs. 5,000 – 800 = Rs. 4,200 x 35% = Rs. 1,470
Page 268 of 328
Paper1: Advanced Financial Reporting [RTP 2020 December]
13. Provisions, Contingent Liabilities, and Contingent Assets (Numerical) – Financial Reporting
Standards
Bravo Limited
Relevant financial statement amounts
For the year ended 30 September 2013
PPE Prov. PL
1 April 2008 [130 + 19] 149 19
Depreciation 149 / 20 x 4.5 years (33.53) -
Interest 19 x 1.10^4.5 -19 - 10.18
1 Oct 2012 115.47 29.18

Depreciation 149 / 20 x 0.5 years (3.73) - 3.73


Interest (19 x 1.10^5 ) - (19 x 1.104.5) 1.42 1.42
30.6
Increase in provision (5.6) (5.6)
106.15 25
Depreciation 106.15 / 15 x 0.5 years 3.54 3.54
Interest 25 x 1.100^0.5 – 25 1.22 1.22
102.61 26.22 9.91

14. Provisions, Contingent Liabilities, and Contingent Assets (Logical) – Financial Reporting
Standards
(a) No provision would be recognised as the decision has not been communicated.
(b) A provision would be made in the 20X9 financial statements.
(c) A provision for such costs is appropriate.
(d) No present obligation exists and under IAS 37 no provision would be appropriate. This is
because the entity could avoid the future expenditure by its future actions, maybe by changing
its method of operation.

15. Accounting Policies, Changes in Accounting Estimates and Errors (Numerical) – Financial
Reporting Standards

G Ltd – Statement of profit or loss and other comprehensive income (extracts)


2012 2011
Profit before interest and tax 27,000+3,000 30,000 20,600
Interest 0 0
Profit before tax 30,000 20,600
Tax @ 30% (9,000) (6,180)
Profit after tax 21,000 14,420

G Ltd – Statement of changes in equity


Share Restated Total
capital R/E
Balance as at January 1, 2011 (previously reported) 10,000 20,000 30,000
Change in accounting policy (net of tax @ 30%) 6,300 6,300
Balance as at January 1, 2011 (Restated) 10,000 26,300 36,300
Profit for the year ended 2011 14,420 14,420
Balance as at December 31, 2011 10,000 40,720 50,720
Profit for the year ended 2012 21,000 21,000
Balance as at December 31, 2012 10,000 61,720 71,720
Page 269 of 328
Paper1: Advanced Financial Reporting [RTP 2020 December]
16. Related Parties (Logical) – Financial Reporting Standards
(i) Mr. Sharp is a related party as he is son (close family member) of the CEO (member of key
management personnel) of the company. [IAS 24.9(a)(iii)]

The fee for training Rs. 0.5 million paid to Mr. Sharp shall be disclosed along with nature of
related party relationship.

(ii) Blue Bank Limited is not a related party as former employee does not create any such
relation. [IAS 24.11(c)(i)]
No disclosure is required under IAS 24.

(iii) Red Supplies Limited is not a related party as significant volume of business does not create
related party relationship unless significant influence can be exerted. [IAS 24.11(d)]
No disclosure is required under IAS 24.

(iv) CEO is related party being member of key management personnel. [IAS 24.9(a)(iii)]
The amount of loan granted along with terms and conditions including rate of interest and
outstanding balance shall be disclosed.

(v) Generally nephews are not considered close family members of a person. However, if Mr.
Clear is able to influence his nephews, he shall be considered a related party. [IAS
24.9(a)(iii)]
If considered related party, the mobilization advance of Rs. 9 million shall be disclosed.

(vi) Brown (Pvt) Limited is a related party being a subsidiary. [IAS 24.9(b)(i)] Nature of related
party relationship shall be disclosed. Awarding a contract itself is not a transaction but the
contractual commitment needs to be disclosed.

(vii) The active participation of director nominated by Yellow Limited indicates the significant
influence over Purple Limited. [IAS 24.9(b)(ii)]
No disclosure is required under IAS 24.

17. Share Based Payment (Numerical) – Financial Reporting Standards


The journal entries are:
Date Particulars Dr. $ Cr. $
31.12.04 Staff costs 506,000
Equity W1 506,000

30.06.05 Staff costs 1,078,000


Equity W2 1,078,000

30.06.05 Equity W3 2,880,000


Staff costs (Profit or loss) 144,000
Cash W3 3,024,000

W1 2,000 shares x 23 managers x $33 = $1,518,000 x 1/ 3 years = $506,000

W2 At 30 June 20X5 the entity should recognise a cost based on the amount of options it had
vested on that date. The total cost is:
2,000 x 24 managers x $33 = $1,584,000
$1,584,000 – 506,000 = $1,078,000
Page 270 of 328
Paper1: Advanced Financial Reporting [RTP 2020 December]

W3 The compensation paid is 2,000 x 24 x $63 = $3,024,000


Of this, the amount attributable to the fair value of the options cancelled is:

2,000 x 24 x $60 (FV of option, not of the underlying share) = $2,880,000

This is deducted from equity as a share buyback.

The remaining $144,000 ($3,024,000 less $2,880,000) is charged to profit or loss.

18. Earning Per Shares (Numerical) – Financial Reporting Standards

2012 2011
BEPS = $650,000 W1 $591,773
4,000,000 4,000,000

=$0.163 =$0.148

DEPS = $650,000 + 70,000 W3 $591,773 + 52,500 W3


4,000,000+1,550,000W2 4,000,000+1,162,500W3

=$0.13 =$0.125
W1
2011, PAT – pref. dividend $700,000 – 50,000 = $650,000
2012 $641,773 – 50,000 = $591,773

W2
Maximum number of potential ordinary shares $1,250,000 / 100 x 124 =1,550,000

W3 2012 2011

IEPS= $100,000 x 70% $75,000 x 70%


1,550,000 1,550,000 x9/12

=$0.045 =$0.045

19. Financial Instruments (Numerical) – Financial Reporting Standards


Part a
The carrying amount of loan shall be restated to revised amount of loan’s recoverable amount of
Rs. 7.5 m x (1.09)-3 = Rs. 5.79 and impairment loss of Rs. 4.21 m (i.e. Rs. 10 m – 5.79) shall be
charged to profit or loss.

The amount of interest income for the year is Rs. 10 m x 9% = Rs. 0.9m. There is no effect on
interest as interest income of year 2012 has to be charged before impairment loss.

Part b
(i) The carrying amount of loan shall be restated to revised amount of loan’s recoverable
amount of Rs. 336,000 x (1.12)-1 = Rs. 300,000 and impairment loss of Rs. 200,000 (i.e.
Rs. 500,000 – 300,000) shall be charged.
(ii) The amount of interest income for the year is Rs. 300,000 x 12% = Rs. 36,000.
Page 271 of 328
Paper1: Advanced Financial Reporting [RTP 2020 December]
20. Business Combination and Consolidations (Numerical) – Financial Reporting Standards

Goodwill W3 Rs. 6.3 million


Consolidated retained earnings W5 Rs. 66.95 million
Non-controlling interest W4 Rs. 40.52 million

W1 GROUP STRUCTURE
Beta Limited Acquisition date: 01-JUL-2014 Group 60% NCI 40%

W2 Net Assets of Subsidiary


At Acquisition date At Reporting date
Share capital 50 50
Retained earnings [Now 18 + 6] 18 24
+- Fair value adjustment effect (Land) 20 20
Incorrect impairment of plant charges 10
+- Fair value adjustment effect (Plant) (10) (10)
- PURP Inventory (S is seller) 9 x 20/120 ( 1.5)
- Impairment of goodwill (FULL) (0.7)
78 91.8

Post-acquisition change (at reporting date – at acquisition date) 13.8

W3 Goodwill Rs. m
Investment 50
+NCI at acquisition (W4) 35
85
- NA of subsidiary at acquisition (78)
7
- + (Impairment) / Transfer of negative goodwill to W5 10% (0.7)
6.3

W4 Non-Controlling Interest Rs. m


NCI at acquisition [at fair value] 35
+ NCI share of post-acquisition [13.8 x 40%] 5.52
40.52

W5 Group Reserves Rs. m


Parent’s reserves (100%) 40 + 20 60
- PURP Inventory (P is seller) {5+3 in transit} x 20/120 (1.33)
Subsidiary’s post-acquisition x G% [13.8 x 60%] 8.28
66.95
Paper1:AdvancedFinancial Reporting [RTP 2021 June]
Page 272 of 328

1. NAS 34- Interim Financial Reporting – Numerical and conceptual


ABC Limited manufactures automobile parts. ABC Limited has shown a net profit of Rs.
20,00,000 for the third quarter of 20X1.

Following adjustments are made while computing the net profit:


(i) Bad debts of Rs. 1,00,000 incurred during the quarter. 50% of the bad debts have been
deferred to the next quarter.
(ii) Additional depreciation of Rs. 4,50,000 resulting from the change in the method of
depreciation.
(iii) Exceptional loss of Rs. 28,000 incurred during the third quarter. 50% of exceptional loss
have been deferred to next quarter.
(iv) Rs. 5,00,000 expenditure on account of administrative expenses pertaining to the third
quarter is deferred on the argument that the fourth quarter will have more sales; therefore
fourth quarter should be debited by higher expenditure. The expenditures are uniform
throughout all quarters.

Ascertain the correct net profit to be shown in the Interim Financial Report of third quarter
to be presented to the Board of Directors.

2. NFRS 8 Operating Segments (Theory)


a. List out the quantitative threshold that shall require an entity to report separately information
about an operating segment as provided under NFRS 8 Operating Segment.

b. Shazad Industries Ltd has recently acquired four large subsidiaries. These subsidiaries
manufacture products which are of different lines from those of the parent company. The
parent company manufactures plastics and related products whereas the subsidiaries
manufacture the following:
Product Location
Subsidiary 1 Textiles Karachi
Subsidiary 2 Car products Lahore
Subsidiary 3 Fashion garments Peshawar
Subsidiary 4 Furniture items Multan

The directors have purchased these subsidiaries in order to diversify their product base but do
not have any knowledge of the information required in the financial statements regarding
these subsidiaries other than the statutory requirements.

Required
i. Explain to the directors the purpose of segmental reporting of financial information.
ii. Explain to the directors the criteria which should be used to identify the separate reportable
segments. (You should illustrate your answer by reference to the above information)
iii. Critically evaluate NFRS 8, Operating segments, setting out any problems with the
standard.

3. Events after the end of reporting period (NAS 10)


Finance Manager of RR Limited approached you to discuss implications of following events on
the financial statements of the company for the year ended June 30, 2005, which are to be placed
before the Board of Directors for approval on August 28, 2005:

a. Trade debts as at June 30, 2005 include a debt of Rs. 500,000 recoverable from Mr. P, who
was declared insolvent on August 05, 2005.
b. A computer software having carrying value of Rs. 1.5 million had been giving operational
problems since May, 2005. It became totally inoperative in July, 2005. It took 25 days and a
cost of Rs. 31,000 for rectification.
Paper1:AdvancedFinancial Reporting [RTP 2021 June]
Page 273 of 328

c. Investments of the company amounting to Rs.10 million at the year-endwere disposed offor
Rs. 6 million in response to a market crash on July 27, 2005.
d. At the year end, the company had 950 laptops of a good brand each costing Rs. 65,000. There
was rising trend of prices in the market, which influenced the company’s sale policy and these
computers were retained in stock till July 25, 2005 when market price started falling and
within one week’s time declined to Rs. 68,000. This situation forced the management to start
selling. However, the whole stock could be sold till August 22, 2005 and fetched total sale
proceeds of Rs.40.85 million.
e. On May 28, 2005, the head of sales department had placed his suggestion to the Chief
Executive Officer (CEO) for a free after-sale-service offer for two years to customers,
effective April 15, 2005. However, the CEO approved the scheme on July 15, 2005 and it was
announced by the company on the same date. It is expected that service cost attributable to
sales made during April15, 2005 to June 30, 2005 would be Rs. 150,000.

Required:
Suggest appropriate accounting treatment in each case with proper reasoning.

4. Income Taxes (NAS-12) - Numerical


Mars Limited (ML) is engaged in the manufacturing of chemicals. On July 1, 2014 it obtained a
motor vehicle on lease from a bank. Details of the lease agreement are as follows:
(i) Cost of motor vehicle is Rs.1,600,000.
(ii) Installments of Rs. 480,000 are to be paid annually in advance.
(iii) The lease term and useful life is 4 years and 5 years respectively.
(iv) The interest rate implicit in the lease is 13.701%.

ML follows a policy of depreciating the motor vehicles over their useful life, on the straight- line
method. However, the tax department allows only the lease payments as a deduction from taxable
profits.

The tax rate applicable to the company is 30%. ML’s accounting profit before tax for the year
ended June 30, 2015 is Rs. 4,900,000.

There are no temporary differences other than those evident from the information provided above.

Required:
a. Prepare journal entries in the books of Mars Limited for the year ended June 30, 2015 to
record the above transactions including tax and deferred tax.
b. Prepare a note to the financial statements related to disclosure of lease liability, in accordance
with the requirements of NFRS.
(Ignore comparative figures.)

5. Inventories – NAS-2- Numerical


M/S Lasani Associates have provided you with the following working papers regarding inventory
on hand at 31 December 2002. The entity is a manufacturer of two product lines:

Cost Rs. NRV Rs. Write Down Rs.


Raw Materials 200,000 150,000 50,000
-air cooler parts 80,000 50,000 -
-Fans parts 120,000 100,000 -
Work In progress 160,000 170,000 -
-air cooler parts 60,000 50,000 -
-Fans parts 100,000 120,000 -
Finished goods 320,000 330,000 -
-air cooler parts 160,000 110,000 -
Paper1:AdvancedFinancial Reporting [RTP 2021 June]
Page 274 of 328

-Fans parts 160,000 220,000 -


680,000 650,000 50,000

Due to the strengthening of the local currency, the parts used in the manufacture of both the air
coolers and fans became cheaper. As a direct result thereof, the net realisable value of both the
finished air coolers and fans also dropped.

Required:
Critically analyze, whether the write down has been calculated correctly.

6. PPE- Conceptual and Numerical (NAS-16)


Adjustments Limited has carried out a review of its non-current assets.
a. A lathe was purchased on 1 January 2009 for Rs. 150,000. The plant had an estimated useful
life of twelve years, residual value of nil. Depreciation is charged on the straight line basis.
On 1 January 2015, when the asset’s net book value is Rs. 75,000, the directors decide that
the asset’s total useful life is only ten years.

b. A grinder was purchased on 1 January 2012 for Rs. 100,000. The plant had an estimated
useful life of ten years and a residual value of nil. Depreciation is charged on the straight line
basis. On 1 January 2015, when the asset’s net book value is Rs. 70,000, the directors decide
that it would be more appropriate to depreciate this asset using the sum of digits approach.
The remaining useful life is unchanged.

c. The company purchased a fifty year lease some years ago for Rs. 1,000,000. This was being
depreciated over its life on a straight line basis. On 1 January 2015, when the net book value
is Rs. 480,000 and twenty-four years of the lease are remaining, the asset is revalued to Rs.
1,500,000. This revised value is being incorporated into the accounts.
Required
Explain the effects of these changes on the depreciation for the year to 31 December 2015.

7. Intangible Assets - Conceptual (NAS-38)


Company X buys 100% of Company Y. Company Y has spent Rs. 600,000 on a research and
development project. This amount has all been expensed as the NAS 38 criteria for capitalising
costs incurred in the development phase of a project have not been met. Company Y has
knowhow as the result of the project.

Company X estimates the fair value of Company Y’s knowhow which has arisen as a result of
this project to be Rs. 500,000.
Discuss the accounting treatment.

8. Intangible Assets - Conceptual (NAS-38)


(i) Briefly explain the term “biological asset” and state when a biological asset is recognised
in the financial statements under the International Financial Reporting Standards.
(ii) The Dairy Company (TDC) owns three farms and has a stock of 3,200 cows. During the
year ended 30 June 2015, 300 animals were born, all of which survived and were still
owned by TDC at year-end. Of those, 225 are infants whereas 75 are nine-month old
having market values of Rs. 26,000 and Rs. 53,000 per animal respectively. The
incidental costs are 2% of the transaction price.
Required:
In accordance with the requirements of the International Financial Reporting Standards, discuss
how the gain in respect of the new born cows should be recognized in TDC’s financial statements
Paper1:AdvancedFinancial Reporting [RTP 2021 June]
Page 275 of 328

for the year ended 30 June 2015. (Show all necessary computations)

9. Non current Assets Held for sale- Conceptual (NFRS-5)


PK Limited had sugar and textile businesses. However, half way through the year ended 31
December 2010, the textile business was closed and the assets sold off, incurring losses on the
disposal on non-current assets of $76,000 and redundancy cost of $37,000. The directors
reorganized the continuing sugar business at a cost of $98,000 and directors consider this an
exceptional item. In addition to these costs, the following operating results have been
summarized:
Sugar Textile
$ $
Revenue 650,000 320,000
Cost of sales 320,000 150,000
Distribution 60,000 90,000
Administration 120,000 110,000
Other information (related to continuing operations only) is as follows:
$
Finance cost 17,000
Tax 31,000

Required:
Draft the income statement for the year ended 31 December 2010.

10. Exploration for and Evaluation of Mineral Resources


If an entity chooses an accounting policy to capitalize E&E expenditure, what approaches are
acceptable?

11. Impairment of assets (Numerical – NAS 36)


The assistant financial controller of the Hussain Associates Ltd group has identified the matters
below which she believes may indicate impairment of one or more assets.
Hussain Associates Ltd owns and operates an item of plant that cost Rs.640,000 and had
accumulated depreciation of Rs.400,000 at 1 October 2015. It is being depreciated at 12½% on
cost.
On 1 April 2016 (exactly half way through the year) the plant was damaged when a factory
vehicle collided into it. Due to the unavailability of replacement parts, it is not possible to repair
the plant, but it still operates, albeit at a reduced capacity. It is also expected that as a result of the
damage the remaining life of the plant from the date of the damage will be only two years.

Based on its reduced capacity, the estimated present value of the plant in use is Rs.150,000. The
plant has a current disposal value of Rs.20,000 (which will be nil in two years’ time), but Hussain
Associates Ltd has been offered a trade-in value of Rs.180,000 against a replacement machine
which has a cost of Rs.1 million (there would be no disposal costs for the replaced plant). Hussain
Associates Ltd is reluctant to replace the plant as it is worried about the long-term demand for the
product produced by the plant. The trade-in value is only available if the plant is replaced.

Required
Prepare extracts from the statement of financial position and statement of profit or loss of Hussain
Associates Ltd in respect of the plant for the year ended 30 September 2016. Your answer should
explain how you arrived at your figures.
Paper1:AdvancedFinancial Reporting [RTP 2021 June]
Page 276 of 328

12. Provisions, Contingent Liabilities and Contingent Assets (Conceptual –NAS 37)
Oval Limited (OL) deals in medicines and surgical instruments. OL is in the process of finalizing
its financial statements for the year ended 31 December 2018. Following matters are under
consideration:
(i) OL sells instruments A-1 and B-1 with 1-year warranty. These units are purchased from a
manufacturer Star Limited (SL). The details of warranty are as under:

A-1: SL provides warranty services to the customers and recovers 50% of the cost from
OL. However, in case of SL’s default, the warranty services would have to be provided by OL.

B-1: OL provides warranty services to the customers and recovers the entire cost from SL.
On 31 December 2018, it is estimated that total cost of Rs. 4 million and Rs. 7 million would be
incurred in next year for providing warranty services for A-1 and B-1 respectively sold in 2018.

(ii) In October 2018, OL was sued by a customer for Rs. 18 million on account of supply of
substandard surgical instruments.

By end of the year, OL communicated to the customer via email to pay Rs. 5 million. In
respect of the remaining amount of the claim, OL’s lawyers anticipate that there is 70%
probability that the court would award Rs. 6 million and 30% probability that the amount
would be Rs. 4 million.

OL lodged a claim with the supplier in December 2018. The supplier principally accepted
the claim to the extent of Rs. 9 million. However, OL is still negotiating with the supplier
and it is probable that OL would recover a further sum of Rs. 3 million.

(iii) OL has imported 7,000 units of a medicine at a cost of Rs. 70 million. However, in
November 2018, a study was published in a medical journal which reveals that results of
an alternate medicine are much better. At year end, 5000 units were in stock. On 25
January 2019, 4000 units were sold at Rs. 8,000 per unit. OL also paid 10% commission.
Required:
Discuss how the above issues should be dealt with in the financial statements of OL for the year
ended 31 December 2018. Support your answers in the context of relevant NFRSs.

13. Accounting policies, estimates and error (Conceptual –NAS 08)


Duncan Company has previously written off any expenditure on borrowing costs in the period in
which it was incurred.

The company has appointed new auditors this year. They have expressed the view that the
previous recognition of borrowing costs in the statement of profit or loss was in error.

The company has decided to correct the error retrospectively in accordance with NAS 8.The
financial statements for 2014 and the 2015 draft financial statements, both reflecting the old
policy, show the following.

Statement of changes in equity (extract)

2014 2015
Retained earnings Retained earnings
$000 $000
Opening balance 22,500 23,950
Profit after tax for the period 3,200 4,712
Dividends paid (1,750) (2,500)
Closing balance 23,950 26,162
Paper1:AdvancedFinancial Reporting [RTP 2021 June]
Page 277 of 328

Borrowing costs written off were $500,000 in 2014 and $600,000 in 2015.

The directors have calculated that borrowing costs, net of depreciation which should have been
included in property, plant and equipment had the correct policy been applied, are as follows.
$000
At 30 December 2013 400
At 31 December 2014 450
At 31 December 2015 180

Had the correct policy been in force depreciation of $450,000 would have been charged in 2014
and $870,000 in 2015.

Required
Show how the change in accounting policy must be reflected in the statement of changes in equity
for the year ended 31 December 2015. Work to the nearest $000.

14. Related Party (NAS-24) - Logical


During the year ended June 30, 2008, Baber Limited (BL) has carried out several transactions
with the following individuals/entities:
(i) AK Associates provides information technology services to BL. One of the directors of
BL is also the partner in AK Associates.
(ii) SS Bank Limited is the main lender. By virtue of an agreement it has appointed a
nominee director on the Board of BL.
(iii) Mr. Zee who supplies raw materials to BL, is the brother of the Chief Executive Officer
of the company.
(iv) JB Limited is the distributor of BL’s products and have exclusive distribution rights for
the province of Punjab.
(v) Mr. Tee is the General Manager-Marketing of BL and is responsible for all major
decisions made in respect of sales prices and discounts.
(vi) BL’s gratuity fund is administered by the Trustees appointed by the company.
(vii) MM Limited is the leading supplier of BL and supplies 60% of BL’s raw materials.
(viii) Ms. Vee who conducted various training programmes for the employees of the company,
is the wife of BL’s Chief Executive Officer.
Required:
Comment as to whether the above individuals/entities are ‘related parties’ of the company or not.
Support your arguments with references from International Accounting Standards.

15. Foreign Exchange (NAS-21- Numerical)


Copper Limited (CL) entered into following transactions during the year ended 30 June 2019:
(i) On 1 October 2018, CL imported a machine from China for USD 250,000 against 60%
advance payment which was made on 1 July 2018. The remaining payment was made on
1 April 2019.

(ii) On 1 January 2019, CL sold goods to a Dubai based company for USD 40,000 on credit.
CL received 25% amount on 1 April 2019, however, the remaining amount is still
outstanding.
Following exchange rates are available:
Date 1 Jul 2018 1 Oct 2018 1 Jan 2019 1 Apr 2019 30 Jun 2019 Average
1 USD Rs. 121 Rs. 124 Rs. 137 Rs. 140 Rs. 163 Rs. 135
Required: Prepare journal entries in CL’s books to record the above transactions for the year
ended 30 June 2019.
Paper1:AdvancedFinancial Reporting [RTP 2021 June]
Page 278 of 328

16. Financial Instruments (NAS32, NAS39, NFRS 9)


Part A
MES Solutions has entered into the following transactions involving the sale of several of its
financial assets during the last year.
(a) Sale of a financial asset for Rs.20,000. There are no conditions attached to the sale and no
other rights and obligations are retained by MES.
(b) Sale of an investment in shares for Rs.20,000. However, MES retains a call option to
repurchase the shares at any time at the current fair value on the date of the repurchase of the
shares.

(c) Sale of a part of its short term receivables for Rs.200,000. According to the terms of the sale,
it promises to pay Rs.6,000 to compensate the buyer, CCE, for any defaults on payment. The
expected credit losses on this transaction are significantly lower than Rs.6,000 and there are
no significant risks.

(d) MES enters into a total return swap with EMW, the essence of which will return any increases
in the fair value of the shares worth Rs.20,000 sold to MES and compensate EMW for any
decreases in the fair value of the shares.

Required: State, to what extent derecognition of these assets is appropriate in the financial
statements in each of the above cases.

Part B
Consider the following cases:
(a) B Limited owes S Limited Rs.25,000. B Limited has set this amount aside in a special trust
that it will not use for any other purpose but to pay S Limited.
(b) AIM Limited pays Object Limited Rs.25,000 in discharge of an earlier obligation.
(c) A put option written by J&J Limited expires.

Required: In which cases would you, as an accountant, derecognize the above financial
liabilities?

17. Financial Instruments (NAS32, NAS39, NFRS 9)-Numerical


On 31 October 20X1, Bling had inventories of gold which cost $6.4m to buy and which could be
sold for $7.7m. The management of Bling are concerned about the risk of fluctuations in future
cash inflows from the sale of this gold.

To mitigate this risk, Bling entered into a futures contract on 31 October 20X1 to sell the gold for
$7.7m. The contracts mature on 31 March 20X2.

The hedging relationship was designated and documented at inception as a cash flow hedge. All
effectiveness criteria are complied with.

On 31 December 20X1, the fair value of the gold was $8.6m. The fair value of the futures
contract had fallen by $0.9m.

There is no change in fair value of the gold and the futures contract between 31 December 20X1
and 31 March 20X2. On 31 March 20X2, the inventory is sold for its fair value and the futures
contract is settled net with the bank.

Required:
(a) Discuss the accounting treatment of the hedge in the year ended 31 December 20X1.
(b) Outline the accounting treatment of the inventory sale and the futures contract settlement on
31 March 20X2.
Paper1:AdvancedFinancial Reporting [RTP 2021 June]
Page 279 of 328

18. Revenue (NFRS-15) – Numerical and Logical


Part(a)
Define ‘performance obligation’. List any six examples of promised goods and services as per
NFRS 15 ‘Revenue from Contracts with Customers’.

Part(b)
On 1 October 2017, Galaxy Telecommunications (GT) entered into a contract with a bank for
supplying 20 smart phones to the bank staff with unlimited use of mobile network for one year.
The contract price per smart phone is Rs. 34,650 and the price is payable in full within 10 days
from the date of contract. At the end of the contract, the phones will not be returned to GT.

The entire amount received as per contract was credited by GT to advance from customers
account. The smart phones were delivered on 1 November 2017.If sold separately, GT charges
Rs. 18,000 for a smart phone and a monthly fee of Rs. 1,800 for unlimited use of mobile network.

Required:
Prepare adjusting entry for the year ended 31 December 2017 in accordance withNFRS 15
‘Revenue from Contracts with Customers’.

19. Share Based Payment (NFRS-2) – Numerical


Management of Star Company is worried about excessive turnover of employees. The HR
Director has suggested to grant 250 cash share appreciation rights (SARs) to each of its 400
employees subject to condition that the employees continue to work for the entity for three years.
The board approved the proposal from July 1, 2009. The management expects that 20 employees
will leave each year. During three years, following employees left the company:

Years No. of employees


2009-2010 20
2010-2011 24
2011-2012 30

On June 30, 2012, 125 employees exercised their rights. The fair value of the share appreciation
rights for the year in which liability exists are shown below, together with the intrinsic value at
the date of exercise:

Year end Fair value (Rs.) Intrinsic value (Rs.)


2009-2010 12.50 11.50
2010-2011 14.00 14.50
2011-2012 16.50 17.00

Required:
Calculate the amount to be presented in the statement of financial position and statement of
comprehensive income for three years from 2010 to 2012.

20. Consolidated Financial statements (NFRS-10)


Hamachi Ltd acquired 90% of Saba Ltd’s Rs. 1 ordinary shares on 1 April 2014 paying Rs. 3.00
per share. The balance on Saba Ltd’s retained earnings at this date was Rs. 800,000. On 1 October
2015, Hamachi Ltd acquired 30% of Anogo Ltd’s Rs. 1 ordinary shares for Rs. 3.50 per share.
The statements of financial position of the three companies at 31 March 2016 are shown below:

Hamachi Ltd Saba Ltd Anogo Ltd


000 000 000 000 000 000
Non current assets
PPE 8,050 3,600 1,650
Investments 4,000 910 -
Paper1:AdvancedFinancial Reporting [RTP 2021 June]
Page 280 of 328

12,050 4,510 1,650


Current Assets
Inventory 830 340 250
Account Receivable 520 290 350
Bank 240 1,590 - 630 100 700
Total Assets 13,640 5,140 2,350
Equity and Liabilities
Equity
Ordinary Share (Rs. 1) 5,000 1,200 600
Retained Earning 6,000 1,400 800
Profit for the year 2016,
March 1,500 7,500 900 2,300 600 1,400
12,500 3,500 2,000
Non Current Liabilities
10% Loan notes 500 240 -
Current Liabilities
Account payable 420 960 200
Tax liability 220 250 150
Overdraft - 640 190 1,400 - 350
Total Equity and Liabilities 13,640 5,140 2,350

The following information is relevant


i. Fair value adjustments on 1 April 2014 Saba Ltd owned an investment property that had a
fair value of Rs. 120,000 in excess of its carrying value (book value). The value of this
property has not changed since acquisition. This property is included within investments in
the balance sheet.

Just prior to its acquisition, Saba Ltd was successful in applying for a six-year licence to
dispose of hazardous waste. The licence was granted by the government at no cost, however
Hamachi Ltd estimated that the licence was worth Rs. 180,000 at the date of acquisition.

ii. In January 2016 Hamachi Ltd sold goods to Anogo Ltd for Rs. 65,000. These were
transferred at a mark-up of 30% on cost. Two thirds of these goods were still in the
inventory of Anogo Ltd at 31 March 2016.

iii. To facilitate the consolidation procedures the group insists that all inter company current
account balances are settled prior to the year-end. However a cheque for Rs. 40,000 from
Saba Ltd to Hamachi Ltd was not received until early April 2016. Inter company balances
are included in accounts receivable and payable as appropriate.

iv. Anogo Ltd is to be treated as an associated company of Hamachi Ltd.

v. An impairment test at 31 March 2016 on the consolidated goodwill of Saba Ltd concluded
that it should be written down by Rs. 468,000. No other assets were impaired.

Required
a. Prepare the consolidated statement of financial position of Hamachi Ltd as at 31 March 2016.
b. Discuss the matters to consider in determining whether an investment in another company
constitutes associated company status.
Paper1:AdvancedFinancial Reporting [RTP 2021 June]
Page 281 of 328

Answer/Hints
1. NAS 34- Interim Financial Reporting – Numerical and conceptual
In the instant case, the quarterly net profit has not been correctly stated. As per NAS 34, Interim
Financial Reporting, the quarterly net profit should be adjusted and restated as follows:

(i) The treatment of bad debts is not correct as the expenses incurred during an interim
reporting period should be recognised in the same period. Accordingly, Rs 50,000 should
be deducted from Rs. 20,00,000.
(ii) Recognising additional depreciation of Rs. 4,50,000 in the same quarter is correct and is
in tune with NAS 34.
(iii) Treatment of exceptional loss is not as per the principles of NAS 34, as the entire amount
of Rs. 28,000 incurred during the third quarter should be recognized in the same quarter.
Hence Rs. 14,000 which was deferred should be deducted from the profits of third quarter
only.
(iv) As per NAS 34 the income and expense should be recognised when they are earned and
incurred respectively. As per para 39 of NAS 34, the costs should be anticipated or
deferred only when:
a. it is appropriate to anticipate or defer that type of cost at the end of the financial year,
and
b. costs are incurred unevenly during the financial year of an enterprise.

Therefore, the treatment done relating to deferment of Rs. 5,00,000 is not correct as expenditures
are uniform throughout all qua rters.

Thus considering the above, the correct net profits to be shown in Interim Financial Report of the
third quarter shall be Rs. 14,36,000 (Rs. 20,00,000 -Rs. 50,000 – Rs.14,000 – Rs. 5,00,000).

2. NFRS 8 Operating Segments (Theory)


a. An entity shall report separately information about an operating segment that meets any of the
following quantitative thresholds:

Its reported revenue, including both sales to external customers and intersegment sales or
transfers, is 10 per cent or more of the combined Revenue, internal and external, of all
operating segments.

The absolute amount of its reported profit or loss is 10 per cent or more of the greater, in
absolute amount, of (i) the combined reported profit of all operating segments that did not
report a loss and (ii) the combined reported loss of all operating segments that reported a loss.

Its assets are 10 per cent or more of the combined assets of all operating Segments. Operating
segments that do not meet any of the quantitative thresholds may be considered reportable,
and separately disclosed, if management believes that information about the segment would
be useful to users of the financial statements.

b. Part (a)
The purposes of segmental information are:
(i) to provide users of financial statements with sufficient details for them to be able to
appreciate the different rates of profitability, different opportunities for growth and
Paper1:AdvancedFinancial Reporting [RTP 2021 June]
Page 282 of 328

different degrees of risk that apply to an entity’s classes of business and various
geographical locations.
(ii) to appreciate more thoroughly the results and financial position of the entity by
permitting a better understanding of the entity’s past performance and thus a better
assessment of its future prospects.
(iii) to create awareness of the impact that changes in significant components of a
business may have on the business as a whole.

Part (b)

NFRS 8 defines an operating segment as a component of an entity:

- that engages in business activities from which it may earn revenues and incur expenses
(including revenues and expenses relating to transactions with other components of the
same entity).
- whose operating results are regularly reviewed by the entity’s chief operating decision
maker to make decisions about resources to be allocated to the segment and assess its
performance.
- for which discrete financial information is available.

In order to identify the separate reportable segments, the following criteria should be adopted:

i. The reported revenue of the segment in Shazad Industries Ltd, including both sales to
external customers and inter-segment sales, is ten percent or more of the combined
revenue of its four operating segments.
ii. The Assets of the segment in Shazad Industries Ltd are ten percent or more of the
combined assets of its four operating segments.
iii. The reported profit or loss of the segment in Shazad Industries Ltd should be ten
percent or more of the greater, in absolute amount, of:
- the combined reported profit of all its operating segments that did not report a
loss and
- the combined reported loss of all operating segments that reported losses.

Part (c)

NFRS 8 lays down some very broad and inclusive criteria for reporting segments. Unlike
earlier attempts to define segments in more quantitative terms, segments are defined largely in
terms of the breakdown and analysis used by management. This is, potentially, a very
powerful method of ensuring that preparers provide useful segmental information.

There will still be problems in deciding which segments to report, if only because
management may still attempt to reduce the amount of commercially sensitive information
that they produce.

The growing use of executive information systems and data management within businesses
makes it easier to generate reports on an ad hoc basis. It would be relatively easy to provide
management with a very basic set of internal reports and analyses and leave the individual
Paper1:AdvancedFinancial Reporting [RTP 2021 June]
Page 283 of 328

managers to prepare their own more detailed information using the interrogation software
provided by the system.

If such analyses become routine then they would be reportable under NFRS 8, but that would
be very difficult to check and audit.

There are problems in the measurement of segmental performance if the segments trade with
each other. Disclosure of details of inter-segment pricing policy is often considered to be
detrimental to the good of a company. There is little guidance on the policy for transfer
pricing.

Different internal reporting structures could lead to inconsistent and incompatible segmental
reports, even from companies in the same industry.

3. Events after the reporting period (NAS 10)


a. The bankruptcy of a customer that occurs after the end of reporting period usually confirms
that the loss existed at the end of reporting period. Assuming that the MR. P was doubtful at
the end of reporting period, the carrying amount of trade debts should be adjusted and a
provision should be recognised.

Date Particulars Dr. Rs Cr. Rs


30.06.2005 Doubtful debt expense (P&L) 500,000
Provision for doubtful debts 500,000
b. It is a non adjusting event as there was no obligation to rectify the problems at the end of
reporting period. Impairment test should also consider whether the software was impaired or
not at the end of reporting period. The following disclosures may be made:
Nature : Rectification of computer software
Financial effect : Rs. 31,000

c. The event of market crash occurred after the end of reporting period; therefore, it is a non
adjusting event. However, the following disclosures may be made:
Nature : Decline in market value of investments
Financial effect : Rs. 4,000,000

d. The price of laptops started falling after the end of the reporting period which indicates a new
condition of stock obsolescence. Therefore, it is a non adjusting event. However, the
following disclosure may be made:
Nature : Decline in prices of inventories
Financial effect : Rs. 20,900,000 i.e. (Rs. 65,000 x 950 units) –Rs. 40,850,000

e. The company had no obligation to provide warranty services at the end of reporting period
but the company had sold the items by then. In accordance with accrual basis, warranty cost
of Rs. 150,000 (which relates to the year end June 30, 2005) should be accounted for:
Paper1:AdvancedFinancial Reporting [RTP 2021 June]
Page 284 of 328

Date Particulars Dr. Rs Cr. Rs


30.06.2005 Warranty expenses (P&L) 150,000
Provision for warranty 150,000

4. Income Taxes (NAS-12) - Numerical

Part (a)

Date Particulars Dr. Rs. Cr. Rs.


01.07.14 Motor vehicle –cost 1,600,000
Lease liability 1,600,000
(capitalize the leased asset and corresponding liability)

01.07. 14 Lease liability 480,000

Bank 480,000

(Recording of first lease payment)

30.06. 15 Interest expense 153,451

Interest payable 153,451(W3)

30.06.15 Depreciation 400,000

Accumulated depreciation 400,000

(Rs. 1,600,000 / 4 years assuming that ownership is not be transferred)

30.06.15 Tax expense 1,492,035

Current tax payable 1,492,035(W1)

30.06.15 Deferred tax asset 22,035

Tax expense 22,035

W1 Current tax Rs.

Accounting profit 4,900,000

Add: Depreciation on leased motor vehicle 400,000


Paper1:AdvancedFinancial Reporting [RTP 2021 June]
Page 285 of 328

Add: Interest expense on lease 153,451

Less: Lease payment (480,000)

Taxable profit 4,973,451

Current tax @ 30% 1,492,035

CA TB Temp. Diff

W2 Deffered tax Rs. Rs. Rs.

Motor Vehicle 1,200,000 0 1,200,000 T

Lease liability(Non current + Current) 1,120,000 0 1,120,000 D

Interest payable 153,451 0 153,451 D

Net 73,451 D

Deferred tax asset @30% 22,035


Paper1:AdvancedFinancial Reporting [RTP 2021 June]
Page 286 of 328

Part (b)

Statement of comprehensive Income (extracts) 2015Rs.

Depreciation expense 400,000

Interest expense 153,451

Statement of financial position (extracts) 2015

Non-current assets Rs.

Motor Vehicle (Right of use) 1,600,000

Accumulated depreciation (400,000)

1,200,000

Non-current liabilities

Lease liability 793,451

Current liabilities

Lease liability 326,549

Interest payable 153,451

Disclosure

Undiscounted cash payments Rs.

Up to 1 year 480,000

2-5 years 960,000

More than 5 year -

Total 1,440,000

Working 3 Lease (Payments in advance) Schedule

Payment Liability at Principal Liability after Interest @


time beginning Rental Element payment 13.701%

01.07.14 1,600,000 480,000 480,000 1,120,000 153,451

01.07.15 1,120,000 480,000 326,549 793,451


Paper1:AdvancedFinancial Reporting [RTP 2021 June]
Page 287 of 328

5. Inventories – NAS-2- Numerical

The calculation of possible write-downs of inventory must not be done based on the classifications
(raw materials, work-in-progress and finished goods) but should be assessed on an item-by-item
basis.

Although both items of raw materials have net realisable values that are lower than cost, raw
materials should not be written-down unless the reason for the drop in the NRV of the raw materials
has also resulted in the NRV of the related finished product also dropping.

Since the NRV of the finished air coolers has dropped below cost, air coolers (raw materials) should
be written-down to their net realisable value (the NRV in this case is usually the net replacement
cost).
Despite the NRV of the finished fans having dropped, the NRV of the fans has not dropped below
cost. The fans (raw materials) should therefore not be written-down.
Calculation of write down Cost Rs. NRV Rs. NRV Loss Rs.

Raw Material 80,000 50,000 30,000

WIP 60,000 50,000 10,000

Finished goods 160,000 110,000 50,000

Raw Materials 120,000 100,000 -

WIP 100,000 120,000 -

Finished goods 160,000 220,000 -

6. PPE- Conceptual and Numerical (NAS-16)


Part (a)

The lathe was purchased in 2009 and was originally being written off over an estimated useful life of
twelve years. As at 1 January 2015 six of the years have elapsed with a further six years remaining. It
was decided that the machine will now only be usable for a further fouryears.

NAS 16 requires that where the original estimate of useful life is revised, adjustments should be made
in current and future periods (not in prior periods). The unamortized cost of the asset should be
charged to revenue over the remaining useful life of the asset. The net book value of Rs. 75,000
should therefore be charged over the remaining four years of useful life, giving an annual depreciation
charge of Rs. 18,750. The revision is not a change in accounting policy, or a fundamental error but a
change in accounting estimate. It is therefore not appropriate to deal with any excess depreciation by
adjusting opening retained earnings.
Paper1:AdvancedFinancial Reporting [RTP 2021 June]
Page 288 of 328

Part (b)

The grinder was purchased in 2012 and was originally being depreciated on a straight line basis. It has
now been decided to depreciate this on the sum of digits basis.

NAS 16 requires that depreciation methods be reviewed periodically and if there is a significant
change in the expected pattern of economic benefits, the method should be changed. Depreciation
adjustments should be made in current and future periods. This change might be appropriate if, for
instance, usage of the machine is greater in the early years of an asset’s life when it is still new and
consequently it is appropriate to have a higher depreciation charge. If the change is implemented, the
unamortized cost (the net book value) of the asset should be written off over the remaining useful life
commencing with the period in which the change is made.

The depreciation charge for the remaining life of the asset will therefore be as follows.

Year No of digits Depreciation


2015 7 (7/28 x 70,000) 17,500
2016 6 (6/28 x 70,000) 15,000
2017 5 12,500
2018 4 10,000
2019 3 7,500
2020 2 5,000
2021 1 2,500
½ x 7 (7 + 1) 28 70,000

Disclosure will need to be made in the accounts of the details of the change, including the effect on
the charge in the year.

Part (c)

NAS 16’s allowed alternative treatment in respect of measurement of property plant and equipment
(subsequent to initial recognition), is that of revaluation. Revaluation is made at fair value. Where any
item of property plant or equipment is revalued, the entire class to which the asset belongs should be
revalued. Revaluations must be kept up to date. Where there are volatile movements in fair value, the
revaluation should be performed annually. Where there are no such movements, revaluations every
three to five years may be appropriate.

Accumulated depreciation at the date of revaluation is either:

- Restated proportionately with the change in the gross carrying amount so that the carrying
amount after the revaluation equals the revalued amount (e.g. where revaluations are made to
depreciated replacement cost using indices)
- Eliminated against the gross carrying amount of the assets and the net amount restated to the
revalued amount of the asset (e.g. where buildings are revalued to their market value).

NAS 16 requires that the subsequent charge for depreciation should be based on the revalued amount.
The annual depreciation will therefore be Rs.62,500, i.e.Rs.1,500,000 divided by the 24 years of
Paper1:AdvancedFinancial Reporting [RTP 2021 June]
Page 289 of 328

remaining life. There will then be a difference between the revalued depreciation charge and the
historical depreciation charge. The resulting excess depreciation may be dealt with by a movement in
reserves, i.e. by transferring from the revaluation reserve to retained earnings a figure equal to the
depreciation charged on the revaluation surplus each year.

7. Intangible Assets - Conceptual (NAS-38)

Company Y’s The in-process research and development are not recognised in Company Y’s
financial financial statements (NAS 38 prohibits the recognition of internally generated
statements assets).

From the Company X group viewpoint, the in-process research and development
Company X are a purchased asset. Part of the consideration paid by Company X to buy
group viewpoint Company Y was to buy the knowhow resulting from the project and it should be
recognised in the consolidated financial statements at its fair value of Rs. 500,000.

8. Intangible Assets - Conceptual (NAS-38)


(i) “Biological asset” is a living animal or plant. An entity shall recognize a biological asset if all
the following conditions are met:
▪ The entity controls the asset as a result of past event;
▪ It is probable that future economic benefits associated with the asset will flow to the
entity;
▪ The fair value or cost of the asset can be measured reliably.

(ii) The new born cows are biological assets and should be measured at fair value less costs to sell,
both on initial recognition and at each reporting period.

The gains on initial recognition and the gains from change in this value should be recognized
in profit or loss for the period in which it arises. The total gains to be recognized in the year
ended 30 June 2015 is as follows:
Rs.
New born [26,000 × 225× (100%-2%)] 5,733,000
9 month old [53,000 × 75 × (100%-2%)] 3,895,500
9,628,500
Paper1:AdvancedFinancial Reporting [RTP 2021 June]
Page 290 of 328

9. Non current Assets Held for sale- Conceptual (NFRS-5)


PK Limited

Income statement

For the year ended 31 December 2010

Revenue 650,000

Cost of sales (320,000)

Gross Profit 330,000

Administrative expenses (120,000)

Distribution expenses (60,000)

Exceptional items (reorganization) (98,000)

Operating profit 52,000

Finance Costs (17,000)

Profit before tax 35,000

Taxation (31,000)

Profit after tax 4,000

Loss on Discontinued Operations (W1) (143,000)

Loss for the period (139,000)


’’’’

W1 Analysis of loss on discontinued operations $

Revenue 320,000

Cost of sales (150,000)

Gross Profit 170,000

Administrative expenses (110,000)

Distribution expenses (90,000)

Operating Loss (30,000)

Loss on Disposal (76,000)

Redundancy Costs (37,000)

Loss on Discontinued Operations (143,000)


Paper1:AdvancedFinancial Reporting [RTP 2021 June]
Page 291 of 328

10. Exploration for and Evaluation of Mineral Resources (NFRS-6)


A number of different methods of capitalizing E&E expenditure have historically been used by
entities in the extractives sector, in particular ‘successful efforts’, ‘area-of-interest’ and ‘full cost
accounting’. A partial capitalisation approach is also commonly followed in practice.

a) Successful efforts

Under this approach, in general only those costs that lead directly to the discovery, acquisition, or
development of specific, discrete mineral reserves are capitalised. Costs that are known, when they are
incurred, to fail to meet this criterion are generally charged to profit or loss as incurred. However,
some interpretations of the successful efforts method would result in capitalising the cost of
unsuccessful development wells. This might be where an entity views a specified area as being a
single cost centre (this may be smaller than an area-of-Interest – see below).

b) Area-of-Interest

An area of interest is an individual geological area which is considered to constitute a favourable


environment for the presence of a mineral deposit or an oil or natural gas field. Under this approach,
all E&E expenditure relating to an area of interest are grouped and capitalised, to the extent that the
costs are expected to be recouped either through the successful development and exploitation of the
area, or by its sale.

c) Full cost

The full cost method generally results in capitalising all costs incurred in prospecting, acquiring
mineral interests, exploration, appraisal, development and construction which are then accumulated in
large cost centres. There are certain aspects of full cost accounting that are not consistent with the
requirements of NFRS. This includes capitalizing all pre licence acquisition costs (which are not
within the scope of NFRS 6, as these do not form E&E activities), the need to classify E&E assets as
tangible or intangible which is often not carried out for large asset pools, and the need to test E&E
assets for impairment at the point at which E&E activity ceases and the assets are reclassified under
NAS 16 or NAS 38(which is typically not possible, as full cost accounting does not disaggregate cost
pools in the required level of detail).

d) Partial capitalisation

Under this approach, only some costs that are eligible for capitalisation are included in an entity’s
E&E asset. In some jurisdictions, a common approach is to capitalise initial acquisition costs for a
particular mining asset and to expense all subsequent costs.

11. Impairment of assets (Numerical – NAS 36)


Paper1:AdvancedFinancial Reporting [RTP 2021 June]
Page 292 of 328

Impairment of plant

The plant had a carrying amount of Rs.240,000 on 1 October 2015. The accident that may have
caused impairment occurred on 1 April 2016 and an impairment test would be done at this date. The
depreciation on the plant from 1 October 2015 to 1 April 2016 would be Rs.40,000 (640,000 x
121/2% x 6/12) giving a carrying amount of Rs.200,000 at the date of impairment. An impairment test
requires the plant’s carrying amount to be compared with its recoverable amount. The recoverable
amount of the plant is the higher of its value in use of Rs.150,000 or its fair value less costs to sell. If
Hussain Associates Ltd trades in the plant it would receive Rs.180,000 by way of a part exchange, but
this is conditional on buying new plant which Hussain Associates Ltd is reluctant to do. A more
realistic amount of the fair value of the plant is its current disposal value of only Rs.20,000. Thus the
recoverable amount would be its value in use of Rs.150,000 giving an impairment loss of Rs.50,000
(Rs.200,000 – Rs.150,000).

The remaining effect on income would be that a depreciation charge for the last six months of the year
would be required. As the damage has reduced the remaining life to only two years (from the date of
the impairment) the remaining depreciation would be Rs.37,500 (Rs.150,000/ 2 years x 6/12).Thus
extracts from the financial statements for the year ended 30 September 2016 would be:

Statement of financial position

Non-current assets Rs

Plant (150,000 – 37,500) 112,500

Statement of profit or loss

Plant depreciation (40,000 + 37,500) 77,500

Plant impairment loss 50,000

12. Provisions, Contingent Liabilities and Contingent Assets (Conceptual –NAS 37)
(i) As on 31 December 2018, OL should recognize a provision for warranty service to be
provided as there is a present obligation as a result of a past event (sale of A-1 and B-1 in
2018). The amount of provision would be:
o Rs. 2 million (4×50%) in respect of A-1 as OL is liable to SL for 50% cost of services.
o Rs. 7 million (entire cost) in respect of B-1 as OL is responsible to the customers for
providing warranty services.

OL is required to disclose a contingent liability for remaining warranty cost of A-1 (which
should be incurred by SL) as OL would be responsible for it in case of SL’s default. (Joint
and several liability).
Paper1:AdvancedFinancial Reporting [RTP 2021 June]
Page 293 of 328

Further OL should recognize a separate asset (receivable) to the extent that reimbursements
from SL in respect B-1 are virtually certain. In the statement of profit or loss, the expense
relating to warranty services may be presented net of the amount recognized as receivable
(reimbursement).

(ii) As on 31 December 2018, OL is required to record a liability of Rs. 5 million as this has
already been approved by OL. In respect of remaining amount of the claim, a provision of Rs.
6 million shall be made as it is most likely that OL would require to pay this amount as
advised by OL’s lawyers.

Further OL should recognize a separate asset (receivable) to the extent of Rs. 9 million as it is
accepted in principle by the supplier. Therefore, it will be taken as ‘virtually certain to be
received’. In the statement of profit or loss, the expense relating to the provision may be
presented net of amount recognized as receivable (reimbursement).

However, recovery of the claim to the extent of Rs. 3 million is probable, therefore, a
contingent asset would be disclosed.

(iii) Introduction of new alternative drug with better results is an indication of reduction in value
of existing medicine kept in stock. It is more evident by subsequent sales of such units at
lower price i.e. Rs. 8,000 with 10% commission to distributors. According to NAS 2,
inventory should be recorded at lower of cost or NRV (i.e. estimated selling price less
estimated costs necessary to make the sale). So OL is required to carry entire stock of this
medicine at NRV i.e. Rs. 36 million [5,000×7,200 (8,000 – 800)].

13. Accounting policies, estimates and error (Conceptual –NAS 08)


Duncan

Statement of Changes in Equity (Extracts)

For the year ended June 30, 2015

2015$000 2014$000

Opening balance as reported 22,500

Change in accounting policy W2 400

Restated balance 24,400 22,900

Profit after tax for the year W1 4,442 3,250

Dividend paid (2,500) (1,750)

Closing balance 26,342 24,400


Paper1:AdvancedFinancial Reporting [RTP 2021 June]
Page 294 of 328

Workings:

W1 – Revised Profit 2015$000 2014$000

Given 4,712 3,200

Add back: Expenditure for the year 600 500

Less: Depreciation (870) (450)

Revised profit 4,442 3,250

W2 – Impact of prior period adjustment 2015 2014


2013

b/d impact 450 400

Borrowing cost 600 500

Depreciation (870) (450)____________

180 450 400

14. Related Party Disclosure (NAS-24) – Logical


Answer 7

(i) AK Associates is a related party due to association of (Partner) of one of its key
management personnel (director). [NAS 24.9(b)(vi)]
(ii) SS Bank Limited is not a related party just being a lender (provider of finance) as
generally nominee directors of lenders cannot exert significant influence. [NAS 24.11(c)]
(iii) Mr. Zee can only be treated as related party if he is in a position to influence his brother
i.e. CEO of BL. (Although the definition of close family member does not specifically
cover brother, but in substance, transactions are being carried out with the brother of
CEO.) [NAS24.9(a)(iii)]
(iv) JB Limited is not a related party as distributor cannot be treated as related party only by
virtue of resulting economic dependence. [NAS 24.11(a)]
(v) Mr. Tee is a related party being a member of key management personnel (is responsible
for all major decisions of sales and discounts). [NAS 24.9(a)(iii)]
(vi) Gratuity fund is a relate party being post-employment benefit plan for employees of BL.
[NAS24.9(b)(v)]
(vii) MM Limited is not a related party as suppliers cannot be treated as related party only
byvirtue of resulting economic dependence. [NAS 24.11(a)]
(viii) Ms. Vee is a related party being close family member (wife) of member of key
management personnel (CEO). [NAS 24.9(a)(iii)]

15. The effects of Changes in Foreign Exchange Rates (NAS-21- Numerical)


Paper1:AdvancedFinancial Reporting [RTP 2021 June]
Page 295 of 328

Copper Limited
General Journal

Transaction i
Date Particulars Dr. (Rs.) Cr. (Rs.)
1-Jul-18 Advance payment (Machine) 18,150,000
Bank ($250,000 x 60% x 121) 18,150,000

1-Oct-18 PPE (Machine) 30,550,000

Advance payment 18,150,000

Payable ($250,000 x 40% x 124) 12,400,000

1-Apr-19 Payable 12,400,000

Exchange loss (PL) 1,600,000

Bank ($250,000 x 40% x 140) 14,000,000

Transaction ii
Date Particulars Dr. (Rs.) Cr. (Rs.)
1-Jan-19 Trade receivables 5,480,000
Sales ($40,000 x 137) 5,480,000

1-Apr-19 Bank ($40,000 x 25% x 140) 1,400,000

Exchange gain (PL) 30,000

Trade receivables (Rs. 5,480,000 x 25%) 1,370,000

30-Jun-19 Trade receivables [$40,000 x 75% (163 – 137)] 780,000

Exchange gain (PL) 780,000

16. Financial Instruments (NAS32, NAS39, NFRS 9)


Part A

(a) This financial asset should be derecognized in the financial statements of MES since all the
risks and rewards of ownership have been transferred to the purchaser.

(b) With the sale of the financial asset, all the risks and rewards of ownership have been
transferred to the purchaser of the asset. While MES has retained a call option on the shares,
the exercise price of the asset is the fair value of the asset as on the date of repurchase. This
makes the value of the call option zero and therefore no risks and rewards of ownership are
retained by MES. As a result, this asset should be derecognized on its sale.
Paper1:AdvancedFinancial Reporting [RTP 2021 June]
Page 296 of 328

(c) As MES agrees to compensate CCE for any defaults on payment, it retains the risk of
ownership. As a result, it should not derecognize the financial asset in its financial statements.

(d) Even after the swap, MES should continue to recognise the asset in its accounts as risk and
rewards have not been practically transferred.

Part B

Although B Limited has set aside the amount in a trust meant only for the purpose of paying S
Limited, it will still have to recognise the liability since it has not completely discharged the
obligation of paying S Limited.
(a) In this case, the obligation has been discharged and there is no further liability. Therefore, the
liability shall be derecognized.
(b) Since the put option has expired, the entity will not be able to pay anything and can therefore
derecognize the put option in its books of accounts.

17. Financial Instruments (NAS32, NAS39, NFRS 9)-Numerical


Part (a)

Between 1 October 20X1 and 31 December 20X1, the fair value of the futures contract had fallen by
$0.9m. Over the same time period, the hedged item (the estimated cash receipts from the sale of the
inventory) had increased by $0.9m ($8.6m – $7.7m).

Under a cash flow hedge, the movement in the fair value of the hedging instrument is accounted for
through other comprehensive income. Therefore, the following entry is required:

Dr Other comprehensive income $0.9m

Cr Derivative $0.9m

The loss recorded in other comprehensive income will be held within equity.

Part (b)

The following entries are required:

Dr Cash $8.6m

Cr Revenue $8.6m

Dr Cost of sales $6.4m

Cr Inventory $6.4m
Paper1:AdvancedFinancial Reporting [RTP 2021 June]
Page 297 of 328

To record the sale of the inventory at fair value

Dr Derivative $0.9m

Cr Cash $0.9m

To record the settlement of the futures contract

Dr Profit or loss $0.9m

Cr OCI $0.9m

To recycle the losses held in equity through profit or loss in the same period as the hedged item
affects profit or loss.

18. Revenue from Contract from Customers (NFRS-15) – Numerical and Logical

Part (a)
Performance obligation: A performance obligation is a promise in a contract with a customer to
transfer to the customer either:
a good or service (or a bundle of goods or services) that is distinct; or
a series of distinct goods or services that are substantially the same and that have the same
pattern of transfer to the customer.

Examples of promised goods and services


Goods produced by an entity for sale
Resale of goods purchased by an entity
Resale of rights to goods or services purchased by an entity
Performing a contractually agreed-upon task for a customer
Standing ready to provide goods or services
Providing a service of arranging for another party to transfer goods or services to the customer
Granting rights to goods or services to be provided in the future that a customer can resell
Constructing, manufacturing or developing an asset on behalf of a customer
Granting licences
Granting options to purchase additional goods/services

Part (b)
Adjusting entry
Debit
Credit
Advance from customers 378,000
Revenue (Smart phones) 18000/39600 x 34650 x 20 315,000
Revenue (Network-usage) 21600/39600 x 34650 x [20 x 2/12] 63,000

Smartphone 18,000 + Network usage 21,600(i.e.Rs.1800x12months] =Total 39,600


Paper1:AdvancedFinancial Reporting [RTP 2021 June]
Page 298 of 328

19. Share Based Payment (NFRS-2) – Numerical

Year # of Employees # of Options Fair value(Rs.) Vesting Period Liability(Rs.) Cash paid (Rs.) Expense (Rs.)
2010 340 x 250 x 12.50 x 1/3 = 354,167 - 354,167
2011 336 x 250 x 14.00 x 2/3 = 784,000 - 429,833
2012 201 x 250x 16.50 x 3/3 = 829,125 531,250 * 576,375

* 125 employees x 250 options x Rs. 17 intrinsic value = Rs. 531,250The liability shall be presented in statement of
financial position and the expense shall be presented in statement of comprehensive income.

20. Consolidated Financial statements (NFRS-10)

Part a
Hamachi Group
Consolidated Statement of Financial Position as at 31 March 2016

Non-current assets Rs.000


PPE 8,050 + 3,600 11,650
Goodwill W3 702
License 180 –60 120
Investment in associates W6 717
Investments including IP (Other) 4,000 + 910 –3,240 –630 + 120 1,160
14,349

Current assets

Inventory 830 + 340 1,170

Accounts receivables 520 + 290 –40 770

Bank 240 + 0 + 40 280

2,220

16,569

Equity
Share capital 5,000
Group RE W5 8,415
NCI W4 374
13,789

Non-current liabilities

10% loan notes 500 + 240 740

740

Current liabilities
Paper1:AdvancedFinancial Reporting [RTP 2021 June]
Page 299 of 328

Accounts payable 420 + 960 1,380

Taxation 220 + 250 470

Overdraft 190 + 0 190

2,040

16,569

W1 GROUP STRUCTURE

Saba Limited Acquisition date: 01-Apr-2014 Group 90% NCI 10%

Anogo Limited Acquisition date: 01-Oct-2015 Group 30%

W2 Net Assets of Subsidiary

At Acquisition date At Reporting date

Share capital 1,200 1,200

Retained earnings 800 2,300

+-Fair value adjustment effect(IP) 120 120

+-Fair value adjustment effect (License) 180 120

2,300 3,740

Post-acquisition change (at reporting date –at acquisition date) 1,440

W3 Goodwill

Investment [Rs. 3 x 1,200 x 90%] 3,240

+NCI at acquisition (W4) 230

3,470

-NA of subsidiary at acquisition (2,300)

1,170
Paper1:AdvancedFinancial Reporting [RTP 2021 June]
Page 300 of 328

-+ (Impairment) / Transfer of negative goodwill to W5 (468)

702

W4Non-Controlling Interest Rs. 000

NCI at acquisition [2,300 x 10%] 230

+ NCI share of post-acquisition[1,440 x 10%] 144

374

W5 Group Reserves

Parent’s reserves (100%) 7,500

-Impairment of goodwill (PARTIAL) (468)

Subsidiary’s post-acquisition x G%[1,440 x 90%] 1,296

+ -Effect of associate or joint venture (from W6) 87

8,415

W6 Investment in Associate or Joint venture

Cost of investment 630

+-share of profit and OCI since acquisition[600 x 30% x 6/12] 90

-share of unrealized profit in inventory[65 x 2/3 x 30/130 x 30%] (3)

717

Part b
NAS 28 Investments in Associates and Joint Ventures defines associates. In order for an investment to
be classified as an investment in an associate the investor must have ‘significant influence’ over the
investee. Significant influence is presumed to exist where there is a holding of 20% or more of the
voting power unless the investor can clearly demonstrate that this is not the case. Conversely a
holding of less than 20% is presumed not to be an associate, unless it can be clearly demonstrated that
the investor can exercise significant influence. The voting rights can be held directly or through
subsidiaries.

NAS 28 says that a majority holding by one investor does not preclude another investor having
significant influence. An investing company owning a majority holding in another company normally
has control over the investee and would thus class it as a subsidiary. In normal circumstances it is
difficult to see how a company could be controlled by one entity and be significantly influenced by a
Paper1:AdvancedFinancial Reporting [RTP 2021 June]
Page 301 of 328

different entity unless ‘control’ was passive. The 20%test is not definitive and the following other
evidence should be considered.

Does the investing company:

have representation on the Board of the investee?

participate in the policy making processes (operational and financial); have material transactions
with the investee?

interchange managerial personnel with the investee; or provide technical expertise to the investee
Paper1:AdvancedFinancial Reporting [RTP 2021 December]
Page 302 of 328

Questions:

1. Numerical and Conceptual - Contract Modifications (Revenue From Customers – NFRS 15)
On 1 January 2015, SL promises to sell 120 products to CL for $12,000 ($100 per product). The
products are to be transferred equally on January 31 and February 28. CL paid $12,000 on
inception of contract.

The 60 products were delivered on January 31.

On February 15, the contract is modified to require the delivery of an additional 30 products to
CL on March 10.CL paid additional amount on modification date i.e. Feb 15, 2015.

Situation 1
The price of the contract modification for the additional 30 products is an additional $2,850 or
$95 per product. The pricing for the additional products reflects the stand-alone selling price of
the products and the additional products are distinct from the original products.

Situation 2
The parties initially agree on a price of $80 per product. However, the customer discovers that the
initial 60 products transferred to the customer contained minor defects that were unique to those
delivered products. The entity promises a partial credit of $15 per product to compensate the
customer for the poor quality of those products. The entity and the customer agree to incorporate
the credit of $900 ($15 credit × 60 products) into the price that the entity charges for the
additional 30 products.

Required: Pass Journal entries.

2. Numerical and Conceptual - Estimating Variable Consideration (Revenue From Customers


– NFRS 15)
An entity enters into a contract with a customer to build a customised asset. The promise to
transfer the asset is a performance obligation that is satisfied overtime. The promised
consideration is $2.5 million, but that amount will be reduced or increased depending on the
timing of completion of the asset.

Specifically, for each day after 31 March 20X7 that the asset is incomplete, the promised
consideration is reduced by $10,000. For each day before 31 March 20X7 that the asset is
complete, the promised consideration increases by $10,000.

In addition, upon completion of the asset, a third party will inspect the asset and assign a rating
based on metrics that are defined in the contract. If the asset receives a specified rating, the entity
will be entitled to an incentive bonus of $150,000.

Required:
(a) Which method of estimation should be used for each component of variable consideration?
(b) Calculate the transaction price if entity estimates 60% chances that there will be delay of two
days (40% chance that it will build on due date) and 70% chances that specified rating shall
be received.
(c) Calculate the transaction price if entity estimates 35% chances to build 5 days before due
date, 40% chances to build on due date and 25% chances that delay of two days will be made.
There is 45% chance that specified rating shall be received.
Paper1:AdvancedFinancial Reporting [RTP 2021 December]
Page 303 of 328

3. Numerical - The Effects of Changes in Foreign Exchange Rates

K Limited commenced business on January 1, 2006 with an opening share capital of $2 million.
The income statement and closing balance sheet follow:

SPL & OCI for the year ended December 31, 2006
$m
Revenue 32
Cost of sales (10)
Gross profit 22
Distribution costs (8)
Administrative expenses (2)
Profit before tax 12
Tax expense (4)
Profit for period 8

Statement of financial position as at December 31, 2006


$m
Share capital 2
Retained earnings 8
10

Trade payables 4
Total equity and liabilities 14

Freehold land (acquired December 31, 2006) 8


Inventories 4
Trade receivables 2
Total assets 14

The functional currency is the $, but the entity wishes to present its financial statements using the
Euro as its presentation currency. The entity translates the opening share capital at the closing
rate. The exchange rates in the period were:
$1=
January 01, 2006 €1.0
December 31, 2006 €2.0
Average rate €1.5
Required: Translate the financial statements from the functional currency to presentation
currency?
4. Theory - Agriculture (NAS 41)
a. Is land related to agricultural activity a biological asset in terms of NAS 41?

b. What are the circumstances where an entity can depart from using fair value?

5. Numerical and Conceptual - Intangible Assets (NAS 38)


Zinc Limited (ZL), a broadcasting company, uses revaluation model for subsequent measurement
of its intangible assets, wherever possible. Following information pertains to ZL’s intangible
assets:
Paper1:AdvancedFinancial Reporting [RTP 2021 December]
Page 304 of 328

(i) On 1 January 2018, ZL bought an incomplete research and development project from Bee
Tech at its fair value of Rs. 90 million. The purchase price was analysed as follows:
Rs. in million
Research 30
Development 60

Subsequent expenditures incurred on this project are as follows:


Rs. in million
Further research to identify possible markets 10
Development 48
Recognition criteria for capitalization of development was met on 1 March 2018. All
costs are incurred evenly from 1 January 2018 till project completion date i.e. 31 August
2018. It is expected that newly developed technology will provide economic benefits to
ZL for the next 10 years.

On 31 December 2018, ZL received an offer of Rs. 170 million for its developed
technology.

(ii) On 31 December 2018, ZL launched its new website for online streaming of TV shows,
movies and web series. The website’s content is also used to advertise and promote ZL’s
products. The website was developed internally and met the criteria for recognition as an
intangible asset. Directly attributable costs incurred for the website are as follows:
Undertaking feasibility studies 3
Evaluating alternative products 1
Acquisition of web servers 16
Acquisition cost of operating system of web servers 7
Registration of domain names 2
Stress testing to ensure that website operates in intended manner 3
Designing the appearance of web pages 5
Development cost of new content related to:
▪ online streaming 11
▪ advertising and promoting ZL’s products 8
Advertising of the website 6

(iii) During 2018, the licensing authority intimated that broadcasting license of one of ZL’s
channels will not be further renewed.
ZL had obtained this license for indefinite period on 1 January 2012 by paying Rs. 150
million, subject to renewal fee of Rs. 0.3 million at every five years. Upto last year, this
license was expected to contribute to ZL’s cash inflows for indefinite period.
As on 31 December 2018, the recoverable amount of this license was assessed as Rs. 105
million.

Required:
In accordance with the requirements of IFRSs, prepare a note on intangible assets, for inclusion in
ZL’s financial statements for the year ended 31 December 2018 in respect of the above intangible
assets. (‘Total’ column is not required)
Paper1:AdvancedFinancial Reporting [RTP 2021 December]
Page 305 of 328

6. Numerical and Conceptual - Financial Instruments


Chive has a firm commitment to buy an item of machinery for CU2m on 31 March 20X2. The
Directors are worried about the risk of exchange rate fluctuations.

On 1 October 20X1, when the exchange rate is CU2:$1, Chive enters into a futures contract to
buy CU2m for $1m on 31 March 20X2.

At 31 December 20X1, CU2m would cost $1,100,000. The fair value of the futures contract has
risen to $95,000. All effectiveness criteria have been complied with.

Required:
Explain the accounting treatment of the above in the financial statements for the year ended 31
December 20X1 if:
(a) Hedge accounting was not used.
(b) On 1 October 20X1, the futures contract was designated as a fair value hedge of the
movements in the fair value of the firm commitment to purchase the machine.

7. Numerical - Impairment of Assets


The assistant financial controller of the Hussain Associates Ltd group has identified the matters
below which she believes may indicate impairment of one or more assets.

Hussain Associates Ltd owns and operates an item of plant that cost Rs.640,000 and had
accumulated depreciation of Rs.400,000 at 1 October 2015. It is being depreciated at 12½% on
cost.

On 1 April 2016 (exactly half way through the year) the plant was damaged when a factory
vehicle collided into it. Due to the unavailability of replacement parts, it is not possible to repair
the plant, but it still operates, albeit at a reduced capacity. It is also expected that as a result of the
damage the remaining life of the plant from the date of the damage will be only two years.

Based on its reduced capacity, the estimated present value of the plant in use is Rs.150,000. The
plant has a current disposal value of Rs.20,000 (which will be nil in two years’ time), but Hussain
Associates Ltd has been offered a trade-in value of Rs.180,000 against a replacement machine
which has a cost of Rs.1 million (there would be no disposal costs for the replaced plant). Hussain
Associates Ltd is reluctant to replace the plant as it is worried about the long-term demand for the
product produced by the plant. The trade-in value is only available if the plant is replaced.

Required
Prepare extracts from the statement of financial position and statement of profit or loss of Hussain
Associates Ltd in respect of the plant for the year ended 30 September 2016. Your answer should
explain how you arrived at your figures.

8. Conceptual - Provisions, Contingent Liabilities and Contingent Assets


Faraz is a chartered accountant and employed as Finance Manager of Gladiator Limited (GL). He
has recently returned after a long medical leave and has been provided with draft financial
statements of GL for the year ended 30 June 2017. Following figures are reflected in the draft
financial statements:
Rs. in million
Profit before tax 125
Total assets 1,420
Total liabilities 925

While reviewing the financial statements, he noted the following issues:


Paper1:AdvancedFinancial Reporting [RTP 2021 December]
Page 306 of 328

As at 30 June 2017, dismantling cost relating to a plant has increased from initial estimate of Rs.
30 million to Rs. 40 million. Further, fair value of the plant on that date was assessed at Rs. 112
million (net of dismantling cost). No accounting entries have been made in respect of increase in
dismantling liability and revaluation of the plant.

The plant had a useful life of 5 years when it was purchased on 1 July 2015. The carrying value of
plant and related revaluation surplus included in the financial statements are Rs. 135.4 million
(after depreciation for the year ended 30 June 2017) and Rs. 3.15 million (after transferring
incremental depreciation for the year ended 30 June 2017) respectively.

The appropriate discount rate is 8%.

Required: Determine the revised amounts of profit before tax, total assets and total
liabilities after incorporating the impact of above adjustments, if any.

9. Conceptual - Related Party Disclosures (NAS 24)


The following related party transactions were carried out by Golden Limited (GL) during the first
year of its operation i.e. year ended December 31, 2009.
(i) Inventory costing Rs. 15 million was sold for Rs. 18 million to Platinum Limited (PL)
which owns 60% shares in GL. It is GL’s policy to add 30% margin on cost. Outstanding
liability at year end, in respect of these purchases was Rs. 6.5 million.
(ii) PL provided administrative services to GL. The cost of these services, if billed in the
open market, would have amounted to Rs. 350,000. No entries were made to record these
transactions, as it was agreed that the services would be provided free of charge.
(iii) A property was sold to Silver Limited (SL), an associated company, at its fair market
value of Rs. 10 million. 50% of the amount was settled prior to year end. GL reimbursed
Rs.500,000 to SL on account of transfer and other incidental charges related to this
property.
(iv) An interest free loan of Rs. 2 million was granted to an executive director of the company
under the terms of employment. During the year, Rs. 200,000 were repaid by the
executive director.
(v) On July 1, 2009 GL obtained a short term loan of Rs. 25 million from one of its major
shareholder, at the prevailing annual interest rate of 12%. The principal as well as the
accrued mark-up were outstanding at the close of the year.

Required:
Prepare a note on related party transactions for inclusion in GL’s financial statements for the year
ended December 31, 2009 showing disclosures as required under NAS 24 (Related Party
Disclosures).

10. Logical - Events After The Reporting Period (NAS 10)


While preparing its financial statements for the year ended 31st March, 20X1, XYZ Ltd. made a
general provision for bad debts @ 5% of its debtors. In the last week of February, 20X1 a debtor
for Rs. 2 lakhs had suffered heavy loss due to an earthquake; the loss was not covered by any
insurance policy. Considering the event of earthquake, XYZ Ltd. made a provision @ 50% of the
amount receivable from that debtor apart from the general provision of 5% on remaining debtors.
In April, 20X1 the debtor became bankrupt. Can XYZ Ltd. provide for the full loss arising out of
insolvency of the debtor in the financial statements for the year ended 31st March, 20X1?

Would the answer be different if earthquake had taken place after 31st March, 20X1, and
therefore, XYZ Ltd. did not make any specific provision in context that debtor and made only
general provision for bad debts @ 5% on total debtors?
Paper1:AdvancedFinancial Reporting [RTP 2021 December]
Page 307 of 328

11. Logical - Fair Value Measurement (NFRS 13)


Company J acquires land in a business combination. The land is currently developed for industrial
use as a factory site. Although the land‘s current use is presumed to be its highest and best use
unless market or other factors suggest a different use, Company J considers the fact that nearby
sites have recently been developed for residential use as high-rise apartment buildings.

On the basis of that development and recent zoning and other changes to facilitate that
development, Company J determines that the land currently used as a factory site could be
developed as a residential site (e.g., for high -rise apartment buildings) and that market
participants would take into account the potential to develop the site for residential use when
pricing the land.

Determine the highest and best use of the land.

12. Conceptual - Presentation of Financial Statements


OMN Ltd has a subsidiary MN Ltd. OMN Ltd provides a loan to MN Ltd at 8% interest to be
paid annually. The loan is required to be paid whenever demanded back by OMN Ltd. How
should the loan be classified in the financial statements of OMN Ltd? Will it be any different for
MN Ltd?

13. Numerical - Cancellation- Equity Settled Share based payment


Anara Fertilisers Limited issued 2,000 share options to its 10 directors for an exercise price of
Rs.100.The directors are required to stay with the company for next 3 years.

Fair value of the option estimated Rs. 130


Expected number of directors to vest the option 8

During the year 2, there was a crisis in the company and Management decided to cancel the
scheme immediately. It was estimated further as below-

Fair value of option at the time of cancellation was Rs. 90


Market price of the share at the cancellation date was Rs. 99

There was a compensation which was paid to directors and only 9 directors were currently in
employment. At the time of cancellation of such scheme, it was agreed to pay an amount of
Rs. 95 per option to each of 9 directors.

How the cancellation would be recorded?

14. Conceptual - Property, Plant and Equipment


An entity has a nuclear power plant and a related decommissioning liability. The nuclear power
plant started operating on 1st April, 2XX1. The plant has a useful life of 40 years. Its initial cost
was Rs. 1,20,000 which included an amount for decommissioning costs of Rs 10,000, which
represented Rs 70,400 in estimated cash flows payable in 40 years discounted at a risk-adjusted
rate of 5 per cent. The entity’s financial year ends on 31st March. On March, 2X11, the net
present value of the decommissioning liability has decreased by Rs 8,000. The discount rate has
not yet changed.

How the entity will account for the above changes in decommissioning liability in the year 2X11,
if it adopts cost model?

15. Numerical - Borrowing Costs (NAS 23)


On 1st April, 20X1, A Ltd. took a 8% loan of ` 50,00,000 for construction of building A which is
repayable after 6 years ie on 31st March 20X7. The construction of building was completed on
31st March 20X3. A Ltd. started constructing a new building B in the year 20X3-20X4, for which
Paper1:AdvancedFinancial Reporting [RTP 2021 December]
Page 308 of 328

he used his existing borrowings. He has outstanding general purpose loan of Rs 25,00,000,
interest on which is payable @ 9% and Rs 15,00,000, interest on which is payable @ 7%. Is the
specific borrowing transferred to the general borrowings pool once the respective qualifying asset
is completed? Why.

16. Theory - Integrated Reporting


a. State the categories defined in the International IR Framework for capitals. Comment
whether an organisation has to follow these categories rigidly.
b. Can a Not-for Profit organisation do the Integrated Reporting as per the Framework?
c. Can an Integrated reporting be done in compliance to the requirements of the local laws to
prepare a management commentary or other reports?

17. Numerical - Employee Benefits (NAS 19)


A Ltd. prepares its financial statements to 31st March each year. It operates a defined benefit
retirement benefits plan on behalf of current and former employees. A Ltd. receives advice from
actuaries regarding contribution levels and overall liabilities of the plan to pay benefits. On 1st
April, 20X1, the actuaries advised that the present value of the defined benefit obligation was Rs.
6,00,00,000. On the same date, the fair value of the assets of the defined benefit plan was Rs.
5,20,00,000. On 1st April, 20X1, the annual market yield on government bonds was 5%. During
the year ended 31st March, 20X2, A Ltd. made contributions of Rs. 70,00,000 into the plan and
the plan paid out benefits of Rs. 42,00,000 to retired members. Both these payments were made
on 31st March, 20X2.

The actuaries advised that the current service cost for the year ended 31st March, 20X2 was Rs.
62,00,000. On 28th February, 20X2, the rules of the plan were amended with retrospective effect.
These amendments meant that the present value of the defined benefit obligation was increased by
Rs. 15,00,000 from that date.

During the year ended 31st March, 20X2, A Ltd. was in negotiation with employee
representatives regarding planned redundancies. The negotiations were completed shortly before
the year end and redundancy packages were agreed. The impact of these redundancies was to
reduce the present value of the defined benefit obligation by Rs. 80,00,000. Before 31st March,
20X2, A Ltd. made payments of Rs. 75,00,000 to the employees affected by the redundancies in
compensation for the curtailment of their benefits. These payments were made out of the assets of
the retirement benefits plan.

On 31st March, 20X2, the actuaries advised that the present value of the defined benefit
obligation was Rs. 6,80,00,000. On the same date, the fair value of the assets of the defined
benefit plan were Rs. 5,60,00,000.

Examine and present how the above event would be reported in the financial statements of A Ltd.
for the year ended 31st March, 20X2 as per NAS.

18. Numerical - Income Taxes (NAS 12)


An entity is finalizing its financial statements for the year ended 31st March, 20X2. Before 31st
March, 20X2, the government announced that the tax rate was to be amended from 40 per cent to
45 per cent of taxable profit from 30th June, 20X2.

The legislation to amend the tax rate has not yet been approved by the legislature. However, the
government has a significant majority and it is usual, in the tax jurisdiction concerned, to regard
an announcement of a change in the tax rate as having the substantive effect of actual enactment
(i.e. it is substantively enacted).

After performing the income tax calculations at the rate of 40 per cent, the entity has the
following deferred tax asset and deferred tax liability balances:
Paper1:AdvancedFinancial Reporting [RTP 2021 December]
Page 309 of 328

Deferred tax asset Rs. 80,000


Deferred tax liability Rs. 60,000

Of the deferred tax asset balance, Rs. 28,000 related to a temporary difference. This deferred tax
asset had previously been recognised in OCI and accumulated in equity as a revaluation surplus.

The entity reviewed the carrying amount of the asset in accordance with para 56 of NAS 12 and
determined that it was probable that sufficient taxable profit to allow utilisation of the deferred tax
asset would be available in the future.

Show the revised amount of Deferred tax asset & Deferred tax liability and present the necessary
journal entries.

19. Numerical - Earning Per Share (NAS 33)


Calculate Subsidiary’s and Group’s Basic EPS and Diluted EPS, when
Parent:
Profit attributable to ordinary equity holders of the parent entity:
12,000 (excluding any earnings of, or dividends paid by, the subsidiary)

Ordinary shares outstanding: 10,000

Instruments of subsidiary owned by the parent:


800 ordinary shares
30 warrants exercisable to purchase ordinary shares of subsidiary
300 convertible preference shares

Subsidiary:
Profit Rs. 5,400
Ordinary shares outstanding 1,000
Warrants 150, exercisable to purchase ordinary shares of the subsidiary

Exercise price Rs. 10

Average market price of one ordinary share Rs20

Convertible preference shares 400, each convertible into one ordinary share

Dividends on preference shares Re.1 per share

Other Information
No inter-company eliminations or adjustments were necessary except for dividends.
Ignore income taxes. Also, ignore classification of the components of convertible
financial instruments as liabilities and equity or the classification of related interest
and dividends as expenses and equity as required by NAS 32.

20. Consolidated Financial Statements


L Company was formed more than 30 years ago. L and its subsidiaries (D Company and
M Company) deal in various goods. The companies have branches in 5 countries on the
continent of Asia.
The following draft statements of financial position relate to L, D and M as at Ashadh end 2078:
Paper1:AdvancedFinancial Reporting [RTP 2021 December]
Page 310 of 328

L Company D Company M Company

(Rs. in '000) (Rs. in '000) (Rs. in '000)

Assets

Non-current Assets

Property, plant and equipment 23,534 21,000 12,600

Investment in:

D Company 7,300 - -

M Company 2,150 8,600 -

32,984 29,600 12,600


Current Assets 19,466 10,920 9,660

Total Assets 52,450 40,520 22,260

Equity and Liabilities


Equity

Equity shares of Re. 0.50 each 9,660 7,980 7,560

Other components of equity 4,410 3,570 3,150

Retained earnings 6,250 (1,050) 2,520

Total Equity 20,320 10,500 13,230

Liabilities

Non-current Liabilities 25,200 21,930 7,140


Current Liabilities 6,930 8,090 1,890

Total Liabilities 32,130 30,020 9,030

Total Equity and Liabilities 52,450 40,520 22,260

The following information is relevant in the preparation of group accounts:


i) L acquired 80% of D company equity shares and 15% of M company equity shares on 1
Shrawan 2074 for a cash consideration of Rs. 7,300,000 and Rs. 2,150,000 respectively. On
that date, the fair value of D company identifiable net assets stood at Rs. 8,980,000. On the
same date, other components of equity and retained earnings were Rs. 630,000 and Rs.
210,000 respectively. The excess in fair value relates to non-depreciable land.
ii) D Company acquired 60% of M company equity shares on Ashadh end 2075 for a cash
consideration of Rs. 8,600,000. The fair values of identifiable net assets stood at Rs.
9,764,000 and Rs. 9,384,000 on Ashadh end 2075 and 1 Shrawan 2074 respectively. On
Ashadh end 2075, other components of equity and retained earnings were Rs. 1,890,000 and
Rs. 714,000 respectively. Other components of equity and retained earnings were Rs.
1,610,000 and Rs. 614,000 respectively on 1 Shrawan 2074. The deficit in fair values relates
to an item of plant. The plant had an economic useful life of five years on 1 Shrawan 2074.
The fair value of L’s 15% holding at M at Ashadh end 2075 was still Rs. 2,150,000.
Paper1:AdvancedFinancial Reporting [RTP 2021 December]
Page 311 of 328

iii) On Ashadh end 2078, L disposed all of its 100% equity interest in N company for a cash
consideration of Rs. 3,800,000. L had acquired 100% equity interest in N on 1 Shrawan 2070
for cash consideration of Rs. 3,640,000. The fair value of N’s identifiable net assets stood at
Rs. 4,800,000 and 3,600,000 on Ashadh end 2078 and 1 Shrawan 2070 respectively.
Goodwill had not been impaired up to the date of disposal. This transaction was correctly
recorded in L's individual books.
iv) D and M have not issued any additional shares since their respective acquisition dates.
Further, fair value adjustments referred to in (i) and (ii) above have not been incorporated in
the above statements of financial position.
v) Investment in D and M were classified as financial assets through other comprehensive
income in accordance with NFRS 9 “Financial Instrument”. The investments are shown in the
statement of financial position of L at cost. This is because changes in their respective fair
values were insignificant.
vi) Goodwill in D and M has not been impaired as at Ashadh end 2078.
vii) It is the group policy to value non-controlling interest using proportion of net assets method.

Required:

(a) Prepare the consolidated statement of financial position of L group as at Ashadh end
2078.
(b) Calculate profit on disposal of 100% equity interests in N included in individual books of
L as at Ashadh end 2078.
Paper1:AdvancedFinancial Reporting [RTP 2021 December]
Page 312 of 328

Answers:

1. Numerical and Conceptual - Contract Modifications (Revenue from Contracts with


Customers – NFRS 15)
Situation 1
This is separate contract that does not affect accounting for existing contract.
Date Particulars Dr. $ Cr. $
01.01.15 Bank 12,000
Advance (Unearned revenue) 12,000
(cash received from customer)

31.01.15 Advance (Unearned revenue) 6,000


Revenue 6,000
(on delivery of first 60 items)

15.02.15 Bank 2,850


Advance (Unearned revenue) 2,850
(cash received from customer for additional items)

28.02.15 Advance (Unearned revenue) 6,000


Revenue 6,000
(on delivery of next 60 items)

10.03.15 Advance (Unearned revenue) 2,850


Revenue 2,850
(on delivery of additional 30 items)

Situation 2
It is termination of original contract and creation of new one.
Date Particulars Dr. $ Cr. $
01.01.15 Bank 12,000
Advance (Unearned revenue) 12,000
(cash received from customer)
31.01.15 Advance (Unearned revenue) 6,000
Revenue 6,000
(on delivery of first 60 items)

15.02.15 Bank 1,500


Revenue (reversed) 900
Advance (Unearned revenue) 2,400
(cash received from customer for additional items)

28.02.15 Advance (Unearned revenue) 5,600


Revenue 5,600
(on delivery of next 60 items)(6000 + 2400)/90 items x 60

10.03.15 Advance (Unearned revenue) 2,800


Revenue 2,800
(on delivery of 30 items) (6000 + 2400)/90 items x 30
Paper1:AdvancedFinancial Reporting [RTP 2021 December]
Page 313 of 328

2. Numerical and Conceptual - Estimating Variable Consideration (Revenue from Contracts


with Customers – NFRS 15)

Part (a)
The entity decides to use the expected value method to estimate the variable consideration associated
with the daily penalty or incentive (ie $2.5 million, plus or minus $10,000 per day). This is because it
is the method that the entity expects to better predict the amount of consideration to which it will be
entitled.
The entity decides to use the most likely amount to estimate the variable consideration associated with
the incentive bonus. This is because there are only two possible outcomes ($150,000 or $0) and it is
the method that the entity expects to better predict the amount of consideration to which it will be
entitled.
Part (b) $
2,500,000 x 40% 1,000,000
(2,500,000 -20,000) x 60% 1,488,000
150,000 150,000
2,638,000

Part (c) $
2,500,000 x 40% 1,000,000
(2,500,000 + 50,000) x 35% 892,500
(2,500,000 -20,000) x 25% 620,000
150,000 0_______
2,512,500

3. Numerical - The Effects of Changes in Foreign Exchange Rates

Statement of Comprehensive Income for the year ended December 31, 2006
(at average rates)
€1.5=$1
Revenue 48
Cost of sales (15)
Gross profit 33
Distribution costs (12)
Administrative expenses (3)
Profit before tax 18
Tax expense (6)
Profit for period 12
Paper1:AdvancedFinancial Reporting [RTP 2021 December]
Page 314 of 328

Other comprehensive income (see note below) 4


Total comprehensive income 16

Statement of financial position as at December 31, 2006


€2.0=$1
Share capital (closing rate) 4
Retained earnings (as calculated above) 12
Exchange difference (see note below) 4
20

Trade payables 8
Total equity and liabilities 28

Freehold land (acquired December 31, 2006) 16


Inventories 8
Trade receivables 4
Total assets 28
Note:
The retained earnings if translated into Euros would be €16 million. As the income statement has
been translated using the average rate, the profit as per income statement is €12 million, creating an
exchange difference of €4 million.
4. Theory - Agriculture (NAS 41)

c. No. Land owned by the entity and used for agricultural activity is subject to the recognition
and measurement principles of NAS 16, ‘Property, plant and equipment‘. Land owned by a
third party and rented to the entity for the purposes of agricultural activity is likely to be the
third party’s investment property and is accounted for in accordance with NAS 40,
‘Investment Property’.

d. There are two occasions where the standard permits departure from current fair value: at the
early stage of an asset’s life; and when fair value cannot be measured reliably on initial
recognition.

The first exemption is a practical expedient. The standard allows that cost may approximate
fair value where little biological transformation has taken place since the initial cost was
incurred (for example, for fruit tree seedlings planted immediately before the balance sheet
date). The same applies when the impact of the biological transformation on price is not
expected to be material (for example, for the initial growth in a 30-year pine plantation cycle)
[NAS 41 para 24].

The second exemption – that fair value cannot be reliably measured – is almost never
relevant. The standard includes a presumption that fair value can be measured reliably for a
biological asset. That presumption can be rebutted only on initial recognition for a biological
Paper1:AdvancedFinancial Reporting [RTP 2021 December]
Page 315 of 328

asset for which market-determined prices or values are not available and for which alternative
estimates of fair value are determined to be clearly unreliable.

In determining whether an estimate is ‘clearly unreliable’, a history of large variations in


outcome of the biological transformation process is not relevant, as this should be factored
into the measurement model. Similarly large fluctuations in the price of the final produce are
not a justification for an estimate to be clearly unreliable. The fact that the asset has a very
long production cycle and there is no forward market price is not an excuse not to measure the
asset at fair value. Only when the asset is unique or of a very special nature may estimates be
unreliable.

The term ‘clearly unreliable’ is not used elsewhere in the IFRS literature, and based on the
objective of the standard it is a high hurdle to clear.

In the event that the estimate of its fair value is deemed to be clearly unreliable, that
biological asset is measured at its cost less any accumulated depreciation and any
accumulated impairment losses [NAS 41 para 30]. Note that determining whether an asset is
impaired requires an estimate of its value.

As the exemption is only available on initial recognition, to rebut the presumption an existing
preparer must either have been gifted an asset that cannot be valued or be able to demonstrate
that the price paid for the asset was not an arm’s length market price. A first-time adopter can
only use this exemption until such time as the asset has a market price or can be valued using
a valuation technique. Once the biological asset has been fair valued, the cost model no longer
applies.

5. Numerical and Conceptual - Intangible Assets (NAS 38)

Zinc Limited
Notes to the financial statements
For the year ended 31 December 2018
R&D Website License
INTANGIBLE ASSETS
----------- Rs. in million -----------

Cost

As at 1 Jan 150

Separate acquisition 90 N1

Development 36 N2 21 N4
As at 31 Dec
126 21 150

Accumulated amortization / Impairment

As at 1 Jan

Amortisation for the year 4.2 N3 37.5 N5

Impairment loss 7.5 N6


As at 31 Dec
4.2 45
Paper1:AdvancedFinancial Reporting [RTP 2021 December]
Page 316 of 328

Carrying amount
121.8 21 105

Useful life 10 NA 4

Amortisation method Straight line NA Straight line

N1: Separately acquired R&D can be capitalised.


N2: Development costs to be capitalised since criteria met i.e. Rs. 48 x 6/8 months = Rs. 36
N3: 90 + 36 = 126 / 10 years x 4/12 = Rs. 4.2
N4: 2 domain + 3 testing + 5 design + 11 streaming = Rs. 21
N5: 150 / 4 years = Rs. 37.5
N6: (150 – 37.5) – 105 = Rs. 7.5

6. Numerical and Conceptual - Financial Instruments


Part a
The futures contract is a derivative and is measured at fair value with all movements being accounted
for through profit or loss. The fair value of the futures contract at 1 October 20X1 was nil. By the year
end, it had risen to $95,000. Therefore, at 31 December 20X1, Chive will recognise an asset at
$95,000 and a gain of $95,000 will be recorded in profit or loss.
Part b
If the relationship had been designated as a fair value hedge then the movement in the fair value of the
hedging instrument (the future) and the fair value of the hedged item (the firm commitment) since
inception of the hedge are accounted for through profit or loss. The derivative has increased in fair
value from $nil at 1 October 20X1 to $95,000 at 31 December 20X1. Purchasing CU2 million at 31
December 20X1 would cost Chive $100,000 more than it would have done at 1 October 20X1.
Therefore the fair value of the firm commitment has fallen by $100,000.
At year end, the derivative will be held at its fair value of $95,000, and the gain of $95,000 will be
recorded in profit or loss.
The $100,000 fall in the fair value of the commitment will also be accounted for, with an expense
recognised in profit or loss.
Dr Derivative $95,000
Cr Profit or loss $95,000

Dr Profit or loss $100,000


Cr Firm commitment $100,000
The gain on the derivative and the loss on the firm commitment largely net off. There is a residual
$5,000 ($100,000 – $95,000) net expense in profit or loss due to hedge ineffectiveness. Nonetheless,
financial statement volatility is far less than if hedge accounting had not been used.
Paper1:AdvancedFinancial Reporting [RTP 2021 December]
Page 317 of 328

7. Numerical - Impairment of Assets


Impairment of plant
The plant had a carrying amount of Rs. 240,000 on 1 October 2015. The accident that may have
caused impairment occurred on 1 April 2016 and an impairment test would be done at this date. The
depreciation on the plant from 1 October 2015 to 1 April 2016 would be Rs. 40,000 (640,000 x
121/2% x 6/12) giving a carrying amount of Rs. 200,000 at the date of impairment. An impairment
test requires the plant’s carrying amount to be compared with its recoverable amount. The recoverable
amount of the plant is the higher of its value in use of Rs. 150,000 or its fair value less costs to sell. If
Hussain Associates Ltd trades in the plant it would receive Rs. 180,000 by way of a part exchange,
but this is conditional on buying new plant which Hussain Associates Ltd is reluctant to do. A more
realistic amount of the fair value of the plant is its current disposal value of only Rs. 20,000. Thus the
recoverable amount would be its value in use of Rs. 150,000 giving an impairment loss of Rs. 50,000
(Rs.200,000 – Rs.150,000).
The remaining effect on income would be that a depreciation charge for the last six months of the year
would be required. As the damage has reduced the remaining life to only two years (from the date of
the impairment) the remaining depreciation would be Rs.37,500 (Rs.150,000/ 2 years x 6/12).Thus
extracts from the financial statements for the year ended 30 September 2016 would be:
Extracts of Financials
Statement of financial position

Non-current assets Rs.


Plant (150,000 – 37,500) 112,500

Statement of profit or loss


Rs.
Plant depreciation (40,000 + 37,500) 77,500
Plant impairment loss 50,000

8. Conceptual - Provisions, Contingent Liabilities and Contingent Assets

Net profit Total assets Total liabilities


As per question 125 1,420 925
Revaluation of plant (see below) 8.35
Increase in provision (see below) 7.94
Revised amounts 125 1,428.35 932.94

PPE Prov. PL RS/OCI


As given & 30 x PVIF(8%, 3yrs) 135.4 23.81 - 3.15
Paper1:AdvancedFinancial Reporting [RTP 2021 December]
Page 318 of 328

Increase in provision - 7.94 4.79 (3.15)


Revaluation effect 8.35 - (4.79) 3.56
143.75* 31.75# Nil 3.56

#June 30, 2016 = 40* PVIF(8%, 3yrs) =31.75


*112 + 31.75 = 143.75
9. Conceptual - Related Party Disclosures (NAS 24)

Golden Limited Notes to the Financial Statements


For the year ended December 31, 2009
Note: Related party Relationships
Platinum Limited is the parent company which holds majority shares of the company.
Note: Related party transactions
The transaction with related parties are carried out in the ordinary course of business at commercial
rates except stated otherwise.

Parent Major
Company Associates KMP Shareholders

Transactions: ‘000

Sales 18,000 - - -

Sales discount 1,500 - - -

Sale of property - 10,000 -

Reimbursement of expenses - property -


sale - 500

Interest free loans granted - - 2,000 -

Short term borrowings acquired - - - 25000

Interest on short term borrowings - - - 1500

Balances

Trade receivables 6500 5000 -

Loans to staff - - 1800 -

Loans payable - - - 25000

Interest payable on loan 12% - - - 1500

• Sales to related parties have been made at 20% mark up as against GL's policy to sell at a
markup of 30%.
• Administrative services are provided by the parent company free of cost as per the agreement.
Market value of these services is Rs. 350,000.
Paper1:AdvancedFinancial Reporting [RTP 2021 December]
Page 319 of 328

• In respect of sale of property, a buyer is required to bear all costs incurred on transfer. But in
this case the company has reimbursed the costs to SL

The interest free loan has been granted to the executive director as per the terms of employment.
10. Logical - Events After The Reporting Period (NAS 10)

As per the definition of ‘Events after the Reporting Period’ and paragraph 8 of
NAS 10, Events after the Reporting Period, financial statements should be adjusted
for events occurring after the reporting period that provide evidence of conditions that
existed at the end of the reporting period. In the instant case, the earthquake took place
in February 20X1 (i.e. before the end of the reporting period). Therefore, the condition
exists at the end of the reporting date though the debtor is declared insolvent after the
reporting period. Accordingly, full provision for bad debt amounting to ` 2 lakhs should
be made to cover the loss arising due to the bankruptcy of the debtor in the financial
statements for the year ended 31st March, 20X1. In this case, assuming that the
financial statements are approved by the approving authority after April, 20X1, XYZ Ltd
should provide for the remaining amount as a consequence of declaration of this debtor
as bankrupt.
In case, the earthquake had taken place after the end of the reporting period, i.e., after 31st March,
20X1, and XYZ Ltd. had not made any specific provision for the debtor who was declared bankrupt
later on, since the earthquake occurred after the end of the reporting period no condition existed at the
end of the reporting period. The company had made only general provision for bad debts in the
ordinary business course – without taking cognizance of the catastrophic situation of an earthquake.
Accordingly, bankruptcy of the debtor in this case is a non-adjusting event. As per para 21 of NAS
AS 10, if non-adjusting events after the reporting period are material, their non-disclosure could
influence the economic decisions that users make based on the financial statements. Accordingly, an
entity shall disclose the following for each material category of non-adjusting event after the reporting
period:
(a) the nature of the event; and
(b) an estimate of its financial effect, or a statement that such an estimate cannot be
made.”

If the amount of bad debt is considered to be material, the nature of this non -adjusting event, i.e.,
event of bankruptcy of the debtor should be disclosed along with the estimated financial effect of the
same in the financial statements.
11. Logical - Fair Value Measurements (NFRS 13)

The highest and best use of the land is determined by comparing the following:
• The value of the land as currently developed for industrial use (i.e., an assumption
that the land would be used in combination with other assets, such as the factory, or
with other assets and liabilities); and
• The value of the land as a vacant site for residential use, taking into account the costs of
demolishing the factory and other costs necessary to convert the land to a vacant site.
The value under this use would take into account risks and uncertainties about whether
the entity would be able to convert the asset to the alternative use (i.e., an assumption
that the land would be used by market participants on a stand -alone basis).

The highest and best use of the land would be determined on the basis of the higher of these
values. In situations involving real estate appraisal, the determination of highest and best use
Paper1:AdvancedFinancial Reporting [RTP 2021 December]
Page 320 of 328

might take into account factors relating to the factory operations (e.g., the factory‘s operating
cash flows) and its assets and liabilities (e.g., the factory‘s working capital).
12. Conceptual - Presentation Of Financial Statements

The demand feature might be primarily a form of protection or a tax -driven feature of the loan.
Both parties might expect and intend that the loan will remain outstanding for the foreseeable
future. If so, the instrument is, in substance, long-term in nature, and accordingly, OMN Ltd
would classify the loan as a non-current asset. However, OMN Ltd would classify the loan as a
current asset if both the parties intend that it will be repaid within 12 months of the reporting
period. MN Ltd would classify the loan as current because it does not have the right to defer
repayment for more than 12 months, regardless of the intentions of both the parties. The
classification of the instrument could affect initial recognition and subsequent measurement.

This might require the entity‘s management to exercise judgement, which could require disclosure
under judgements and estimates.

13. Numerical - Cancellation- Equity Settled Share based payment


Year 1 Year 2
Expected directors to vest 8 9
Fair value of option 130 130
Number of options 2,000 2,000
Total 20,80,000 23,40,000

Expense weightage 1/3 Full


Expense for the year 6,93,333 16,46,667*
*Remaining amount since cancelled

Cancellation compensation
Number of directors 9
Amount agreed to pay 95
Number of options/ director 2,000
Compensation amount (9 x 95 x 2,000) Also refer working notes 1 and 2 17,10,000

WN. 1 Amount to be deducted from Equity


Number of directors 9
Fair value of option (at the date of cancellation) 90
Number of options / director 2,000
Total 16,20,000

WN 2. Amount transferred to Profit and Loss


Total cancellation compensation 17,10,000
Less: To deduct from Equity (16,20,000)
Balance transferred to Profit and Loss 90,000

14. Conceptual - Property, Plant and Equipment


On 31st March, 2X11, the plant is 10 years old. Accumulated depreciation is Rs 30,000 (120,000
x 10 / 40 years). Due to unwinding of discount @ 5% over the 10 years, the amount of
decommissioning liability has increased from ` 10,000 to ` 16,300 (approx.). On 31st March,
Paper1:AdvancedFinancial Reporting [RTP 2021 December]
Page 321 of 328

2X11, the discount rate has not changed. However, the entity estimates that, as a result of
technological advances, the net present value of the decommissioning liability has decreased by
Rs 8,000. Accordingly, the entity adjusts the decommissioning liability from Rs 16,300 to Rs
8,300. On this date, the entity passes the following journal entry to reflect the change:

Provision for decommissioning liability Dr. 8,000


To Asset 8,000

Following this adjustment, the carrying amount of the asset is Rs. 82,000 (1,20,000 –8,000 –
30,000), which will be depreciated over the remaining 30 years of the asset’s life giving a
depreciation expense for the next year of Rs. 2,733 (82,000/30). The next year’s finance cost for
unwinding of discount will be 415 (8,300 × 5 per cent).

15. Numerical - Borrowing Costs (NAS 23)


Yes. If specific borrowings were not repaid once the relevant qualifying asset was completed,
they become general borrowings for as long as they are outstanding.

The borrowing costs that are directly attributable to obtaining qualifying assets are those
borrowing costs that would have been avoided if the expenditure on the qualifying asset had not
been made. If cash was not spent on other qualifying assets, it could be directed to repay this
specific loan. Thus, borrowing costs could be avoided (that is, they are directly attributable to
other qualifying assets).

• When general borrowings are used for qualifying assets, NAS 23 requires that, borrowing
costs eligible for capitalisation is calculated by applying a capitalisation rate to the
expenditures on qualifying assets.

• The amount of borrowing costs eligible for capitalisation is always limited to the amount of
actual borrowing costs incurred during the period.

16. Theory - Integrated Reporting


a. Various categories of capital are:
- Financial
- Manufactured
- Intellectual
- Human
- Social and Relationship
- Natural
Organizations preparing an integrated report are not required to adopt this categorization or to
structure their report along the above lines of the capitals.
b. The Framework is written primarily in the context of private sector, for-profit companies of
any size but it can also be applied, adapted as necessary, by public sector and not-for-profit
organizations.

c. An integrated report may be prepared in response to existing compliance requirements. For


example, an organization may be required by local law to prepare a management commentary
or other report that provides context for its financial statements. If that report is also prepared
in accordance with this Framework, it can be considered an integrated report.
If the report is required to include specified information beyond that required by this
Framework, the report can still be considered an integrated report if that other information
does not obscure the concise information required by this Framework.
Paper1:AdvancedFinancial Reporting [RTP 2021 December]
Page 322 of 328

17. Numerical - Employee Benefits (NAS 19)


All figures are Rs. in ’000.
On 31st March, 20X2, A Ltd. will report a net pension liability in the statement of financial position.
The amount of the liability will be 12,000 (68,000 – 56,000).
For the year ended 31st March, 20X2, A Ltd. will report the current service cost as an operating cost
in the statement of profit or loss. The amount reported will be 6,200. The same treatment applies to
the past service cost of 1,500.
For the year ended 31st March, 20X2, A Ltd. will report a finance cost in profit or loss based on the
net pension liability at the start of the year of 8,000 (60,000 – 52,000). The amount of the finance cost
will be 400 (8,000 x 5%).
The redundancy programme represents the partial settlement of the curtailment of a defined benefit
obligation. The gain on settlement of 500 (8,000 – 7,500) will be reported in the statement of profit or
loss.
Other movements in the net pension liability will be reported as remeasurement gains or losses in
other comprehensive income.
For the year ended 31st March, 20X2, the remeasurement loss will be 3,400 (Refer W. N.)
Working Notes
Remeasurement of gain or loss
Liability at the start of the year (60,000 – 52,000) 8,000
Current service cost 6,200
Past service cost 1,500
Net finance cost 400
Gain on settlement (500)
Contributions to plan (7,000)
Remeasurement loss (balancing figure) 3,400
Liability at the end of the year (68,000 – 56,000) 12,000

18. Numerical - Income Taxes (NAS 12)


Calculation of Deductible temporary differences:
Deferred tax asset = Rs. 80,000
Existing tax rate = 40%
Deductible temporary differences = 80,000/40% = Rs 2,00,000

Calculation of Taxable temporary differences:


Deferred tax liability = Rs. 60,000
Existing tax rate = 40%
Deductible temporary differences = 60,000 / 40% = Rs. 1,50,000
Paper1:AdvancedFinancial Reporting [RTP 2021 December]
Page 323 of 328

Of the total deferred tax asset balance of Rs 80,000, Rs 28,000 is recognized in OCI
Hence, Deferred tax asset balance of Profit & Loss is Rs 80,000 - Rs 28,000 = Rs 52,000

Deductible temporary difference recognized in Profit & Loss is Rs 1,30,000 (52,000 / 40%)
Deductible temporary difference recognized in OCI is Rs. 70,000 (28,000 / 40%)

The adjusted balances of the deferred tax accounts under the new tax rate are:
Previously credited to OCI-equity 70,000 x 0.45 31,500
Previously recognised as Income 1,30,000 x 0.45 58,500
90,000

Deferred tax liability


Previously recognized as expense 1,50,000 x 0.45 67,500

The net adjustment to deferred tax expense is a reduction of Rs. 2,500. Of this amount, Rs 3,500 is
recognised in OCI and Rs. 1,000 is charged to P&L.
The amounts are calculated as follows:

Carrying Increase
Carrying
amount (decrease)
amount
at 40% in deferred tax
at 45%
expense

Deferred tax assets


Previously credited to OCI-equity 31,500 28,000 (3,500)
Previously recognised as Income 58,500 52,000 (6,500)
90,000 80,000 (10,000)

Deferred tax liability


Previously recognized as expense 67,500 60,000 7,500
Net adjustment (2,500)

Journal Entries
Paper1:AdvancedFinancial Reporting [RTP 2021 December]
Page 324 of 328

Deferred tax asset Dr. 10,000


Deferred tax expense Dr. 1,000
To OCI –revaluation surplus 3,500
To Deferred tax liability 7,500
19. Numerical - Earning Per Share (NAS 33)

Subsidiary’s earnings per share

Basic EPS Rs5.00 calculated: Rs. 5,400 (a) – Rs. 400 (b)
1,000 (c)

Diluted EPS Rs 3.66 calculated: _______Rs 5,400 (d)______


(1,000 + 75 (e) + 400(f))

Notes:
(a) Subsidiary's profit attributable to ordinary equity holders.
(b) Dividends paid by subsidiary on convertible preference shares.
(c) Subsidiary's ordinary shares outstanding.
(d) Subsidiary's profit attributable to ordinary equity holders (Rs 5,000) increased by Rs 400
preference dividends for the purpose of calculating diluted earnings per share.
(e) Incremental shares from warrants, calculated: [(Rs20 – Rs 10) ÷ Rs 20] × 150.
(f) Subsidiary's ordinary shares assumed outstanding from conversion of convertible
preference shares, calculated: 400 convertible preference shares × conversion factor
of 1.

Consolidated earnings per share

Basic EPS Rs 1.63 calculated: Rs12,000(a) + Rs 4,300(b)


10,000(c)

Diluted EPS Rs. 1.61 calculated: Rs 12,000 + Rs 2,928(d) + Rs 55(e) +Rs.1098(f)


10,000

a) Parent's profit attributable to ordinary equity holders of the parent entity.


b) Portion of subsidiary's profit to be included in consolidated basic earnings per share,
calculated: (800 × Rs 5.00) + (300 × Re 1.00).
c) Parent's ordinary shares outstanding.
d) Parent's proportionate interest in subsidiary's earnings attributable to ordinary shares,
calculated: (800 ÷ 1,000) × (1,000 shares × Rs 3.66 per share).
e) Parent's proportionate interest in subsidiary's earnings attributable to warrants,
calculated: (30 ÷ 150) × (75 incremental shares × Rs 3.66 per share).
f) Parent's proportionate interest in subsidiary's earnings attributable to convertible
preference shares, calculated: (300 ÷ 400) × (400 shares from conversion × Rs 3.66 per
share).
Paper1:AdvancedFinancial Reporting [RTP 2021 December]
Page 325 of 328

20. Consolidated Financial Statements

(a)
L Group

Consolidated Statement of Financial Position as at Ashadh end 2078

Assets (Rs. in '000)

Non-Current assets

Property, plant and equipment [23,534 + 21,000 + 12,600 + 160 (W.N.1) -


400(W.N.1) + 300(W.N.1)] with FV adjustments 57,194

Goodwill (W.N.1) 2,995

60,189

Current assets [19,466 + 10,920 + 9,660] 40,046

Total Assets 100,235

Equity and Liabilities

Equity

Equity shares of Re. 0.50 each 9,660

Other components of equity (W.N.3) 7,556

Retained earnings (W.N.4) 6,569

Non – controlling interests (W.N.5) 5,270

Total Equity 29,055

Liabilities

Non- current liabilities (25,200 + 21,930 + 7,140) 54,270


Current liabilities (6,930 + 8,090 + 1,890) 16,910

Total Liabilities 71,180

Total Equity and Liabilities 100,235

Workings:
Rs. in '000 Rs. in '000

W.N. 1) Goodwill

D Company

Investment at cost 7,300

Fair value of Non - controlling interest (20% ×Rs. 8,980,000) 1,796

Fair value of net assets acquired:


Paper1:AdvancedFinancial Reporting [RTP 2021 December]
Page 326 of 328

Equity shares 7,980

Other components of Equity 630

Retained earnings 210

Fair value adjustment (Balancing figure) 160 8,980

Goodwill 116

Less: Impairment loss -

116

M Company

Investment at cost : Direct 15% 2,150


Indirect (80% X 8,600) 6,880

Non - controlling interest (37%×Rs. 9,764,000) 3,613

Fair value of net assets acquired:


Equity shares 7,560

Other components of Equity 1,890

Retained earnings 714

Fair value adjustment (Balancing figure) -400 9,764

Goodwill 2,879

Less: Impairment loss -

2,879
Total Net Goodwill (116+2,879) 2,995

Excess depreciation of fair value adjustment: (400 X 3/4yrs) 300

W.N. 2)

Calculation of Loss - Disposal of 100% in N Company

Rs. in '000

Consideration received 3,800

Less: FV of net assets at disposal -4,800

Goodwill (3,640 - 3,600) -40

Loss on disposal -1,040

W.N. 3)

Other components of equity

Rs. in '000
Paper1:AdvancedFinancial Reporting [RTP 2021 December]
Page 327 of 328

L Company 4,410

Share of post acquisition in:

D Company [80% X (3,570 - 630)] 2,352

M Company [63% X (3,150 - 1,890)] 794

7,556

W.N. 4)

Retained Earnings

Rs. in '000

L Company 6,250
Share of post-acquisition in:

D Company [80% X (-1,050 - 210)] -1,008

M Company [63% X (2,520 - 714 + 300(W.N.1)] 1,327


N Company [100% X (4,800 - 3,600)] 1,200

Reversal of profit – (W.N 1) FV adjustment -160

Loss on disposal of N company (W.N. 2) -1,040

6,569

W.N. 5)

Non - controlling interests

Rs. in '000 Rs. in '000


D Company

At acquisition 1,796

Less: Share of investment in M Company (20% X 8,600) -1,720

Share of post-acquisition:

Other component of equity (20% X 2,940 i.e. 3570-630) 588

Retained earnings (20% X - 1,260 i.e.-1050-210) -252 412

M Company

At acquisition 3,613

Share of post-acquisition:

Other component of equity (37% X 1,260 i.e. 3150-1890) 466

Retained earnings (37% X - 2,106 i.e 2,520-714+300 FV


adjustment) 779 4,858
Paper1:AdvancedFinancial Reporting [RTP 2021 December]
Page 328 of 328

Total 5,270

(b) Calculation of profit on disposal of 100% equity interest in N company.


Rs. in '000
Consideration received 3,800
Cost of investment sold 3,640
160
RTP CAP III June 2022 © The Institute of Chartered Accountants of Nepal

PAPER 1: Advanced Financial Reporting

3|Page
RTP CAP III June 2022 © The Institute of Chartered Accountants of Nepal

Revision Questions:

Conceptual Framework & NAS-1


1. An important aspect of the Accounting Standards Board’s Conceptual Framework for Financial Reporting is
that transactions should be recorded on the basis of faithful representation.
Required:
Explain why it is important that financial statements should show faithful representation, and describe
circumstances where the recognition criteria for assets and liabilities may differ from the passing of legal title
over items.

2. Damauli’s activities include the production of maturing products which take a long time before they are ready
to retail. Details of one such product are that on 1st Shrawan 2075 it had a cost of Rs 5 million and a fair value
of Rs 7 million. The product would not be ready for retail sale until 31st Ashad 2078.
On 1 Shrawan 2075 Damauli entered into an agreement to sell the product to Kalika finance for Rs 6 million.
The agreement gave Damauli the right to repurchase the product at any time up to 31 Ashad 2078 at a fixed
price of Rs 7,986,000, at which date Damauli expected the product to retail for Rs 10 million. The compound
interest Damauli would have to pay on a three-year loan of Rs 6 million would be:
Rs
Year 1 600,000
Year 2 660,000
Year 3 726,000
This interest is equivalent to the return required by kalaika finance.
Required:
Assuming the above figures prove to be accurate, prepare extracts from the statement of profit or loss of
Damauli for the three years to 31 Ashad 2078 in
respect of the above transaction:
(i) Reflecting the legal form of the transaction
(ii) Reflecting the faithful representation of the transaction.
Note: Statement of financial position extracts are NOT required.
(iii) Comment on the effect the two treatments have on the statement of profit or loss and the statements of
financial position and how this may affect an assessment of Damauli’s performance.

NFRS 16 – Property Plant and Equipment


3. Himalaya enters into a contract with an airport operator to use some space in the airport to sell its goods from
portable stall for a three year period. Himalaya owns the portable Stall. The contract stipulates the amount of
space and states that the space may be located at any one of several departure areas within the airport. The
airport operator can change the location of the space allocated to Himalaya at any time during the period of
use, and the costs that the airport operator would incur to do this would be minimal. There are many areas in
the airport that are suitable for the portable Stall. Does the contract contain a lease?

4. NAS 16 – Property, Plant and Equipment You are the auditor of a large manufacturing company called
Yoyo Limited, which has a December year-end. With the implementation of International Financial Reporting
Standards, the accountant of Yoyo Ltd feels that he has lost touch with the basic principles governing the
accounting treatment of certain elements. He has approached you regarding three issues that need clarification
before the current financial statements for the year ended 31 December 20X0 may be finalized.

Issue a) Internal manufacture of machine B


Included in non-current assets in Yoyo Limited’s statement of financial position at 31st December 20X0 is
‘Machine B’ at a carrying amount of C 400 000. Machine B was manufactured by Yoyo Limited during April
20X0. The machine was completed and available for use on 1May 20X0 and was brought into commercial
production on 31May 20X0. Included in the cost of C 400 000 were the following amounts:

• Raw materials of C 150 000 (including C 20,000 materials that were destroyed when a strike by the workers
ended in a warehouse being set alight).
4|Page
RTP CAP III June 2022 © The Institute of Chartered Accountants of Nepal

• Depreciation of other machinery used in the manufacture of Machine B: C 80 000.


• Labour costs of C 100 000.
• Administration overheads of C 70 000

Issue b) Acquisition of a crane


Yoyo Limited owns only one crane, which is included in property, plant and equipment at C 500 000 at 31st
December 20X0. This crane was acquired at 31 December 20X0 by exchanging the previously owned crane
(with a carrying amount of C 500 000) stationed at the Durban harbour mouth for the newly acquired crane
situated at the Richard’s Bay harbour mouth. The accountant is aware that the fair value of the newly acquired
crane is actually C 400 000 but believes that no adjustment is required since this is considered to be an
‘exchange of similar assets’. He added that although it would appear that the company made a loss in the
exchange of the two cranes, this is offset by the savings in not having to
physically move the Durban crane to the Richard’s Bay harbour where the crane would need to be stationed
from now on. Relocation costs were expected to be approximately C 100 000.

Issue c) Repainting of administration building


The administration building was repainted during the current year ended 31st December 20X0 at a cost of C
300 000. The cost of painting was capitalised to the building on the grounds that the cost of painting was
‘unavoidable’ since the directors believed that the building was ‘looking shabby’ and that this was
‘detrimentally affecting businesses.

Required:
Critically analyze each of the above issues, explaining whether the treatment is correct or incorrect and
justifying your advice with references to Nepal Financial Reporting Standards.
Your answer must be broken down into three separate discussions under the headings given.

NAS 2 – Inventories
5. On 31 Ashad 20X1, the inventory of ABC includes spare parts which it had been supplying to a number of
different customers for some years. The cost of the spare parts was Rs 10 million and based on retail prices
at 31 Ashad 20X1, the expected selling price of the spare parts is Rs 12 million. On 15 Shrawan 20X1, due
to market fluctuations, expected selling price of the spare parts in stock reduced to Rs 8 million. The estimated
selling expense required to make the sales would Rs 0.5 million. Financial statements were authorized by
Board of Directors on 20th Shrawan 20X1.

As at 31st Ashad 20X2, Directors noted that such inventory is still unsold and lying in the warehouse of the
company. Directors believe that inventory is in a saleable condition and active marketing would result in an
immediate sale. Since the market conditions have improved, estimated selling price of inventory is Rs 11
million and estimated selling expenses are same Rs 0.5 million.

What will be the value inventory at the following dates?


(a) 31st Ashad 20X1
(b) 31st Ashad 20X2

NFRS 5 – Non Current Assets Held for Sale and Discontinued Operation.
6. Riverside Resort Pvt Ltd (RRP) operates chain of luxury holiday villas in Nepal’s different location. The
directors have requested your advice on the following matter.
RRP’s policy is to carry the holiday villas at their re-valued amount, which, based on the most recent valuation
in 20X0, was Rs. 20m (historical cost was Rs. 10m). RRP is unsure how frequently a revaluation of such
properties is required and so has instructed a surveyor to provide an up-to-date valuation as at 31 December
Year 4. This valuation has provided the following information:
Rs ‘Million
Replacement Cost 17
Value in Use 28
Open Market Value 25
One of the villas has received very few bookings over the past two years and so a decision was reached to
exclude it from the Year 5 brochure. It is currently up for sale. The villa has a carrying value of Rs. 1.25m. Its

5|Page
RTP CAP III June 2022 © The Institute of Chartered Accountants of Nepal

value in use is only Rs.0.85m and its expected market value is Rs. 1m, before expected agents and solicitor’s
fees of Rs. 50,000. The directors are unsure as to the accounting treatment of this villa. A number of potential
buyers have expressed an interest in the property, and it is hoped that a deal will be negotiated in the first few
months of Year 5.
RRP’s accounting policy is to not charge depreciation on the villas. Its justification is that the villas are
maintained to a high standard and have useful lives of at least 50 years.
Required
Explain the accounting issues relating to the villas referring to relevant NFRS guidance. Where possible,
numerical information relating to the 31 December Year 4 financial statements should be provided.

NAS 38 – Intangible Assets


7. Deurali is a bio-technology company performing research for pharmaceutical companies. The finance director
has contacted your financial consulting company to arrange a meeting to discuss issues relevant to the
preparation of the financial statements for the year to 31 June 2016. Your initial telephone conversation has
provided the necessary background information. On 1st August 2015 Brooklyn began investigating a new
bio-process. On 1st September 2016, the new process was widely supported by the scientific community and
the feasibility project was approved. A grant was then obtained relating to future work. Several
pharmaceutical companies have expressed an interest in buying the ‘know how’ when the project completes
in June 2017. The nominal ledger account set up for the project shows that the expenditure incurred between
1st August 2015 and 30th June 2016 was Rs. 300,000 per month.
Required
Prepare notes for your meeting with the finance director which explain and justify the accounting treatment of
the issue.

NAS 36 –Impairment of assets


8. The accountant of Orfeo Co is preparing financial statements for the year ended 31 December 20X7.
Before these can be completed a number of issues need to be resolved.
(i) Orfeo Co’s head office building was revalued on 1 July 20X7, giving rise to a surplus of $100,000.
The building had an original cost of $1m on 1 January 20X0 and a 50-year life at that date. The useful life of
the building remains unchanged.
(ii) During the year one of Orfeo Co’s machines broke down and could not be fixed. The carrying amount of
the machine at that date was $30,000. The accountant must now consider the issue of impairment.

Because of the loss of production caused by the damaged machine, Orfeo Co lost customers and it was decided
that the whole factory unit was impaired by $120,000. Orfeo Co’s accountant has to decide how to allocate
this impairment loss.
The carrying amounts of the assets of the factory unit at the date of the impairment review, including the
damaged machine, were:
$
Goodwill 20,000
Factory building 440,000
Plant and machinery 160,000
Net current assets 100,000
720,000
What will be the carrying amount of plant and machinery when the impairment loss has been allocated?
What amount should be charged as depreciation on the building in (i) for the year ended 31 December 20X7?

NFRS 13 – Fair value measurement


9. Mehran, a public limited company, has just acquired a company, which comprises a farming and mining
business. Mehran wishes advice on how to fair value some of the assets acquired. One such asset is a piece
of land, which is currently used for farming. The fair value of the land if used for farming is $5 million. If the
land is used for farming purposes, a tax credit currently arises annually, which is based upon the lower of
15% of the fair market value of land or $500,000 at the current tax rate. The current tax rate in the jurisdiction
is 20%. Mehran has determined that market participants would consider that the land could have an alternative
use for residential purposes. The fair value of the land for residential purposes before associated costs is
thought to be $7.4 million. In order to transform the land from farming to residential use, there would be legal

6|Page
RTP CAP III June 2022 © The Institute of Chartered Accountants of Nepal

costs of $200,000, a viability analysis cost of $300,000 and costs of demolition of the farm buildings of
$100,000. Additionally, permission for residential use has not been formally given by the legal authority and
because of this, market participants have indicated that the fair value of the land, after the above costs, would
be discounted by 20% because of the risk of not obtaining planning permission.
Required: Discuss the way in which Mehran should fair value the above assets with reference to the
principles of NFRS 13 Fair Value Measurement

NAS 8 - Accounting Policies, Changes in Accounting Estimates & Error


10. During 20X2, Beta Ltd. discovered that some products that had been sold during 20X1 were incorrectly
included in inventory at March 31, 20X1 at Rs 6,500.
(1) Beta’s accounting records for 20X2 show sales of Rs 1, 04,000, cost of goods sold of Rs 86,500 (including
Rs 6,500 for the error in opening inventory), and income taxes of Rs 5,250.
(2) In 20X1, Beta Ltd. reported:
• Sales of Rs 73,500
• Cost of goods sold of Rs 53,500
• Profit before income taxes of Rs 20,000
• Income taxes of Rs 6,000
• Profit of Rs 14,000
(3) 20X1 opening retained earnings was Rs 20,000 and closing retained earnings was Rs 34,000.
(4) Beta’s income tax rate was 30 per cent for 20X2 and 20X1. It had no other income or expenses.
(5) Beta Ltd. had Rs 5,000 of share capital throughout, and no other components of equity except for retained
earnings. Its shares are not publicly traded and it does not disclose earnings per share.
Required
You are required to prepare relevant extract from the statement of profit and loss and statement of changes in
equity. Also, what should be disclosed in the notes?

NAS 19 –- Employee Benefits


11. At 1 January 20X2 the fair value of the assets of a defined benefit plan were valued at $1,100,000 and the
present value of the defined benefit obligation was $1,250,000. On 31 December 20X2, the plan received
contributions from the employer of $490,000 and paid out benefits of $190,000. The current service cost for
the year was $360,000 and a discount rate of 6% is to be applied to the net liability/(asset). After these
transactions, the fair value of the plan's assets at 31 December 20X2 was $1.5m. The present value of the
defined benefit obligation was $1,553,600.
Required
a. Calculate the gains or losses on re-measurement through OCI and the return on plan assets and illustrate
b. how this pension plan will be treated in the statement of profit or loss and other comprehensive income and
statement of financial position for the year ended 31 December 20X2.

NFRS 2 – Share based payment


12. You are the accountant of Alpha Co. located in Kathmandu Your assistant is responsible for preparing the
financial statements for your review. He is in the process of preparing the first draft of the financial statements
for the year ended 31 Ashad 20X5. While he has a sound basic knowledge of accounting he is unsure about
certain aspects of financial reporting. He has asked you the following question regarding financial reporting.
'As you know on 1 Shrawan 20X4 our company granted 50 executives call options to purchase up to 5,000
shares each on 1 Shrawan 20X6. This was partly a means of deterring them from leaving as the options only
vest if the executives are still employed on 1 Shrawan 20X6. Personnel tell me that they estimate 90% of the
executives will remain with us for the two year period and exercise their options in full. I am very confused by
some of the financial information I have been given regarding these options:
(i) The option price is $20 per share.
(ii) The market value of each share was $15 on 1 Shrawan 20X4 and $18 on 31 Shrawan 20X5. It is $19 today.
(iii) The market value of the share option was $2 on 1 Shrawan 20X4 and $2.20 on 31 Shrawan 20X5. It is
$2.25 today.
I have made no accounting entries in the draft financial statements regarding these options. I base this on the
fact that the option price is greater than the current market value so is unlikely to be exercised. Is my thinking
correct on this?'

7|Page
RTP CAP III June 2022 © The Institute of Chartered Accountants of Nepal

Required
Explain with reasons whether the proposed treatment is correct or not set out correct treatment if appropriate.

NAS 37 – Provisions, Contingent Liabilities and Contingent Assets


13. Discuss whether the following provisions have been accounted for correctly under NAS 37 Provisions,
contingent liabilities and contingent assets. WorldWide Nuclear Fuels, a public limited company, disclosed
the following information in its financial statements for the year ending 30 November 20X9.
Provisions and long-term commitments
Provision for decommissioning the group's radioactive facilities is made over their useful life and covers
complete demolition of the facility within 50 years of it being taken out of service together with any associated
waste disposal. The provision is based on future prices and is discounted using a current market rate of interest.
Provision for decommissioning costs
Rs m
Balance at 1 December 20X8 675
Adjustment arising from change in price levels charged to reserves 33
Charged in the year to profit or loss 125
Adjustment due to change in knowledge (charged to reserves) 27
Balance at 30 November 20X9 860
There are still decommissioning costs of Rs 1,231m (undiscounted) to be provided for in respect of the group's
radioactive facilities as the company's policy is to build up the required provision over the life of the facility.

NAS 33 – Earning Per Share


14. Calculate the diluted EPS according to NAS 33 Earnings per share from the following information.
X, a public limited company – accounting data year ended 31 May 20X7
Net profit after tax and non-controlling interest Rs 18,160,000
Ordinary shares of Rs 1 (fully paid) Rs 40,000,000
Average fair value for year of ordinary shares Rs 1.50
(i) Share options have been granted to directors giving them the right to subscribe for ordinary shares
between 20X8 and 20Y0 at Rs 1.20 per share. 2,000,000 options were outstanding at 31 May 20X7.
(ii) The company has Rs 20m of 6% convertible bonds in issue. The terms of conversion of the bonds per
Rs 200 nominal value of bond at the date of issue (1 May 20X6) were:
Number of
Conversion date shares
31 May 20X7 24
31 May 20X8 23
31 May 20Y0 22
None of the bonds have as yet been converted. The bonds have been issued at a discount of 1% and
the company has written off the discount against the statement of profit or loss.
(iii) There are 1,600,000 convertible preferred shares in issue. The cumulative dividend is 10 paisa per
share and each preferred share can convert into two ordinary shares. The preferred shares can be
converted in 20X9.
(iv) Assume a tax rate of 33%.

NFRS 8 – Operating Segments


15. Prepare a segmental report for publication in B Ltd. from the following details of the company‘s three
divisions and the head office:
Rs.(‟000)
Forging Shop Division
Sales to Bright Bar Division 4,575
Other Domestic Sales 90
Export Sales 6,135
10,800
Bright Bar Division
Sales to Fitting Division 45
Export Sales to Ruwanda 300
345

8|Page
RTP CAP III June 2022 © The Institute of Chartered Accountants of Nepal

Fitting Division
Export Sales to India 270

Particulars Head Office Forging Shop Bright Bar Fitting


Division Division Division Division
Rs. (‗000) Rs. (‗000) Rs. (‗000) Rs. (‗000)
Pre-tax operating result - 240 30 (12)
Head office cost reallocated - 72 36 36
Interest costs - 6 8 2
Fixed assets 75 300 60 180
Net current assets 72 180 60 135
Long-term liabilities 57 30 15 180

NAS 41 – Agriculture
16. Moon Ltd prepares financial statements to 31 March each year. On 1 April 20X1 the company carried out the
following transactions:
— Purchased a land for Rs 50 Lakhs.
— Purchased 200 dairy cows (average age at 1 April 20X1 two years) for Rs 10 Lakhs.
— Received a grant of Rs 1 million towards the acquisition of the cows. This grant was nonrefundable.

For the year ending 31 March 20X2, the company has incurred following costs:
— Rs 6 Lakh to maintain the condition of the animals (food and protection).
— Rs 4 Lakh as breeding fee to a local farmer.
On 1 October 20X1, 100 calves were born. There were no other changes in the number of animals during the
year ended 31 March 20X2. As of 31 March 20X2, Moon Ltd had 3,000 litres of unsold milk in inventory. The
milk was sold shortly after the year end at market prices.
Information regarding fair values are as follows:
Item Fair Value less cost to sell
1 April 20X1 1 October 20X1 31 March 20X2
Rs Rs Rs
Land 50 Lakhs 60 Lakhs 70 lakhs
New born calves (per 1,000 1,100 1,200
calf)
Six Month Old calves 1,100 1,200 1,300
(per calf)
Two years old cows 5,000 5,100 5,200
(per cow)
Three years old cows 5,200 5,300 5,500
(per cow)
Milk (per litre) 20 22 24

Prepare the extract from the balance sheet and statement of profit & loss that would be reflected in
the financial statement of the entity for the year ended 31st march 20X2.

NAS 12 – Income Taxes


17. Your client is a full tax free enterprise for the first ten years and is in the second year of operation. Depreciation
temporary differences resulting in deferred tax liability in year 1 & 2 are Rs. 100 million and Rs. 200 million
respectively. From the third year and onwards, it is expected that the temporary difference would reverse each
year by Rs. 5 million. Determine deferred tax liability at the end of second year and the charge to the P & L
account, if any. Assume tax rate @ 25%.

9|Page
RTP CAP III June 2022 © The Institute of Chartered Accountants of Nepal

NAS 24 – Related Party Disclosure


18.
(a) Power Limited is a producer of electricity. Transmission Limited regularly purchases electricity from
Power Limited. Power Limited whose financial year ends on March 31, 20X2, acquired 100%
shareholding of Transmission Limited on July 15, 20X1. However, the entire shareholding is disposed of
on March 21, 20X2. Power Limited and Transmission Limited had transactions when Transmission
Limited was a subsidiary of Power Limited and also in the period when it was not a subsidiary of Power
Limited.
Required
For which period, related party disclosure should Power Limited make in its financial statements for the
year ended March 31, 20X2 with respect to transactions with Transmission Limited.

(b) Government G directly controls Entity 1 and Entity 2. It indirectly controls Entity A and Entity B through
Entity 1, and Entity C and Entity D through Entity 2. Person X is a member of the key management
personnel in Entity 1.
Required
Examine the entity to whom the exemption for disclosure to be given and for transaction with whom.

Human Resource Accounting


19. Elaborate the relevance and purpose of Human Capital Reporting, and from the following detail, compute the
total value of human resources of the employee groups according to the Lav and Schwartz Model:
Particulars Skilled Employees Unskilled employees
Annual Average earning of an employee till the Rs.50,000 Rs.30,000
retirement age
Age of retirement 65 Years 62 Years
Discount Rate 15% 15%
No of Employees 20 25
Average Age 62 years 60 years

Value Added Statement


20. Describe the necessity/advantage of Value Added Statement.

NFRS 15 – Revenue from Contracts with Customers


21. On 1 December 20X1, Bristow provides a service to a customer for the next 12 months. The consideration is
$12 million. Bristow is entitled to an extra $3 million if, after twelve months, the number of mistakes made
falls below a certain threshold.
Required:
Discuss the accounting treatment of the above in Bristow’s financial statements for the year ended 31
December 20X1 if:
(a) Bristow has experience of providing identical services in the past and it is highly probable that the number
of mistakes made will fall below the acceptable threshold.
(b) Bristow has no experience of providing this service and is unsure if the number of mistakes made will fall
below the threshold.

NFRS – 9 Financial Instrument


22.
(a) On 1 December 20X4, Ambush sold goods on credit to Tray for Rs 600,000. Tray has a credit limit with
Ambush of 60 days. Ambush applies NFRS 9 Financial instruments , and uses a pre-determined matrix for the
calculation of allowances for receivables as follows.
Days overdue Expected loss
provision
Nil 1%
1 to 30 5%
31 to 60 15%
61 to 90 20%

10 | P a g e
RTP CAP III June 2022 © The Institute of Chartered Accountants of Nepal

90 + 25%
Tray had not paid by 31 January 20X5, and so failed to comply with its credit term, and Ambush learned that
Tray was having serious cash flow difficulties due to a loss of a key customer. The finance controller of Tray
has informed Ambush that they will receive payment.
Ignore sales tax.
Required
Show the accounting entries on 1 December 20X4 and 31 January 20X5 to record the above, in accordance
with the expected credit loss model in NFRS 9.

(b) Aarti, a public limited company, produces artefacts made from precious metals. (i) Aarti has entered into
three derivative contracts during the year ended 30 November 20X4, details of which are as follows:
Initial Fair Value Reason
Recognition at at the Year
Fair Value End
A Nil $20m Aarti believes that oil prices are due to rise in the future so
(liability) during the year has entered into oil futures contract to buy oil at
a fixed price. Aarti has no exposure to oil prices in the course
of its business. In fact, oil prices have fallen resulting in the loss
at the year-end.
B $1 m $9m Aarti has an investment in equity designated to be measured at
(liability) fair value through other comprehensive income. Aarti is
concerned the investment will fall in value and it wishes to
cover this risk. Thus during the year it has entered into
derivative B to cover any fall in value and designated this as a
hedging instrument as part of a fair value hedge. In fact, the
asset has risen in value by $8.5 million.
C Nil $ 25 m Aarti is concerned about the potential for raw material prices to
(Asset) rise. It wishes to cover this risk that future costs will rise over
the next two to three years. Thus it has entered into derivative
C – a futures contract. This arrangement has been designated as
a cash flow hedge. At the year-end the raw material prices have
risen, potentially giving the company an increased future cost
of $24 million.
All designated hedges meet the effectiveness criteria outlined in NFRS 9 Financial Instruments.
Outline the hedge effectiveness criteria according to NFRS 9 Financial Instruments and discuss how each of
the three derivatives would be accounted for in the financial statements for the year ended 30 November
20X4.

Consolidation of Financial Statement/Accounting for Group Companies


23. he following draft statements of financial position relate to Glove, Body and Fit, all public limited
companies, as at 31 May 20X7.
Glove Body Fit
Rs (m) Rs (m) Rs (m)
Assets
Non-Current Assets
Property Plant & Equipment 260 20 26
Investment in Body 60
Investment in Fit 30
Available for sale investments 10
Current Assets 65 29 20
Total Assets 395 79 46
Ordinary Shares 150 40 20
Other reserves 30 5 8
Retained Earnings 135 25 10
Total Equity 315 70 38

11 | P a g e
RTP CAP III June 2022 © The Institute of Chartered Accountants of Nepal

Non-Current Liabilities 45 2 3
Current Liabilities 35 7 5
Total Liabilities 80 9 8

Total Equity & Liabilities 395 79 46

The following information is relevant to the preparation of the group financial statements.
(a) Glove acquired 80% of the ordinary shares of Body on 1 June 20X5 when Body's other reserves were Rs
4 million and retained earnings were Rs 10 million. The fair value of the net assets of Body was Rs 60
million at 1 June 20X5. Body acquired 70% of the ordinary shares of Fit on 1 June 20X5 when the other
reserves of Fit were Rs 8 million and retained earnings were Rs 6 million. The fair value of the net assets
of Fit at that date was Rs 39 million. The excess of the fair value over the net assets of Body and Fit is due
to an increase in the value of non-depreciable land of the companies. There have been no issues of ordinary
shares in the group since 1 June 20X5.
(b) Body owns several trade names which are highly regarded in the market place. Body has invested a
significant amount in marketing these trade names and has expensed the costs. None of the trade names
has been acquired externally and, therefore, the costs have not been capitalised in the statement of financial
position of Body. On the acquisition of Body by Glove, a firm of valuation experts valued the trade names
at Rs 5 million and this valuation had been taken into account by Glove when offering Rs 60 million for
the investment in Body. The valuation of the trade names is not included in the fair value of the net assets
of Body above. Group policy is to amortise intangible assets over ten years.
(c) On 1 June 20X5, Glove introduced a new defined benefit retirement plan. At 1 June 20X5, there were no
unrecognized actuarial gains and losses. The following information relates to the retirement plan.
31 May 20X6 31 May 20X7
Rs m Rs m
Un-recognized actuarial losses to date 3 5
Present value of obligation 20 26
Fair value of plan assets 16 20
The expected average remaining working lives of the employees in the plan is ten years at 31 May 20X6 and
31 May 20X7. Glove wishes to defer actuarial gains and losses by using the 'corridor' approach. The defined
benefit liability is included in non-current liabilities.
(d) Glove has issued 30,000 convertible bonds with a three year term repayable at par. The bonds were issued
at par with a face value of Rs 1,000 per bond. Interest is payable annually in arrears at a nominal interest
rate of 6%. Each bond can be converted at any time up to maturity into 300 shares of Glove. The bonds
were issued on 1 June 20X6 when the market interest rate for similar debt without the conversion option
was 8% per annum. Glove does not wish to account for the bonds at fair value through profit or loss. The
interest has been paid and accounted for in the financial statements. The bonds have been included in non-
current liabilities at their face value of Rs 30 million and no bonds were converted in the current financial
year.
(e) On 31 May 20X7, Glove acquired plant with a fair value of Rs 6 million. In exchange for the plant, the
supplier received land, which was currently not in use, from Glove. The land had a carrying value of Rs 4
million and an open market value of Rs 7 million. In the financial statements at 31 May 20X7, Glove had
made a transfer of Rs 4 million from land to plant in respect of this transaction.
(f) Goodwill has been tested for impairment at 31 May 20X6 and 31 May 20X7 and no impairment loss
occurred.
(g) It is the group's policy to value the non-controlling interest at acquisition at its proportionate share of the
fair value of the subsidiary's identifiable net assets.
(h) Ignore any taxation effects.
Required
Prepare the consolidated statement of financial position of the Glove Group at 31 May 20X7 in accordance
with Nepal Financial Reporting Standards (NFRS).

12 | P a g e
RTP CAP III June 2022 © The Institute of Chartered Accountants of Nepal

Suggested Answers/Hints:
1.
For financial statements to achieve a faithful representation, information should be complete, neutral and free
from bias. A key component for recognizing an asset is control, whether an asset is a resource controlled by
an entity. For an asset to control a resource, it is often considered that they are exposed to the majority of the
risks and rewards associated with that asset.
Examples where the risks and rewards may differ from the passing of legal title:
• Finance lease agreements:
Under a finance lease, legal title may not pass to the lessee, but the risks and rewards associated with the asset
pass to them. Therefore the asset is controlled by the lessee and should be recorded in their books, along with
the associated liability.
• Sale and repurchase agreements:
These often mean that an entity ‘sells’ an asset to a third party, but continued to enjoy the future benefits
embodied in that asset. Under faithful representation this transaction would not be represented faithfully by
recording it as a sale (in all probability this would be a financing transaction, as control over the asset is lost).
• Consignment inventory:
This is where goods are supplied (usually by a manufacturer) to a retailer under terms which mean the legal
title to the goods remains with the supplier until a specified event (say payment in three months). If the retailer
bears the risk of the item not selling, then it is likely that the goods should be treated as their inventory and
should therefore be derecognized in the books of the manufacturer.
• Sale and finance leaseback:
Under a finance leaseback, the risks and rewards associated with the asset would remain with the selling
company. Therefore the asset would be brought back under a finance lease, with the corresponding liability.

2.
Extracts from the statement of profit or loss
(i) Reflecting the legal form:
31 Ashad 31 Ashad 31st Ashad
Year ended: 2076 2077 2078 Total
Rs 000 Rs 000 Rs 000 Rs 000
Revenue 6,000 nil 10,000 16,000
Cost of sales (5,000) nil (7,986) (12,986)
–––––– –––––– –––––– ––––––
Gross profit 1,000 nil 2,014 3,014
Finance costs nil nil nil nil
–––––– –––––– –––––– ––––––
Net profit 1,000 nil 2,014 3,014
–––––– –––––– –––––– ––––––

(ii) Reflecting faithful representation:


31 Ashad 31 Ashad 31 Ashad
Year ended: 2076 2077 2078 Total

Revenue nil nil 10,000 10,000


Cost of sales (nil) nil (5,000) (5,000)
–––––– –––––– –––––– ––––––
Gross profit nil nil 5,000 5,000
Finance costs (600) (660) (726) (1,986)
–––––– –––––– –––––– ––––––
Net profit (600) (660) 4,274 3,014
–––––– –––––– –––––– ––––––

(iii) It can be seen from the above that the two treatments have no effect on the total net profit reported in the
statement of comprehensive incomes, however, the profit is reported in different periods and the classification
of costs is different. In effect the legal form creates some element of profit smoothing and completely hides
13 | P a g e
RTP CAP III June 2022 © The Institute of Chartered Accountants of Nepal

the financing cost. Although not shown, the effect on the statements of financial position
is that recording the legal form of the transaction does not show the inventory, nor does it show the in-substance
loan. Thus recording the legal form would be an example of off balance sheet (statement of financial position)
financing. The effect on an assessment of Damauli using ratio analysis may be that recording the legal form
rather than the substance of the transaction would be that interest cover and
inventory turnover would be higher and gearing lower. All of which may be considered as reporting a more
favorable performance.

3.
The contract does not contain a lease because there is no identified asset. The contract is for space in the airport,
and the airport operator has the practical right to substitute this during the period of use because:
• There are many areas available in the airport that would meet the contract terms, providing the operator with
a practical ability to substitute
• The airport operator would benefit economically from substituting the space because there would be minimal
cost associated with it. This would allow the operator to make the most effective use of its available space,
thus maximizing profits.

4.
Issue a) Internal manufacture of Machine B
All costs necessarily incurred in bringing the asset ‘to a location and condition for it to be capable of operating
in a manner intended by management’ should be capitalised. This means that the following costs may not be
capitalised (all other costs were correctly capitalised):
• The C 20 000 materials destroyed – all unnecessary wastage is expressly not allowed to be capitalised per
NAS 16
• Administration overheads of C70 000 – unless it can be proved that these costs were directly linked to
the manufacture of the machine.
The capitalisation of the other costs is correct on the grounds that:
• Raw materials, labour and the depreciation of other machinery used in the manufacture of machine B
would be classified as costs necessarily incurred in bringing the asset to a location and condition enabling
it to be used.

Property, plant and equipment must be depreciated from the date on which it first becomes available for use
and thus this machine should be depreciated from 1 May 20X0. This has obviously not been done since the
carrying amount at year-end equals the cost of manufacture.

Issue b) Acquisition of a crane


A newly acquired asset should be brought into the accounting records at the fair value of the asset given away
adjusted for cash and cash equivalents, where appropriate. Where this fair value is not available, or the fair
value of the acquired asset is considered to be more clearly evident, the fair value of the newly acquired asset
should be used instead (in this case, C400 000). The only time that the carrying amount of the asset given away
may be used is if neither of the fair values was available or there was no economic substance to the transaction
(i.e. the assets are very similar).

If the difference between the carrying amount (C500 000) and the available fair value (C400 000) is considered
to be material, this will mean than either:
• the carrying amount of the crane that was given away should first be reduced to C400 000, being its true
fair value (i.e. there is an impairment loss), or
• the carrying amount of the crane that was given away was a true reflection of its fair value (C500 000) and
therefore a genuine loss on exchange of C100 000 (C500 000 – C400 000) was incurred. The new crane
should then be recognized at C400 000, this fair value being considered to be more clearly evident.

Although costs necessarily incurred in bringing the asset to a location/ condition that enables it to be used must
be capitalised, costs that are avoided may not be capitalised since they have not been incurred. The relocation
costs that were avoided should, therefore, not be considered in the debate regarding the measurement of newly
acquired crane.
14 | P a g e
RTP CAP III June 2022 © The Institute of Chartered Accountants of Nepal

If it is decided that the C500 000 carrying amount of the crane that was given away should be reduced to C400
000 (being a better reflection of its fair value) this reduction may be effected either as a change in depreciation
(if it is believed that the useful life was shorter than originally estimated) or as an impairment loss (if it is found
that there was an unexpected drop in the asset’s recoverable amount). The respective journal entries would be
as follows:
Debit Credit
a change in estimated depreciation:
Depreciation 100 000
Accumulated depreciation 100 000
or
an impairment loss:
Impairment loss 100 000
Accumulated depreciation & impairment loss 100 000

Issue c): Repainting of the administration building.


The cost of painting would either be considered to be:
• subsequent expenditure: a replacement or renewal of a component; or
• Subsequent expenditure: other.
Paint would only be considered to be a separate component of the building if it is material and has a different
useful life or a different pattern of economic benefits. If it were a separate component, then it would also need
to meet the recognition criteria. It is submitted that since there is no definite cause and effect between the cost
of the paint and the potential for future economic benefits to flow, the recognition criteria would not be met.
If it were to be argued that the paint was ‘subsequent expenditure: other’, the cost of the paint would only be
capitalised if ‘the standard of performance assessed immediately before the expenditure was made’ is
improved. It is submitted that the extra paint maintains the expected level of future economic benefits (thus
avoiding an impairment rather than improving the asset) and should therefore be classified as ‘maintenance
costs’ and be expensed.

5.
As per NAS 2 ‘Inventories’, inventory is measured at lower of ‘cost’ or ‘net realisable value’. Further, as per
NAS 10: ‘Events after Balance Sheet Date’, decline in net realisable value below cost provides additional
evidence of events occurring at the balance sheet date and hence shall be considered as ‘adjusting events’.
(a) In the given case, for valuation of inventory as on 31 Ashad 20X1, cost of inventory would be Rs 10 million
and net realisable value would be Rs 7.5 million (i.e. Expected selling price Rs 8 million- estimated selling
expenses Rs 0.5 million). Accordingly, inventory shall be measured at Rs 7.5 million i.e. lower of cost and net
realisable value. Therefore, inventory write down of Rs 2.5 million would be recorded in income statement of
that year.

(b) As per para 33 of NAS 2, a new assessment is made of net realizable value in each subsequent period. It
Inter alia states that if there is increase in net realizable value because of changed economic circumstances, the
amount of write down is reversed so that new carrying amount is the lower of the cost and the revised net
realizable value. Accordingly, as at 31 Ashad 20X2, again inventory would be valued at cost or net realisable
value whichever is lower. In the present case, cost is Rs 1 million and net realisable value would be Rs 10. 5
million (i.e. expected selling price Rs 11 million – estimated selling expense Rs 0.5 million). Accordingly,
inventory would be recorded at Rs 10 million and inventory write down carried out in previous year for Rs 2.5
million shall be reversed.

6.
NAS 16 allows property, plant and equipment to be re-valued or left at historical cost. Revaluation should be
based on the fair value . Revaluation is not required every year, but must be conducted when it is believed that
the fair value differs materially from the carrying value.
The method of accounting for the villa that is to be sold is covered by NFRS 5 which requires that where, at
the end of a reporting period, an asset is held for sale it should be reclassified, re-measured and no longer
depreciated. An asset is only classified as held for sale where the following conditions are all met:
(i) The asset is available for sale in its present condition.
(ii) The sale is believed to be highly probable:
15 | P a g e
RTP CAP III June 2022 © The Institute of Chartered Accountants of Nepal

• Appropriate level of management is committed to the sale;


• There is an active programme underway to find a buyer;
• The asset is marketed at a realistic price.
• Completion of sale expected within 12 months of classification.

From the limited information provided it appears that these conditions have been met and therefore, under the
rules of NFRS 5, the villa should be re-measured to the lower of its carrying value and its fair value minus
costs to sell.

Therefore, the villas should be valued at 31 December Year 4 as follows:


Fair value Carrying value
Rs. Rs.
All villas 25.00 20.00
Property held for sale (1.00) (1.25)
Properties to be retained 24.00 18.75
Note:
(i) The villas to be retained should be re-valued to Rs. 24m, resulting in an increase in the revaluation
reserve of Rs. 5.25m (24-18.75).

(ii) The villa to be sold should be written down from its carrying value to its fair value minus costs to
sell of Rs.0.95m (Rs. 1m – 50,000). This impairment of Rs. 300,000 (1.25m – 0.95m) will be charged
against the revaluation reserve for this asset. If there is insufficient revaluation reserve, then the write
down must be charged to profit or loss.

(iii) The villa held for sale must be re-classified from ‘Non-current assets’ to ‘Current assets’ as a
separate line item.

Depreciation should not be charged when an asset has been classified as held for sale. However, the
other villas should be depreciated. NAS 16 states that expenditure on repairs and maintenance does not
remove the need to depreciate an asset. The villas have a finite useful life and therefore must be
depreciated. If the residual value of these assets is greater than the carrying value then the depreciation
charge will be zero. It is not acceptable therefore to have a policy of non-depreciation on such assets,
and a prior year adjustment should be made to correct the error if the error is material.

7.
NAS 38 on intangible assets requires that research and development be considered separately:
• research – which must be expensed as incurred
• development – which must be capitalized where certain criteria are met.
It must first be clarified how much of the Rs. 3 million incurred to date (10 months at Rs. 300,000) is simply
research and how much is development. The development element will only be capitalized where the NAS 38
criteria are met. The criteria are listed below together with the extent to which they appear to be met.
• The project must be believed to be technically feasible. This appears to be so as the feasibility has
been acknowledged.
• There must be an intention to complete and use/sell the intangible. Completion is scheduled for June
2017
• The entity must be able to use or sell the intangible. Interest has been expressed in purchasing the
knowhow on completion
• It must be considered that the asset will generate probable future benefits. Confirmation is required
from Brooklyn as to the extent of interest shown by the pharmaceutical companies and whether this is
of a sufficient level to generate orders and to cover the deferred costs.
• Availability of adequate financial and technical resources must exist to complete the project. The
financial position of Brooklyn must be investigated. A grant is being obtained to fund further work
and the terms of the grant, together with any conditions, must be discussed further.
• Able to identify and measure the expenditure incurred. A separate nominal ledger account has been
set up to track the expenditure.
If all of the above criteria are met, then the development element of the Rs. 3m incurred to date must be

16 | P a g e
RTP CAP III June 2022 © The Institute of Chartered Accountants of Nepal

capitalised as an intangible asset. Amortisation will not begin until commercial production commences.
8.
$ $
Original cost 1.1.X0 1,000,000
Depreciation to 31.12.X6 ( 1,000,000 X 7/50) (140,000)
860,000
Depreciation to 30.6.X7 ((1,000,000/50) X 6/12) (10,000) 10,000
850,000
Revaluation surplus 100,000
950,000
Depreciation to 31.12.X7 (950,000 X 0.5/42.5) (11,176) 11,176
Total depreciation year to 31.12.X7 21,176

Original amount Impairment Post-impairment


Goodwill 20,000 (20,000) –
Building 440,000 (54,000) 386,000
Plant and machinery 160,000 (30,000 + 16,000) 114,000
Net current assets 100,000 –
720,000 120,000

9.
NFRS 13 Fair Value Measurement requires the fair value of a non-financial asset to be measured based on its
highest and best use. This is determined from the perspective of market participants. It does not matter whether
the entity intends to use the asset differently. The highest and best use takes into account the use of the asset
which is physically possible, legally permissible and financially feasible. NFRS 13 allows management to
presume that the current use of an asset is the highest and best use unless market or other factors suggest
otherwise.
If the land zoned for agricultural use is currently used for farming, the fair value should reflect the cost structure
to continue operating the land for farming, including any tax credits which could be realised by market
participants. Thus the fair value of the land if used for farming would be $5.1 million ($5m + (20% × $0.5m)).
The agricultural land appears to have an alternative use as market participants have considered its alternative
use for residential purposes. A use of an asset need not be legal at the measurement date, but it must not be
legally prohibited in the jurisdiction. If used for residential purposes, the value should include all costs
associated with changing the land to the market participant’s intended use. In addition, demolition and other
costs associated with preparing the land for a different use should be included in the valuation. These costs
would include the uncertainty related to whether the approval needed for changing the usage would be
obtained, because market participants would take that into account when pricing value of the land if it had a
different use. Thus the fair value of the land if used for residential purposes would be $5.44 million (($7.4m –
$0.2m – $0.3m – $0.1m) × 80%). In this situation, the presumption that the current use is the highest and best
use of the land has been overridden by the market factors which indicate that residential development is the
highest and best use. Therefore the fair value of the land would be $5.44 million.

10.
Beta Ltd.
Extract from the statement of profit and loss
(Amount in - )
20X2 Restated 20X1
Sales 104,000 73,500
Cost of goods sold (80,000) (60,000)
Profit before income taxes 24,000 13,500
Income taxes (7,200) (4,050)
Profit 16,800 9,450

17 | P a g e
RTP CAP III June 2022 © The Institute of Chartered Accountants of Nepal

Beta Ltd.
Statement of changes in equity
(Amount in - )
Share Capital Retained Earnings Total
Balance as at March 31, 20X0 5,000 20,000 25,000
Profit for the year ended March 31,
20X1, as restated — 9450 9450
Balance as at March 31, 20X1 5,000 29,450 34,450
Profit for the year ended March 31, 20X2 — 16,800 16,800
Balance as at March 31, 20X2 _ 5000 46,250
51,250
Extract from the notes:
Some products that had been sold in 20X0-20X1 were incorrectly included in inventory at March 31, 20X1 at
Rs 6,500. The financial statements of March 31, 20X1 have been restated to correct this error. The effect of
the restatement on those financial statements is as summarized above. There is no effect in March 31, 20X2.

11.
It is always useful to set up a working reconciling the assets and obligation:
Assets Obligation
$ $
Fair value/present value at 1/1/X2 1,100,000 1,250,000
Interest (1,100,000 × 6%)/(1,250,000 × 6%) 66,000 75,000
Current service cost 360,000
Contributions received 490,000
Benefits paid (190,000) (190,000)
Return on plan assets excluding amounts in 34,000 -
net interest (balancing figure)
(OCI) (re-measurement)
Loss on re-measurement (balancing figure) (OCI) - 58,600
1,500,000 1,553,600
The following accounting treatment is required.
(a) In the statement of profit or loss and other comprehensive income, the following amounts will be
recognised.

In profit or loss:
$
Current service cost 360,000
Net interest on net defined benefit liability (75,000 – 66,000) 9,000
In other comprehensive income (34,000 – 58,000) 24,000

(b) In the statement of financial position, the net defined benefit liability of $53,600 (1,553,6 00 –1,500,000)
will be recognised.

12.
Not making any accounting entries in respect of the granting of options is contrary to the requirements of
NFRS 2 Share-based payment. NFRS 2 requires that share-based transactions made in return for goods or
services are recognised in the financial statements.

The granting of options to the senior executives is a share-based payment under NFRS 2 and will need
to be recognised as a remuneration expense. The amount to be charged as an expense is measured at the fair
value of the goods or services provided as consideration for the share based payment or at the fair value of the
share-based payment, whichever can be more reliably measured.
In the case of employee share options, the market value of the options on the day these are granted is used as
this can be measured more reliably. The market value of the share options at the day these were granted, 1
18 | P a g e
RTP CAP III June 2022 © The Institute of Chartered Accountants of Nepal

Shrawan 20X4 was $2 each.

The exercise price for the option at $20 per share is above the market price on the date of issue on 1 Shrawan
20X4. This, however, does not mean that the option has zero market value. It has no intrinsic value, but it has
what is referred to as time value relating to expectations of a share price increase over time.
The options vest at the end of the two year period. The company expects that 90% of the options will vest as
it is estimated that 90% of the executives will remain in employment for the two-year period and thus be able
to exercise their options in full.
The remuneration expense will be 5,000 X $2 X 50 X 90% = $450,000 and as this vest over a two-year period,
the entry to income for the current year to 31 March 20X5 will relate to half that amount:
1/2 X $450,000 = $225,000
$ $
DEBIT Share-based payment remuneration expense 225,000
CREDIT Equity – share-based payment reserve 225,000
When the shares are issued a transfer will be made from that reserve together with any further proceeds (if
any) of the shares and will be credited to the share capital and share premium accounts.

13.
The company is building up the provision over the life of the asset using the 'units of production' method. NAS
37 requires a provision to be the best estimate of the expenditure required to settle the obligation at the end of
the reporting period. The provision should be capitalised as an asset if the expenditure provides access to future
economic benefits; otherwise, it should be immediately charged to the statement of profit or loss
NAS 16 Property, plant and equipment has been amended to cater for debits set up when assets are created as
a result of provisions. Such assets are written off over the life of the facility and normal impairment rules will
apply. The decommissioning costs of Rs 1,231m (undiscounted) not yet provided for will be included as a
provision (at the discounted amount) in the statement of financial position and a corresponding asset created.
The discounting method used is inconsistent. NAS 37 suggests the use of a pre-tax rate reflecting current
market assessments of the time value of money and risks. The discount rate should not reflect risks which have
been included by adjusting future cash flows.
The company also makes reserve adjustments for changes in price levels. This adjustment comprises two
elements chargeable to the statement of profit or loss, not reserves:
(i) Adjustments to the provision caused by changes in discount rates.
(ii) An interest element representing the 'unwinding' of the discount, which should be classified as part
of interest expenses in the statement of profit or loss. Any subsequent change in the provision should be
recognized in profit or loss for the year, but the company is treating the adjustment of Rs 27m as a movement
on reserves.

14.
Diluted EPS at 31 May 20X7
No of shares Earnings EPS
Rs'000s Rs'000 Paisa
Basic 40,000 18,000 45.00
Options (Note 1) (W) 400
40,400 18,000 44.55
Convertible preferred shares (Note 2) 3,200 160
43,600 18,160 41.65
Convertible loan stock (Note 3) 2,300 804
Discount (Rs 20m X 1%) 200
45,900 19,164 41.75
Notes:
Again, the dilutive effect of each item must be considered, looking at the cumulative effect each time.
1 Options- Diluted EPS is less than 45p, so the effect is dilutive.
2 Convertible preferred shares- Diluted EPS is less than 44.55p, so the effect is dilutive.
3 Convertible loan stock- Diluted EPS is higher than 41.65p, so this is not dilutive and should be ignored when
calculating diluted EPS.

19 | P a g e
RTP CAP III June 2022 © The Institute of Chartered Accountants of Nepal

Working
Numbers of options at option price = 2m X Rs 1.20 = $1.24m
Number assumed issued at fair value = Rs 2.4m X Rs 1.50 = 1.6m
So, Number issued free = 2m – 1.6m = 400,000

15.
B Ltd.
Segmental Report
(Rs.‟000)

Particulars Forging Bright Fittings Inter Total


Shop Bar Segment
Elimination
Segment Revenue
Sales
Domestic 90 - - - 90
Export 6,135 300 270 - 6705
External Sales 6225 300 270 - 6795
Inter Segment Sales 4575 45 - 4620 -
Total Revenue 10,800 345 270 4620 6,795

Segment Result (given) 240 30 -12 258


Head Office Expenses -144
Pre-Tax Operating 114
Interest Expenses -16
Profit before tax 98
B Ltd.
Segmental Report
Information in relation to assets and liabilities:
(Rs.‟000)

Particulars Forging Bright Fittings Inter Total


Shop Bar Segment
Elimination
Fixed Asset 300 60 180 540
Net Current Assets 180 60 135 375
Segment Assets 480 120 315 - 915
Unallocated corporate Assets (75+72) 147
Total Assets 1062
Segment Liabilities 30 15 180 225
Unallocated corporate Liabilities 57
Total Liabilities 282
Sales Revenue by geographical market
Rs “000
Home Sales Export Sales Export to Export to Total
(by forging Ruwanda India
shop division)
External Sales 90 6,135 300 270 6,795

20 | P a g e
RTP CAP III June 2022 © The Institute of Chartered Accountants of Nepal

16.
Moon Ltd.
Extract from the Statement of Profit & Loss
Income WN Amount
Change in fair value of purchased dairy cow WN 2 1,00,000
Government Grant WN 3 10,00,000
Change in the fair value of newly born calves WN 4 1,30,000
Fair Value of Milk WN 5 72,000
Total Income 13,02,000
Less: Expenses
Maintenance Costs WN 2 6,00,000
Breeding Fees WN 2 4,00,000
Total Expense (10,00,000)
Net Income 3,02,000
Extracts from Balance Sheet
Property, Plant and Equipment:
Land WN1 50,00,000
Dairy Cow WN 2 11,00,000
Calves WN 4 1,30,000
62,30,000
Inventory
Milk WN 5 72,000
72,000

Working Notes:
1. Land: The purchase of the land is not covered by NAS 41. The relevant standard which would apply to this
transaction is NAS 16. Under this standard the land would initially be recorded at cost and depreciated over
its useful economic life. This would usually be considered to be infinite in the case of land and so no
depreciation would be appropriate. Under Cost Model no recognition would be made for post-acquisition
changes in the value of land. The allowed alternative treatment under Revaluation Model
would permit the land to be revalued to market value with the revaluation surplus taken to the other
comprehensive income. We have followed the Cost Model.
2. Dairy Cows: Under the ‘fair value model’ laid down in NAS 41 the mature cows would be recognised in
the Balance Sheet at 31 March 20X2 at the fair value of 200 x Rs 5,500 = Rs 11,00,000.
Increase in price change 200 x (5,200-5,000) = 40,000
Increase in physical change 200 x (5,500-5,200) = 60,000
The total difference between the fair value of matured herd and its initial cost (Rs 11,00,000 – Rs 10,00,000 =
a gain of Rs 1,00,000) would be recognised in the profit and loss along with the maintenance costs and breeding
fee of Rs 6,00,000 and Rs 4,00,000 respectively.
3. Grant: Grand relating to agricultural activity is not subject to the normal requirement of NAS 20. Under
NAS 41 such grants are credited to income as soon as they are unconditionally receivable rather than being
recognised over the useful economic life of the herd. Therefore, Rs 10,00,000 would be credited to income of
the company.
4. Calves: They are a biological asset and the fair value model is applied. The breeding fees are charged to
income and an asset of 100 x Rs 1,300 = Rs 1,30,000 recognised in the Balance sheet and credited to Profit
and loss.
5. Milk: This is agricultural produce and initially recognised on the same basis as biological assets. Thus the
milk would be valued at 3,000 x Rs 24 = Rs 72,000. This is regarded as ‘cost’ for the future application of
NAS 2 to the unsold milk

17.
Governments often use tax holidays as a great incentive to attract new investors into their country. Although
tax holidays are not specifically defined in NFRS, companies must apply the standard NAS 12,as tax holidays
are provided for a limited period of time.

21 | P a g e
RTP CAP III June 2022 © The Institute of Chartered Accountants of Nepal

The basic rule for measuring deferred tax is to apply the tax rate expected to apply in the period when the asset
is realized or liability settled.
Here, the temporary differences resulting in deferred tax liability in Year 1 & 2 are Rs. 100 million and Rs.
200 million respectively. As given in the question, the asset will be realized only after the tax holiday period
(as it is expected that the whole temporary difference will not be reversed during the tax free period). Therefore,
normal tax rate, i.e. 25% will be used to determine the deferred tax liability arising after the tax free period.
Temporary difference (Rs. million)
Year 1 Year 2 Year 3 Year 4 Year 5 Year 6 Year 7 Year 8 Year 9 Year 10
100 200 195 190 185 180 175 170 165 160
Therefore, deferred tax liability to be recognized at the end of second year = 160 x 25% = Rs. 40 million
Charge to Profit & Loss A/c on Year 2 =Rs. 40 million – liability recognized in Year 1
Alternatively,
Of the Rs.100 million in Year 1, Rs 40 million will reverse in the tax holiday period. Therefore, deferred tax
liability will be created on Rs. 60 million at the tax rate of 25%, i.e., Rs.15 million. In the second year, the
entire Rs. 200 million will reverse only after the tax holiday period therefore, deferred tax charge in the profit
and loss account will be Rs. 50 million and deferred tax liability in the balance sheet will be Rs. 65 million.

18.
(a)
Power Limited should in its financial statements for the year ended March 31, 20X2 make related party
disclosures for the period from July 15, 20X1 to March 21, 20X2 when Transmission Limited was its
subsidiary.
(b)
For Entity A’s financial statements, the exemption of NAS 24 applies to:
(a) transactions with Government G; and
(b) transactions with Entities 1 and 2 and Entities B, C and D. However, that exemption does not apply to
transactions with Person X.

19.
Relevance and purpose of Human Capital Reporting:
Now a day, financial reporting system has been changed from traditional approach to new comprehensive
approach. In this approach, all quantifiable resources including human resources are taken in to consideration.
Relevance: It enables the comparison of Human Capital and Non-human Capital employed in a firm, which
shows that the company is labour intensive or capital intensive; and it also enables the comparison of pay scale
with industry average.
Purpose: It provides scope for planning and decision making in relation to proper manpower planning.
Calculation of Value of Human Resources of the employee groups:
i) For skilled employees:
Value of Human resources =
(Avg. Annual earnings ×No. of employee)×PVIFA for additional years of service than
average age
= Rs. 50,000×20× 2.283 (Annuity @ 15% for 3 Yrs)
= Rs. 2,283,000
ii) For Unskilled employees:
Value of Human resources = Rs. 30,000×25× 1.626 (Annuity @ 15% for 2Yrs)
= Rs. 1,219,500
Total value of HR Resources = Rs. 2,283,000 + Rs. 1,219,500 = Rs. 3,502,500

20.
The analysis of value added can be classified in at least three fields of research: management control (internally
oriented), financial reporting (externally oriented), and social reporting (externally oriented).
The first field emphasizes the role of value added as an indicator of efficiency among the tools to appraise the
“economic productivity”. Therefore, the value added measurement is used as one of the performance indicators
in the management control system, particularly in the industrial sector, with the main purpose of controlling
costs and the performance of productive factors, especially labour.

22 | P a g e
RTP CAP III June 2022 © The Institute of Chartered Accountants of Nepal

The second field of analysis looks at value added reporting as additional information to the traditional income
statement, which is focused on earnings and net profit. An externally oriented value added statement can
synthesize the contribution of the whole business in different sectors, not only the industrial one.
The third approach considers the value added statement as an form of social reporting. It is a means of
communication in the overall business reporting process which is added to the traditional and official annual
report.

21.
The $12 million consideration is fixed. The $3 million consideration that is dependent on the number of
mistakes made is variable. Bristow must estimate the variable consideration. It could use an expected value or
a most likely amount. Since there are only two outcomes, $0 or $3 million, then a most likely amount would
better predict the entitled consideration.
(a) Bristow expects to hit the target. Using a most likely amount, the variable consideration would be valued
at $3 million. Bristow must then decide whether to include the estimate of variable consideration in the
transaction price.
Based on past experience, it seems highly probable that a significant reversal in revenue recognised would not
occur. This means that the transaction price is $15 million ($12m + $3m).
As a service, it is likely that the performance obligation would be satisfied over time. The revenue recognised
in the year ended 31 December 20X1 would therefore be $1.25 million ($15m × 1/12).
(b) Depending on the estimated likelihood of hitting the target, the variable consideration would either be
estimated to be $0 or $3 million.
Whatever the amount, the estimated variable consideration cannot be included in the transaction price because
it is not highly probable that a significant reversal in revenue would not occur. This is because Bristow has no
experience of providing this service. Therefore, the transaction price is $12 million.
As a service, it is likely that the performance obligation would be satisfied over time. The revenue recognised
in the year ended 31 December 20X1 would be $1 million ($12m × 1/12).

22.
(a)
In the case of trade receivables such as this, that is trade receivables that do not have an NFRS 15
financing element, NFRS 9 allows a simplified approach to the expected credit loss method. The loss
allowance is measured at the lifetime expected credit losses, from initial recognition.
On 1 December 20X4
The entries in the books of Ambush will be:
DEBIT Trade receivables Rs 600,000
CREDIT Revenue Rs 600,000
(Being initial recognition of sales)

An expected credit loss allowance, based on the matrix given, would be calculated as follows:
DEBIT Expected credit losses Rs 6,000
CREDIT Allowance for receivables Rs 6,000
(Being expected credit loss: $600,000 X 1%)

On 31 January 20X5
Applying Ambush's matrix, Tray has moved into the 5% bracket, because it has exhausted its 60-day credit
limit. Despite assurances that Ambush will receive payment, the company should still increase its credit loss
allowance to reflect the increased credit risk. Ambush will therefore record the following entries on 31
January 20X5:
DEBIT Expected credit losses Rs 24,000
CREDIT Allowance for receivables Rs 24,000
(Being expected credit loss: Rs 600,000 X 5% – $6,000)

(b)
Hedge effectiveness
If an entity chooses to hedge account, then it must assess at inception and at each reporting date whether the
hedge meets all effectiveness criteria. These criteria are as follows:
23 | P a g e
RTP CAP III June 2022 © The Institute of Chartered Accountants of Nepal

• There must be an economic relationship between the hedged item and the hedging instrument
• The effect of credit risk does not dominate the value changes that arise from that relationship
• The hedged ratio should be the same as that resulting from the quantity of the hedged item that the entity
actually hedges and the quantity of the hedging instrument that the entity actually uses.

NFRS 9 Financial Instruments says that hedge effectiveness relates to expectations and therefore the
assessment of effectiveness must be forwards-looking.
Derivatives
All derivatives have to be initially recognised at fair value, i.e. at the consideration given or received at
inception of the contract. Derivatives A and C appear to have no purchase price, so are initially recognised at
nil. Derivative B will be initially recognised at its fair value of $1m.
Derivative A: Artwright has entered into this derivative for speculative purposes. NFRS 9 requires that all
derivatives not designated as part of a hedge accounting arrangement are accounted for at fair value through
profit or loss. The loss of $20 million that has been incurred has to be immediately recognised profit or loss.

Derivative B: If a fair value hedge is effective, then the movement in the fair value of the item and the
instrument since the inception of the hedge are normally recognised in profit or loss. However, if the hedged
item is an investment in shares that has been designated to be measured at fair value through other
comprehensive income (FVOCI), then the fair value movement on the hedged item and the hedging instrument
are recognised in other comprehensive income.
The hedged item is an investment of shares designated to be measured at FVOCI. Therefore, the following
entries are required at the reporting date:

Dr Financial asset $8.5m


Cr Other comprehensive income $8.5m

Dr Other comprehensive income $10m


Cr Derivative $10m.
Derivative C: If a cash flow hedge is effective, then the movement in the fair value of the instrument is
accounted for through other comprehensive income. However, if the movement on the instrument exceeds the
movement on the item, then the excess is recognised in profit or loss.
The following entry is required:
Dr Derivative $25m
Cr Other comprehensive income $24m
Cr Profit or loss $1m
When the raw materials are purchased, the gains recognised in other comprehensive income can be reclassified
against the carrying amount of the inventory.

23.
Clove Group
Consolidated Statement of Financial Position as at 31 May 20X7
Rs (m)
Assets
Non Current Assets
Property Plant & Equipment 320.0
260+20+26+6 (W2)+5 (W2)+3 (W5)
Goodwill (W 6) 10.1
Other Intangibles: trade Name (W 2) 4.0
Available for sale investments 10.0
344.2
Current Assets: 65 + 29 + 20 114.0
Total Assets 458.1
Equity & Liabilities
Ordinary Shares 150.0
Other reserves (W 8) 30.8

24 | P a g e
RTP CAP III June 2022 © The Institute of Chartered Accountants of Nepal

Retained Earnings (W 8) 150.8


Equity Component of Convertible Debt (W 4) 1.6
333.2
Non-Controlling Interest ( W 7) 28.9
Total Equity 362.1

Non-Current Liabilities (W 10) 49.0


45 + 2 + 3 + 0.1 (W 3) – 30 + 28.9 (W 4)
Current Liabilities: 35 + 7 + 5 47
Total Liabilities 96

Total Equity & Liabilities 458.1


Workings

1. Group Structure
Body Fit
Glove’s Effective holding in Body 80%
Glove’s Effective holding in Fit 56%
(70% X 80%)

NCI 20% 44%

2. Fair Value Adjustment of Subsidiaries


On At Reporting Post
Acquisition Date Acquisition
Date ( Rs’ m) ( Rs’ m) ( Rs’ m)
Body
Land: 60-(40+10+4) 6 - 6
Brand name (note) 5 (1) 4
11 1 10

On At Reporting Post
Acquisition Date Acquisition
Date ( Rs’ m) ( Rs’ m) ( Rs’ m)
Fit
Land: 39-(20+8+6) 5 - 5
5 - 5

Note. The trade name is an internally generated intangible asset. While these are not normally recognised under
NAS 38 Intangible assets, NFRS 3 Business combinations allows recognition if the fair value can be measured
reliably. Thus this Glove should recognise an intangible asset on acquisition (at 1 June 20X5). This will reduce
the value of goodwill. The trade name is amortised over ten years, of which two have elapsed: Rs 5m × 2/10
= Rs 1m.
So the value is Rs (5 – 1)m = Rs 4m in the consolidated statement of financial position.

3. Defined Pension Scheme


Rs m
Present value of obligation 26.0
Fair value of plan assets (20.0)
Unrecognised actuarial losses (note) (Rs 3m – Rs 0.1m (note)) (2.9)
3.1
Note. recognised actuarial losses

Corridor amounts:
10% of present value of obligation: 10% × Rs 20m = Rs 2m

25 | P a g e
RTP CAP III June 2022 © The Institute of Chartered Accountants of Nepal

10% of fair value of plan assets: 10% × Rs 16m = Rs 1.6m


∴ Use Rs 2m
Rs m
Unrecognised losses at 1 June 20X6 3
Less 10% of P.V. of obligation (2)
Excess 1

Amortised over ten years ... Rs 1m/10 = Rs 0.1m

Accounting entries:
DEBIT Retained earnings Rs 0.1m
CREDIT Unrecognised actuarial losses Rs 0.1m

4. Convertible Bond
Under NFRS 9, the bond must be split into a liability and an equity component:
Rs’000 Rs’000
Proceeds: 30,000 X Rs 1,000 30,000
Present Value of principal in 3 years time Rs 30 m X 1/(1.08)3 23,815
Present value of Interest Annuity:
Rs 30m X 6% = 1,800,000
X 1/(1.08) 1,667
2 1,543
X 1/(1.08)
X 1/(1.08)3 1,429
Liability Component (28,454)
Equity Component 1,546

Rounded to Rs 1.6m
Balance of liability at 31 May 20X7
Rs ’000
Balance b/d at 1 June 20X6 28,454
Effective interest at 8% 2,276
Coupon interest paid at 6% (1,800)
Balance c/d at 31 May 20X7 28,930

5. Exchange of Assets
The cost of the plant should be measured at the fair value of the asset given up, rather than the carrying value.
An adjustment must be made to the value of the plant, and to retained earnings.

Rs ’000
Fair value of land 7
Carrying value of land (4)
∴Adjustment required 3

DEBIT Plant Rs 3m
CREDIT Retained earnings Rs 3m

6. Goodwill
Glove in Body Body in fit
Rs (m) Rs (m) Rs (m) Rs (m)
Consideration Transferred 60 30
Indirect Holding Adjustment (30 X 20%) (6)
NCI (PSNA)
[65 X 20%] 13
[39 X 44%] 17.16
73 41.16

26 | P a g e
RTP CAP III June 2022 © The Institute of Chartered Accountants of Nepal

FV of net asset acquired (65) (39)


Goodwill 8 2.16

7. Non-Controlling Interest
Body Fit
Net Asset as per question 70.0 38
Cost of investment in fit (30) -
Fair value Adj (w 2) 10 5
50 43
X 20% X 44%
10.00 18.92

8. Retained Earning
Glove Body Fit
Rs m Rs m Rs m
Per question 135.00 25 10
Fair value movement (W2) - (1) -
Pension scheme (W3) (0.10)
Convertible bonds (W4) (2.3 – 1.8) (0.50)
Assets exchange:
Adjustment to plant (W5) 3.00
Less pre-acquisition (10) (6)
14 4
Share of Body
80% × 14 11.20
Share of Fit
56% x 4 2.24
150.84

9. Other reserve

Glove Body Fit


Rs m Rs m Rs m
Per question 30.0 5 8
Less pre-acquisition (4) (8)
1 -
Share of body
80% × 1 0.8
Share of Fit
56% × 0 0.0
30.8

10. Non-current liabilities


Note. This working is for additional information. To save time, you should do yours on the face of the
consolidated position statement
Glove Body
Rs m Rs m
Non-current liabilities per question:
Glove 45
Body 2
Fit 3
50.0
Unrecognised actuarial losses (W3) 0.1
Proceeds of convertible bond (30.0)
Value of liability component 28.9
49.0
27 | P a g e
Paper-1
Advanced Financial Reporting

3
Q. No. 1 Consolidated Financial Statements
Kanchenjunga Ltd. has investment in Koshi Ltd., Mechi Ltd. and Api Ltd. The Statements of
financial position of Kanchenjunga, Koshi and Mechi as at 32 Ashadh 2079 are as under:
Rs.
000"
Assets Kanchenjunga Koshi Mechi
Non-Current Assets
Property, Plant and Equipment 53,100 60,500 63,250
Investment in Subsidiaries
Koshi 57,000
Mechi 46,400
Investment in Api 3,400
Financial assets 9,500 - -
169,400 60,500 63,250
Current Assets 44,250 39,100 11,200

Total 213,650 99,600 74,450

Equity and Liabilities


Equity Share Capital (Rs. 100 per share) 82,500 36,000 35,000
Retained Earnings 59,000 44,000 18,200
Other Components of Equity 6,400 3,900 2,950
147,900 83,900 56,150
Non-Current Liabilities
Term Loan 53,900 9,450 8,600
Defined Benefit Obligation 3,250 - -
Current Liabilities 8,600 6,250 9,700
Total 213,650 99,600 74,450
Consider following information for preparation of the group financial statements:
a. On 1 Shrawan 2078, Kanchenjunga acquired 70% of equity shares of Koshi Ltd. On that date,
the retained earnings and other components of equity of Koshi Ltd. were Rs. 39,000,000 and
Rs. 3,200,000 respectively. At 1 Shrawan, 2078, the fair value of identifiable net assets of
Koshi Ltd. was Rs. 80,000,000.
It is group policy to value the non-controlling interest at fair value and, at the date of
acquisition, this was Rs. 24,250,000. The excess in fair value of identifiable net assets is due
to land.
b. On 1 Shrawan 2077, Kanchenjunga acquired 40% of the equity shares of Mechi Ltd. for cash
consideration of Rs. 21,000,000. At this date, carrying amount and fair value of the
identifiable net assets of Mechi Ltd. was Rs. 51,600,000. Kanchenjunga treated Mechi Ltd.
as an associate and equity method was followed to account for interest in Mechi Ltd. up to
Ashadh end, 2078.

4
On 1 Shrawan 2078, Kanchenjunga took control of Mechi Ltd., by further acquiring 45%
interest for cash consideration of Rs. 25,000,000 and included this amount in carrying amount
of investment in Mechi Ltd.
On 1 Shrawan 2078, the retained earnings and other components of equity of Mechi Ltd. were
Rs. 14,650,000 and 2,950,000 respectively and the fair value of identifiable net assets was Rs.
53,100,000. The difference between the carrying amounts and the fair values was in relation
to plant with a remaining useful life of 5 years.
The share prices of Kanchenjunga and Mechi were Rs. 500 and Rs. 160 per share respectively
on 1 Shrawan 2078. The fair value of original 40% holding and the fair value of non-
controlling interest should both be estimated using the market value of shares.
c. Kanchenjunga has been holding 25% equity shares in Api Ltd. since couple of years. Api Ltd.
made profits during the FY 2078/79 of Rs. 1,000,000, which can be assumed to have accrued
evenly. Api does not have any other comprehensive income. On Poush end 2078,
Kanchenjunga sold 10% equity interest for cash of Rs. 2,100,000. Kanchenjunga was unsure
about accounting of this disposal and so has deducted the proceeds from the carrying amount
of the investment at 1 Shrawan 2078 which was Rs. 5,500,000.
The fair value of the remaining 15% shareholding was estimated to be Rs. 3,250,000 at Poush
end, 2078 and Rs. 3,350,000 at 32 Ashadh, 2079. Kanchenjunga no longer exercises
significant influences and has designated the remaining shareholding as fair value through
other comprehensive income.
d. There has been no impairment of goodwill.
e. Kanchenjunga operates a defined benefit pension scheme. On 32 Ashadh, 2079, the company
announced that it was to close down a business division and agreed to pay each of its 150
staffs a cash payment of Rs. 2,500 to compensate them for loss of pension arising from wage
inflation. It is estimated that the closure will reduce the present value of the pension obligation
by Rs. 290,000. Kanchenjunga is unsure of how to deal with such settlement and curtailment
and has not yet recorded anything in its financial statements.
Required:
Prepare the consolidated statement of financial position of Kanchenjunga Group as at 32 Ashadh,
2079 in accordance with Nepal Financial Reporting Standards.

Q. No. 2 NFRS 2 Share-based Payment


Info Developers Ltd. grants 130 stock options to each of its 800 employees on 1 January 2018.
The grant condition is that employees have to remain in the service over the next three years. The
exercise date is 31st December, 2021and the vesting date is 31st December, 2020. The company
estimates that fair value of each option on the grant date is Rs. 30. The nominal value and exercise
price per share is Rs. 100 and Rs. 135 respectively.
On 31st December 2018, the exercise price of the company has dropped to Rs. 130 and the company
has repriced its stock option which will vest at the end of 3rd year i.e. 31st December, 2020. The
company estimates that at the date of repricing, fair value of each of the original options granted
(i.e. before taking into account the repricing) is Rs. 24 and that the fair value of each repriced stock
option is Rs. 28.

5
Following information are also relevant:
31 Dec, 2018 31 Dec, 2019 31 Dec, 2020
Number of employees Left 50 45 30
Estimated No. of employees
expected to leave in next year(s) 80 35 0
On 31st December 2021, 650 employees exercised the option.
Required:
Compute the amount of expenses to be recognized by the company for the year 2018, 2019 and
2020 and also give extract of ESOP outstanding Account as appeared in in the books of account
of Info Developers ltd. in the year 2021.

Q. No. 3 NFRS 5 Non-Current Assets Held for Sale and Discontinued Operations
Rohit Ltd and Kohli Ltd are engaged in similar nature of business. Due to this, on 01.01.2022,
both of them mutually agreed that it would be in the best interest of both of them that Kohli Ltd
be acquired by Rohit Ltd. 1 Jan 2022. The acquired entity, Kohli Ltd, however is itself
a holding entity with two wholly owned subsidiaries, Rahul and Dhawan.
Dhawan is however, acquired exclusively with a view to resale and meets the criteria for
classification as held for sale. Rohit Ltd.’s year end is 32 Dec 2022.
On 1 Jan 2022 the following information is relevant:
❖ The identifiable liabilities of Dhawan have a fair value of Rs.10 Lakhs
❖ The acquired assets of Dhawan have a fair value of Rs.20 Lakhs
❖ The expected costs of selling Dhawan are 1 Lakh
❖ On 32 Dec 2022, the assets of Dhawan have a fair value of Rs. 15 Lakhs.
❖ The liabilities have a fair value of Rs. 6 Lakhs and the selling costs still remain at 1 Lakh.
Discuss how Dhawan Ltd will be treated in the Rohit Group financial statements on acquisition
and at 32 Dec 2022.

Q. No. 4 NFRS 8 Operating Segments


The management of Ichiban Ltd has identified operating segments based on following categories.
Information for these segments is provided below:
In ‘000
Segment External Revenue Internal Revenue Total Revenue Profit/ (Loss) Assets
Pradesh 1 250 250 500 100 3,500
Pradesh 2 46 44 90 (30) 400
Pradesh 3 200 - 200 50 1,000
Pradesh 4 270 280 550 130 4,000
Pradesh 5 45 35 80 (20) 600
Pradesh 6 45 50 95 15 900
Pradesh 7 10 10 20 10 200
Total 866 669 1,535 255 10,600

6
The management wants to know according to NFRS 8, which segments need to be reported?

Q. No. 5 NFRS 9 Financial Instruments


Dillibazar Ltd is a key supplier for Satdobato Ltd. On 1 Nov 2022, Dillibazar Ltd sold goods on
credit to Satdobato Ltd for Rs. 500,000. Satdobato Ltd has a credit limit with Dillibazar Ltd of 60
days and this limit is quite normal in any transactions between them. Dillibazar Ltd applies NFRS
9 Financial instruments, and uses a pre-determined matrix for the calculation of allowances for
receivables as follows.
Days overdue Expected loss provision
Nil 1%
1 to 30 5%
31 to 60 15%
61 to 90 20%
90 + 25%
Satdobato Ltd has, however, not paid the sum by 31 Dec 2022, and so failed to comply with its
credit term, and Dillibazar Ltd learned that Satdobato Ltd was having serious cash flow difficulties
due to downfall of its products. Despite this fact, the finance controller of Satdobato Ltd has
informed Dillibazar Ltd that they will receive payment soon and they need not to worry about it.
Ignore sales tax.
Required:
Show the accounting entries on 1 Nov 2022 and 31 Dec 2022 to record the above, in accordance
with the expected credit loss model in NFRS 9.

Q. No. 6 NFRS 9/NFRS 13/ NAS 32


The directors of Pawan, a public limited company, are worried about the challenging market
conditions of Nepal which the company is facing. The markets are very much volatile and illiquid
and the central government is also injecting liquidity into the economy. Thus, the directors are
concerned about the significant shift towards the use of fair values in financial statements. NFRS
9 Financial instruments in conjunction with NFRS 13 Fair value measurement defines fair value
and requires the initial measurement of financial instruments to be at fair value. The directors are
uncertain of the relevance of fair value measurements in these current market conditions.
Required
(a) Briefly discuss how the fair value of financial instruments is measured, commenting on the
relevance of fair value measurements for financial instruments where markets are volatile and
illiquid.
(b) Further they would like advice on accounting for the following transaction within the financial
statements for the year ended 31 Dec 20X8.
Pawan issued one lakh convertible bonds on 1 Jan 20X6. The bonds had a term of three years
and were issued with a total fair value of Rs.10m which is also the par value. Interest is paid
annually in arrears at a rate of 6% per annum and bonds, without the conversion option,
attracted an interest rate of 9% per annum on 1 Jan 20X6. The company incurred issue costs

7
of Rs. 100,000. If the investor did not convert to shares they would have been redeemed at par.
At maturity all of the bonds were converted into 2.5 million ordinary shares of $1 of Pawan.
No bonds could be converted before that date. The directors are uncertain how the bonds should
have been accounted for up to the date of the conversion on 31 Dec 20X8 and have been told
that the impact of the issue costs is to increase the effective interest rate to 9.38%.

Q. No. 7 NFRS 13 Fair Value Measurement


a. What is an “observable” input for NFRS 13 purposes, and where it can be found?
b. What is the relationship between the input data hierarchy and the choice of measurement
technique? Must this be a standard technique?

Q. No. 8 NFRS 15 Revenue from Contracts with Customers


Kabindra Ltd is a renowned trader and supplier of Gym equipment having all kinds of fitness
equipment available in the market. On 01 Jan 2022, Kabindra Ltd sold a Gym Equipment to a
Rabindra for a total price of Rs. 200,000. Kabindra Ltd invoiced the Rabindra for Rs. 200,000 on
01 Jan 2022.
However, Rabindra made a payment of Rs. 200,000 to Kabindra Ltd only on 20 Jan 2022. The
terms of sale also included an arrangement where Kabindra Ltd would service and maintain the
machine for a two-year period from starting from 01 Jan 2022. Kabindra Ltd would normally
charge an annual fee of Rs. 5,000 for a service and maintenance arrangement of this nature. The
normal selling price of the machine without a service and maintenance arrangement was Rs.
190,000.
Explain and show how the event would be accounted for in the financial statements of Kabindra
Ltd for the year ended 32 December 2022.

Q. No. 9 NFRS 16 Leases


Lumbini Ltd has entered into a 10-year lease contract for a building as a lessee with Bagmati Ltd.
The contract also provides for an option to extend the lease for a further five years. Lease payments
are Rs. 200,000 per year during the initial term and Rs. 250,000 per year during the optional period.
Lumbini ltd also incurred initial direct costs associated with the lease at the commencement date
2079.04.01 at Rs.30, 000.
Lumbini Ltd concluded that it is not reasonably certain to exercise the option to extend the lease
at the commencement date and, therefore, determined that the lease term is for 10 years only. The
interest rate implicit in the lease could not be readily determined at the commencement of the lease.
However, Lumbini Ltd.'s incremental borrowing rate is 8.5% per annum.
Determine the Right of use Assets (ROU) and lease liability at the commencement of the lease and
after 1 year from the date of commencement if:
a. The lease payments are made at the end of the year.
b. The lease payments are made in advance
Also, segregate the above lease liability at the end of year 1 into current and non-current lease
liability.

8
Q. No. 10 NAS 2 Inventories
Bhandari Retail Shop Pvt. Ltd. is engaged in retail business of consumable goods. The company
has obtained VAT registration to carry out its retail business. As on Ashadh end 2079, the company
holds 1,000 kg goods as inventories. The following expenditure was incurred by the company in
relation to the inventories of goods:
Particulars Rs. Per kg
Cost of per kg of Goods 500
Less: Trade discount received 10
Add: VAT paid 63.7
Billing price by the vendor 553.7
Transportation cost per kg 20
You are asked to calculate the cost of per kg of goods to the company and its valuation when the
net realizable value is Rs. 580.

Q. No. 11 NAS 12 Income Taxes


ABC Ltd has purchased Machinery in the past for Rs 50 lakhs which now has carrying amount of
Rs 25 Lakhs. However, during the current fiscal year the machinery is revalued to 60 Lakhs. No
adjustment of revaluation is made for tax purpose. The cumulative depreciation of the machine for
tax purpose is Rs 35 lakhs and the tax rate is 30%. If the asset is sold for more than the cost, the
profit to the extent of accumulated depreciation will be included in taxable profit and the proceeds
in excess of the original cost will not be taxable.
In Lakhs
Particulars Accounts Tax Books
Cost 50 50
Accumulated Depreciation (25) (35)
Carrying value/ Tax Base 25 15
Revalued To 60 -
Revaluation By 35 -
Calculate the amount of Deferred Tax Assets or Deferred Tax Liability of the assets if:
i) The asset is used in the business to recover the revalued amount,
ii) The asset is sold immediately at revalued amount.

Q. No. 12 NAS 19 Employee Benefits


At 1 Shrawan, 2078, the fair value of the Plan Assets was Rs. 1,000,000. The Plan paid benefits
of Rs. 1,90,000 and received contributions of Rs. 490,000 on Poush end, 2078. The company
computes the fair value of Plan Assets to be Rs. 1,500,000 as on Ashadh end, 2079 and the present
value of the Defined Benefit Obligation amounts to Rs. 1,479,200 on the same date. Actuarial
losses on defined benefit obligation were Rs. 6,000.

9
Compounding happens half-yearly. The normal interest rate for 6 months period is 10% per
annum, while the effective interest rate for 12 months period is based on the following data:
At 1 Shrawan, 2078, the company made the following estimates based on market prices at that
date:
Particulars %
Interest and Dividend Income after tax payable by the fund 9.25
Add: Realized and Unrealized Gains on Plan Assets (after tax) 2.00
Less: Administration Cost (1.00)
Expected Rate of Return 10.25
Determine actual return and expected return on plan asset. Also compute amount to be recognized
in ‘Other Comprehensive Income’ in this case.

Q. No. 13 NAS 23 Borrowing Costs


MS Ltd began to construct a new building in Imadol and Kamalbinayak on 1st January 2022.It had
incurred various expenditures during the construction at various dates as given below. For such
the company had used the general borrowings availed from the banks at the given interest rates.
The company had also obtained Rs.1.75 lakh special loan to finance the construction of the
building on 1st January 2022 at an interest rate of 8%.
The company’s general borrowing outstanding during the year are:
Amount (Rs.) Rate of Interest
500,000.00 11%
900,000.00 13%
The expenditure that were made on the building projects were as follows:
Date Amount (Rs.)
1-Jan-12 20,000.00
1-Feb-12 157,000.00
1-Oct-12 275,000.00
1-Dec-12 76,000.00
Both buildings were completed by 31st December 2022. Following the principles prescribed in
NAS 23 Borrowing Costs, calculate the amount of interest to be capitalized in the cost of building,
also calculate the total cost of building considering all the above facts and figures.

Q. No. 14 NAS 24 Related Party Disclosure


a. Argentina Ltd has owned 60% of the equity shares of France Ltd and 70% of the equity shares
of Brazil Ltd for many years. On 1 January 2022, France Ltd entered into a lease agreement
with Brazil Ltd. Under the terms of the lease, France Ltd would lease one of its unused
warehouses, with a remaining useful life of 20 years, to Brazil Ltd for five years. Consideration
payable by Brazil Ltd would be Rs. 25,000 a year in arrears. Market rentals for similar sized
warehouses tend to be around Rs. 250,000 per year

10
Discuss the correct treatment of the above transaction in France Ltd.'s financial statements for
the year ended 30 June 2022.
b. At the end of FY 2078/79, there are three shareholders in FinTax Ltd. The finance director has
60%, and the operation director has 30% stake in the company. The third owner is a passive
investor who does not participate in the management and operation of the company. All
ordinary shares carry equal voting rights. Further, the spouse of the finance director is serving
as the sales director of the company and their son also works there as an intern and receives a
salary of Rs. 700,000 p.a. which is normal compensation package prevailing in the industry.
The finance director and his wife have set up an investment company, InvoMax Ltd. They
jointly own InvoMax and their shares in InvoMax will eventually be transferred to their son
when he has finished the internship with FinTax.
Further, on 1 Chaitra 2078, FinTax obtained a loan of Rs. 20 Lakhs from a local bank, by
keeping the private property of finance director as a collateral.
Required:
Advise FinTax Ltd. for the identification and disclosure of the company’s related parties in
preparing its separate financial statements for the FY 2078/79.

Q. No. 15 NAS 33 Earnings per Share


Hamro Bank Ltd. really wants to know its actual basic earnings per share and has given you with
the following information to compute the basic earnings per share
• Net Profit: Rs. 43 Crores for Year 2077/78 and Rs. 63 Crores for Year 2078/79
• No. of shares outstanding prior to Right issue: 30,000,000 shares
• Right issue: One new share for each four outstanding, i.e. 7,500,000 shares
• Right issue price: Rs. 250
• Last date to exercise right: 30/6/2078
• Fair rate of equity share immediately prior to exercise of right on 30/6/2078: Rs. 300
Required
Compute the basic earnings per share for the years 2077/78 and 2078/79 of Hamro Bank Ltd

Q. No. 16 NAS 36 Impairment of Assets


a. Grow Ltd., several years ago, acquired a Strategic Business Unit (SBU), which can generate
cash inflows independently but, at 32 Ashadh 2079, the CFO of the company was concerned that
the value of the SBU had declined because of reduction in sales owing to entry of new competitors
in the market. On that date, the carrying values of the assets under SBU before any impairment
review were as under:
Rs. in Lakhs
Goodwill 6
Property, Plant and Equipment 20
Other Assets 38
Total 64
11
The fair values of the property, plant and equipment and the other assets at 32 Ashadh 2079 were
Rs. 20 Lakhs and Rs. 34 Lakhs respectively and their costs to sell were Rs. 2 lakhs and 6 lakhs
respectively.
It is expected that the SBU will generate following cash flows in next five years:

Rs. in Lakhs
Pre-tax Cash Post-tax Cash
Financial Year flows flows
FY 2079/80 16 10
FY 2080/81 14 10
FY 2081/82 10 6
FY 2082/83 6 3
FY 2083/84 26 20
The pre-tax discount rate for the SBU is 12% and the post-tax discount rate is 9%. Grow Ltd. has
no plans to expand the capacity of the SBU and believes that a reorganization would bring cost
savings but, as yet, no plan has been approved.
Give advice to the CFO whether the SBU’s value is impaired and suggest how it should be
reflected in the financial statements for the year ended on 32 Ashadh, 2079 in accordance with
relevant Nepal Financial Reporting Standards.
b. NB Ltd has two machineries in its books. The carrying value of first machinery after 2 years of
use is Rs. 30 lakhs with remaining life of 3 years. The machinery if sold now would generate Rs.
25 lakhs cash flows net of all the selling costs. However, due to accidents that occur in the factory
in the same day, the company now expects the machinery to generate less cash flow than
anticipated for the rest of its useful life. The estimated cash flow for the next year would be Rs. 5
lakhs with an estimated growth rate of 3% pa thereafter for the remaining term.
The expected growth rate for the following years is estimated to be 3% pa with the discount rate
of 10%.
The second machinery was acquired as on 01.04.2075 for 18 lakhs which had estimated useful life
of 5 years. On 01.04.2078, the carrying value of the machinery was reassessed to Rs. 12.4 lakhs
and the gain arising out of revaluation was credited to revaluation reserve. However, during the
year 2078.79, due to the change in the market conditions, the recoverable amount was ascertained
to be only Rs. 2.6 lakhs as on Ashadh end 2079. NB Ltd had followed the policy of writing down
the revaluation gain by the increased charges of depreciation resulting from revaluation.
Both the fixed assets are subjected to Straight line depreciation (SLM) with nil residual value each.

Q. No. 17 NAS 38 Intangible Assets


On 1 Shrawan 2074, Jaftra Pvt. Ltd. acquired a trademark, Lilivin, for a line of sport wear for Rs.
50 Lakhs. Initially, Jaftra expected to continue marketing and receiving cash flows from the Lilivin
product-line indefinitely. However, it decided to amortise the trademark over 10 years, using the
straight-line method because of the complexity in ascertaining its useful life.
In Shrawan 2078, one of the rival companies unexpectedly revealed a technological breakthrough
which is expected to result in a product which, when launched, will significantly reduce the
demand for the Lilivin product-line. The demand for the Lilivin product-line is expected to remain
high until Ashadh 2081, when the rival company is expected to launch its new product.
12
At 32 Ashadh 2079, Jaftra assessed the recoverable amount of the trademark at Rs. 10 Lakhs and
has intention to continue the production of Lilivin product till Ashadh end, 2081.
Advise Jaftra on how to reflect its trademark in the financial statements for the year ended on 32
Ashadh 2079.

Q. No. 18 Value Added Statement


A1Soft Ltd. has been preparing value added statements from past few years. The company has
decided to introduce an incentive system based on value added in order to enhance the employee
performance. The incentive system proposes that the best index performance (from company’s
view) i.e. employee costs to value added for the last five years will be used as the target index for
future calculations of the bonus to be earned.
After the target index is determined, any actual improvement in the index will be rewarded, to
employer and employees in the ratio of 1:2. The bonus is given at the end of the period, after the
profit for the year is ascertained.
From the following details, find out the bonus to be paid to the employees, if any, for 2022:
Value Added Statements for 5 Years
Rs. '000
Year 2017 2018 2019 2020 2021
Sales 4,200 5,700 6,900 7,800 9,000
Less: Cost of bought in
goods and services 1,920 3,000 3,750 4,200 4,800
Value Added 2,280 2,700 3,150 3,600 4,200
Employee Costs 975 1,140 1,260 1,476 1,680
Depreciation 390 465 540 660 840
Taxes 480 570 630 750 840
Interest on Debentures 60 60 60 60 60
Dividend 150 225 300 360 450
Retained Earnings 225 240 360 294 330
Value Added 2,280 2,700 3,150 3,600 4,200

Summarized Statement of Profit or Loss for 2022


Particulars Rs. "000 Rs. "000
Sales 10,950
Expenses
Cost of materials 3,750
Wages 1,050
Production staff salaries 300
Production expenses 1,050
Depreciation of Machinery 750
Administrative salaries 450
Administrative expenses 450
Administrative depreciation 300
13
Debenture interest 60
Sales Department's Salaries 90
Sales expenses 300
Depreciation Sales Department 90
Total Expenses 8,640
Profit 2,310

Q. No. 19 Public Financial Management


What is public financial management system? Explain the initiative taken by the government of
Nepal to strengthen PFM system.

Q. No. 20 Short note:


a. Opportunity Cost in the context of Human Resource Accounting
b. Sustainability Reporting

14
Answers:
Q. No. 1 Consolidated Financial Statements
Kanchenjunga Group
Statement of Financial Position
As at 32 Ashadh 2079

Rs. '000
Assets Amount
Non-Current Assets
Goodwill 3,950
Property, Plant and Equipment 179,050
[53,100+60,500+63,250+1,800+500-100]
Financial assets 12,850
[9,500+3,350]
195,850
Current Assets 94,550

Total 290,400
Equity and Liabilities
Equity Fund
Equity Share Capital (Rs. 100 per share) 82,500
Retained Earnings 66,072.50
Other Components of Equity 7,115
155,687.50
Non-Controlling Interest 34,877.50
Non-Current Liabilities
Term Loan 71,950
Defined Benefit Obligation 2,960
Current Liabilities 24,925
Total 290,400

15
\Working Notes:

Working Note 1: Group Structure

Subsidiary (70%) Kanchenjunga Ltd.


2078.04.01

Associates (25%)
Associates (40%)
Koshi Ltd. Poush end, 2078
1 Shrawan, 2078

Subsidiary (85%) Financial Asset (15%)

Mechi Ltd. Api Ltd.

Working Note 2: Net Assets Calculation


Koshi Ltd.
000"
Particulars At Acquisition Date At Reporting Date
Equity Share capital 36,000 36,000
Retained Earnings 39,000 44,000
Other Components of Equity 3,200 3,900
Fair Value adjustment-Land (Bal. Fig.) 1,800 1,800
Total 80,000 85,700
Increase in Net assets 5,700

Mechi Ltd.
000"
Particulars At Acquisition Date At Reporting Date
Equity Share capital 35,000 35,000
Retained Earnings 14,650 18,200
Other Components of Equity 2,950 2,950
Fair Value adjustment-Plant (Bal. Fig.) 500 500
Additional Depreciation on Plant (500/5) (100)
Total 53,100 56,550
Increase in Net assets 3,450

16
Working Note 3: Goodwill Calculation
000"
Koshi Ltd. Mechi Ltd.
Cash consideration 57,000 25,000
Fair value of Existing Holding* 22,400
Fair Value of Non-controlling Interest** 24,250 8,400

Less: Fair Value of Identifiable Net Assets at


acquisition date (80,000) (53,100)
Goodwill 1,250 2,700
3,950
For Mechi
*(350,000*0.4*160)
**(350,000*0.15*160)

Working Note 4: Gain (loss) on existing holding (40%) at Mechi while acquiring control
000"
Amount
Cost of investment 21,000
Share in increase in Net Assets 400
(14650+2950+35000-51600)*0.4
Carrying amount as on 2078.04.01 21,400
Fair Value of Existing Holding 22,400
Gain 1,000
This amount of gain should be debited to investment in Mechi Ltd and credited to Profit or loss
account at the time of acquiring control over Mechi Ltd.

Working Note 5: Disposal of Shares of Api Ltd.


000"
Amount
Carrying amount as at 2078.04.01 5,500
Share of profit till the date of disposal 125
(1,000*0.5*0.25)
Carrying amount at the date of disposal (A) 5,625

Fair value of remaining interest on Poush end, 2078 3,250


Proceeds from disposal 2,100
(B) 5,350

Loss on disposal (A-B) 275


The remaining 15% holding shall be restated at its fair value of Rs. 3,350,000 at 32 Ashadh 2079
and gain of Rs. 100,000 shall be recognized as other components of equity.

17
Working Note 6: Non-controlling Interest
000"
Koshi Ltd. Mechi Ltd.
Fair Value of NCI at Acquisition date 24,250 8,400
Share in Post-acquisition increment in net assets 1,710 517.50
[5700*30%]
[3450*15%]
25,960 8,917.50
Total NCI 34,877.50

Working Note 7: Pension adjustment


000"
Amount
Estimated settlement on pension liability 375
[150@ Rs. 2,500)
Estimated curtailment gain 290

Loss to be charged against retained earnings 85


Pension liability w.r.t. settlement should be included within current liabilities whereas Defined
Benefit obligation should be reduced by curtailment gain.

Working Note 8: Group Retained Earnings


000"
Amount
Kanchenjunga 59,000
Share in post-acquisition retained earnings
Koshi Ltd. [5700-(3900-3200)]*0.7 3,500
Mechi Ltd. [3450*0.85] 2,932.50

Loss due pension adjustment (85)


Loss on disposal of interest in Api Ltd. (275)
Gain on existing holding at Mechi at the date of
acquisition 1,000
Total 66,072.50

Working Note 9: Other Components of Equity


000"
Amount
Kanchenjunga 6,400
Share in post-acquisition increase in OCI:
Koshi Ltd. 490
Mechi Ltd. -
Share in profit of Api till the date of disposal 125

18
Gain on Fair value of Investment in Api 100
Total 7,115.00

Q. No. 2 NFRS 2 Share-based Payment


NFRS 2 Share-based Payment states that equity settled share-based payment transactions related
to employees should be measured at the fair value of the equity instruments granted at the grant
date. So, fair value of stock option at the grant date i.e. Rs. 30 each has to be used to measure the
transaction in the given case of Info Developers Ltd.
Further, when any modification occurs to the terms of equity settled share-based payment, an entity
must continue to recognize the grant date fair value of equity instruments in profit or loss account,
and make adjustments for change in fair values prospectively by spreading the difference between
fair value of the original arrangement (current fair value) and fair value of new arrangement over
the period from the date of modification to the vesting date.
We know that:
Equity = (No. of expected employees at vesting date)*(stock option per employee)*fair value of
option)*Year elapsed/Vesting period
Staff Expense for the year = Closing Equity – Opening Equity
Computation of Expenses to be recognized by the company:
Particulars Calculation Equity Staff Expenses
Year 2018 [(800-50-80)*130*30*1/3] 871,000 871,000

Year 2019
Under Original Arrangement [(750-45-35)*130*30*2/3] 1,742,000 871,000
Modification [(750-45-35)*130*(28-24)*1/2] 174,200 174,200
Total 1,916,200 1,045,200

Year 2020
Under Original Arrangement [(705-30)*130*30*3/3] 2,632,500 890,500
Modification [(705-30)*130*4*2/2] 351,000 176,800
Total 2,983,500 1,067,300

ESOP Outstanding Account


Date Particulars Amount Date Particulars Amount
31.12.2021 To Share capital 8,450,000 01.01.2021 By Balance b/d 2,983,500
[650*130*100] 31.12.2021 By Bank 10,985,000
31.12.2021 To Securities Premium 5,408,000 [650*130*130]
[650*130*(30+30+4)]
31.12.2021 To Retained Earnings 110,500
Total 13,968,500 Total 13,968,500

19
Q. No. 3 NFRS 5 Non-Current Assets Held for Sale and Discontinued Operations
On acquisition, the assets and liabilities of Dhawan are measured at fair value less costs to sell in
accordance with IFRS 5:
Particulars Rs. In Lakhs
Assets 20
less: Selling Cost (1)
19
Liabilities (10)
Net Fair Value less cost to sell 9

At the reporting date, the assets and liabilities of Dhawan are remeasured to update the fair value
less cost to sell.
Particulars Rs. In Lakhs
Assets 15
less: Selling Cost (1)
14
Liabilities (6)
Net Fair Value less cost to sell 8
The fair value less cost to sell has decreased from Rs. 9 Lakhs on 1 Jan to 8 Lakhs on 32 Dec. This
1 lakh reduction in the fair value must be presented in the consolidated statement of profit or loss
as a part of the single line item entitled “Discontinued operations”. Also included in this line is the
post-tax profit or loss earned/incurred by Dhawan in the Jan-Dec 2022 period.
The assets and liabilities of Dhawan must be disclosed separately on the face of the statement of
financial position. Dhawan’s assets will appear below the subtotal for the Rohit group’s current
assets:
Non-current assets classified as held for sale Rs. 14 Lakhs.
Dhawan’s liabilities will appear below the subtotal for the Rohit group’s current liabilities:
Liabilities directly associated with non-current assets classified as held for sale Rs. 6 Lakhs.
No Other disclosure is required.

Q. No. 4 NFRS 8 Operating Segments


Step 1: The 10% Test**
Segment 10% total Revenue test 10% result test 10% Assets Test Reportable?
Pradesh 1 Yes Yes Yes Yes
Pradesh 2 No No No No
Pradesh 3 No Yes No Yes
Pradesh 4 Yes Yes Yes Yes

20
Pradesh 5 No No No No
Pradesh 6 No No No No
Pradesh 7 No No No No
** From Working Notes below
Based on the 10% tests, Pradesh 1, 3 and 4 are reportable. However, we must check whether they
comprise at least 75% of the company's external revenue.
Step 2: The 75% Test: ‘000
External
Segments revenue
Pradesh 1 250
Pradesh 3 200
Pradesh 4 270
Total 720
The external revenue of reportable segments is 83.14% (720/866) of total external revenue. The
75% test is met and no other segments need to be reported.
Hence, the reportable segments are Pradesh 1, 3 and 4.
Working Notes (All figures in ‘000)
W. No. 1 10% of total Sales
10% of 15,35,000 153.5

Hence, all the segments having total sales above 153.50 thousand are reportable
W. No. 2 10% of result
10% of profit-making segments
10% of (100+50+130+15+10) 30.5
10% of profit-making segments
10% of (30+20) 5

Therefore, all the segments making profit or loss greater than 30.5 thousand are
reportable.
W. No. 3 10% of Total Assets
10% of 10.6 million 1,060

All the segments exceeding 1060 thousands assets are to be reported.

Q. No. 5 NFRS 9 Financial Instruments


In the case of trade receivables such as this, that is trade receivables that do not have an NFRS 15
financing element, NFRS 9 allows a simplified approach to the expected credit loss method. The
loss allowance is measured at the lifetime expected credit losses, from initial recognition. On 1
Nov 2022 the entries in the books of Dillibazar Ltd will be:
Dr. Trade receivables Rs. 500,000
Cr. Revenue Rs. 500,000
21
Being initial recognition of sales an expected credit loss allowance, based on the matrix given,
would be calculated as follows:
Dr. Expected credit losses Rs. 5,000 *
Cr. Allowance for receivables Rs. 5,000
*(Being expected credit loss: Rs. 500,000*1%)
On 31 Dec 2022, applying Dillibazar Ltd.'s matrix, Satdobato Ltd has moved into the 5% bracket,
because it has exhausted its 60-day credit limit. Despite assurances that Dillibazar Ltd will receive
payment, the company should still increase its credit loss allowance to reflect the increased credit
risk. Dillibazar Ltd will therefore record the following entries on 31 Dec 2022:
Dr. Expected credit losses Rs. 20,000 **
Cr. Allowance for receivables Rs. 20,000
**Being expected credit loss: (Rs. 500,000*5% – Rs. 5,000)

Q. No. 6 NFRS 9/NFRS 13/ NAS 32


The fair value of an asset is the price that would be received to sell an asset or paid to transfer a
liability in an orderly transaction between market participants at the measurement date (NFRS 13
Fair value measurement). NFRS 13 states that valuation techniques must be those which are
appropriate and for which sufficient data are available. Entities should maximize the use of
relevant observable inputs and minimize the use of unobservable inputs. The standard establishes
a three-level hierarchy for the inputs that valuation techniques use to measure fair value.
Level 1 Quoted prices (unadjusted) in active markets for identical assets or liabilities that the
reporting entity can access at the measurement date
Level 2 Inputs other than quoted prices included within Level 1 that are observable for the asset
or liability, either directly or indirectly, e.g. quoted prices for similar assets in active markets or
for identical or similar assets in non-active markets or use of quoted interest rates for valuation
purposes
Level 3 Unobservable inputs for the asset or liability, i.e. using the entity's own assumptions about
market exit value
The standard believes that fair value is the most appropriate measure for most financial
instruments because it is the most relevant. However, it may be less reliable. There is more scope
for manipulation. Particular difficulties arise where quoted prices are unavailable. If this is the case
– and it frequently is – there is more reliance on estimates.
Not all markets are liquid and transparent. Where a market is illiquid, it is particularly difficult to
apply fair value measurement, because the information will not be available. In addition, not all
markets are stable; some are volatile. Fair valuing gives a measurement at a particular point in
time, but in a volatile market this measure may not apply long term. It needs to be considered
whether an asset is to be actively traded or held for the long term. Disclosure is important in helping
to deal with some of the problems of fair value, particularly as it provides an indicator of a
company's risk profile.
Convertible bond
Some financial instruments contain both a liability and an equity element. In such cases, NAS 32
requires the component parts of the instrument to be classified separately, according to the

22
substance of the contractual arrangement and the definitions of a financial liability and an equity
instrument.
One of the most common types of compound instrument, as here, is convertible debt. This creates
a primary financial liability of the issuer and grants an option to the holder of the instrument to
convert it into an equity instrument (usually ordinary shares) of the issuer. This is the economic
equivalent of the issue of conventional debt plus a warrant to acquire shares in the future.
Although in theory there are several possible ways of calculating the split, the following method
is recommended:
(1) Calculate the value for the liability component.
(2) Deduct this from the instrument as a whole to leave a residual value for the equity component.
The reasoning behind this approach is that an entity's equity is its residual interest in its assets
amount after deducting all its liabilities.
The sum of the carrying amounts assigned to liability and equity will always be equal to the
carrying amount that would be ascribed to the instrument as a whole.
The equity component is not re-measured. However, the liability component is measured at
amortized cost using an effective interest rate (here 9.38%).
It is important to note that the issue costs (here $m) are allocated in proportion to the value of the
liability and equity components when the initial split is calculated.
Step 1: Calculate Liability Element
* A 9% discount rate is used, which is the market rate for similar bonds without the conversion
rights:
Present value of interest at end of: Rs'000
Year 1 (31 Dec 20X6) (Rs.10m × 6%) × 0.9174 550
Year 2 (31 Dec 20X7) (Rs.10m × 6%) × 0.8417 505
Year 3 (31 Dec 20X8) (Rs.10m × (Rs.10m × 6%)) × 0.7722 8,185
Total liability component 9,241
Total equity element 759
Proceeds of issue 10,000

Step 2: Allocate Issue Costs


Liability Equity Total
Details Rs'000 Rs'000 Rs'000
Proceeds 9,241 759 10,000
Issue cost (92) (8) (100)
Total 9,148 752 9,900
The Double entry is:
Rs'000 Rs'000
Dr. Cash 10,000 -

23
Cr. Liability - 9,241
Cr. Equity - 759

Rs'000 Rs'000
Dr. Liability 92 -
Dr. Equity 8 -
Cr. Cash - 100

Step 3: Re-measure liability using effective interest rate


Details Rs'000
Cash – 1.1.20X6 (net of issue costs per Step 2) 9,148
Effective interest to 31.12.20X6 (9.38% * 9,148) 858
Coupon paid (6% * Rs.10m) (600)
At 1.1.20X7 9,406
Effective interest to 31.12.20X7 (9.38% * 9,406) 882
Coupon paid (6% * Rs.10m) (600)
At 1.1.20X8 9,689
Effective interest to 31.12.20X8 (9.38% * 9,689) 911
Coupon paid (6% * Rs.10m) (600)
At 31.12.20X8 10,000
Step 4: Conversion of bond
On conversion of the bond on 31 Dec 20X8, Pawan will issue 2.5 million ordinary shares. The
consideration for these shares will be the original equity component (net of its share of issue costs)
together with the balance on the liability.
Details Rs'000
Share capital – 2.5 million at $1 2,500
Share premium 8,251
Equity and liability components (10,000 + 759 – 8) 10,751

Q. No. 7 NFRS 13 Fair Value Measurement


a. For the purpose of measuring the fair value of an asset or liability, NFRS 13 requires the use
of a measurement technique, which maximizes the use of relevant observable inputs, while
considering the complete set of characteristics of the asset or liability.
NFRS 13 defines “observable” inputs as “inputs that are developed using market data, such as
publicly available information about the actual events or transactions, and that reflect the
assumptions that market participants would use when pricing the asset or liability.”
The standard gives some examples of markets in which observable inputs can be identified:
24
Exchange markets: The prices observed directly derive from the transactions and are generally
representative of fair value.
Dealer Markets: Dealers are agents prepared to engage directly in buying and selling.
Therefore, they provide a price at which they are willing to buy and a price at which they are
willing to sell.
Brokered Markets: Brokers are intermediaries who do not trade on their own account. Brokers
match buyers and sellers to make the transaction possible. It is sometimes possible to obtain
prices from completed transactions.
Principal-to-Principal Markets: Transactions are negotiated independently with no
intermediaries and may therefore be used as observable inputs. However, limited information
about these transactions is available publicly.
b. NFRS 13 attributes more importance to the quality of the inputs than the measurement model,
which is chosen with a view to maximizing the use of observable inputs.
NFRS 13 does not expressively require a “Standard” Model to be used in order for the
measurement to be considered as level 1 or 2.
However, it should be noted that the standard does require the use of technique which is:
• Appropriate; and
• Likely to be used by a market participant in estimating the price.
Hence, these two requirements mean that in practice an entity will use standard approaches
whenever they are available.

Q. No. 8 NFRS 15 Revenue from Contracts with Customers


NFRS 15 Revenue from contracts with Customers regards a transaction such as this as being made
up of two separately identifiable performance obligations
– The supply of the equipment and the supply of the servicing agreement.
The total revenue of Rs. 200,000 would need to be allocated between the two separate
performance obligations in proportion to their stand-alone selling prices. The selling price of the
machine is Rs. 190,000 and the normal selling price of the supply of services is Rs.10, 000 (2 ×
Rs. 5,000). The total stand-alone selling prices therefore total Rs.200, 000.
Revenue of Rs. 190,000 (Rs. 200,000 × 190,000/200,000) is allocated to the supply of the
equipment. The balance of revenue of Rs.10, 000 is allocated to the supply of services. On 01 Jan
2022, Kabindra Ltd would recognize revenue from the supply of the machine of Rs.190, 000. On
the same date Kabindra Ltd would recognize a receivable of Rs. 200,000. The balance of Rs 10,000
would initially be recognized as deferred income.
On 20 Jan 2022, the receivable of Rs. 200,000 would be de-recognized when the payment was
received from the Rabindra. In the year ended 32 December 2022, service revenue of Rs.5, 000
(Rs. 10,000 X 1/2) can be recognized. The closing balance of deferred income on 32 December
2022 will be Rs. 5,000 (Rs. 10,000- Rs.5, 000) which will be shown as a current liability as this
refers to service revenue to be recognized in the year ended 32 December 2023.

25
Q. No. 9 NFRS 16 Leases
When the payments are made at the end of the year:
At the commencement date, a lessee shall measure the lease liability at the present value of the
lease payments that are not paid at that date. The lease payments shall be discounted using the
interest rate implicit in the lease, if that rate can be readily determined. If that rate cannot be readily
determined, the lessee shall use the lessee’s incremental borrowing rate. (Para 26 of NFRS 16:
Leases) Hence, Lumbini Ltd should discount the cash flows associated with the lease with the
incremental borrowing rate i.e. 8.5%.
The carrying amount of the right-of-use-asset at the commencement date would be Rs.
13,42,269.61 (Working note 1) (Rs. 13,12,269.61 + Rs.30,000 initial direct costs) and
consequently the annual depreciation charge will be Rs. 13,42,26.96 (Rs. 13,42,269.61 x 1/10).
The lease liability will be measured using amortized cost principles.
Working Note 1.
Year Cash flows PVIF @8.5% PV
1 200,000.00 0.9217 184,331.80
2 200,000.00 0.8495 169,891.06
3 200,000.00 0.7829 156,581.62
4 200,000.00 0.7216 144,314.86
5 200,000.00 0.6650 133,009.08
6 200,000.00 0.6129 122,589.02
7 200,000.00 0.5649 112,985.27
8 200,000.00 0.5207 104,133.89
9 200,000.00 0.4799 95,975.94
10 200,000.00 0.4423 88,457.08
Present value 1,312,269.61

Working Note 2:
Year Balance b/fwd Finance cost (8.5%) Payment Balance c/fwd
1 13,42,269.61 114,092.92 (200,000) 12,56,362.53
2 12,56,362.53 106,790.81 (200,000) 11,63,153.34
At the end of year one, the carrying amount of the right-of-use-asset will be 12,08,042.65 (Rs.
13,42,269.61 less Rs. 13,42,26.96 depreciation).
The interest cost of Rs. 114,092.92 will be taken to the statement of profit or loss as a finance cost.
The total lease liability at the end of year one will be Rs. 12,56,362.53. As the lease is being paid
off over 10 years, some of this liability will be paid off within a year and should therefore be
classed as a current liability.
Hence Non-Current liability will be Rs. 11,63,153.34 and Current Liability will be Rs. 93,209.19
(Rs. 12,56,362.53 – Rs. 11,63,153.34)

26
Where payment are made in advance:
Where payment are made in advance, the carrying amount of the right-of-use-asset at the
commencement date would be Rs. 1,453,812.53 (Working note 3) (Rs. 1,423,812.53 + Rs.30,000
initial direct costs) and consequently the annual depreciation charge will be Rs. 1,453,81.25 (Rs.
1,453,812.53 x 1/10). At the end of year one, the carrying amount of the right-of-use-asset will be
Rs.13,08,431.28 (Rs. 1,453,812.53 less Rs. 1,453,81.25 depreciation).
The initial lease payment of Rs. 200,000 would actually be included as part of the cost of the right-
of-use asset rather than the lease liability. This is because the cost of the right-of-use asset should
include the initial measurement of the lease liability plus any lease payments made at or before the
commencement date.
The total lease liability at the end of year one will be Rs. 13,27,836.59.(Working note 4) Where
payments are made in advance, the non-current liability would be Rs. 12,23,702.70 and current
liability would be Rs. 104,133.89 (Rs. 13,27,836.59 - 12,23,702.70)
Working Note 3.
Year Cash flows PVIF @8.5% PV
1 200,000.00 1.0000 200,000.00
2 200,000.00 0.9217 184,331.80
3 200,000.00 0.8495 169,891.06
4 200,000.00 0.7829 156,581.62
5 200,000.00 0.7216 144,314.86
6 200,000.00 0.6650 133,009.08
7 200,000.00 0.6129 122,589.02
8 200,000.00 0.5649 112,985.27
9 200,000.00 0.5207 104,133.89
10 200,000.00 0.4799 95,975.94
Present value 1,423,812.53
Working Note 4.
Year Balance b/fwd Payment Subtotal Finance cost (8.5%) Balance c/fwd
1 1,423,812.53 (200,000) 12,23,812.53 104,024.06 13,27,836.59
2 13,27,836.59 (200,000) 11,27,836.59 95,866.11 12,23,702.70

Q. No. 10 NAS 2 Inventories


As per NAS 2, inventories shall be measured at the lower of cost and net realizable value. The cost
of inventories shall comprise all costs of purchase, costs of conversion and other costs incurred in
bringing the inventories to their present location and condition. The cost of purchase of inventories
comprise the purchase price, import duties and other taxes (other than those subsequently
recoverable by the entity from the taxing authorities,), and transport, handling and other costs
directly attributable to the acquisition of finished goods, materials and services. Trade discounts,
rebates and other similar items are deducted in determining the costs of purchase.

27
Computation table for cost per kg of goods purchased:
Particulars Rs. Per kg
Normal price per kg of goods 500
Less: Trade discount (10)
Cost to the Company 490
Add: Transportation cost to the Company 20
Cost of purchase per kg** 510
** As the company is registered with VAT, the amount of VAT paid at the time of purchase will
be netted off against the VAT amount that will be collected from the buyer and hence the VAT
paid will not form part of cost of goods.
Hence, the inventory of goods should be valued at cost i.e. Rs. 510 per kg.

Q. No. 11 NAS 12 Income Taxes


The tax base of the assets is the difference between the cost and the accumulated depreciation for
tax purpose i.e. Rs 50 Lakhs – 35 Lakhs = Rs 15 Lakhs. Hence, there is a taxable temporary
difference of Rs 45 Lakhs i.e. the difference between the revalued amount and the tax base (Rs 60
Lakhs – 15 Lakhs). The tax base of the assets is less than the carrying amount and hence will result
in Deferred Tax Liability. The Deferred Tax Liability in the two given case will be calculated as
below:
i) If the entity expects to recover the carrying amount by using the assets in the business, it must
generate taxable income of Rs 45 Lakhs. The taxable temporary difference of Rs 45 Lakhs
will be subject to tax. On this basis there will be Deferred Tax Liability of Rs 13.5 Lakhs i.e.
30% of Rs 45 Lakhs.
ii) If the entity expects to recover the carrying amount by selling the assets immediately the
proceeds from the sale of the assets will be equivalent to the revalued amount i.e. Rs 60 Lakhs.
In the given case, the profit on tax base will be Rs 45 Lakhs i.e. Rs 60 Lakhs – Rs 15 Lakhs.
The profit to the extent of accumulated depreciation for tax purpose i.e. Rs 35 Lakhs will be
taxed at 30% and the balance will not be taxable.
Hence, Deferred Tax Liability in the given case will be Rs 10.5 Lakhs i.e. 30% of Rs 35 Lakhs.

Q. No. 12 NAS 19 Employee Benefits


Computation of Expected Return on Plan Assets
Particulars Amount Rs.
Return on Rs. 10 Lakhs for FY 2078/79 @ 10.25% 102,500
Add: Return on Rs. 3 Lakhs for 6 months @ 10% p.a. 15,000
[(490,000-190,000)*10%*6/12]
Expected Return on Plan Assets 117,500

Computation of Actual Return on Plan Assets


Particulars Amount Rs.
Fair Value of Plan Assets as at Ashadh end, 2079 1,500,000
Less: Fair Value of Plan Assets as at Shrawan 1, 2078 (1,000,000)

28
Less: Contribution received during the year (490,000)
Add: Benefits paid during the year 190,000
Actual Return on Plan Assets 200,000

Computation of Net Actuarial Gain


Particulars Amount Rs.
Actual Return on Plan Assets 200,000
Less: Expected Return on Plan Assets (117,500)
Actuarial gain on Plan Assets 82,500
Less: Actuarial loss on Defined Benefit Obligations (6,000)
Net Actuarial gain to be recognized in 'Other Comprehensive
Income' 76,500

Q. No. 13 NAS 23 Borrowing Costs


1. Calculation of amount eligible for capitalization of borrowing cost
Cost (Rs.) Date Months Eligible cost
20,000.00 1-Jan-12 12 20,000.00
157,000.00 1-Feb-12 11 143,916.67
275,000.00 1-Oct-12 3 68,750.00
76,000.00 1-Dec-12 1 6,333.33
Total Cost Rs. 239,000.00

2. Calculation of weighted average rate of interest on general borrowings


General Borrowings Rate of Interest (p.a.) Total interest (Rs.)
250,000.00 8% 20,000.00
300,000.00 10% 30,000.00
550,000.00 9.09% 50,000.00
Weighted average interest rate = 9.09%
3. Calculation of interest eligible for capitalization
Specific borrowing (175000*8%) Rs. 14,000.00
General borrowings (239000*9.09%) Rs. 21,727.27
Total Rs. 35,727.27

4. Total amount to be capitalized for building and total cost of the buildings:
Cost of building (20000+157000+275000+76000) = Rs. 528,000.00
Amount of interest = Rs. 35,727.27
Total cost of both buildings = Rs. 563,727.27

29
Q. No. 14 NAS 24 Related Party Disclosure
a. A finance lease is a lease where the risks and rewards of ownership are transferred from lessor
to lessee. This lease between France Ltd and Brazil Ltd is only for a fraction of the asset’s
remaining useful life and the lease payments are insignificant. The lease is therefore an operating
lease. France Ltd should recognize lease income on a straight line basis over the lease term.
Therefore, Rs. 12,500 (Rs. 25,000 × 6/12) should be recognized in the current year’s statement of
profit or loss, as well as a corresponding entry to accrued income on the statement of financial
position.
A related party transaction is defined by NAS 24 as a transfer of resources, services or obligations
between a reporting entity and a related party. An entity is related to the reporting entity if they
are under joint control. An entity must disclose if it has entered into any transactions with a related
party.
France Ltd and Brazil Ltd are under joint control of Argentina Ltd, so this means that they are
related parties. Disclosure is required of all transactions between France Ltd and Brazil Ltd during
the financial period.
France Ltd must disclose details of the leasing transaction and the income of Rs. 12,500 from
Brazil Ltd during the year. Disclosures that related party transactions were made on terms
equivalent to an arm’s length transaction can only be made if they can be substantiated. The lease
rentals are only 10% of normal market rate meaning that this disclosure cannot be made.

b. NAS 24 Related Party Disclosures requires an entity to identify and disclose facts related to
existence of related parties and transactions and outstanding balances with them in its financial
statements in order to draw the attention of users of financial statements on the possible impact
that may have on the financial performance and position of the entity due to such relationship.
The standard further states that a person or a close family member of such person is related to a
reporting entity if that person:
➢ has control or joint control over the reporting entity;
➢ has significant influence over the reporting entity; or
➢ is a member of the key management personnel of the reporting entity or of a parent of
the reporting entity.
In case of FinTax, the finance director is a related party, as he owns more than half of the voting
power (60%). In the absence of evidence to the contrary, he controls FinTax and is a member of
the key management personnel.
The sales director is also a related party of FinTax as she is a member of the key management
personnel and a close family member (spouse) of a finance director. Their son being a close family
member also meets the criteria for being a related party of the company.
The operation director is also a related party as he owns more than 20% of the voting right in the
company. In the absence of evidence to the contrary, the operation director has significant
influence over FinTax and is a member of key management personnel.
Further, an entity is related to a reporting entity if the entity is controlled or jointly controlled by a
person identified as a related party. Considering this criterion, it can be concluded that InvoMax
Ltd. is a related party of FinTax.

30
In the absence of evidence to the contrary, the third owner of the company is not a related party.
The person is a passive investor who does not appear to exert significant influence over the FinTax.
The loan from the bank, which has been secured through the private property of finance director,
shall be disclosed in the Financial Statements of FinTax, by detailing the facts in Notes that the
loan has been obtained by keeping the personal property of finance director as a collateral.

Q. No. 15 NAS 33 Earnings per Share


Computation table for Basic Earnings per share
Particulars Year ended as on
31/3/2078 32/3/2079
EPS for the year 2077/78 as originally reported:
[Net Profit of the year attributable to equity shareholders / Weighted Rs. 14.33
average number of equity shares outstanding during the year]
(Rs. 430,000,000/30,000,000)

EPS for the year 2077/78 restated for right issue:


[Rs. 430,000,000/{30,000,000 X 1.03(as per working note No. 2)}] Rs. 13.91
EPS for the year 2078/79 including effects of right issue:
Rs. 630,000,000
(30,000,000 X 1.03X 3/12) + (37,500,000 X 9/12)
Rs. 630,000,000 Rs. 17.57
35,850,000 (approx..)

Working Notes
1. Computation of theoretical ex-rights fair value per share

= (Rs. 300 X 30,000,000 shares) + (Rs. 250 X 7,500,000 shares)


30,000,000 shares + 7,500,000 shares
= Rs. 10,875,000,000 / 37,500,000
= Rs. 290
2. Computation of adjustment factors
= Fair value per share prior to exercise of right / Theoretical ex-right value per share (as per
Working Note No. 1)
= Rs. 300/ Rs.290
= 1.03 (approx.)

31
Q. No. 16 NAS 36 Impairment of Assets
a. NAS 36 Impairment of Assets requires that assets be carried at no more than their recoverable
amount. Therefore, entities should test all assets within the scope of the standard if there is
potential impairment when indicators of impairment exist. If fair value less cost to sell or value
in use is more than carrying amount, the asset is not impaired. It further says that in measuring
value in use, the discount rate used should be the pre-tax rate which reflects current market
assessments of the time value of money and the risks specific to the asset. The discount rate
should not reflect risks which future cash flows have been adjusted and should equal the rate of
return which investors would require if they were to choose an investment which would
generate cash flows equivalent to those expected from the asset. Therefore, pre-tax cash flows
and pre-tax discount rates should be used to calculate value in use. Discounting post-tax cash
flows with a post-tax discount rate could give the same result in any entity were it not for any
temporary difference and/or tax losses which might exist.
Rs. in Lakhs
Financial Year Pre-tax Cash flows DF@12% Discounted cash flows
FY 2079/80 16 0.8929 14.29
FY 2080/81 14 0.7972 11.16
FY 2081/82 10 0.7118 7.12
FY 2082/83 6 0.6355 3.81
FY 2083/84 26 0.5674 14.75
Value in Use 51.13
The SBU is impaired by the amount by which its carrying amount exceeds its recoverable
amount which is the higher of an asset’s fair value less cost to sell and its value in use. The fair
value less cost to sell of the SBU is:
Rs. in Lakhs
Assets Fair Value Cost to sell FV less Cost to sell
Property, Plant and Equipment 20 2 18
Other Assets 34 6 28
SBU's Fair value less cost to sell 46
Therefore, recoverable amount of the SBU is Rs. 51.13 Lakhs.
Impairment loss of SBU = Carrying amount – Recoverable amount
= 64 – 51.13
= Rs. 12.87 lakhs
Now, Grow Ltd. has to allocate this amount, first to the goodwill and then to other remaining
assets on pro rata basis based on their carrying amount provided that their individual value
should not be less than fair value less cost to sell.
Allocation of Impairment loss:
Rs. in Lakhs
Allocation of Value after
Assets Carrying Amount Impairment Loss Impairment
Goodwill 6 6 0.00
Property, Plant and Equipment 20 2.37 17.63
Other Assets 38 4.50 33.50
32
Total 64 12.87 51.13
Since carrying amount of an asset cannot be reduced below its fair value less cost to sell,
allocation of impairment loss as shown in above table shall be adjusted for property, plant and
equipment and other assets by Rs. 0.37 lakhs (Rs. 18 lakhs – Rs. 17.63 Lakhs). So, impairment
loss on property, plant and equipment should be reduced by Rs. 0.37 lakhs and that of other
assets should be increased by such amount. Hence, final allocation of impairment loss and
carrying amount of assets shall be as under:
Rs. in Lakhs
Allocation of Value after
Assets Carrying Amount Impairment Loss Impairment
Goodwill 6 6 0.00
Property, Plant and Equipment 20 2.00 18.00
Other Assets 38 4.87 33.13
Total 64 12.87 51.13
The amount of impairment loss Rs. 12.87 lakhs shall be shown under Statement of Profit or loss
of the Grow Ltd.

b. NAS 36 prescribes the procedures that an entity should apply to ensure that its assets are carried
at no more than their recoverable amount. An asset is carried at more than its recoverable amount
if its carrying amount exceeds the amount to be recovered through use or sale of the asset. If this
is the case, the asset is described as impaired and the Standard requires the entity to recognize an
impairment loss. The Standard also specifies when an entity should reverse an impairment loss
and prescribes disclosures.
To ascertain whether the asset is impaired, the company should firstly calculate the value in use of
the asset in order to determine the higher amount among the value in use and fair value less cost
to sell.
Calculation of value in use of the first machinery:
Amount in Lakhs
Year Future cash flows PV @ 10% Discounted Cash flows
1 9.00 0.9091 8.18
2 9.27 0.8264 7.66
3 9.54 0.7513 7.17
Total 23.02
The impairment loss would be calculated by comparing the carrying value (Rs. 30 lakhs) with the
higher of value in use (Rs. 23.02 lakhs) and net selling price (Rs. 25 Lakhs)
Thus the impairment loss would be (Rs. 30 Lakhs - Rs. 25 Lakhs = Rs. 5 lakhs) and the new
carrying value would be Rs. 25 Lakhs.
For the second machinery:
Amount in Lakhs
Amount
Particulars (Rs.)
Cost of the machine as on 01.04.2075 18

33
Depreciation for 3 years i.e. up to 01.04.2078 (18/5)*3 Years 10.8
Carrying amount as on 01.04.2078 7.2
Add: Upward revaluation (Credited to revaluation reserve) (12.5-7.2) 5.3
Revaluated amount as on 01.04.2078 12.5
Less: Depreciation for 1 year (12.5/2)*1 6.25
Carrying amount as on Ashadh End 2079 6.25
Less: Recoverable amount 2.6
Impairment loss 3.65
Less: Balance in revaluation reserve as on Ashadh End 2079
Balance in revaluation reserve as on 01.04.2078 less amount equal to
additional depreciation transferred from revaluation reserve for 2078.79 i.e. -2.65
[5.3 - (5.3/2)*1] 2.65
Impairment loss to be debited to profit and loss account 1

Q. No. 17 NAS 38 Intangible Assets


NAS 38 Intangible Assets states that the cost less residual value of an intangible asset with a finite
useful life should be amortized on a systematic basis over that useful life, that the amortization
method should reflect the pattern of benefits and that it should be reviewed at least annually.
The amortization method should be reviewed at least annually and, if the pattern of consumption
of benefits has changed, the amortization method should be changed prospectively as a change in
estimate under NAS 8 Accounting Policies, changes in Accounting Estimates and Errors.
Expected future reductions in sales could be indicative of a higher rate of consumption of future
economic benefits embodied in an asset. Hence, the trademark would be amortized over 3 year
period till Ashadh 2081.
Further, NAS 36 Impairment of Assets states that an entity should assess at the end of each
reporting period whether there is any indication that an asset may be impaired. If any such
indication exists, the entity should estimate the recoverable amount of the asset. Irrespective of
whether there is any indication of impairment, an entity shall also test an intangible asset with an
indefinite useful life or an intangible asset not yet available for use for impairment annually by
comparing its carrying amount with its recoverable amount. This impairment test may be
performed at any time during the reporting period, provided that it should be performed same
time every year. Hence, Jaftra should test the trademark for impairment.
Following adjustments need to be made in the books of account Jaftra Pvt. Ltd. for the period
ended on 32 Ashadh, 2079:
A. Amortization expenses
Amortization Expenses Dr. ……………………….. 1,000,000
To Intangible asset (Trademark) …………………………. 1,000,000
(Being recognizing amortization of trademark during the period)
B. Impairment Loss
Impairment Loss on Trademark Dr. …………………………… 1,000,000
To Intangible asset (Trademark) …………………………………. 1,000,000
(Being impairment loss recognized with respect to trademark)

34
Working Notes:
Cost of trademark (Rs.) 5,000,000
Expected useful life (Years) 10
Amortization Expenses per year (Rs.) 500,000

Accumulated amortization till 1 Shrawan 2078


(For 4 years) (Rs.) 2,000,000

Carrying amount of trademark as at 1 Shrawan 2078 3,000,000


[50 Lakhs-20Lakhs]
The useful life of the trademark is reduced to 3 years and therefor it has to be amortized over that
period.
Amortization Expenses for the period (Rs.) 1,000,000
[Rs. 3,000,000/3yrs]

Carrying Amount as at 32 Ashadh, 2079 2,000,000


[Rs. 30 Lakhs- Rs. 10 Lakhs]

Recoverable Amount (Rs.) 1,000,000


Impairment Loss (Rs.) 1,000,000
[Carrying Amount-Recoverable Amount]

Q. No. 18 Value Added Statement


Statement showing Value Added and amount of Bonus paid to Employees

Rs. '000
Year 2017 2018 2019 2020 2021
Employee Cost 975 1,140 1,260 1,476 1,680
Value Added 2,280 2,700 3,150 3,600 4,200
Percentage 42.76% 42.22% 40.00% 41.00% 40.00%
Target Index = 40%
Value Added Statement for 2022
Rs. "000
Particulars Amount Amount
Sales 10,950
Less: Cost of bought in goods and
services
Cost of Materials 3,750
Production Expenses 1,050
Administrative Expenses 450
Sales Expenses 300 (5,550)
Value Added 5,400
35
Employee Cost
Wages 1,050
Production staff salaries 300
Administrative salaries 450
Sales Department's Salaries 90
Total Employee Cost 1,890

Bonus Calculation:
Employee cost as per target index 2,160
[5400*40%]
Actual Cost 1,890
Saving in employee cost 270
Employee Share 180
[270*2/3]
i.e. Rs. 180,000.00

Q. No. 19 Public Financial Management


PFM refers to the set of laws, rules, systems and processes used by sovereign nations (and sub-
national governments), to mobilise revenue, allocate public funds, undertake public spending,
account for funds and audit results. It encompasses a broader set of functions than financial
management and is commonly conceived as a cycle of six phases, beginning with policy design
and ending with external audit and evaluation. A large number of actors engage in this “PFM
cycle” to ensure it operates effectively and transparently, whilst preserving accountability.
The PFM and budgetary policies of the Government of Nepal during the Nineties were directed
towards economic liberalization, privatization, poverty reduction and decentralization. Policies
and programs of the budget were mainly concerned with agriculture, modernization, employment
promotion, women's empowerment, financial sector reform, government expenditure
management, tax reform, good governance, social service and the development of basic and
physical infrastructure. PFM system of Nepal, like most developing countries, continued to be
dominated by the traditional objectives of control and accountability rather than a concern for
allocating limited public sector resources to well defined programs and projects that were intended
to serve a set of national objectives.
The extension of the budget coverage involved a combination of formal and informal incorporation
of expenditure activities. The other formal extension involved the incorporation of foreign
assistance programs, which were previously outside the budget. Planning the allocation of scarce
resources was not given due priority. The pattern of government expenditure followed more or
less the uniform course till the 1990s. Public expenditure and revenue both increased; but the
expenditure increase trend was greater than the revenue. The inadequate mobilization of domestic
resources through government revenue resulted in a serious problem of widening resource gap in
Nepal. Foreign aid was the main source of development financing and deficit financing continued
to increase. Planning, budgeting, and implementation had inherent problems such as lack of
capacity, co-ordination and monitoring. In spite of a number of initiatives taken, one of the main
problems of Nepal has been the lack of proper domestic resource mobilization.

36
Several factors have contributed in varying degrees to the lack of effectiveness of public spending
in Nepal. The institutional factors played major role in the over-programming (having too many
programs in scarce resources) of the budget, its lack of focus and prioritization and implementation
problems. The lacks of ownership of projects/ programs at various levels and the absence of
accountability, also undermined the quality and effectiveness of public spending. Managing the
national budget became increasingly difficult for Government of Nepal to further their objectives
of poverty alleviation.
Public Expenditure Management is one of the key activities of any government in the world. There
is a growing concern to make PFM system predictable, transparent and accountable anywhere in
the world. PFM in general incorporates a credible planning system, management of government
revenues, budget execution, expenditure management, debt management, reimbursement,
procurement and other important aspects of financial management such as accounting, recording,
financial reporting and auditing and external scrutiny of the financial transactions. Improving
governance and enhancing accountability are considered as the critical agenda of the Government
of Nepal in the endeavour of institutionalizing good governance practices in the country. Hence,
strengthening Public Financial Management has been accepted as one of the key elements of the
GoN’s strategy for improving the overall governance, optimizing outputs from public resources
and ensuring inclusive and broad-based development.
Poor planning, ever increasing indiscipline in budget execution, ineffective expenditure control
and lack of transparency mainly in public procurement pose significant fiduciary risks to almost
all development projects both at centre and local level. The GoN’s recent initiatives such as
Financial Administration Reform Program, Strengthening PFM Project, Government Financial
Statistics (GFS) based new codes and classification of revenues and expenditures, implementation
of Treasury Single Account (TSA) system, strategy to implement International Accounting and
Reporting Standards (NAPSAS), Public Expenditure and Financial Accountability (PEFA)
initiative and other capacity building programs for PFM have resulted some positive impacts in
strengthening PFM system in general and financial good governance in particular in Nepal.

Q. No. 20 Short notes:


a. Opportunity cost method is one of the economic value models used for measurement and
valuation of human assets. As per this model, opportunity cost is the value of an employee in
his/her alternative use. This opportunity cost is used as a basis for estimating the value of
Human Resources.
Opportunity cost value may be established by competitive bidding within the firm so that in
effect, managers must bid for any scarce employee. A human asset will have a value only if it
is a scarce resource, that is, when its employment in one division denies it to another division.
This method excludes employees of the type of which can be readily hired from outside the
firm. Also, it is in very rare cases that managers would like to bid for an employee.
b. Sustainability reporting is a form of non-financial reporting through which companies convey
their progress toward goals on a variety of sustainability parameters, including environmental,
social and governance metrics, as well as risks and impacts they may face, at the moment or in
the future.
In fact, sustainability reporting gives an overview of a company’s economic, environmental
and social impacts. By considering this, an organization is able to measure, understand and
assess its performances in the larger context. As a result, it becomes able to set new
goals and manage changes needed, in order to be integrated in a new sustainable global
37
economy. All in all, sustainability report helps to enhance the trustworthiness of a company
among its customers, employees and other stakeholders, including investors.

38
Revision Test Paper (RTP), December 2023 CAP III – Group I

Paper 1 - Advanced Financial Reporting

The Institute of Chartered Accountants of Nepal 3


Revision Test Paper (RTP), December 2023 CAP III – Group I

Consolidated Financial Statements


1. X Limited is a company carrying on the business of dairy products and is having a subsidiary
Y Limited. Balance Sheets of both the companies as at 16 July 2023 are as under:

Particulars Notes X Limited (Rs.) Y Limited (Rs.)


I. Equity and Liabilities:
1. Shareholder's Funds:
a. Share Capital 1 12,500,000 2,900,000
b. Reserves and Surplus 2 2,562,500 1,625,000
2. Current Liabilities
a. Trade Payables 3 2,415,000 1,592,500
Total 17,477,500 6,117,500
II. Assets:
1. Non-Current Assets
a. Property, Plant and
Equipment 10,850,000 3,125,000
b. Investments 4 2,550,000 -
2. Current Assets:
a. Inventories 2,400,000 1,596,000
b. Trade Receivables 5 1,240,000 1,320,000
c. Cash and Cash Equivalents 437,500 76,500
Total 17,477,500 6,117,500

Notes to Accounts:
Note X Limited Y Limited
No. (Rs.) (Rs.)
1 Share Capital
Authorized, Issued, Subscribed and Paid-up:
125,000 Equity Shares of Rs. 100 each 12,500,000
29,000 Equity Shares of Rs. 100 each 2,900,000
2 Reserve and Surplus
General Reserve 1,000,000 600,000
Profit and Loss Account 1,562,500 1,025,000
2,562,500 1,625,000
3 Trade Payables
Trade Payables 2,275,000 1,177,500
Bills Payables 140,000 415,000
2,415,000 1,592,500
4 Investments (20,300 Shares in Y Limited) 2,550,000
5 Trade Receivables
Trade Receivables 900,000 820,000
Bills Receivables 340,000 500,000
1,240,000 1,320,000

The Institute of Chartered Accountants of Nepal 4


Revision Test Paper (RTP), December 2023 CAP III – Group I

X Limited has provided the following information:

1. X Limited has acquired the shares in Y Limited in two lots on two different dates during
the financial year 2022/23.
The relevant information at the time of acquisition of shares was as under:
No. of shares acquired Balance in General Reserve Balance in Profit or Loss
a/c
st
1 Acquisition – 17,400 400,000 125,000
2nd Acquisition – 2,900 425,000 510,000

2. Bills receivable of X Ltd includes Rs.75,000 being acceptance from Y Ltd.


3. Both the companies have proposed dividend of 10% for the year ended on 15 July 2023
but it has not been provided in the books of accounts.
4. Y Limited's inventory includes stock of Rs.725,000 purchased from X Ltd. X Ltd. sells
goods at mark up of 25% on its costs.

You are required to prepare the Consolidated Balance Sheet of X Limited along with the Notes
to Accounts.

Preparation of Financial Statements


2. The trial balance of ABC Ltd, as at 31 Ashadh 2080 is provided below:

Rs. '000 Rs. '000


Revenue 320,700
Cost of sales 215,700
Closing inventories (31/03/2080) (Note i) 15,750
Operating expenses 33,600
Income from Investment Property 1,800
Finance costs 6,300
Land and building at valuation (Note iii) 94,500
Plant and equipment at cost (Note iii) 54,000
Investment property (Note iii) 24,000
Accumulated depreciation on plant and equipment 25,200
(01/04/2079)
Trade receivables 20,250
Bank 11,850
Trade payables 17,700
Ordinary shares @RS. 0.25 30,000
10% Redeemable preference shares @ Rs. 1.00 15,000
Deferred Tax (Note iv) 7,800
Revaluation surplus 31,500
Retained earnings (01/03/2080) 26,250
475,950 475,950

The following information is provided:

The Institute of Chartered Accountants of Nepal 5


Revision Test Paper (RTP), December 2023 CAP III – Group I

i) An inventory count at 31 Ashadh 2080 amounted to Rs. 15,750,000. This includes


damaged goods with a cost of Rs. 1,200,000. These will require remedial work costing Rs.
675,000 and could be sold for Rs.1,425,000.
ii) Finance cost is made up of the full year’s preference and ordinary dividends paid.
iii) Non-Current Assets
• Land and Building were revalued at Rs. 22,500,000 and Rs. 72,000,000 respectively on
1 Shrawan 2079, resulting in revaluation gain of Rs. 11,000,000 for the current year. At
that date, the remaining life of the building was 15 years. Depreciation is on straight line
basis. Ignore deferred tax implication.
• Depreciation on Plant and Equipment is at 12.5% on reducing balance basis.
• Investment Property: On 31 Asar 2080, a qualified surveyor valued the property at Rs.
20,250,000. ABC Ltd uses fair value model under NAS 40: Investment Property to
value its investment property.
• It is the policy of the company to charge depreciation on full year basis.
iv) The directors have estimated the provision for income tax for the year ended 31 Ashadh
2080 at Rs. 12,000,000. The deferred tax for the year ended 31 Ashadh 2080 is to be
adjusted so that the tax base of the company’s net assets is Rs. 18,000,000 less than the
carrying amount. Assume the rate of tax is 30%.
v) On 1 Baishakh 2080, ABC Ltd imported a piece of equipment from a Japanese supplier
for ¥ 1million and agreed to settle the bill in six months’ time. The relevant exchange rates
are provided below:

Date Exchange Rate

1 Baishakh 2080 ¥1:Rs. 6.20

31 Ashadh 2080 ¥1: Rs. 6.50

1 Kartik 2080 ¥1: Rs. 6.40

No entries have been made for the above transaction. Any exchange difference on
translation should be debited or credited to operating expenses.

Required: Prepare for ABC Ltd


a) Statement of Comprehensive Income for the year ended 31 Ashadh 2080.
b) Statement of Financial Position as at 31 Ashadh 2080.

NAS – 12 Income Taxes


3. An entity is finalising its financial statements for the year ended 31st Ashadh 2080. Before 31st
Ashadh 2080, the government announced that the tax rate was to be amended from 40 per cent
to 45 per cent of taxable profit from 30th Ashwin 2080.

The legislation to amend the tax rate has not yet been approved by the legislature. However,
the government has a significant majority and it is usual, in the tax jurisdiction concerned, to

The Institute of Chartered Accountants of Nepal 6


Revision Test Paper (RTP), December 2023 CAP III – Group I

regard an announcement of a change in the tax rate as having the substantive effect of actual
enactment (i.e. it is substantively enacted).

After performing the income tax calculations at the rate of 40 per cent, the entity has the
following deferred tax asset and deferred tax liability balances:

Deferred tax asset Rs.80,000

Deferred tax liability Rs. 60,000

Of the deferred tax asset balance, Rs. 28,000 related to a temporary difference. This deferred
tax asset had previously been recognized in OCI and accumulated in equity as a revaluation
surplus.

The entity reviewed the carrying amount of the asset in accordance with para 56 of NAS 12
and determined that it was probable that sufficient taxable profit to allow utilisation of the
deferred tax asset would be available in the future.

Show the revised amount of Deferred tax asset & Deferred tax liability and present the
necessary journal entries.

NAS 20 Accounting for Government Grants and Disclosure of Government Assistance

4. On 1 Shrawan 2079, Kalanki Ltd received a Government grant of Rs.8 million towards the
purchase of new plant with a gross cost of Rs.64 million. The plant has an estimated life of 10
years and is depreciated on a straight-line basis. One of the terms of the grant is that the sale
of the plant before 31 Ashadh 2083 would trigger a repayment on a sliding scale as follows:

Intended sale in the year ended: Repayment amount

31 Ashadh 2080 100%

31 Ashadh 2081 75%

31 Ashadh 2082 50%

31 Ashadh 2083 25%

The directors propose to credit the statement of profit or loss with Rs.2 million (Rs.8 million
@ 25%) being the amount of the grant they believe has been earned in the year ended 31
Ashadh 2080. Kalanki Ltd accounts for government grants as a separate item of deferred credit
in its statement of financial position. Kalanki Ltd has no intention of selling the plant before
the end of its useful economic life.

Required: Explain with computations, the appropriate accounting treatment of the above
transaction in accordance with NAS 20 Accounting for Government Grants and Disclosure of

The Institute of Chartered Accountants of Nepal 7


Revision Test Paper (RTP), December 2023 CAP III – Group I

Government Assistance in the financial statements of Kalanki Ltd for the year ended 31
Ashadh 2080.

NFRS 16 Leases

5. Kathmandu Ltd is a manufacturing company located in the Bagmati Province. It intends to


expand its production to take advantage of emerging economic activities in the new region. On
1 Shrawan 2079, the company entered into a lease agreement for a production equipment which
has a useful economic life of 8 years. The lease term is for four years and Kathmandu Ltd
agrees to pay annual rent of Rs.50,000 commencing on 1 Shrawan 2079 and annually
thereafter. The interest rate implicit in the lease is 7.5% and the lessee's incremental borrowing
rate is 10%. The present value of lease payments not yet paid on 1 Shrawan 2079 is Rs.130,026.
Kathmandu Ltd paid legal fees of Rs.1,000 to set up the lease.

Required: Prepare extracts for the Statement of Financial Position and Statement of Profit or
Loss for 2079/80 and 2080/81, showing how Kathmandu Ltd should account for this
transaction.

6. NFRS 15 Revenue from Contracts with Customers

Fashion Stores is a company in Nepal that manufactures clothes under its brand ‘Naya Look’
and exports them to clothes shop operators in India, the UK and Australia.

With a view of expanding its sales to other international retail markets, Fashion Stores recently
entered into an agreement with Unlimited Fashion, which operates a website to facilitate online
clothes sales and purchases in the international market.

Based on the above agreement:

• Fashion Stores displays its Naya Look brand products on the website operated by
Unlimited Fashion and the selling price will be decided by Fashion Stores.

• A 5% commission should be paid to Unlimited Fashion for each of the sales transactions.

• Once an order is placed on the website by a customer, it will be automatically


communicated to Fashion Stores and the shipments/delivery should be done through
Fashion Stores’s centralised warehouse.

• Upon the delivery of goods from the warehouse, the legal title of the goods is transferred
to Unlimited Fashion. However, the goods need to be insured by Fashion Stores.

• Customers will be given one week upon receipt of the goods to return any dissatisfied
goods to Unlimited Fashion. These goods need to be returned back to Fashion Stores by
Unlimited Fashion without a penalty.

The Institute of Chartered Accountants of Nepal 8


Revision Test Paper (RTP), December 2023 CAP III – Group I

Evaluate whether Unlimited Fashion is acting as an agent or principal in this agreement, based
on the guidance given in NFRS 15 Revenue from Contracts with Customers.

NAS 24 – Related Party Disclosure

7. Pokhara Ltd (Pokhara) holds 56% of the voting shares of Nepal Ltd. Shyam holds 78% voting
shares of Pokhara. However, Shyam does not hold any directorship in either company.
Pratik, Kalum and Menaka are directors at Nepal Ltd. Mennan, Geetha and Kalum are directors
at Pokhara Ltd. Nepal Ltd has a 52% stake in Solex (Pvt) Ltd (Solex). Kamal, Anusha and
Priya are directors at Solex.
Other relevant information is given below.
• Geetha is married to Gayan who is the head of marketing at Winwin Pvt Ltd.
• Nepal Ltd owns 50% of Phenix Pvt Ltd (Phenix). Balance 50% of Phenix is owned by Jaya.
The contractual arrangement between them specifies that at least 51% of the voting rights are
required to make decisions about the relevant activities of the arrangement.

Comment on each party mentioned in the scenario above to identify whether they are related
parties to Nepal Ltd.

NFRS 13 – Fair value measurement

8. ABC Ltd acquired an entity (Entity A) and is required to measure fair values of the following
item by applying the requirements of NFRS 13 – Fair value measurement
Investment in an equity instrument

Entity A holds an investment in an equity instrument, which is pledged as collateral to support


a debt obligation of the entity. Until the debt is fully settled, the entity cannot sell this
instrument. The debt is due for repayment one year after the date of acquisition of Entity A by
ABC Ltd. It is highly likely that the entity will settle the debt on the due date. The following
details at the acquisition date are provided to you. Market price of the equity instrument: Rs.
13 million Discount (%) on the market price for the restriction period of one year: 7%

NAS 36 Impairment of Assets

9. Phillips acquired 75% of Little Ltd (Little) by purchasing 7.5 million of its voting shares on 7
May 2018 when the fair value of a share was Rs. 250. The goodwill recognised on the
acquisition was Rs. 375 million after fair valuing the non-controlling interest (NCI). The
carrying value of the investment in Little and the goodwill have not been impaired up to 31
March 2022. However, due to a decline in market conditions Phillips decided to carry out an
impairment review on 31 March 2023 for the investment in Little.

The Institute of Chartered Accountants of Nepal 9


Revision Test Paper (RTP), December 2023 CAP III – Group I

The fair value of a share of Little as at 31 March 2023 was Rs. 225 and the estimated transaction
cost (at the time of disposal) was 1.5% of the fair value. The net asset value of Little as at the
year-end was Rs. 275 per share.

Little operates as a single cash-generating unit (CGU) and the following information relates to
the value-in-use assessment.

Year Pre-tax cash flows (Rs. In million)


2023/24 275
2024/25 325
2025/26 368
2026/27 412
2027/28 494
2027/28 onwards 1% terminal growth rate

Further, fair value of interest bearing borrowings and cash and cash equivalents balance of
Little as at the year-end were Rs. 275 million and Rs. 136 million respectively. These balances
have been considered in deriving net assets of Little. The pre-tax market rate of return for Little
was 15% per annum. The business income of the company is taxed at 24%.
Required:
Advise Phillips on the impairment assessments to be carried out in preparing the separate
financial statements and consolidated financial statements of Phillips as at 31 March 2023.
NAS 16: Property, Plant and Equipment and NAS 36: Impairment of Assets

10. B Ltd acquired 100% of a subsidiary, M Ltd, on 1 January 2021. The carrying amount of the
assets of M Ltd in the consolidated financial statements of the B group at 31 December 2021,
immediately before an impairment review, were as follows:

Particulars Rs. In million


Goodwill 1.4
Brand name 2
Property, plant and equipment 6
Current assets (at recoverable amount) 2.4
11.8

The recoverable amount of M Ltd was estimated at Rs. 9.6 million at 31 December 2021 and
the impairment of the investment in M Ltd was deemed to be Rs.2.2 million. B Ltd applies
NAS 16: Property, Plant and Equipment and NAS 36: Impairment of Assets in preparing
its financial statements.

Required:

Assuming M Ltd represents a cash generating unit, show the financial reporting treatment of
the brand name at 31 December 2021 in the books of B Ltd following the impairment review.

The Institute of Chartered Accountants of Nepal 10


Revision Test Paper (RTP), December 2023 CAP III – Group I

NAS 19 – Employee Benefits

11. On 1 Shrawan 2079, the fair value of the assets of A Ltdʼs defined benefit plan were valued at
Rs.20,40,000 and the present value of the defined obligation was Rs.21,25,000. On 31 Ashadh
2080 the plan received contributions from A Ltd amounting to Rs. 4,25,000 and paid out
benefits of Rs. 2,55,000. The current service cost for the financial year ending 31 Ashadh 2080
is Rs. 5,10,000. An interest rate of 5% is to be applied to the plan assets and obligations.

The fair value of the plan assets at 31 Ashadh 2080 was Rs.23,80,000, and the present value
of the defined benefit obligation was Rs.27,20,000. Provide a reconciliation from the opening
balance to the closing balance for Plan assets and Defined benefit obligation. Also show how
much amount should be recognized in the statement of profit and loss, other comprehensive
income and balance sheet?

NAS 2 Inventories

12. From the following details of Amatya Ltd, you are required to compute the closing inventory:

Particulars
Raw Material – A
Closing Balance 600 units
Rs. Per unit
Cost Price 250
Freight inward 30
Handling charges 15
Tax refund 20
Replacement cost 180
Finished Goods – B
Closing Balance 1500 units
Rs. Per unit
Material Consumed 250
Direct Labour 70
Direct Overhead 30

Total fixed overhead for the year was Rs.300,000 on a normal capacity of 30,000 units while
actual production has been of 25,000 units

Calculate the value of closing inventory when

i) Net Realizable Value of the Finished Goods B is Rs. 450 per unit
ii) Net Realizable Value of the Finished Goods B is Rs. 340 per unit

The Institute of Chartered Accountants of Nepal 11


Revision Test Paper (RTP), December 2023 CAP III – Group I

NFRS 8 Operating Segments

13. M Ltd. Group has three divisions A, B and C. Details of their turnover, results and net assets
are given below:
Rs. '000
Division A
Sales to B 3,050
Other Sales (Domestic) 60
Export Sales 4,090
7,200
Division B
Sales to C 30
Export Sales to Europe 200
230
Division C
Export sales to US 180

Head Office Division A Division B Division C(Rs.


(Rs. '000) (Rs. '000) (Rs. '000) '000)
Operating Profit 160 20 (8)
or Loss Before
tax
Reallocated cost 48 24 24
from Head office
Interest Costs 4 5 1
Fixed assets 50 200 40 120
Net current assets 48 120 40 90
Long term 38 20 10 120
liabilities
Prepare Segmental Report for publication in M Ltd

Value Added Statement


14. EV Ltd have announced bonus for staff @ 50 percent of any value added earned in excess of
Rs.1.15 per Re.1 of labour. EV Ltd's results for the year ended 30th Ashadh 2080 are as follows:
Rs.'000
Sales 5000
Cost of Sales
Cost of Production:
Materials 1000
Direct Labour 1200
Factory Overhead (Note 1) 1500 3700
Work in progress:
Opening 140
Closing 160 (20)
Finished good stock

The Institute of Chartered Accountants of Nepal 12


Revision Test Paper (RTP), December 2023 CAP III – Group I

Opening 400
Closing 440 (40) 3640
Gross Profit 1360
Selling & administration overhead (Note 1) 870
Profit 490
Royalties received 10
Net Profit 500
Taxation 200
Net profit after tax 300
Dividends 100
Profit 200

Note 1:
Factory Selling &
Administration
Wages & Salaries 400 470
Equipment Leasing 50 10
Depreciation 200 50
Other rents, rates, etc. 850 340
EV Ltd is having beta factor of 1.1 and has the following capital structure:
Equity Shareholders' Fund 700
9% Preference Share Capital 200
Rate of tax is 40%
Risk Free Rate is 4%
Market rate of return is 6%
Company is planning to introduce EVA based bonus which will be 10% of EVA. Such
bonus will be shared by employees in ratio of their emoluments. Find the amount of Bonus
receivable by an employee whose annual wage bill of Rs.5000 under both plans of VA and
EVA.

NAS 33 Earnings per Share


15. At 30 June 20X1, the issued share capital of an entity consisted of 1,500,000 ordinary
shares of Rs. 1 each. On 1 October 20X1, the entity issued Rs. 1,250,000 of 8% convertible
loan stock for cash at par. Each Rs. 100 nominal of the loan stock may be converted, at
any time during the years ended 20X6 to 20X9, into the number of ordinary shares set
out below:
30 June 20X6: 135 ordinary shares;
30 June 20X7: 130 ordinary shares;
30 June 20X8: 125 ordinary shares; and 30 June 20X9: 120 ordinary shares.
If the loan stocks are not converted by 20X9, they would be redeemed at par.
This illustration assumes that the written equity conversion option is accounted for as a
derivative liability and marked to market through profit or loss. The change in the options’

The Institute of Chartered Accountants of Nepal 13


Revision Test Paper (RTP), December 2023 CAP III – Group I

fair value reported in 20X2 and 20X3 amounted to losses of Rs. 2,500 and Rs. 2,650
respectively. It is assumed that there are no tax consequences arising from these losses.
The profit before interest, fair value movements and taxation for the year ended 30 June
20X2 and 20X3 amounted to Rs. 825,000 and Rs. 895,000 respectively and relate wholly
to continuing operations. The rate of tax for both periods is 33%.
Calculate Basic and Diluted EPS.

NAS 40 Investment Property


16. Define Investment Property as per NAS 40 and state whether the following properties will
meet the definition of Investment Property as per the accounting standard:
• Owned by the company and leased out under an operating lease
• Held under Finance Lease and Leased out under operating lease
• Held under Finance Lease and leased out under Finance Lease
• Property acquired with a view for development and re-sale

NFRS 9 – Financial Instrument


17. M Ltd purchased Rs. 30 million 10% debentures on 1 April 2019. These debentures are
redeemable on 31 March 2024 at Rs. 31 million. Interest on these debentures is paid annually
on 31 March each year. M Ltd intends to hold the debentures in order to collect the interest
payments and receive repayment on maturity. However, it may sell these debentures if the
opportunity of buying one with a greater return arises. Effective interest rate of these
debentures is 10.54%. Fair value of the debentures as at 31 March 2020 was Rs. 30.5 million.
As at 31 March 2021, the debentures had been impaired by Rs. 300,000. Fair value of the
debentures as at 31 March 2021 was Rs. 29.9 million. On 1 April 2021, M Ltd sold the
debentures at Rs. 29.9 million.
Required: Advise on the accounting treatment to be applied for the above debentures together
with the calculations required up to 31 March 2021.

NAS 38 Intangible Assets


18. An entity has an intangible asset in the form of a product protected by patented technology
which is expected to be a source of net cash inflows for at least 15 years. It has been recognized
in the books on initial date at Rs.1,200,000. The entity has a commitment from a third party to
purchase the patent in five years for 60 percent of the fair value of the patent at the date of
acquisition, and the entity intends to sell the patent in five years. Company is amortizing the
asset in 15 years considering its residual value to be zero. Annual amortization charted to Profit
or Loss account is Rs.80,000. You are required to state whether the accounting treatment done
by the company is in accordance with the NAS 38? If not, calculate the annual amortization of
the intangible asset and also the amount at which it will be reflected in the balance sheet.

NAS 34 Interim Financial Reporting


19. What are the components of interim financial statements (condensed or complete)?

The Institute of Chartered Accountants of Nepal 14


Revision Test Paper (RTP), December 2023 CAP III – Group I

20. Write short notes on:


a. Expected Credit Loss Model
b. Reversal of Impairment Loss as per NAS 36
c. De-recognition of financial assets
d. Distinction between the treatment of contingent consideration classified as equity and
contingent consideration classified as either asset or liability
e. Distinguish between Joint Operation and Joint arrangement as explained under NRFS 11 Joint
Arrangements

The Institute of Chartered Accountants of Nepal 15


Revision Test Paper (RTP), December 2023 CAP III – Group I

Answers
1. Consolidated Financial Statements
Consolidated Balance Sheet of X Ltd and its subsidiary Y Ltd
as on 16th July 2023
Particulars Notes X Limited (Rs.)
I. Equity and Liabilities:
1. Shareholder's Funds:
a. Share Capital 1 12,500,000
b. Reserves and Surplus 2 3,146,500
2. Non-controlling Interest (WN 2) 1,357,500
3. Current Liabilities
Trade Payables 3 3,932,500
Total 20,936,500
II. Assets:
1. Non-Current Assets
a. Property, Plant and Equipment 4 13,975,000
b. Investments 5 111,500
2. Current Assets:
a. Inventories 6 3,851,000
b. Trade Receivables 7 2,485,000
c. Cash and Cash Equivalents 8 514,000
Total 20,936,500
Notes to Accounts:
Note No. Rs. Rs.
1 Share Capital
Authorized, Issued, Subscribed and Paid-up:
125,000 Equity Shares of Rs. 100 each 12,500,000
2 Reserve and Surplus
General Reserve (WN 4) 1,137,500
Profit and Loss Account (WN 4) 2,009,000 3,146,500
3 Trade Payables
Trade Payables

The Institute of Chartered Accountants of Nepal 16


Revision Test Paper (RTP), December 2023 CAP III – Group I

X Ltd 2,275,000
Y Ltd 1,177,500 3,452,500
Bills Payables
X Ltd 140,000
Y Ltd 415,000
Less: Mutual Owings (75,000) 340,000 480,000
3,932,500
4 Property, Plant and Equipment
X Ltd 10,850,000
Y Ltd 3,125,000 13,975,000
5 Intangible Assets
Goodwill (WN 3) 111,500
6 Inventories
X Ltd 2,400,000
Y Ltd 1,596,000
3,996,000
Less: Unrealized Prrofit (145,000) 3,851,000
7 Trade Receivables
X Ltd 900,000
Y Ltd 820,000 1,720,000
Bills Receivables
X Ltd 340,000
Less: Mutual Owings (75,000) 265,000
Y Ltd 500,000 765,000
2,485,000
8 Cash and Cash Equivalents
X Ltd 437,500
Y Ltd 76,500 514,000

Note: Since dividend has been proposed by both the companies, it has been considered as
dividends were declared after the reporting date. As per the NAS 10, no adjustment is required
to be made in the financial statements if dividend is declared after the reporting date. However,
the disclosure is required in the Notes to the Account.

The Institute of Chartered Accountants of Nepal 17


Revision Test Paper (RTP), December 2023 CAP III – Group I

9. 10% dividend has been proposed by the group companies as follows:


X Ltd Rs. 1,250,000
Non-controlling Interest of Y Ltd (2,900,000*10%*30%) Rs.87,000
Rs.13,37,000
However, this being a non-adjusting event has not been recognized in the Consolidated
Financial Statement ended on 15.07.2023
Working Notes:
2. Analysis of Reserves and Surplus of Y Ltd
Pre- Post-acquisition
acquisition
General Profit & Loss
Profits
Reserve Account
General Reserve 400,000 200,000
Profit & Loss Account 125,000 900,000
For Lot 1 (A) 525,000 200,000 900,000
Pre-acquisition for 2nd Lot
General Reserve (425,000-400,000) 25,000
Profit & Loss Account (510,000- 385,000
125,000
X Ltd (70% of A) 367,500 140,000 630,000
Adjustment for pre-acquisition 2,500 (2,500)
General Reserve for Lot 2 (10%)
Adjustment for pre-acquisition 38,500 (38,500)
Profit or Loss Account for Lot 2
(10%)
X Ltd 408,500 137,500 591,500
Non-controlling Interest (30% of A 157,500 60,000 270,000
Ltd)

3. Non-controlling Interest
Amount (Rs.)
Share Capital (30%) 870,000
Add: Share of pre-acquisition profit of Y Ltd 157,500
Add: Share of post-acquisition General Reserve 60,000
Add: Share of post-acquisition Profit or Loss Account 270,000
1,357,500

The Institute of Chartered Accountants of Nepal 18


Revision Test Paper (RTP), December 2023 CAP III – Group I

4. Cost of Control/Goodwill
Amount (Rs.)
Cost of Investments 2,550,000
Less: Share Capital (70%) (2,030,000)
Less: Share of pre-acquisition profit (408,500)
Goodwill 111,500

5. Consolidated General Reserve & Profit or Loss Account


General Reserve Profit or Loss Account
X Ltd. 1,000,000 1,562,500
Less: Unrealized Profit (145,000)
Add: Share in post- 137,500 591,500
acquisition profit of Y Ltd
1,137,500 2,009,000

Preparation of Financial Statements


2. Solution
ABC Limited
Statement of Profit or Loss and Other Comprehensive Income
For The Year Ended 31/03/2080
Particulars Rs. '000
Revenue 320,700
Cost of sales (225,325)
Gross Profit 95,375
Income from Investment Property 1,800
Total Income 97,175
Operating expenses (33,600 +300 exchange loss) (33,900)
Fair value loss on investment property (24,000-20,250) (3,750)
Profit before interest and tax 59,525
Finance cost (6,300-4,800 ordinary dividends) (1,500)
Profit before tax 58,025
Taxation (9,600)
Profit after tax 48,425

The Institute of Chartered Accountants of Nepal 19


Revision Test Paper (RTP), December 2023 CAP III – Group I

Other comprehensive income:


Gain on revaluation -
PPE 11,000
Total comprehensive income 59,425

ABC Limited
Statement of Financial Position
as at 31/03/2080
Particulars Rs. '000 Rs. '000
Non-current assets
Property plant and equipment 120,325
Investment property 20,250
140,575
Current assets
Inventory (15,750-450) 15,300
Trade receivables 20,250
Bank 11,850
47,400
Total Assets 187,975
Current liabilities
Trade payables (17,700+6,500(w6)) 24,200
Current tax liability 12,000
36,200
Non-current liabilities
Deferred tax liability 5,400
10% Redeemable Preference shares 15,000
56,600
Equity
Stated capital (ordinary shares @ .25) 30,000
Retained earnings (26,250 +48,425-4,800 dividends) 69,875
Revaluation surplus 31,500
131,375

The Institute of Chartered Accountants of Nepal 20


Revision Test Paper (RTP), December 2023 CAP III – Group I

Total Equity and Liabilities 187,975


Working Notes:
1. Depreciation
Rs.’000
Buildings (72,000/15) 4,800
Old plant and equipment (12.5% x (54,000-25,200)) 3,600
New plant and equipment (12.5% x 6,200(w5)) 775
9,175
2. Cost of Sales
Rs.’000
Per trial balance 215,700
Write down (below) 450
Depreciation charge for the year 9,175
225,325
Inventory
Cost 15,750
Damaged goods:
Cost 1,200
NRV (1,425-675) (750)
Write down (450)
Value at year end 15,300
3. Property Plant and Equipment
Land and Building Plant and Equipment Total
Cost/Valuation 83,500 54,000
(1/04/2079)
Accumulated - 25,200
depreciation
NBV (1/4/2079) 83,500 28,800
Revaluation gain 11,000 - 11,000
Addition - 6,200 6,200
Depreciation 4,800 4,375 9,175
NBV (31/03/2080) 89,700 30,625 120,325
Depreciation charge on building per annum = 72 m/15 years = 4.8 m

The Institute of Chartered Accountants of Nepal 21


Revision Test Paper (RTP), December 2023 CAP III – Group I

Depreciation charge on Plant and Machinery is 12.5% x (54 m-25.2 m) = 3.6 m


4. Taxation
i. Income tax
Current charge for the year 12,000
Decrease in deferred Tax (2,400)
Charge to SPL 9,600
ii. Deferred Tax
Balance b/d 7,800
Decrease in deferred Tax (2,400)
Balance c/d 5,400
Note: Deferred tax liability at year end = 30% x 18,000,000 (temporary timing difference) =
Rs.5,400,000.
4. Foreign currency translation
At 1/01/2080 Rs’000
Dr: Equipment (100,000 x 6.2) 6,200
Cr: Payables 6,200
Note: the non-monetary item is not subject to retranslation but the foreign currency payable
should be retranslated as follows:
Payables Rs.’000
At 1/1/2080 6,200
Exchange loss (balance) 300
At 31/03/2080 (1,000,000 x 6.50) 6,500
5. Finance cost
Finance cost is stated less the ordinary dividends of Rs. 4,800,000 (i.e. 6,300,000 less
preference dividends of Rs. 1,500,000), as well as the net interest on defined benefit plan Rs.
10,000 (i.e. Rs. 20,000 interest cost – Rs. 10,000 return on plan assets).
Rs.’000
Per trial balance 6,300
Dividends (ordinary) (4,800)
10% x15,000 Preference shares 1,500

NAS – 12 Income Taxes


3. Calculation of Deductible temporary differences:
Deferred tax asset = Rs. 80,000

The Institute of Chartered Accountants of Nepal 22


Revision Test Paper (RTP), December 2023 CAP III – Group I

Existing tax rate = 40%


Deductible temporary differences = 80,000/40%
= Rs. 2,00,000
Calculation of Taxable temporary differences:
Deferred tax liability = Rs. 60,000
Existing tax rate = 40%
Deductible temporary differences = 60,000 / 40%
= Rs. 1,50,000
Of the total deferred tax asset balance of Rs.80,000, Rs. 28,000 is recognized in OCI
Hence, Deferred tax asset balance of Profit & Loss is Rs.80,000 - Rs.28,000 = Rs.52,000
Deductible temporary difference recognized in Profit & Loss is Rs. 1,30,000 (52,000 / 40%)
Deductible temporary difference recognized in OCI is Rs.70,000 (28,000 / 40%)
The adjusted balances of the deferred tax accounts under the new tax rate are:
Deferred tax asset Rs.
Previously credited to OCI-equity 70,000 x 0.45 31,500
Previously recognised as Income 1,30,000 x 0.45 58,500
90,000
Deferred tax liability
Previously recognized as expense 1,50,000 x 0.45 67,500
The net adjustment to deferred tax expense is a reduction of Rs. 2,500. Of this amount,
Rs.3,500 is recognised in OCl and Rs.1,000 is charged to P&L.
The amounts are calculated as follows:
Carrying Carrying Increase
amount at amount at /(decrease)
45% 40%
in deferred tax
expense
Deferred tax assets
Previously credited to OCI-equity 31,500 28,000 (3,500)
Previously recognised as Income 58,500 52,000 (6,500)
90,000 80,000 (10,000)
Deferred tax liability
Previously recognized as expense 67,500 60,000 7,500
Net adjustment (2,500)

The Institute of Chartered Accountants of Nepal 23


Revision Test Paper (RTP), December 2023 CAP III – Group I

An alternative method of calculation is:


DTA shown in OCI Rs. 70,000 x (0.45 - 0.40) 3,500
DTA shown in Profit or Loss Rs.1,30,000 x (0.45-0.40) 6,500
DTL shown in Profit or Loss Rs.1,50,000 x (0.45 -0.40) 7,500
Journal Entries
Deferred tax asset Dr 3,500
To OCI –revaluation surplus 3,500
Deferred tax asset Dr 6,500
To Deferred tax expense 6,500
Deferred tax expense Dr 7,500
To Deferred tax liability 7,500
Alternatively, a combined journal entry may be passed as follows:
Deferred tax asset Dr. 10,000
Deferred tax expense Dr. 1,000
To OCI –revaluation surplus 3,500
To Deferred tax liability 7,500
NAS 20 Accounting for Government Grants and Disclosure of Government Assistance
4. Government grants related to non-current assets should be credited to the statement of profit
or loss over the life of the asset to which they relate, not in accordance with the schedule of
any potential repayment. The directors’ proposed treatment is implying that the government
grant is a liability which decreases over four years. This is not correct as there would only be
a liability if the directors intended to sell the related plant, which they do not intend to do so.
Therefore, in the year ended 31st Asar 2080, Rs.800,000 (8 million/10 years) should be credited
to the statement of profit or loss and Rs.7·2 million should be shown as deferred income
(Rs.800,000 current and Rs.6·4 million non-current) in the statement of financial position.
NFRS 16 Leases
5. Solution
Kathmandu Ltd
Statements of financial position extract as at 31st Ashadh 2081
Particulars 2079/80 2080/81
Non-Current Asset
Right-of-use asset 181,026 181,026
Accumulated Depreciation (45,257) (90,514)
135,769 90,512
Non-current liabilities

The Institute of Chartered Accountants of Nepal 24


Revision Test Paper (RTP), December 2023 CAP III – Group I

Lease Liability 89,778 46,511


Current Liabilities
Capital Sum 40,248 43,267
Financial Charge 9,752 6,733
50,000 50,000

Kathmandu Ltd
Statements of Profit or Loss extract for the year ended 31st Ashadh 2081
Particulars 2079/80 2080/81
Expenses
Depreciation (181,026/4) (45,257) (45,257)
Finance Costs (9,752) (6,733)
Working Notes
WN-1: Lease Liability
Year Opening Lease Liability for Implicit Closing
Balance payments the year Interest @ Balance
7.5%
2079/80 130,026 - 130,026 9,752 139,778
2080/81 139,778 (50,000) 6,733 96,511
89,778
2081/82 96,511 (50,000) 3,488 50,000
46,511
2082/83 50,000 (50,000) - -
-
WN-2: Right of use asset
Lease Liability (Opening) 130,026
Lease payment at 1 January 2020 50,000
Initial Direct Cost 1,000
181,026
Annual Depreciation = 181,026/4
=45,257
NFRS 15 Revenue from Contracts with Customers
6. NFRS 15 provides guidance to assess whether an entity is acting as an agent or a principal in
an arrangement. An entity is a principal if it controls the specified good or service before that
good or service is transferred to the customer.

The Institute of Chartered Accountants of Nepal 25


Revision Test Paper (RTP), December 2023 CAP III – Group I

An entity is an agent if the entity’s performance obligation is to arrange for the provision of
the specified good or service by another party.
In this scenario Unlimited Fashion receives the title to the goods or services before it transfers
them to the customer. However, per paragraph B35 of NFRS 15, obtaining the title to the goods
is not necessarily a factor to determine whether an entity is a principal in a particular
arrangement.
There are indicators provided in paragraph B37 to assess whether an entity is acting as an agent
or a principal in a particular arrangement.
1. The entity is primarily responsible for fulfilling the promise to provide the specified good
or service.
Once an order has been placed it will automatically be communicated to Fashion Stores
and the shipments should be done by Fashion Stores itself. Once a good is returned by a
customer, it needs to be returned back to Fashion Stores. Unlimited Fashion will not take
any responsibility over that. Therefore it appears that the responsibility to fulfill the
promise lies with Fashion Stores and not with Unlimited Fashion.
2. The entity has an inventory risk before the specified good or service is transferred
tocustomer or after the transfer of control to the customer.
Although the title to the goods has been transferred to Unlimited Fashion, the inventories
are insured under the name of Fashion Stores. This indicated Unlimited Fashion does not
assume any inventory risk.
3. The entity has discretion in establishing the price for the specified good or service. The
price of the goods is determined by Fashion Stores and not by Unlimited Fashion. After
the evaluation of the terms of the arrangement against the above criteria, it can be
concluded that Unlimited Fashion is acting as an agent in the arrangement.
NAS 24 – Related Party Disclosure
7. Solution
Party Is the party related Explanation
or not?
Pokhara Ltd Related party 56% of the voting shares implies the
controlling power is with Pokhara. Hence it
is the parent company of Nepal Ltd
Shyam Related party 78% of Pokhara's shares are owned by
Shyam (ultimate controlling party of Nepal
Ltd)
Pratik, Kalum and Related Parties All of them are directors of Nepal Ltd (part
Menaka of Key Management Personnel) (KMP)
Mennan, Geetha and Related Parties All of them are directors of the parent
Kalum company Pokhara Ltd (they are part of Key
Management Personnel) (KMP)
Solex (Pvt) Ltd Related party Nepal Ltd has the controlling stake of Solex,
hence it is a subsidiary company

The Institute of Chartered Accountants of Nepal 26


Revision Test Paper (RTP), December 2023 CAP III – Group I

Kamal, Anusha and May not be related They are directors of a subsidiary company
Priya parties of Nepal Ltd. That relationship alone will not
allow us to consider them as related parties.
Gayan Related party Geeta's husband and therefore the spouse of
a KMP.
Winwin Pvt Ltd May not be related Not enough facts available to determine the
parties relationship.
Phenix Pvt. Ltd Related party A Jointly controlled entity
Jaya May not be related Not enough facts available to determine the
parties relationship,
NFRS 13 – Fair value measurement
8. Fair value measurement is for a particular asset or liability. Therefore, when measuring fair
value, an entity should take into account the characteristics of the asset/liability if market
participants take those into account when pricing the asset/liability at the measurement date.
Restrictions (if any) on the sale or use of the asset is one such characteristic.
In this case, there is a restriction on the sale of the asset since the entity has pledged that asset
as collateral for a borrowing. This restriction is a characteristic of the entity rather than of the
asset (e.g. if the entity settles the loan early, the restriction will not apply).
Therefore, this characteristic would not be considered by the market participants in pricing the
asset (e.g. market participants may require the entity to settle the loan and release the security
before the sale).
Therefore, it is not required to discount the market price for the restriction. The fair value is
Rs. 13 million.
NAS 36 Impairment of Assets
9. Impairment is determined by assessing the recoverable amount of a cash generating unit to
which the investment/goodwill relates. The recoverable amount of a cash generating unit is
determined based on the higher of fair value, less cost to sell and the value in use (VIU)
calculation.
As per NAS 36 it’s mandatory to perform an impairment testing on goodwill at the end of each
financial year
Rs. In million
2023/24 2024/25 2025/26 2026/27 2027/28
Pre-tax Cash Flows 275 325 368 412 494
Terminal Value
(494*1.01)/(0.15-0.010 3,536.86
Year 1 2 3 4 5
Discounting Factor @15% 0.870 0.756 0.658 0.572 0.497
Discounted Cash Flows 239.25 245.70 242.14 235.66 2,003.34

The Institute of Chartered Accountants of Nepal 27


Revision Test Paper (RTP), December 2023 CAP III – Group I

Rs. In million
Enterprise value 2,966.10
Add: Cash and cash equivalents 136
Less: Interest Bearing Liabilities -275
Equity Value 2,827.10
FV less cost to sell
Company (225*10*98.5%) 2216.25
Since the "value in use" (VIU) is higher, it will be taken as the recoverable value in determining
impairment.
Investment in Little Rs. In million
Carrying value as at 31st March 2023 (250*7.5) 1,875
Recoverable value (75%*2,840.88) 2,136.66
Since the recoverable value is higher than the carrying value, no impairment on the investment
is recognized in Little.
Goodwill Rs. In million
Carrying value as at 31st March 2023
Net asset value of Little 2,750
Goodwill 375
3,125
Recoverable Value 2,840.88
Impairment of Goodwill 284.12
Goodwill impairment will be charged to profit or loss as an expense in the year 2022/23.
Rs.90.88 million will be recognized in the Statement of Financial Position as at 31 March 2023
as carrying value of the goodwill.
NAS 16: Property, Plant and Equipment and NAS 36: Impairment of Assets
10. The impairment loss for the Cash Generating Unit is Rs.2.2 million (Rs.11.8 million – Rs.9.6
million). The impairment loss is initially allocated to the goodwill balance of Rs.1.4 million.
The unallocated impairment loss is Rs.0·8 million. This is allocated to the brand and PPE based
on their carrying amounts:
Rs. In million
Brand name 2
Property, plant and equipment 6
8

The Institute of Chartered Accountants of Nepal 28


Revision Test Paper (RTP), December 2023 CAP III – Group I

2/8 *Rs.0.8 = 0.2 loss to be allocated to brand so new carrying amount


= Rs.2 million – Rs.0.2 million
= Rs.1.8 million.
NAS 19 - Employee Benefits
11. Reconciliation of Plan asset and Defined benefit obligations
Particulars Plan Asset Defined Benefit
(Rs.) Obligation (Rs.)
Fair value/present value as at 1 Shrawan 2079 20,40,000 21,25,000
Interest @ 5% 1,02,000 1,06,250
Current service cost 5,10,000
Contributions received 4,25,000 -
Benefits paid (2,55,000) (2,55,000)
Return on gain (assets) (balancing figure) 68,000 -
Actuarial Loss (balancing figure) - 2,33,750
Closing balance as at 31 Ashadh 2080 23,80,000 27,20,000
In the Statement of Profit and loss, the following will be recognised:
Rs.
Current service cost 5,10,000
Net interest on net defined liability (Rs. 1,06,250 – Rs. 1,02,000) 4,250
Defined benefit re-measurements recognized in other comprehensive income:
Rs.
Loss on defined benefit obligation (2,33,750)
Gain on plan assets 68,000
(1,65,750)
In the Balance sheet, the following will be recognised:
Rs.
Net defined liability (Rs. 27,20,000 – Rs. 23,80,000) 3,40,000
NAS 2 Inventories
12. When Net Realizable Value of the Finished Goods B is Rs. 450 per unit
Valuation Quantity Rate Amount
Base (Rs.) (Rs.)
Raw Material – A Cost 600 275 165,000
Finished Goods – B Cost 1500 360 540,000
Total value of closing inventory 705,000
When Net Realizable Value of the Finished Goods B is Rs. 340 per unit

The Institute of Chartered Accountants of Nepal 29


Revision Test Paper (RTP), December 2023 CAP III – Group I

Since Net Realizable Value of Finished Good B is less than its cost i.e. Rs. 360 (Working
Note), Raw Material A is to be valued at replacement cost and finished goods is to be
valued at Net Realizable Value.
Valuation Base Qty Rate (Rs.) Amount
(Rs.)
Raw Material – A Replacement Cost 600 180 108,000
Finished Goods – B Net Realizable Value 1500 340 510,000
Total value of closing inventory 618,000
Working Note
Statement showing cost of Raw Material - A and Finished product – B
Raw Material A Rs.
Cost Price (250-20) 230
Add: Freight inward 30
Add: Handling Charges 15
275
Finished product – B Rs.
Material Consumed 250
Direct Labour 70
Direct Overhead 30
Fixed Overhead (300,000/30000 units) 10
360
NFRS 8 Operating Segments
13. Solution
M Ltd
Segment Report
(Rs. '000)
Division Division Division Eliminations Consolidated
A B C – Inter
Segment
Revenue
External Sales:
Domestic 60 - - - 60
Export 4,090 200 180 - 4,470
Inter-segment Sales 3,050 30 - 3,080 -

The Institute of Chartered Accountants of Nepal 30


Revision Test Paper (RTP), December 2023 CAP III – Group I

Total Revenue 7,200 230 180 3,080 4,530


Result
Segment Result 160 20 8 - 172
Reallocated Cost - - - - 96
from Head Office
Operating Profit - - - - 76
Interest Cost - - - - (10)
Profit before tax - - - - 66
Other Information
Segment Assets:
a. Fixed Assets 200 40 120 - 360
b. Net Current 120 40 90 - 250
Assets
Unallocated - - - - 98
Corporate Assets
Segment Liabilities 20 10 120 - 150
(Long Term)
Unallocated - - - - 38
corporate liabilities
(Long term)
Sale revenue by Geographical Market
Domestic sales by Export Export to Export to Consolidated
Division A Sales of Europe US
Division A
60 4,090 200 180 4530
Value Added Statement
14. Solution
Working Notes
1. Value added based bonus
Value Added Statement
Sales 5000
Add: Royalty income 10
5010
Less: Cost of Production
Materials 1000

The Institute of Chartered Accountants of Nepal 31


Revision Test Paper (RTP), December 2023 CAP III – Group I

Factory overhead (1500-400-200) 900


Work in progress (20)
Finished Stock (40)
Selling & administration overhead 350 2190
Value added by manufacturing and trading activity 2820
Payroll expenses
Direct Labour 1200
Factory overhead 400
Selling overhead 470
2070
Calculation of Bonus
Labour Cost =2070
Value added (Min) =2070*1.15=2380.50
Bonus =(2820-2380.50)*50%
=219.75
Bonus for employee
219.75/2070*5000 =530.79
Working Notes:
Economic Value Added
i) Calculation of Net Operating Profit after Tax
Earning before Interest & Taxes = 500
Less: Tax @ 40% =200
Net Operating Profit after Tax =300
ii) Calculation of Weighted Average Cost of Capital
Kp = 9%
Kc=4%+1.10(6%-4%) =6.2%
[9%*200/900]+[6.2%*700/900]=6.82%
iii) Economic Value Added = (Return on Operating Capital – Weighted
average cost of capital)*Operating Capital
= [(300/900*100)-6.82%]*900
= 238.60
iv) Economic Value added based Bonus = 238.6*10% = 23.86
Employee = 23.86/2070*5000 =57.60

The Institute of Chartered Accountants of Nepal 32


Revision Test Paper (RTP), December 2023 CAP III – Group I

NAS 33 Earnings per Share


15. Solution
Trading results 20X3 20X2

Rs. Rs.

A. Profit before interest, fair value movements and tax 895,000 825,000

B. Interest on 8% convertible loan stock (20X2: 9/12 × (100,000) (75,000)


Rs.100,000)

C. Change in fair value of embedded option (2,650) (2,500)

Profit before tax 792,350 747,500

Taxation @ 33% on (A-B) (262,350) (247,500)

Profit after tax 530,000 500,000

Calculation of basic EPS

Number of equity shares outstanding 1,500,000 1,500,000

Earnings Rs. 530,000 Rs. 500,000

Basic EPS 0.35 0.33

Calculation of diluted EPS


Test whether convertibles are dilutive:
The saving in after-tax earnings, resulting from the conversion of Rs. 100 nominal of loan
stock, amounts to Rs. 100 × 8% × 67% + Rs. 2,650/12,500 = Rs. 5.36 + Rs. 0.21 = Rs. 5.57.
There will then be 135 extra shares in issue.
Therefore, the incremental EPS is Re.0.04 (ie. Rs. 5.57/135). As this incremental EPS is
less than the basic EPS at the continuing level, it will have the effect of reducing the basic
EPS of Re.0.35. Hence the convertibles are dilutive.

20X3 20X2
Adjusted earnings
Rs. Rs.

Profit for basic EPS 530,000 500,000

Add: Interest and other charges on earnings saved as 102,650 77,500


a result of the conversion
(100,000 + (75000+ 2500)
2,650)

The Institute of Chartered Accountants of Nepal 33


Revision Test Paper (RTP), December 2023 CAP III – Group I

Less: Tax relief thereon (33,000) (24,750)

Adjusted earnings for equity 599,650 552,750

Adjusted number of shares


From the conversion terms, it is clear that the maximum number of shares issuable on
conversion of Rs. 1,250,000 loan stock after the end of the financial year would be at the
rate of 135 shares per Rs. 100 nominal (that is, 1,687,500 shares).

20X3 20X2

Number of equity shares for basic EPS 1,500,000 1,500,000

Maximum conversion at date of issue 1,687,500 × 9/12 – 1,265,625

Maximum conversion after balance sheet date 1,687,500

Adjusted capital 3,187,500 2,765,625

Adjusted earnings for equity Rs. 599,650 Rs. 552,750

Diluted EPS (approx.) 0.19 0.20

NAS 40 Investment Property


16. Investment property is property (land or building – or part of a building or both) held (by the
owner or by the lessee as a right-of-use) to earn rentals or for capital appreciation or both,
rather than for:
a. Use in the production or supply of goods or services or for administrative purposes; or
b. Sale in the ordinary course of business
Appropriate classification of given properties on basis of above definition as 'Investment
Property' or 'Not an Investment Property' is done as follows:
i) Owned by the company and leased out under an operating lease : Investment Property
ii) Held under Finance Lease and Leased out under operating lease : Investment Property
iii) Held under Finance Lease and leased out under Finance Lease : Not an Investment
Property
iv) Property acquired with a view for development and re-sale : Not an Investment
Property
NFRS 9 Financial Instruments
17. As per NFRS 9, a financial asset should be measured at fair value through other comprehensive
income (FVTOCI) if both of the following conditions are met.

The Institute of Chartered Accountants of Nepal 34


Revision Test Paper (RTP), December 2023 CAP III – Group I

➢ The financial asset is held within a business model whose objective is achieved by both
collecting contractual cash flows and selling financial assets (Business Model Test); and
➢ the contractual terms of the financial asset give rise on specified dates to cash flows that
are solely payments of principal and interest on the principal amount outstanding (SPPI
Test)
M Ltd intends to hold the debentures in order to collect the interest payments and receive the
repayment on maturity. However, it may sell these debentures if the possibility of buying one
with a greater return arises.
Accordingly, the debentures are held within the business model of collecting contractual cash
flows and selling the financial asset. Therefore, the Business Model Test is satisfied.
The feature of this instrument is similar to a basic lending arrangement. There are no
contractual terms that introduce exposure to risks or volatility in the contractual cash flows that
are unrelated to a basic lending arrangement, such as exposure to changes in equity prices or
commodity prices. Accordingly, the SPPI test is also satisfied.
Accordingly, the investment should be classified as a financial asset measured at FVTOCI.
Initial measurement should be at the transaction price (i.e. Rs. 30 million).
At amortized cost, the debt would be measured as follows:
Calculations done by applying the same interest and cash flows as for the amortised cost model,
but re-measuring the asset to fair value at each period end.
Rs.'000
Opening balance 10.54% Repayments Closing balance
31.03.2020 30,000 3,162 (3,000) 30,162
31.03.2021 30,162 3,179 (3,000) 30,341
31.03.2022 30,341 3,198 (3,000) 30,539
31.03.2023 30,539 3,219 (3,000) 30,758
31.03.2024 30,758 3,242 (34,000) -
Calculations done by applying the same interest and cash flows, but re-measuring the asset to
fair value at each period end.
Rs.'000
Opening 10.54 Repayments Closing Impairment Balance after Fair Fair Value
% balance impairment value
balance changes

31.03.2020 30,000 3,162 (3,000) 30,162 338 30,500


31.03.2021 30,162 3,179 (3,000) 30,341 (300) 30,379 (479) 29,900
For the financial year ended 31 March 2020, a finance cost of Rs. 3.162 million should be
recognized in profit or loss.
A fair value gain of Rs. 0.338 million should be recognized in OCI.

The Institute of Chartered Accountants of Nepal 35


Revision Test Paper (RTP), December 2023 CAP III – Group I

The carrying value of the asset as at 31 March 2020 is Rs. 30.5 million, which is the fair value.
For the financial year ended 31 March 2021, a finance cost of Rs. 3.179 million should be
recognized in profit or loss.
As per NFRS 9, a gain or loss on a financial asset measured at FVTOCI should be recognized
in other comprehensive income, except for impairment gains or losses and foreign exchange
gains and losses, until the financial asset is derecognized or reclassified.
So, the impairment of Rs. 0.3 million should be recognized in profit or loss.
The fair value reduction of Rs. 479 million should be recognized in OCI.
The carrying value of the investment as at 31 March 2021 is the fair value of Rs. 29.9 million.
NAS 38 Intangible Assets
18. For determination of amortization of intangible asset, which has the finite useful life, two
elements need to be determined: useful life and residual value.
Useful life is defined as:
a. The period over which an asset is expected to be available for use by an entity; or
b. The number of production or similar units expected to be obtained from the asset by an
entity
In the given case, since the entity expects that the asset will be available for use for 5 years and
thereafter it will be transferred, the useful life of the asset is 5 years.
For residual value, paragraphs 100-102 of NAS 38 states that the residual value of an intangible
assets with the finite useful life shall be assumed to be zero unless:
a. there is a commitment by a third party to purchase the asset at the end of its useful life; or
b. There is an active market for the asset and
i. residual value can be determined by reference to the market; and
ii. it is probable that such market will exist at the end of the asset's useful life.
The depreciable amount of an asset with a finite useful life is determined after deducting its
residual value. A residual value other than zero implies that an entity expects to dispose of the
intangible asset before the end of its economic life.
An estimate of an asset’s residual value is based on the amount recoverable from disposal using
prices prevailing at the date of the estimate for the sale of a similar asset that has reached the
end of its useful life and has operated under conditions similar to those in which the asset will
be used.
On the basis of above paragraphs, the depreciable amount of the patent will be determined after
deducting the residual value, which is 60% of its fair value at the date of its acquisition.
Accordingly, the patent will be amortized over the useful life of 5 years, with a residual value
equal to 60% of its fair value at the date of acquisition. The patent will also be tested for
impairment in accordance with NAS 36.Therefore the accounting policy of amortizing the
assets over a period of 15 years considering its residual value of zero is not in accordance with
NAS 38.

The Institute of Chartered Accountants of Nepal 36


Revision Test Paper (RTP), December 2023 CAP III – Group I

Computation of correct amount of residual amount and annual amortization


Rs.
Cost of Intangible asset 1,200,000
Residual value (60% of 12 lakhs) 720,000
Depreciable value of intangible asset 480,000
Useful life 5 years
Annual amortization (480,000/5) Rs.96,000 p.a.
NAS 34 Interim Financial Reporting
19. Interim reports shall include interim financial statements (condensed or complete) for periods
as follows:
(a) statement of financial position as of the end of the current interim period and a comparative
statement of financial position as of the end of the immediately preceding financial year.
(b) statements of profit or loss and other comprehensive income for the current interim period
and cumulatively for the current financial year to date, with comparative statements of
profit or loss and other comprehensive income for the comparable interim periods (current
and year-to-date) of the immediately preceding financial year.
(c) statement of changes in equity cumulatively for the current financial year to date, with a
comparative statement for the comparable year-to-date period of the immediately
preceding financial year.
(d) statement of cash flows cumulatively for the current financial year to date, with a
comparative statement for the comparable year-to-date period of the immediately
preceding financial year.
20.
a. Expected Credit Loss Model
Expected Credit Loss (ECL) is the probability-weighted estimate of credit losses (i.e., the
present value of all cash shortfalls) over the expected life of a Financial Instrument. The
concept is particularly important in the context of NFRS 9.
The new impairment model under NFRS 9 provides for allowances for expected credit losses,
marking a shift away from the previous approach based on incurred losses. Financial reporting
thus moves closer to forward-looking credit risk management and means that a model is
required to measure credit risks for all financial assets not measured at fair value.
ECL can be measured either on an individual exposure level or a collective portfolio
level (grouped exposures based on shared credit risk characteristics)
Under the general approach, an entity must determine whether the financial asset is in one of
three stages in order to determine both the amount of ECL to recognize as well as how interest
income should be recognized.
Stage 1 is where credit risk has not increased significantly since initial recognition. For
financial assets in stage 1, entities are required to recognize 12 month ECL and recognize

The Institute of Chartered Accountants of Nepal 37


Revision Test Paper (RTP), December 2023 CAP III – Group I

interest income on a gross basis – this means that interest will be calculated on the gross
carrying amount of the financial asset before adjusting for ECL.
Stage 2 is where credit risk has increased significantly since initial recognition. When a
financial asset transfers to stage 2 entities are required to recognize lifetime ECL but interest
income will continue to be recognized on a gross basis.
Stage 3 is where the financial asset is credit impaired. This is effectively the point at which
there has been an incurred loss event under the NAS 39 model. For financial assets in stage 3,
entities will continue to recognize lifetime ECL but they will now recognize interest income
on a net basis. This means that interest income will be calculated based on the gross carrying
amount of the financial asset less ECL.
b. Reversal of Impairment Loss as per NAS 36
An impairment loss recognised in prior periods for an asset other than goodwill shall be
reversed if, and only if, there has been a change in the estimates used to determine the asset’s
recoverable amount since the last impairment loss was recognised. If this is the case, the
carrying amount of the asset shall be increased to its recoverable amount. That increase is a
reversal of an impairment loss.
A reversal of an impairment loss reflects an increase in the estimated service potential of an
asset, either from use or from sale, since the date when an entity last recognised an impairment
loss for that asset. Paragraph 130 requires an entity to identify the change in estimates that
causes the increase in estimated service potential. Examples of changes in estimates include:
(a) a change in the basis for recoverable amount (ie whether recoverable amount is based on
fair value less costs of disposal or value in use);
(b) if recoverable amount was based on value in use, a change in the amount or timing of
estimated future cash flows or in the discount rate; or
(c) if recoverable amount was based on fair value less costs of disposal, a change in estimate
of the components of fair value less costs of disposal.
An asset’s value in use may become greater than the asset’s carrying amount simply because
the present value of future cash inflows increases as they become closer. However, the service
potential of the asset has not increased. Therefore, an impairment loss is not reversed just
because of the passage of time (sometimes called the ‘unwinding’ of the discount), even if the
recoverable amount of the asset becomes higher than its carrying amount.
c. De-recognition of financial assets
Before evaluating whether, and to what extent, de-recognition is appropriate, an entity
determines whether those requirements should be applied to a part of a financial asset (or a
part of a group of similar financial assets) or a financial asset (or a group of similar financial
assets) in its entirety, as follows:
(a) De-recognition requirements are applied to a part of a financial asset (or a part of a group
of similar financial assets) if, and only if, the part being considered for de-recognition
meets one of the following three conditions.
(i) The part comprises only specifically identified cash flows from a financial asset (or a
group of similar financial assets).

The Institute of Chartered Accountants of Nepal 38


Revision Test Paper (RTP), December 2023 CAP III – Group I

(ii) The part comprises only a fully proportionate (pro rata) share of the cash flows from a
financial asset (or a group of similar financial assets).
(iii) The part comprises only a fully proportionate (pro rata) share of specifically identified
cash flows from a financial asset (or a group of similar financial assets).
(b) In all other cases, de-recognition requirements are applied to the financial asset in its
entirety (or to the group of similar financial assets in their entirety).
In the derecognition requirements, the term ‘financial asset’ refers to either a part of a financial
asset (or a part of a group of similar financial assets) as identified in (a) above or, otherwise, a
financial asset (or a group of similar financial assets) in its entirety.
An entity shall derecognize a financial asset when, and only when:
(a) the contractual rights to the cash flows from the financial asset expire, or
(b) it transfers the financial asset and the transfer qualifies for de-recognition.

d. Distinction between the treatment of contingent consideration classified as equity and


contingent consideration classified as either asset or liability
Contingent consideration classified as a liability or an asset: Contingent consideration
classified as either an asset or a liability (financial or nonfinancial) should be re-measured to
fair value at each reporting date and changes in fair value should be included in profit or loss
in accordance with NAS 39 or NFRS 9.
Contingent consideration classified as equity: Equity-classified contingent consideration is
measured initially at fair value on the acquisition date and is not re-measured subsequent to
initial recognition. Settlement of the equity-classified contingent consideration is accounted
for within equity. In other words, the initial value recognized for an equity contingent
consideration arrangement on the acquisition date is not adjusted, even if the fair value of the
arrangement on the settlement date is different.
e. Distinguish between Joint Operation and Joint arrangement as explained under NRFS 11 Joint
Arrangements
Basis Joint Operation Joint Venture
The terms The contractual arrangement The contractual arrangement
of the provides the parties to the joint provides the parties to the joint
contractual arrangement with rights to the assets, arrangement with rights to the net
arrangement and obligations for the liabilities, assets of the arrangement (i.e. it is
relating to the arrangement. the separate vehicle, not the parties,
that has rights to the assets, and
obligations for the liabilities,
relating to the arrangement).
Rights to The contractual arrangement The contractual arrangement
assets establishes that the parties to the joint establishes that the assets brought
arrangement share all interests (e.g. into the arrangement or
rights, title or ownership) in the assets subsequently acquired by the joint
relating to the arrangement in a arrangement are the arrangement’s
specified proportion (e.g. in assets. The parties have no interests

The Institute of Chartered Accountants of Nepal 39


Revision Test Paper (RTP), December 2023 CAP III – Group I

proportion to the parties’ ownership (i.e. no rights, title or ownership) in


interest in the arrangement or in the assets of the arrangement.
proportion to the activity carried out
through the arrangement that is
directly attributed to them).
Obligations The contractual arrangement The contractual arrangement
for establishes that the parties to the joint establishes that the joint
liabilities arrangement share all liabilities, arrangement is liable for the debts
obligations, costs and expenses in a and obligations of the arrangement.
specified proportion (e.g. in The contractual arrangement
proportion to the parties’ ownership establishes that the parties to the
interest in the arrangement or in joint arrangement are liable to the
proportion to the activity carried out arrangement only to the extent of
through the arrangement that is their respective investments in the
directly attributed to them). The arrangement or to their respective
contractual arrangement establishes obligations to contribute any unpaid
that the parties to the joint or additional capital to the
arrangement are liable for claims arrangement, or both. The
raised by third parties. contractual arrangement states that
creditors of the joint arrangement do
not have rights of recourse against
any party with respect to debts or
obligations of the arrangement.
Revenues, The contractual arrangement The contractual arrangement
expenses, establishes the allocation of revenues establishes each party’s share in the
profit or and expenses on the basis of the profit or loss relating to the
loss relative performance of each party to activities of the arrangement.
the joint arrangement. For example,
the contractual arrangement might
establish that revenues and expenses
are allocated on the basis of the
capacity that each party uses in a plant
operated jointly, which could differ
from their ownership interest in the
joint arrangement. In other instances,
the parties might have agreed to share
the profit or loss relating to the
arrangement on the basis of a
specified proportion such as the
parties’ ownership interest in the
arrangement. This would not prevent
the arrangement from being a joint
operation if the parties have rights to
the assets, and obligations for the
liabilities, relating to the
arrangement.

The Institute of Chartered Accountants of Nepal 40


PAPER 1: ADVANCED FINANCIAL REPORTING
Questions:
1. NAS 34 Interim Financial Reporting
Company A has reported Rs. 60,000 as pre-tax profit in first quarter and expects a loss of Rs.
15,000 each in the subsequent quarters. It has a corporate tax slab of 20 percent on the first Rs.
20,000 of annual earnings and 40 per cent on all additional earnings. Calculate the amount of
tax to be shown in each quarter.
2. NAS 16 Property, Plant and Equipment
Write Short Notes/Answers on following:
a. In what circumstances is the reducing balance method more appropriate than the
straight-line method?
b. Minority Interest
c. Underlying Assumption of Financial Statement
d. Need and Significance of Environmental Accounting
e. Describe the processes of formulation of capital budget of Government of Nepal.
f. Techniques of inflation accounting
3. NAS 10 Events after the Reporting Period
ABC Ltd. received a demand notice on 15th Jestha, 2079 for an additional amount of Rs.
28,00,000 from the Excise Department on account of higher excise duty levied by the Excise
Department compared to the rate at which the company was creating provision and depositing
the same. The financial statements for the year 2078-79 are approved on 10th Bhadra, 2079. In
Shrawan 2079, the company has appealed against the demand of Rs. 28,00,000 and the
company has expected that the demand would be settled at Rs. 15,00,000 only. Show how the
above event will have a bearing on the financial statements for the year 2078-79. Whether these
events are adjusting or non-adjusting events and explain the treatment accordingly.
4. NAS 2 Inventories
A retailer has the following purchases and sales of a particular product line:
Units Purchase Price per Units Selling price
unit Rs.
Purchased Sold per units Rs.
2 Ashadh 2079 100 500 60 530
16 Ashadh 2079 60 503 80 528
30 Ashadh 2079 70 490 50 526
14 Shrawan 2079 50 509 70 524
28 Shrawan 2079 80 512 50 522
11 Shrawan 2079 40 515 40 520
At 32 Ashadh 2079 the physical inventory was 150 units (including of opening balance). The cost of
inventories is determined on a FIFO basis. Selling and distribution costs amount to 5% of selling price
and general administration expenses amount to 7% of selling price.
Required:
Calculate to the nearest Rupee, value of inventory at 32 Ashadh 2079:
i) at cost;
ii) at net realizable value; and
Determine the amount to be included in the financial statements.
5. NAS 8 Accounting Policies, Changes in Accounting Estimates and Errors
During 2079, Dream Ltd., changed its accounting policy for depreciating property, plant and
equipment, so as to apply much more fully a components approach, whilst at the same time
adopting the revaluation model.
In years before 2079, Dream Ltd.’s asset records were not sufficiently detailed to apply
components approach fully. At the end of 2078, management commissioned an engineering
survey, which provided information on the components held and their fair values, useful lives,
estimated residual values and depreciable amounts at the beginning of 2079. However, the
survey did not provide a sufficient basis for reliably estimating the cost of those components
that had not previously been accounted for separately, and the existing records before the
survey did not permit this information to be reconstructed.
Dream Ltd.’s management considered how to account for each of the two aspects of the
accounting change. They determined that it was not practicable to account for the change to
a fuller component approach retrospectively, or to account for that change prospectively from
any earlier date than the start of 2079. Also, the change from a cost model to a revaluation
model is required to be accounted for prospectively. Therefore, management concluded that
it should apply Dream Ltd.’s new policy prospectively from the start of 2079.
Additional information:
(i) Dream Ltd.’s tax rate is 30%
(ii) Property, plant and equipment at the end of 2078:
Cost Rs. 25,000
Depreciation Rs. 14,000
Net book value Rs. 11,000
(iii) Prospective depreciation expense for 2079 (old basis) Rs. 1,500
(iv) Some results of the engineering survey:
Valuation Rs. 17,000
Estimated residual value Rs. 3,000
Average remaining asset life 7 years
Depreciation expense on existing property, plant and equipment for 2079 (new basis) Rs.
2,000
You are required to prepare relevant note for disclosure in accordance with NAS 8.
6. NAS 11 Construction Contracts
On Shrawan 1, 2076, an entity receives a contract to build the tallest building in town. The
building will have 70 floors. The construction period will last about 3 years. Total revenues
are assessed at Rs 40 million. The expected costs are estimated to be Rs. 38 million. At the
end of the fiscal year 2077/78, the cost assessment increases to Rs 38.50 million. Other
information related to the contractis given below:
YEAR 1 YEAR 2 YEAR 3
Cost incurred 11,400,000 20,212,500 38,500,000
Assessed total cost 38,000,000 38,500,000 38,500,000
Contract cost and contract revenue are recognised using the percentage of completion
method. Determine the profit for each of the three fiscal years viz. 2076/77, 2077/78 &
2078/79. Show any additional disclosure that is required in the Notes to accounts in
accordance with NAS 11.
7. NFRS 13: Fair Value Measurement
An asset is sold in 2 different active markets at different prices. An entity enters into
transactions in both markets and can access the price in those markets for the asset at the
measurement date.
In Market A:
The price that would be received is Rs. 26, transaction costs in that market are Rs. 3 and the
costs to transport the asset to that market are Rs. 2.
In Market B:
The price that would be received is Rs. 25, transaction costs in that market are Rs. 1 and the
costs to transport the asset to that market are Rs. 2.
You are required to calculate:
(i) The fair value of the asset, if market A is the principal market, and
(ii) The fair value of the asset, if none of the markets is principal market.
8. NAS 17 Leases
An equipment is leased for 3 years and its useful life is 5 years. Both the cost and the fair
value of the equipment are Rs. 6,000,000. The amount will be paid in 3 installments, and at
the termination of the lease, lessor will get back the equipment. The unguaranteed residual
value at the end of 3 years is Rs. 800,000. The internal rate of return (IRR) of the investment
is 10%. The present value of annuity factor of Re. 1 due at the end of 3rd year at 10% IRR
is 2.4868. The present value of Re. 1 due at the end of 3rd year at 10% rate of interest is
0.7513.
Required:
State with reason whether the lease constitutes finance lease. Calculate unearned finance income.
9. NFRS 2 Share-based payment
An entity which follows its financial year as per the calendar year grants 1,000 share
appreciation rights (SARs) to each of its 40 management employees as on 1st Baishakh
2075. The SARs provide the employees with the right to receive (at the date when the rights
are exercised) cash equal to the appreciation in the entity’s share price since the grant date.
All of the rights vest on 31st Ashadh 2076; and they can be exercised during 2077 and 2078.
Management estimates that, at grant date, the fair value of each SAR is Rs. 11; and it
estimates that overall, 10% of the employees will leave during the two-year period. The fair
values of the SARs at each year end are shown below:
Year Fair value at year end
32 Ashadh 2075 12
31 Ashadh 2076 8
31 Ashadh 2077 13
31 Ashadh 2078 12
10% of employees left before the end of 2076. On 31st December 2077 (when the intrinsic
value of each SAR was Rs. 10), six employees exercised their options; and the remaining
30 employees exercised their options at the end of 2078 (when the intrinsic value of each
SAR was equal to the fair value of Rs. 12).
How much expense and liability are to be recognized at the end of each year? Pass Journal
entries.
10. NAS 33 Earnings Per Share
From the following information compute basic and diluted earnings per share.
Net profit for the year 2079 Rs. 12,00,000
Weighted average number of equity shares outstanding
during the year 2079 500,000 shares
Average fair value of one equity shares during the year 2079 Rs 20.00
Weighted average number of shares options during the year
2079 100,000 Shares
Exercise Price for shares under option during the year 2079 Rs. 15.00
11. NAS 20 Accounting for Government Grants and Disclosure of Government Assistance
A village of artisans in a district got devastated because of an earthquake. A Limited was
operating in that district and was providing employment to the artisans. The government
gave a grant of Rs. 10,00,000 to A Limited so that 100 artisans are rehabilitated over a period
of 3 years. Government releases Rs. 2,00,000. Examine how the Government grant be
realized.
12. NAS 38 Intangible Assets
A company acquired for its internal use a software on 15-06-2079 from USA for USD
100,000. The Exchange rate on that date was Rs.120.00 per USD. The seller allowed trade
discount @ 5%. The other expenses incurred were:
 Custom Duty: 20%
 VAT- 13%
 Other Tax: 5% (Refundable)
 Installation Expenses: Rs.25,000
 Fee for Custom clearance (to agent): Rs.20,000
Compute the cost of Software to be capitalized.
13. NFRS 8 Operating Segments
X Ltd. has identified 4 operating segments for which revenue data is given below:

External Sale (Rs.) Internal Sale (Rs.) Total (Rs.)


Segment A 30,00,000 Nil 30,00,000
Segment B 6,50,000 Nil 6,50,000
Segment C 8,50,000 1,00,000 9,50,000
Segment D 5,00,000 49,00,000 54,00,000
Total Sales 50,00,000 50,00,000 1,00,00,000
Additional information:
Segment C is a new business unit and management expect this segment to make a significant
contribution to external revenue in coming years. Which of the segments would be
reportable under the criteria identified in NFRS 8?
14. NAS 3 Statement of Cash Flows
From the following Summary Cash Account of X Ltd., prepare Cash Flow Statement for the
year ended 32nd Ashadh, 2079 in accordance with NAS 3 using the direct method. The
company does not have any cash equivalents.
Summary Cash Account for the year ended 32.03.2079
Rs. ‟000 Rs. ‟000

Balance on 01.04.2078 50 Payment to Suppliers 2,000


Issue of Equity Shares 300 Purchase of Fixed Assets 200
Receipts from Customers 2,800 Overhead expense 200
Sale of Fixed Assets 100 Wages and Salaries 100
Taxation 250
Dividend 50
Repayment of Bank Loan 300
Balance on 32.03.2079 150
3,250 3,250
15. Development Fund
The Institute for Banking & Research maintains a combined Development Fund inrespect of
which the following information is available for the year ended 32nd Ashadh 2079:

Particulars Rs.
Govt. Grants received for acquisition of land Rs. 6,000,000
Private Grants received for construction of buildings Rs. 3,000,000

Foreign Private Grant for purchase of computing equipment USD 500,000


Transfer from unrestricted fund for purchase of furniture Rs. 1,000,000
Cost of Assets so far acquired:
Land Rs. 5,900,000
Buildings in progress (payments to Contractors) Rs. 1,500,000
Furniture Rs. 300,000
The USD grant has been received into a bank account in USA on 29.03.2079 and is expected
to be utilized therefrom for purchases to be made abroad. The rate of exchange on 32.03.2079
is 1 USD = Rs. 120.
You are required to prepare:
i) A Statement showing changes in the Development Fund for the fiscal year 2078/79; and
ii) Statement of Financial Position of the Development Fund as at 32nd Ashadh 2079.
16. Nepal Rastra Bank, Loan Loss Provision
At the end of FY 2078/79, ABC Commercial Bank Ltd. disclosed following information
about the outstanding loan as classified below:

Class of loan Amount (Rs. in Million)


Pass loan 25,000
Substandard loan 400
Doubtful loan (Overdue up to 1 year) 350
Loss loan (Overdue more than 1 year) 500
Among the pass loan, 10% amount was disbursed by taking personal guarantee (not secured
by the collateral).
Required:
i) Calculate the Loan Loss Provision to be booked by ABC Commercial Bank Ltd. as
at the end of FY 2078/79.
ii) Calculate the level of Non-Performing Loan and state whether it is under the acceptable
limit fixed by Nepal Rastra Bank.
17. Net Assets Value Per Unit
ABC Equity Oriented Mutual Fund has introduced a scheme ―ABC Dhamaka. The current
major details of the mutual fund scheme are as follows:

Particulars Information
Scheme size Rs. 100 crore
Face value of units Rs. 10 per unit

Investment In equity shares


Market value of all investments 50% investments made in commercial banks are
appreciated by 30%, 20% investments made in
hydropower & hotels are appreciated by 15%
and remaining amount of investment made in
insurance sector is depreciated by 20%.

Required:
Calculate the net assets value per unit of ABC Dhamaka. Is there any appreciation in value of
units? If yes, calculate the percentage of appreciation of value of units.
18. Leverage Effect
Consider the following case:

(i) Current cost of capital employed (equity approach) Rs. 1,040,000


(ii) Profit earned after current cost adjustment (equity approach) Rs. 172,000
(iii) 10% long term loan Rs. 450,000
(iv) Normal rate of return:
On equity capital employed 15.6%
On long-term capital employed 13.5%
Required: Find out Leverage effect on Goodwill.
19.
i) Define Economic Value Added (EVA) and its use?
ii) Economic Value Added
The following information is available of a concern. Calculate Economic Value Added
(E.V.A.):
Debt capital 12% Rs. 2,000 crores
Equity capital Rs. 500 crores
Reserve and surplus Rs. 7,500 crores
Capital employed Rs. 10,000 crores
Risk-free rate 9%
Beta factor 1.05
Market rate of return 19%
Equity (market) risk premium 10%
Operating profit after tax Rs. 2,100 crores
Tax rate 30%
20. Consolidated Financial Statements
Prepare the consolidated Balance Sheet as on 32nd Ashadh 2079 of a group of companies
comprising A Limited, B Limited and BB Limited. Their Statement of Financial position on
that date are given below.
in lakhs
A Ltd. B Ltd. BB Ltd.
Assets
Non -Current Assets
Property, Plant and Equipment 320 360 300
Investment:
32 Lakhs shares in B Ltd. 340
24 Lakhs Shares in BB Ltd. 280
Current Assets
Inventories 220 70 50
Financial Assets
Trade Receivables 260 100 220
Bills Receivables 72 30
Cash in hand at Bank 228 40 40
1440 850 640
Equity and Liabilities
Shareholders' Equity
Share capital (10 per shares ) 600 400 320
Other Equity
Reserves 180 100 80
Retained Earnings 160 50 60
Current Liabilities
Financial Liabilities
Trade Payables 470 230 180
Bills Payable
A Ltd. 70
BB Ltd. 30
1440 850 640
The following additional information is available:
(i) A Ltd. holds 80% shares in B Ltd. holds 75% shares in BB Ltd. Their holdings were
acquired on 30th Ashoj, 2078.
(ii) The business activities of all the companies are not seasonal in nature and therefore, it
can be assumed that profits are earned evenly throughout the year.
(iii) On 1st Shrawan, 2078 the following balances stood in the books of B limited and BB
Limited
B Limited BB Limited
Reserves 80 60
Retained earnings 20 30
(iv) 10 lakhs included in the inventory figure of B Limited, is inventory which has been
purchased from BB Limited at cost plus 25%.
(v) The parent company has adopted an accounting policy to measure non-controlling
interested at fair value (quoted market price). Assume market Prices of B Limited and
BB Limited are the same as respective face values.
Answers:
1. NAS 34 interim financial reporting
Amount of income tax expense reported in each quarter would be as below:
Expected total Income = Rs. 15,000 [60,000- (15,000 x 3)]
Expected tax as per slabs = 15,000 x 20% = Rs. 3,000
Average Annual Income tax rate = 3,000/15,000 = 20%
Q1 Q2 Q3 Q4
Profit before tax 60,000 (15,000) (15,000) (15,000)
Tax expense 12,000 (3,000) (3,000) (3,000)
2. NAS 16 Property, Plant and Equipment
a. The reducing balance method of depreciation is used instead of the straight line method
when it is considered fair to allocate a greater proportion of the total depreciable amount
to the earlier years and a lower proportion to the later years on the assumption that the
benefits obtained by the business from using the asset decline over time.
In favour of this method it may be argued that it links the depreciation charge to the costs
of maintaining and running the asset. In the early years these costs are low and the
depreciation charge is high, while in later years this is reversed.
b. Minority interest represents that part of the net results of operations and of net assets of a
subsidiary attributable to interests which are not owned, directly or indirectly through
subsidiaries, by the holding or parent, company. In short, minority interest represents the
claims of the outside shareholders of a subsidiary. Minority interest in the net income of
consolidated subsidiaries for the reporting period are identified and adjusted against the
income of the group in order to arrive at the net income attributable to the shareholders of
the holding company. Minority interest in the net assets of consolidated subsidiaries should
be identified and presented in the consolidated balance sheet separately from liabilities and
the equity of the parents shareholders. Minority interest in the net assets consists of:
a. the amount of equity attributable to minorities at the date on which investment in
a subsidiary is made, and
b. the minorities' share of movements in equity since the date the parent-subsidiary
relationship and into existence.
c. Underlying assumptions of financial statements are those assumptions which are assumed
to have been followed while preparing the statements unless expressly stated thereon.
There are three basic underlying assumptions of financial statements, which are given
below:
Accrual Basis: In order to meet their objectives, financial statements are prepared on the
accrual basis of accounting. Under this basis the effects of transaction and other events are
recognized when they occur (and not as cash or its equivalent is received or paid) and they
are recorded in accounting records and reported in the financial statements of the period
to which they relate. The cash flow information are provided through a separate cash flow
statement to supplement the profit/ loss information derived in accrual basis.
Going Concern: Financial statements are normally prepared on the assumption that an
enterprise is a going concern and will continue its operation for the foreseeable future.
Hence, it is assumed that the enterprise has no intention to liquidate or curtail materially
the scale of its operations. If such an intention or need exists the financial statements are
to be prepared on a different basis and if so the basis used is to be disclosed. Here the
conceptual framework is not sufficiently clear. In case going concern assumption cannot
be followed, what basis should be adopted for the preparation and presentation in the
financial statements is not specifically spelt out in the conceptual framework.
Consistency: In order to achieve comparability of the financial statements of an enterprise
through time, the accounting policies are followed consistently from one period to another,
a change in an accounting policy is made only in certain exceptional circumstances.
d. Environmental accounting is a faithful attempt to identify and bring to light the resources
consumed and the cost rendered reciprocally to the environment by business enterprises.
It is needed for the following issues:
a) Resource Utilization: Natural resources are required to carry on the business
activities of every firm. Also the functioning of an enterprise has some favorable and
some adverse effects on the environment. Hence there is a need for maintaining
accounts of the effect of the activities of a business entity on the environment and
on natural resources.
b) Resource Availability: Environmental accounting is useful for disclosing how much
natural resources are available in the country, their incomes and the costs incurred
to use them and their depreciation, values etc.
c) Social Responsibility: Environmental accounting is useful for measuring industrial
development and social welfare and the fulfillment of social responsibilities by the
business entity. Business entities are urged to be accountable to both stakeholders
and wider society. Profit making is not considered as the sole business / corporate
objective. Qualitative Study: Traditional accounting system is restricted to
quantitative and monetary aspects only. Hence, environmental accounting is
necessary to analyze the effect of environmental resources in the entire business
function of a firm.
d) Environmental Protection: Environmental accounting will help in evaluating the
problem of environmental protection. The business activities of the enterprise
should be recognized as society centered and not only the profit centered.
e) Going concern: Environmental pollution and substantial costs associated with clean-
up activities, fines, compensation, and bad publicity etc. can even significantly
affects the share prices and even the stability of a company. Hence environmental
accounting awareness is required.
f) Social Accounting: Social accounting has been the precursor of environmental
accounting. Social costs also include the use of natural resources and pollution of
environments. Also preservation of environment is a critical factor for sustainable
development. So, environmental accounting deserves special attention of managers,
investors, society, different branches of Government and other stakeholders.
e. The capital budget formulation has four essential steps: budget forecasting, budget ceiling
setting, budget planning, and budget approval.
a. Forecasting
Forecasting is the first step in the budget making process where various macro-
economic projections are made, sources of revenues and tentative areas of expenditure
are identified and based on these economic targets are set. The Resource Committee of
the National Planning Commission (NPC) leads the forecasting process and produces
fiscal aggregates. The committee prepares the forecast on the basis of information and
feedback collected from different agencies viz. Ministry of Finance (MoF), Nepal
Rastra Bank etc.
Although these forecasts may not seem to have a direct linkage with the budget, they do
affect the budget to a huge extent. The forecasting is the basis of the annual budget.
Economic growth is targeted based on these estimates. Forecasting is a crucial step in
budget formulation as its accuracy determines whether the budget will be balanced, in
surplus or in deficit. Forecasting is the first step that paves the path for other budget
formulation activities. It is essential for the forecast to be comprehensive as it determines
the nature and shape of the budget, and the indicators that define the forecastalso define
the economic objectives of the nation.
b. Ceiling Setting
Once a tentative forecast of the budget is prepared, it is presented to the budget
committee comprising of members from the NPC and the MoF. The committee
conducts extensive discussions regarding ceiling setting; an exercise that sets the limits
on budget expenditure. A final shape to the budget is given at this stage. NPC also takes
this opportunity to assess the budget utilisation and the progress of large-scale projects,
which finally defines the scale of the capital budget. It is essential to set a ceiling on
expenditures considering the limited resources. In the absence of such a ceiling, the
request forwarded by the subordinate ministries may not be in line with the annual
budget ceiling. This means that a considerable amount of time would have to bedevoted
to reconciling the ministry specific budget expenditures with the national level ceiling.
Therefore, ceiling setting plays a vital role in limiting the size of the budget based on
sectors and availability of resources.
c. Budget Planning
After the determination of annual budget ceilings, the NPC requests the line ministries
to submit their capital budget estimates in accordance to the guidelines and sectoral
budget ceilings. Along with the NPC, the MoF issues detailed guidelines to the line
ministries that require the budgets to be submitted in the prescribed format and within
the stipulated time.
NPC informs each of the districts of the budget ceilings that need to be adhered to and
the MoF dispatches the budget circulars through the line ministries and the various
departments and divisions at the district level. Once a District Coordination Committee
(DCC) receives these requests and guidelines, it begins its budget planning based on
different sectors while the relevant line ministry sends guidelines to each of the
departments and divisions within the district-level offices. As departments are not given
a sectorial budget ceiling, they generally do not take expenditure limitations into
account; instead they look at project-specific targets as a guide for the budget.
At the local level, a meeting of the village council is called for discussing plans and
programmes to be incorporated into the annual budget. Interest groups such as consumer
committees, NGOs, political leaders and citizens are included in these discussions. Their
plans are then forwarded to the Village Development Committee (VDC) / municipality
where it gets accumulated. Once the VDC scrutinizes and gives a green signal to the
plan it moves up to the District Council which re-examines the budget and forwards it
to the DDC. On the other hand, the district level development offices also forward their
plans and budgets for the upcoming year to the DDC underthe supervision of the
relevant ministry.
Once finalised, these plans are forwarded to the line ministry and then to the NPC. A
tripartite discussion between the NPC, MoF and line ministry is held before finalizing
the budget limits for the ensuing year. Upon receiving the budget requests, the NPC and
the MoF compare various frameworks of the sectoral and national objectives. There is
room for revision of the plans to achieve various programme targets for the upcoming
fiscal year. The sectoral budgets are assessed in line with the budget ceilings.
The budget formulation process displays both a bottom up and a top to bottom approach.
This two way process is essential in creating a synergy between the national level plans
and the local level needs. The actors in the central level authority are well versed with
policies, plans, and guidelines while the local level actors are acquainted with the local
needs, local capacity, and available resources in a particular area. It is when these two
approaches coincide, that a fruitful expenditure plan of the budget can be obtained. This
would ensure a budget that fulfils the demands of both the state and the locals.
d. Budget Approval
The final step in budget process is budget approval. Before the budget is presented in
the parliament, the parliamentarians are presented with a draft budget document for
their perusal. They discuss the overall budget whilst looking at sector-wise specifics.
For instance, the Ministry of Health and Population has the opportunity to discuss and
review the budget allotted for health programs. The comments and queries that arise
during the ministry-specific discussions are discussed in detail until the relevant
ministry and parliamentarians are satisfied with the clarifications provided to their
queries. The budget is presented by the government only after the ministry level budget
discussion ends. If the majority agrees with the budget then it is approved and is put
into implementation.
f. Inflation accounting refers to the process of adjusting the financial statements of a
company to show the real financial position of the company during inflationary period.
i) Current Purchasing Power (CPP) Method – It involves adjustment of financial
accounts to price changes. A general price index is used to convert the values of
various items. It takes into account the purchasing power of money and ignores the
rise and fall in the price of an item. It involves adjustment of historical figures at
current purchasing power which is done through multiplication of the historical figures
by a conversion factor.
ii) Current Cost Accounting (CCA) Method – Under this method assets are shown at
current costs and profits are determined on the basis of costs at the date of sale rather
than the actual cost. CCA is an alternative to the CPP method. The CCA method
matches current revenues with the current cost of the resources which are consumed
in earning them.
3. NAS 10 Events After the Reporting Period
NAS 10 defines ‘Events after the Reporting Period’ as follows:
Events after the reporting period are those events, favourable and unfavourable, that occur
between the end of the reporting period and the date when the financial statements are
approved by the Board of Directors in case of a company, and, by the corresponding approving
authority in case of any other entity for issue.
Two types of events can be identified:
(a) those that provide evidence of conditions that existed at the end of the reporting period
(adjusting events after the reporting period); and
(b) those that are indicative of conditions that arose after the reporting period (non-adjusting
events after the reporting period)
In the instant case, the demand notice has been received on 15th Jestha, 2079, which is between
the end of the reporting period and the date of approval of financial statements. Therefore, it is
an event after the reporting period. This demand for additional amount has been raised because
of higher rate of excise duty levied by the Excise Department in respect of goods already
manufactured during the reporting period. Accordingly, condition exists on 32nd Ashadh, 2079,
as the goods have been manufactured during the reporting period on which additional excise
duty has been levied and this event has been confirmed by the receipt of demand notice.
Therefore, it is an adjusting event. In accordance with the principles of NAS 37, the company
should make a provision in the financial statements for the year 2078-79, at best estimate of
the expenditure to be incurred, i.e., Rs. 15,00,000.
4. NAS 2 Inventories
Cost on a FIFO basis
Date purchased Units Per Unit Cost
Rs. Rs.
30 Ashadh 70 490 34,300.00
16 Ashadh 60 503 30,180.00
2 Ashadh 20 500 10,000.00
150 74,480.00
Net Realisable Value (NRV)
In the given case, it is not possible to pick one particular NRV, because if we do so then the
very spirit of NFRS would be violated; which states in Para 30, that:
“Estimates of NRV are based on the most reliable evidence available at the time of estimates
are made, of the amount of inventories are expected to realize”. These estimates take into
consideration, fluctuations of price or cost directly relating to events occurring after the end of
the period to the extent that such events confirm conditions existing at the end of the period.
From above, it is clear that; to estimate NRV, the date when estimate is made is very necessary.
Hence in the given case the students cannot be expected to pin point NRV, rather they should
be expected to mention above paragraph along with following further explanation.”
“Therefore when NRV estimate is made consideration should be given to fluctuations of price
or cost directly relating to events occurring after the end of the period to the extent that such
events confirm conditions existing at the end of the period.
Assuming that estimates are made on Ashadh end 2079*
Particulars Amount in Rs.
Selling Price per unit as on Ashadh end 2079 526.00
Less: Selling & Distribution Cost 26.30
Net realizable value 499.70
Number of units hold on Ashadh end 2079 150
Net Realizable value 74,955
Amount to be include in financial statements
Lower of cost and net realizable value 74,480
5. NAS 8: Accounting Policies, Changes in Accounting Estimates and Errors
From the start of 2079, Dream Ltd. changed its accounting policy for depreciating property,
plant and equipment, so as to apply much more fully a components approach, whilst at the
same time adopting the revaluation model. Management takes the view that this policy
provides reliable and more relevant information because it deals more accurately with the
components of property, plant and equipment and is based on up-to-date values. The policy
has been applied prospectively from the start of 2079 because it was not practicable to estimate
the effects of applying the policy either retrospectively, or prospectively from any earlier date.
Accordingly, the adoption of the new policy has no effect on prior years. The effect on the
current year is to increase the carrying amount of property, plant and equipment at the start of
the year by Rs. 6,000 (17,000 - 11,000); increase the opening deferred tax provision by Rs.
1,800 (6,000 x 30%); create a revaluation surplus at the start of the year of Rs. 4,200 (6,000 –
1,800); increase depreciation expense by Rs. 500 (2,000 – 1,500); and reduce tax expense by
Rs. 150 (500 x 30%).
6. NAS 11 Construction Contracts
Computation of percentage of completion in each of three fiscal years
Fiscal Year Fiscal Year Fiscal Year
2076/77 2077/78 2078/79
a) Assessed total contract cost (Rs) 38,000,000 38,500,000 38,500,000
b) Actual cost (Rs) 11,400,000 20,212,500 38,500,000
c) Percentage of completion (b/a) 30% 52.50% 100%

Determination of net profit for each of three fiscal years


Particulars To date Recognized Recognized in
(Rs) in Previous current year
year (Rs) (Rs)
a) Fiscal year 2076/77
Revenue (40,000,000*30%) 12,000,000 12,000,000
Cost (38,000,000*30%) 11,400,000 11,400,000
Net Profit (revenue – cost) 600,000 600,000
b) Fiscal year 2077/78
Revenue (40,000,000*52.50%) 21,000,000 12,000,000 9,000,000
Cost (38,500,000*52.50%) 20,212,500 11,400,000 8,812,500
Net Profit (revenue – cost) 787,500 600,000 187,500
c) Fiscal year 2078/79
Revenue (40,000,000*100%) 40,000,000 21,000,000 19,000,000
Cost (38,500,000*100%) 38,500,000 20,212,500 18,287,500
Net Profit (revenue – cost) 1,500,000 787,500 712,500
Disclosure required under the NAS 11
a. the amount of contract revenue recognized as revenue in the period
b. the methods used to determine the contract revenue recognized in the period; and
c. the methods used to determine the stage of completion of contracts in progress.
Further disclosures: An entity shall disclose each of the following for contracts inprogress
at the end of the reporting period:
(a) the aggregate amount of costs incurred and recognized profits (less recognized
losses) to date;
(b) the amount of advances received; and
(c) the amount of retentions
7. NFRS 13: Fair value measurement
(i) If Market A is the principal market
If Market A is the principal market for the asset (i.e., the market with the greatest volume
and level of activity for the asset), the fair value of the asset would be measured using the
price that would be received in that market, after taking into account transport costs.
Fair Value will be:
Particulars Rs
Price receivable 26
Less: Transportation cost (2)
Fair value of the asset 24
(ii) If neither of the market is the principal market
If neither of the market is the principal market for the asset, the fair value of the asset would
be measured using the price in the most advantageous market. The most advantageous
market is the market that maximises the amount that would be received to sell the asset,
after taking into account transaction costs and transport costs (i.e., the net amount that
would be received in the respective markets).
Particulars Rs Rs
Market A Market B
Price receivable 26 25
Less: Transaction cost (3) (1)
Less: Transportation cost (2) (2)
Net Proceeds of the asset 21 22
Since the entity would maximise the net amount that would be received for the asset in
Market B i.e., Rs. 22, the fair value of the asset would be measured using the price in Market
B.
Fair value
Rs
Price receivable 25
Less: Transportation cost (2)
Fair value of the asset 23
8. NAS 17 Leases
(i)
Present value of residual value = Rs. 800,000 × 0.7513 = Rs. 601,040
Present value of lease payments = Rs. 6,000,000 – Rs. 601,040 = Rs. 5,398,960 The present
value of lease payments being 89.98% [5,398,960 / 6,000,000 X 100] of the fair value, i.e.
being a substantial portion thereof, the lease constitutes a finance lease.
(ii) Calculation of unearned finance income
Rs.
Gross investment in the lease [(Rs. 2,171,047* × 3) + Rs. 800,000] 7,313,141
Less: Cost of the equipment 6,000,000
Unearned finance income 1,313,141
Note: - In the above solution, annual lease payment has been determined on the basis that the
present value of lease payments plus residual value is equal to the fair value (cost) of the asset.
*Annual lease payments = Rs 5,398,960 / 2.4868 = Rs. 2,171,047 (approx.)
9. NFRS 2 Share Based Payment
The amount recognized as an expense in each year and as a liability at each yearend) is as
follows:
Year Expense Rs Liability Rs Calculation of Liability
32 Ashadh 2075 2,16,000 2,16,000 = 36 ∗ 1,000 ∗ 12 ∗ 1/2
31 Ashadh 2076 72,000 2,88,000 =36 * 1,000 * 8
31 Ashadh 2077 1,62,.000# 3,90,000 =30 * 1,000 * 13
31 Ashadh 2078 (30,000) ## 0 Liability extinguished
# Expense comprises an increase in the liability of Rs. 102,000 and cash paid to those
exercising their SARs of Rs. 60,000 (6*1,000*10).
## Difference of opening liability (Rs. 3,90,000) and actual liability paid {Rs. 3,60,000
(30*1,000*12)}is recognised to Profit and Loss i.e., Rs. 30,000.
Journal Entries
32 Ashadh 2075
Employee benefits expenses. Dr. 2,16,000
To Share based payment liability
(Fair value of the SAR recognized) 2,16,000
31 Ashadh 2076
Employee benefits expenses. Dr. 72,000
To Share based payment liability
(Fair value of the SAR re-measured) 72,000
31 Ashadh 2077
Employee benefits expenses. Dr. 1,62,000
To Share based payment liability
(Fair value of the SAR recognized) 1,62,000
Share based payment liability Dr. 60,000
To Cash
(Settlement of SAR) 60,000
31 Ashadh 2078
Share based payment liability Employee30,000
benefits expenses. Dr.
To Employee benefits expenses.
30,000
(Fair value of the SAR recognized)
Share based payment liability Dr. 3,60,000
To Cash
(Settlement of SAR) 3,60,000
10. NAS 33 Earning Per Share
Computation of Basic and Diluted Earnings per Share.

Particulars Rs. Shares Earnings per


share
Net profit for the year 2079 Rs. 12,00,000
Weighted average number of equity 500,000
shares outstanding during the year 2079
Basic earnings per share Rs. 2.40
Number of shares under options 100,000
Number of shares that would have been (75,000)
issued at fair value (100,000X15)/20
Diluted Earnings per Share 525,000 Rs. 2.29
(12,00,000/525,000) Rs. 12,00,000
11. NAS 20 Accounting for Government Grants and Disclosure of Government Assistance
A Limited will recognise Rs. 10,00,000 as government grant and set it up as a deferred income
and will recognise it in profit or loss over the period of three years as per the principles
enunciated in NAS 20. Once a government grant is recognised, any related contingent liability
or contingent asset is treated in accordance with NAS 37, Provisions, Contingent Liabilities
and Contingent Assets. The manner in which a grant is received does not affect the accounting
method to be adopted in regard to the grant. Thus, a grant is accounted for in the same manner
whether it is received in cash or as a reduction of a liability to the government or in the form
of a non-monetary asset.
12. NAS 38 Intangible Assets
Calculation of the cost of the software to be capitalized as per NAS 38:
Purchase Price: USD 100,000
Less: Trade Discount (5%) (USD 5,000)
Net purchase Price USD 95,000
× Exchange Rate Rs.120.00
Hence, Purchase price in NRs. Rs. 11,400,000
Add: Import Duty (20%) Rs. 2,280,000
Rs. 13,680,000
Add: VAT (13%) Rs. 1,778,400
Add: Installation Charges Rs. 25,000
Add: Fee for custom clearance Rs. 20,000
Total Amount to be capitalized: Rs. 15,503,400
13. NFRS 8 Operating segments
Threshold amount is Rs. 10,00,000 (Rs. 1,00,00,000 × 10%).
Segment A exceeds the quantitative threshold (Rs. 30,00,000 > Rs. 10,00,000) and hence
reportable segment.
Segment D exceeds the quantitative threshold (Rs. 54,00,000 > Rs. 10,00,000) and hence
reportable segment.
Segment B & C do not meet the quantitative threshold amount and may not be classified as
reportable segment.
However, the total external revenue generated by these two segments A & D represent only
70% (Rs. 35,000/50,000 x 100) of the entity’s total external revenue. If the total external
revenue reported by operating segments constitutes less than 75% of the entity total external
revenue, additional operating segments should be identified as reportable segments until at
least 75% of the revenue is included in reportable segments.
In case of X Ltd., it is given that Segment C is a new business unit and management expect
this segment to make a significant contribution to external revenue in coming years. In
accordance with the requirement of NFRS 8, X Ltd. designates this start-up segment C as a
reportable segment, making the total external revenue attributable to reportable segments 87%
(Rs. 43,50,000/ 50,00,000 x 100) of total entity revenues.
14. NAS 3 Statement of Cash Flows
X Ltd.
Cash Flow Statement for the year ended 32nd Ashadh, 2079
(Using the direct method)
Rs. ’000 Rs. ’000
Cash flows from operating activities
Cash receipts from customers 2,800
Cash payment to suppliers (2,000)
Cash paid to employees (100)
Cash payments for overheads (200)
Cash generated from operations 500
Income tax paid (250)
Net cash from operating activities 250
Cash flows from investing activities
Payment for purchase of fixed assets (200)
Proceeds from sale of fixed assets 100
Net cash used in investing activities (100)
Cash flows from financing activities
Proceeds from issuance of equity shares 300
Bank loan repaid (300)
Dividend paid (50)
Net cash used in financing activities (50)
Net increase in cash 100
Cash at beginning of the period 50
Cash at end of the period 150
15. Development Fund
The Institute for Banking & Research
Statement of Changes in Development Fund
during the Fiscal Year 2078/2079
Rs. Rs.
Receipts
Government grants 6,000,000
Private grants 3,000,000
Foreign private grant (in USD 60,000,000
500,000)
Transfer from unrestricted fund 1,000,000 70,000,000
Deductions/Transfers
Cost of land acquired 5,900,000
Furniture purchased 300,000 6,200,000
Balance as at 32.03.2079 63,800,000

Development Fund
Statement of Financial Position
As at 32.03.2079
Equity & Liabilities Rs.
Fund Balance 63,800,000
63,800,000
Assets
Buildings in progress 1,500,000
Bank balances
Local 2,300,000
Foreign 60,000,000 62,300,000
63,800,000
Bank A/c (Local)

Rs. Rs.
To Government grant 6,000,000 By Land 5,900,000
To Private grant 3,000,000 By Furniture 300,000
To Transfer 1,000,000 By Payments to contractors 1,500,000
for buildings
By Balance c/d 2,300,000
10,000,000 10,000,000
16. Nepal Rastra Bank- Loan Loss Provision
i)
Calculation of the Loan Loss provision of ABC Commercial Bank Ltd.
as on end of FY2078/79 as per the provision of the NRB Directive:

Class of Loan Loan % of Amount of


Outstanding provision provision
(Rs. in million) needed needed (Rs. in
million)
Pass Loan 25,000 1 250
Substandard Loan 400 25 100
Doubtful Loan ( up to 1 year) 350 50 175
Loss Loan (Overdue more than 1 500 100 500
year)
Additional provision for the Pass 2,500 20 500
Loan not secured by collateral
(10% of 25,000)
Total Loan Loss provision 1,525
ii) Calculation of the level of non-performing loan (NPL):
Total non-performing loan (Rs. in million) = 1,250 (400+350+500)
Total Loan (Rs. in million) = 26,250
Therefore,
Level of NPL (%) = 1,250/ 26,250 = 4.76%.
The non-performing loan level of ABC Commercial Bank Ltd. is within the acceptable level
limit fixed by Nepal Rastra Bank, i.e. below 5%.
17. Net Assets Value Per Unit
Calculation of the Net Assets Value (per unit) of “ABC Dhamaka”
Investment in Equity Shares of Face Value of % age of Market Value of
the Investment appreciation the Investment
(Rs. In Crore)
(Rs. in Crore) (Depreciation)
Commercial Banks (50%) 50 30% 65
Hydropower & Hotels (20%) 20 15% 23

Insurance (30%) 30 (20%) 24


Total 100 112
Total No. of Units 10 Crore
NAV Per Unit 112/10= Rs. 11.20 Per unit
There is an appreciation in the value of the units as the NAV per unit has increased than FV.
Here, the Appreciation % = (11.2-10.0)×100/10.0 = 12%
18. Leverage Effect
Amount in Rs.
a. Profit for equity fund after current cost adjustment 172,000

b. Profit as per Long-term fund approach:


Profit for equity fund 172,000
Add: Interest on Long-term loan (Rs. 450,000 x 10%) 45,000 217,000

1,040,000
c. Capital employed as per Long-term fund approach:

Current cost of capital employed (by Equity approach) 1,040,000

Add: 10% Long term loan 450,000


1,490,000
d. Value of Goodwill:
(A) By equity approach:
Capitalized value of Profit as per equity
approach
1,102,564
= 172,000/15.60 ×100
Less: Capital employed as per equity approach (1,040,000)

Value of Goodwill 62,564


(B) By long-term fund approach:
Capitalized value of Profit as per long-term
fund approach = 217,000/13.5 ×100
1,607,407
Less: Capital employed as per Long-term
fund approach (1,490,000)
Value of goodwill 117,407
Leverage effect on Goodwill:
Adverse Leverage effect on goodwill is Rs. 54,843 (i.e. Rs. 117,407 – Rs. 62,564).
19.
i. Economic Value Added (EVA) is primarily a benchmark to measure earnings efficiency.
Though the term "Economic Profit" was very much there since the inception of
"Economics", Stern Stewart & Co., of USA has got a registered Trade Mark for this by the
name "EVA", an acronym for Economic Value Added.
EVA as a residual income measure of financial performance is simply the operating profit
after tax less a charge for the capital, equity as well as debt, used in the business. EVA
includes both profit or loss as well as balance sheet efficiency as well as the ROCE, or
ROE.
In addition, EVA is a management tool to focus managers on the impact of their decisions
in increasing shareholders‟ wealth. These include both strategic decisions such as what
investments to make, which businesses to exit, what financing structure is optimal; as well
as operational decisions involving trade-offs between profit and asset efficiency such as
whether to make in house or outsource, repair or replace a piece of equipment, whether to
make short or long production runs etc.
Most importantly the real key to increasing shareholder wealth is to integrate the EVA
framework in four key areas; to measure business performance; to guide managerial
decision making; to align managerial incentives with shareholders' interests; and to
improve the financial and business literacy throughout the organization.
To better align managers interests with Shareholders – the EVA framework needs to be
holistically applied in an integrated approach – simply measuring EVAs is not enough it
must also become the basis of key management decisions as well as be linked to senior
management's variable compensation.
ii. Economic Value Added
EVA of Concern
E.V.A. = NOPAT – COCE
NOPAT = Net Operating Profit after Tax
COCE = Cost of Capital Employed
COCE = Weighted Average Cost of Capital x Average Capital Employed
= WACC x Capital Employed
Debt Capital Rs.2,000 crores
Equity capital 500 + 7,500 = Rs.8,000 crores
Capital employed = 2,000+8,000 = Rs.10,000 crores
Debt to capital employed = 2,000/10,000 =0.20
Equity to Capital employed = 8,000/10,000 =0.80
Debt cost before Tax 12%
Less: Tax (30% of 12%) 3.6%
Debt cost after Tax 8.4%
According to Capital Asset Pricing Model (CAPM)
Cost of Equity Capital
= Risk Free Rate + Beta x Equity Risk Premium; Or
= Risk Free Rate + Beta (Market Rate – Risk Free Rate)
= 9 + 1.05 x (19-9)
= 9 + 1.05 x 10 = 19.5%
WACC
= Equity to CE x Cost of Equity capital + Debt to CE x Cost of debt
= 0.8x 19.5% + 0.20x 8.40%
= 15.60% + 1.68% = 17.28%
COCE
= WACC x Capital employed
= 17.28% x 10,000 crores = 1728 crores
E.V.A.
= NOPAT – COCE
= Rs. 2,100 – Rs. 1,728 = Rs. 372 crores
20. Consolidated Financial Statements
Consolidated Statement of Financial Position of the Group as on 32nd Ashadh 2079
Particulars Note No. (In lakh)
ASSETS
Non-current assets
Property, plant and equipment 1 980
Current assets
(a) Inventory 2 338
(b) Financial assets
Trade receivable 3 580
Bills receivable 4 2
Cash and cash equivalents 5 308
Total assets 2208
EQUITY & LIABILITIES
Equity attributable to owners of the parent
Share capital 600
Other Equity
Reserves (W.N.5) 194
Retained Earnings (W.N.5) 179.8
Capital Reserve (W.N.3) 188
Non-controlling interests (W.N.4) 166.2
Total equity 1328
LAIBILITES Nil
Non-current liabilities
Current Liabilities
(a) Financial liabilities
(i) Trade payable 6 880

Total Liabilities 880


Total equity and liabilities 2208
Note to Accounts (in lakhs)
Working Notes:

1. Property, Plant & Equipment


A Ltd. 320
B Ltd. 360
BB Ltd. 300 980
2. Inventories
A Ltd. 220
B Ltd. (70-2) 68
BB Ltd. 50 338
3. Trade Receivables
A Ltd. 260
B Ltd. 100
BB Ltd. 220 580
4. Bills Receivables
A Ltd. (72-70) 2
BB Ltd. (30-30) - 2
Cash & Cash equivalents
5. A Ltd. 228
B Ltd. 40
BB Ltd. 40 308
Trade Payable
6. A Ltd. 470
B Ltd. 230
BB Ltd. 180 880

1. Analysis of Reserves and Surplus ( in lakhs)


B Ltd. BB Ltd.
Reserves as on 31.3.2078 80 60
Increase during the year 2078-2079 20 20
Increase for the half year till 30.6.2078 10 10
Balance as on 30.6.2078(A) 90 70
Total Balance as on 32.3.2079 100 80
Post-acquisition balance 10 10
B Ltd. BB Ltd.
Retained Earnings as on 31.3.2078 20 30
Increase during the year 2078-2079 30 30
Increase for the half year till 30.6.2078 15 15
Balance as on 30.6.2078(B) 35 45
Total Balance as on 32.3.2079 50 60
Post-acquisition balance 15 15
Less: Unrealized gain on inventories - (2)
(10x25%)
Post-acquisition balance for CFS 15 13
Total balance on the acquisition date i.e. 125 115
30.06.2078 ( A+B)
2. Calculation of Effective Interest of A Ltd. in BB Ltd.
Acquisition by A Ltd. in B Ltd. = 80%
Acquisition by B Ltd. in BB Ltd. = 75%
Acquisition by Group in BB Ltd. (80%x75%) = 60%
Non-controlling Interest = 40%
3. Calculation of Goodwill / Capital Reserve on the acquisition date
Particulars B Ltd. BB Ltd.
Investment or Consideration 340 (280x80%)
224
Add: NCI at Fair value
(400x20%) 80
(320x40%) - 128
420 352
Less: Identifiable net assets (Share capital
+Increase in the Reserves and surplus till
(400+125) (525) (320+115)
acquisition date)
(435)
Capital Reserves 105 83
Total Capital Reserve (105+83) 188
4. Calculation of Non-Controlling Interest
B Ltd. BB Ltd.
At Fair value (See Note 3) 80 128
Add: Post Acquisition Reserve (See Note 1) (10x20%) 2 (10x40%) 4
Add: Post Acquisition Retained Earnings (See (15x20%) 3 (13x40%) 4
Note 1)
Less: NCI share of investment in SS Ltd. (280x20%) (56)* -
29 137.2
Total (29+137.2) 166.2
Note: The Non-controlling interest in B Ltd. will take its proportion in BB Ltd. so they
have to bear their proportion in the investment by B Ltd. (in BB Ltd.) also.
5. Calculation of Consolidated Equity
Particulars Reserves Retained Earnings
A Ltd 180 160
Add: Share in B Ltd. (10x80%)8 (15x80%)12
Add: Share in BB Ltd. (10x60%)6 (13x60%)7.8
194 179.8

You might also like